ΠŸΠΎΠΊΠ°Π·Π°Ρ‚Π΅Π»ΠΈ стСпСнСй ΠΏΡ€ΠΈ слоТСнии: Π‘Π»ΠΎΠΆΠ΅Π½ΠΈΠ΅ стСпСнСй с ΠΎΠ΄ΠΈΠ½Π°ΠΊΠΎΠ²Ρ‹ΠΌΠΈ показатСлями. Π‘Π»ΠΎΠΆΠ΅Π½ΠΈΠ΅, Π²Ρ‹Ρ‡ΠΈΡ‚Π°Π½ΠΈΠ΅, ΡƒΠΌΠ½ΠΎΠΆΠ΅Π½ΠΈΠ΅, ΠΈ Π΄Π΅Π»Π΅Π½ΠΈΠ΅ стСпСнСй

Π‘ΠΎΠ΄Π΅Ρ€ΠΆΠ°Π½ΠΈΠ΅

Бвойства стСпСнСй ΠΏΡ€ΠΈ слоТСнии. ΠžΡΠ½ΠΎΠ²Π½Ρ‹Π΅ свойства стСпСнСй. ΠŸΡ€ΠΈΠΌΠ΅Ρ€Ρ‹ Π½Π° ΡƒΠΌΠ½ΠΎΠΆΠ΅Π½ΠΈΠ΅ стСпСнСй с ΠΎΠ΄ΠΈΠ½Π°ΠΊΠΎΠ²Ρ‹ΠΌΠΈ показатСлями

КаТдая арифмСтичСская опСрация ΠΏΠΎΡ€ΠΎΡŽ становится слишком Π³Ρ€ΠΎΠΌΠΎΠ·Π΄ΠΊΠΎΠΉ для записи ΠΈ Π΅Ρ‘ ΡΡ‚Π°Ρ€Π°ΡŽΡ‚ΡΡ ΡƒΠΏΡ€ΠΎΡΡ‚ΠΈΡ‚ΡŒ. Когда-Ρ‚ΠΎ Ρ‚Π°ΠΊ Π±Ρ‹Π»ΠΎ ΠΈ с ΠΎΠΏΠ΅Ρ€Π°Ρ†ΠΈΠ΅ΠΉ слоТСния. Π›ΡŽΠ΄ΡΠΌ Π±Ρ‹Π»ΠΎ Π½Π΅ΠΎΠ±Ρ…ΠΎΠ΄ΠΈΠΌΠΎ ΠΏΡ€ΠΎΠ²ΠΎΠ΄ΠΈΡ‚ΡŒ ΠΌΠ½ΠΎΠ³ΠΎΠΊΡ€Π°Ρ‚Π½ΠΎΠ΅ ΠΎΠ΄Π½ΠΎΡ‚ΠΈΠΏΠ½ΠΎΠ΅ слоТСниС, Π½Π°ΠΏΡ€ΠΈΠΌΠ΅Ρ€, ΠΏΠΎΡΡ‡ΠΈΡ‚Π°Ρ‚ΡŒ ΡΡ‚ΠΎΠΈΠΌΠΎΡΡ‚ΡŒ ста пСрсидских ΠΊΠΎΠ²Ρ€ΠΎΠ², ΡΡ‚ΠΎΠΈΠΌΠΎΡΡ‚ΡŒ ΠΊΠΎΡ‚ΠΎΡ€ΠΎΠ³ΠΎ составляСт 3 Π·ΠΎΠ»ΠΎΡ‚Ρ‹Π΅ ΠΌΠΎΠ½Π΅Ρ‚Ρ‹ Π·Π° ΠΊΠ°ΠΆΠ΄Ρ‹ΠΉ. 3+3+3+…+3 = 300. Из-Π·Π° громоздкости Π±Ρ‹Π»ΠΎ ΠΏΡ€ΠΈΠ΄ΡƒΠΌΠ°Π½ΠΎ ΡΠΎΠΊΡ€Π°Ρ‚ΠΈΡ‚ΡŒ запись Π΄ΠΎ 3 * 100 = 300. ЀактичСски, запись Β«Ρ‚Ρ€ΠΈ ΡƒΠΌΠ½ΠΎΠΆΠΈΡ‚ΡŒ Π½Π° сто» ΠΎΠ·Π½Π°Ρ‡Π°Π΅Ρ‚, Ρ‡Ρ‚ΠΎ Π½ΡƒΠΆΠ½ΠΎ Π²Π·ΡΡ‚ΡŒ сто Ρ‚Ρ€ΠΎΠ΅ΠΊ ΠΈ ΡΠ»ΠΎΠΆΠΈΡ‚ΡŒ ΠΌΠ΅ΠΆΠ΄Ρƒ собой. Π£ΠΌΠ½ΠΎΠΆΠ΅Π½ΠΈΠ΅ ΠΏΡ€ΠΈΠΆΠΈΠ»ΠΎΡΡŒ, ΠΎΠ±Ρ€Π΅Π»ΠΎ ΠΎΠ±Ρ‰ΡƒΡŽ ΠΏΠΎΠΏΡƒΠ»ΡΡ€Π½ΠΎΡΡ‚ΡŒ. Но ΠΌΠΈΡ€ Π½Π΅ стоит Π½Π° мСстС, ΠΈ Π² срСдних Π²Π΅ΠΊΠ°Ρ… Π²ΠΎΠ·Π½ΠΈΠΊΠ»Π° Π½Π΅ΠΎΠ±Ρ…ΠΎΠ΄ΠΈΠΌΠΎΡΡ‚ΡŒ ΠΏΡ€ΠΎΠ²ΠΎΠ΄ΠΈΡ‚ΡŒ ΠΌΠ½ΠΎΠ³ΠΎΠΊΡ€Π°Ρ‚Π½ΠΎΠ΅ ΠΎΠ΄Π½ΠΎΡ‚ΠΈΠΏΠ½ΠΎΠ΅ ΡƒΠΌΠ½ΠΎΠΆΠ΅Π½ΠΈΠ΅. ВспоминаСтся старая индийская Π·Π°Π³Π°Π΄ΠΊΠ° ΠΎ ΠΌΡƒΠ΄Ρ€Π΅Ρ†Π΅, ΠΏΠΎΠΏΡ€ΠΎΡΠΈΠ²ΡˆΠ΅ΠΌ Π² Π½Π°Π³Ρ€Π°Π΄Ρƒ Π·Π° Π²Ρ‹ΠΏΠΎΠ»Π½Π΅Π½Π½ΡƒΡŽ Ρ€Π°Π±ΠΎΡ‚Ρƒ ΠΏΡˆΠ΅Π½ΠΈΡ‡Π½Ρ‹Π΅ Π·Ρ‘Ρ€Π½Π° Π² ΡΠ»Π΅Π΄ΡƒΡŽΡ‰Π΅ΠΌ количСствС: Π·Π° ΠΏΠ΅Ρ€Π²ΡƒΡŽ ΠΊΠ»Π΅Ρ‚ΠΊΡƒ ΡˆΠ°Ρ…ΠΌΠ°Ρ‚Π½ΠΎΠΉ доски ΠΎΠ½ просил ΠΎΠ΄Π½ΠΎ Π·Π΅Ρ€Π½ΠΎ, Π·Π° Π²Ρ‚ΠΎΡ€ΡƒΡŽ – Π΄Π²Π°, Ρ‚Ρ€Π΅Ρ‚ΡŒΡŽ – Ρ‡Π΅Ρ‚Ρ‹Ρ€Π΅, ΠΏΡΡ‚ΡƒΡŽ – восСмь ΠΈ Ρ‚Π°ΠΊ Π΄Π°Π»Π΅Π΅. 3. Π’ ΠΎΡΡ‚Π°Π»ΡŒΠ½ΠΎΠΌ, ΠΊΠΎΠ³Π΄Π° Ρ€Π°Π·Π»ΠΈΡ‡Π½Ρ‹Π΅ основания ΠΈ ΠΏΠΎΠΊΠ°Π·Π°Ρ‚Π΅Π»ΠΈ, произвСсти ΠΏΠΎΠ»Π½ΠΎΠ΅ ΡƒΠΌΠ½ΠΎΠΆΠ΅Π½ΠΈΠ΅ нСльзя. Иногда ΠΌΠΎΠΆΠ½ΠΎ частично ΡƒΠΏΡ€ΠΎΡΡ‚ΠΈΡ‚ΡŒ ΠΈΠ»ΠΈ ΠΏΡ€ΠΈΠ±Π΅Π³Π½ΡƒΡ‚ΡŒ ΠΊ ΠΏΠΎΠΌΠΎΡ‰ΠΈ Π²Ρ‹Ρ‡ΠΈΡΠ»ΠΈΡ‚Π΅Π»ΡŒΠ½ΠΎΠΉ Ρ‚Π΅Ρ…Π½ΠΈΠΊΠΈ.

Π€ΠΎΡ€ΠΌΡƒΠ»Ρ‹ стСпСнСй ΠΈΡΠΏΠΎΠ»ΡŒΠ·ΡƒΡŽΡ‚ Π² процСссС сокращСния ΠΈ упрощСния слоТных Π²Ρ‹Ρ€Π°ΠΆΠ΅Π½ΠΈΠΉ, Π² Ρ€Π΅ΡˆΠ΅Π½ΠΈΠΈ ΡƒΡ€Π°Π²Π½Π΅Π½ΠΈΠΉ ΠΈ нСравСнств.

Число c являСтся n -Π½ΠΎΠΉ ΡΡ‚Π΅ΠΏΠ΅Π½ΡŒΡŽ числа a ΠΊΠΎΠ³Π΄Π°:

ΠžΠΏΠ΅Ρ€Π°Ρ†ΠΈΠΈ со стСпСнями.

1. УмноТая стСпСни с ΠΎΠ΄ΠΈΠ½Π°ΠΊΠΎΠ²Ρ‹ΠΌ основаниСм ΠΈΡ… ΠΏΠΎΠΊΠ°Π·Π°Ρ‚Π΅Π»ΠΈ ΡΠΊΠ»Π°Π΄Ρ‹Π²Π°ΡŽΡ‚ΡΡ:

a m Β·a n = a m + n .

2. Π’ Π΄Π΅Π»Π΅Π½ΠΈΠΈ стСпСнСй с ΠΎΠ΄ΠΈΠ½Π°ΠΊΠΎΠ²Ρ‹ΠΌ основаниСм ΠΈΡ… ΠΏΠΎΠΊΠ°Π·Π°Ρ‚Π΅Π»ΠΈ Π²Ρ‹Ρ‡ΠΈΡ‚Π°ΡŽΡ‚ΡΡ:

3. Π‘Ρ‚Π΅ΠΏΠ΅Π½ΡŒ произвСдСния 2-Ρ… Π»ΠΈΠ±ΠΎ большСго числа ΠΌΠ½ΠΎΠΆΠΈΡ‚Π΅Π»Π΅ΠΉ равняСтся ΠΏΡ€ΠΎΠΈΠ·Π²Π΅Π΄Π΅Π½ΠΈΡŽ стСпСнСй этих сомноТитСлСй:

(abc…) n = a n Β· b n Β· c n …

4. Π‘Ρ‚Π΅ΠΏΠ΅Π½ΡŒ Π΄Ρ€ΠΎΠ±ΠΈ равняСтся ΠΎΡ‚Π½ΠΎΡˆΠ΅Π½ΠΈΡŽ стСпСнСй Π΄Π΅Π»ΠΈΠΌΠΎΠ³ΠΎ ΠΈ дСлитСля:

(a/b) n = a n /b n .

5. Возводя ΡΡ‚Π΅ΠΏΠ΅Π½ΡŒ Π² ΡΡ‚Π΅ΠΏΠ΅Π½ΡŒ, ΠΏΠΎΠΊΠ°Π·Π°Ρ‚Π΅Π»ΠΈ стСпСнСй ΠΏΠ΅Ρ€Π΅ΠΌΠ½ΠΎΠΆΠ°ΡŽΡ‚:

(a m) n = a m n .

КаТдая Π²Ρ‹ΡˆΠ΅ΠΏΡ€ΠΈΠ²Π΅Π΄Π΅Π½Π½Π°Ρ Ρ„ΠΎΡ€ΠΌΡƒΠ»Π° Π²Π΅Ρ€Π½Π° Π² направлСниях слСва Π½Π°ΠΏΡ€Π°Π²ΠΎ ΠΈ Π½Π°ΠΎΠ±ΠΎΡ€ΠΎΡ‚.

НапримСр . (2Β·3Β·5/15)Β² = 2Β²Β·3Β²Β·5Β²/15Β² = 900/225 = 4 .

ΠžΠΏΠ΅Ρ€Π°Ρ†ΠΈΠΈ с корнями.

1. ΠšΠΎΡ€Π΅Π½ΡŒ ΠΈΠ· произвСдСния Π½Π΅ΡΠΊΠΎΠ»ΡŒΠΊΠΈΡ… сомноТитСлСй равняСтся ΠΏΡ€ΠΎΠΈΠ·Π²Π΅Π΄Π΅Π½ΠΈΡŽ ΠΊΠΎΡ€Π½Π΅ΠΉ ΠΈΠ· этих сомноТитСлСй:

2. ΠšΠΎΡ€Π΅Π½ΡŒ ΠΈΠ· ΠΎΡ‚Π½ΠΎΡˆΠ΅Π½ΠΈΡ Ρ€Π°Π²Π΅Π½ ΠΎΡ‚Π½ΠΎΡˆΠ΅Π½ΠΈΡŽ Π΄Π΅Π»ΠΈΠΌΠΎΠ³ΠΎ ΠΈ дСлитСля ΠΊΠΎΡ€Π½Π΅ΠΉ:

3. ΠŸΡ€ΠΈ Π²ΠΎΠ·Π²Π΅Π΄Π΅Π½ΠΈΠΈ корня Π² ΡΡ‚Π΅ΠΏΠ΅Π½ΡŒ довольно возвСсти Π² эту ΡΡ‚Π΅ΠΏΠ΅Π½ΡŒ ΠΏΠΎΠ΄ΠΊΠΎΡ€Π΅Π½Π½ΠΎΠ΅ число:

4. Если ΡƒΠ²Π΅Π»ΠΈΡ‡ΠΈΡ‚ΡŒ ΡΡ‚Π΅ΠΏΠ΅Π½ΡŒ корня Π² n Ρ€Π°Π· ΠΈ Π² Ρ‚ΠΎΠΆΠ΅ врСмя возвСсти Π² n -ΡƒΡŽ ΡΡ‚Π΅ΠΏΠ΅Π½ΡŒ ΠΏΠΎΠ΄ΠΊΠΎΡ€Π΅Π½Π½ΠΎΠ΅ число, Ρ‚ΠΎ Π·Π½Π°Ρ‡Π΅Π½ΠΈΠ΅ корня Π½Π΅ помСняСтся:

5. Если ΡƒΠΌΠ΅Π½ΡŒΡˆΠΈΡ‚ΡŒ ΡΡ‚Π΅ΠΏΠ΅Π½ΡŒ корня Π² n Ρ€Π°Π· ΠΈ Π² Ρ‚ΠΎΠΆΠ΅ врСмя ΠΈΠ·Π²Π»Π΅Ρ‡ΡŒ ΠΊΠΎΡ€Π΅Π½ΡŒ n -ΠΎΠΉ стСпСни ΠΈΠ· ΠΏΠΎΠ΄ΠΊΠΎΡ€Π΅Π½Π½ΠΎΠ³ΠΎ числа, Ρ‚ΠΎ Π·Π½Π°Ρ‡Π΅Π½ΠΈΠ΅ корня Π½Π΅ помСняСтся:

Π‘Ρ‚Π΅ΠΏΠ΅Π½ΡŒ с ΠΎΡ‚Ρ€ΠΈΡ†Π°Ρ‚Π΅Π»ΡŒΠ½Ρ‹ΠΌ ΠΏΠΎΠΊΠ°Π·Π°Ρ‚Π΅Π»Π΅ΠΌ. Π‘Ρ‚Π΅ΠΏΠ΅Π½ΡŒ Π½Π΅ΠΊΠΎΡ‚ΠΎΡ€ΠΎΠ³ΠΎ числа с Π½Π΅ΠΏΠΎΠ»ΠΎΠΆΠΈΡ‚Π΅Π»ΡŒΠ½Ρ‹ΠΌ (Ρ†Π΅Π»Ρ‹ΠΌ) ΠΏΠΎΠΊΠ°Π·Π°Ρ‚Π΅Π»Π΅ΠΌ ΠΎΠΏΡ€Π΅Π΄Π΅Π»ΡΡŽΡ‚ ΠΊΠ°ΠΊ Π΅Π΄ΠΈΠ½ΠΈΡ†Ρƒ, Π΄Π΅Π»Π΅Π½Π½ΡƒΡŽ Π½Π° ΡΡ‚Π΅ΠΏΠ΅Π½ΡŒ Ρ‚ΠΎΠ³ΠΎ ΠΆΠ΅ числа с ΠΏΠΎΠΊΠ°Π·Π°Ρ‚Π΅Π»Π΅ΠΌ, Ρ€Π°Π²Π½Ρ‹ΠΌ Π°Π±ΡΠΎΠ»ΡŽΡ‚Π½ΠΎΠΉ Π²Π΅Π»ΠΈΡ‡ΠΈΠ½Π΅ Π½Π΅ΠΏΠΎΠ»ΠΎΠΆΠΈΡ‚Π΅Π»ΡŒΠ½ΠΎΠ³ΠΎ показатСля:

Π€ΠΎΡ€ΠΌΡƒΠ»Ρƒ a m :a n =a m — n ΠΌΠΎΠΆΠ½ΠΎ ΠΈΡΠΏΠΎΠ»ΡŒΠ·ΠΎΠ²Π°Ρ‚ΡŒ Π½Π΅ Ρ‚ΠΎΠ»ΡŒΠΊΠΎ ΠΏΡ€ΠΈ m > n , Π½ΠΎ ΠΈ ΠΏΡ€ΠΈ m n .

НапримСр . a 4:a 7 = a 4 — 7 = a -3 .

Π§Ρ‚ΠΎΠ±Ρ‹ Ρ„ΠΎΡ€ΠΌΡƒΠ»Π° a m :a n =a m — n стала справСдливой ΠΏΡ€ΠΈ m=n , Π½ΡƒΠΆΠ½ΠΎ присутствиС Π½ΡƒΠ»Π΅Π²ΠΎΠΉ стСпСни.

Π‘Ρ‚Π΅ΠΏΠ΅Π½ΡŒ с Π½ΡƒΠ»Π΅Π²Ρ‹ΠΌ ΠΏΠΎΠΊΠ°Π·Π°Ρ‚Π΅Π»Π΅ΠΌ. Π‘Ρ‚Π΅ΠΏΠ΅Π½ΡŒ всякого числа, Π½Π΅ Ρ€Π°Π²Π½ΠΎΠ³ΠΎ Π½ΡƒΠ»ΡŽ, с Π½ΡƒΠ»Π΅Π²Ρ‹ΠΌ ΠΏΠΎΠΊΠ°Π·Π°Ρ‚Π΅Π»Π΅ΠΌ равняСтся Π΅Π΄ΠΈΠ½ΠΈΡ†Π΅.

НапримСр . 2 0 = 1,(-5) 0 = 1,(-3/5) 0 = 1.

Π‘Ρ‚Π΅ΠΏΠ΅Π½ΡŒ с Π΄Ρ€ΠΎΠ±Π½Ρ‹ΠΌ ΠΏΠΎΠΊΠ°Π·Π°Ρ‚Π΅Π»Π΅ΠΌ. Π§Ρ‚ΠΎΠ±Ρ‹ возвСсти Π΄Π΅ΠΉΡΡ‚Π²ΠΈΡ‚Π΅Π»ΡŒΠ½ΠΎΠ΅ число Π° Π² ΡΡ‚Π΅ΠΏΠ΅Π½ΡŒ m/n , Π½Π΅ΠΎΠ±Ρ…ΠΎΠ΄ΠΈΠΌΠΎ ΠΈΠ·Π²Π»Π΅Ρ‡ΡŒ ΠΊΠΎΡ€Π΅Π½ΡŒ n -ΠΎΠΉ стСпСни ΠΈΠ· m -ΠΎΠΉ стСпСни этого числа Π° .

ΠžΠΏΠ΅Ρ€Π°Ρ†ΠΈΠΈ со стСпСнями ΠΈ корнями. Π‘Ρ‚Π΅ΠΏΠ΅Π½ΡŒ с ΠΎΡ‚Ρ€ΠΈΡ†Π°Ρ‚Π΅Π»ΡŒΠ½Ρ‹ΠΌ

,

Π½ΡƒΠ»Π΅Π²Ρ‹ΠΌ ΠΈ Π΄Ρ€ΠΎΠ±Π½Ρ‹ΠΌ ΠΏΠΎΠΊΠ°Π·Π°Ρ‚Π΅Π»Π΅ΠΌ. О выраТСниях, Π½Π΅ ΠΈΠΌΠ΅ΡŽΡ‰ΠΈΡ… смысла.

ΠžΠΏΠ΅Ρ€Π°Ρ†ΠΈΠΈ со стСпСнями.

1. ΠŸΡ€ΠΈ ΡƒΠΌΠ½ΠΎΠΆΠ΅Π½ΠΈΠΈ стСпСнСй с ΠΎΠ΄ΠΈΠ½Π°ΠΊΠΎΠ²Ρ‹ΠΌ основаниСм ΠΈΡ… ΠΏΠΎΠΊΠ°Π·Π°Ρ‚Π΅Π»ΠΈ ΡΠΊΠ»Π°Π΄Ρ‹Π²Π°ΡŽΡ‚ΡΡ :

a m Β· a n = a m + n .

2. ΠŸΡ€ΠΈ Π΄Π΅Π»Π΅Π½ΠΈΠΈ стСпСнСй с ΠΎΠ΄ΠΈΠ½Π°ΠΊΠΎΠ²Ρ‹ΠΌ основаниСм ΠΈΡ… ΠΏΠΎΠΊΠ°Π·Π°Ρ‚Π΅Π»ΠΈ Π²Ρ‹Ρ‡ΠΈΡ‚Π°ΡŽΡ‚ΡΡ .

3. Π‘Ρ‚Π΅ΠΏΠ΅Π½ΡŒ произвСдСния Π΄Π²ΡƒΡ… ΠΈΠ»ΠΈ Π½Π΅ΡΠΊΠΎΠ»ΡŒΠΊΠΈΡ… сомноТитСлСй Ρ€Π°Π²Π½Π° ΠΏΡ€ΠΎΠΈΠ·Π²Π΅Π΄Π΅Π½ΠΈΡŽ стСпСнСй этих сомноТитСлСй.

( abc … ) n = a n Β· b n Β· c n …

4. Π‘Ρ‚Π΅ΠΏΠ΅Π½ΡŒ ΠΎΡ‚Π½ΠΎΡˆΠ΅Π½ΠΈΡ (Π΄Ρ€ΠΎΠ±ΠΈ) Ρ€Π°Π²Π½Π° ΠΎΡ‚Π½ΠΎΡˆΠ΅Π½ΠΈΡŽ стСпСнСй Π΄Π΅Π»ΠΈΠΌΠΎΠ³ΠΎ (числитСля) ΠΈ дСлитСля (знамСнатСля):

( a / b ) n = a n / b n .

5. ΠŸΡ€ΠΈ Π²ΠΎΠ·Π²Π΅Π΄Π΅Π½ΠΈΠΈ стСпСни Π² ΡΡ‚Π΅ΠΏΠ΅Π½ΡŒ ΠΈΡ… ΠΏΠΎΠΊΠ°Π·Π°Ρ‚Π΅Π»ΠΈ ΠΏΠ΅Ρ€Π΅ΠΌΠ½ΠΎΠΆΠ°ΡŽΡ‚ΡΡ:

(a m )

n = a m n .

ВсС Π²Ρ‹ΡˆΠ΅ΠΏΡ€ΠΈΠ²Π΅Π΄Π΅Π½Π½Ρ‹Π΅ Ρ„ΠΎΡ€ΠΌΡƒΠ»Ρ‹ Ρ‡ΠΈΡ‚Π°ΡŽΡ‚ΡΡ ΠΈ Π²Ρ‹ΠΏΠΎΠ»Π½ΡΡŽΡ‚ΡΡ Π² ΠΎΠ±ΠΎΠΈΡ… направлСниях слСва Π½Π°ΠΏΡ€Π°Π²ΠΎ ΠΈ Π½Π°ΠΎΠ±ΠΎΡ€ΠΎΡ‚.

П Ρ€ ΠΈ ΠΌ Π΅ Ρ€. (2 Β· 3 Β· 5 / 15) Β² = 2 Β² Β· 3 Β² Β· 5 Β² / 15 Β² = 900 / 225 = 4 .

ΠžΠΏΠ΅Ρ€Π°Ρ†ΠΈΠΈ с корнями. Π’ΠΎ всСх Π½ΠΈΠΆΠ΅ΠΏΡ€ΠΈΠ²Π΅Π΄Π΅Π½Π½Ρ‹Ρ… Ρ„ΠΎΡ€ΠΌΡƒΠ»Π°Ρ… символ ΠΎΠ·Π½Π°Ρ‡Π°Π΅Ρ‚ арифмСтичСский ΠΊΠΎΡ€Π΅Π½ΡŒ (ΠΏΠΎΠ΄ΠΊΠΎΡ€Π΅Π½Π½ΠΎΠ΅ Π²Ρ‹Ρ€Π°ΠΆΠ΅Π½ΠΈΠ΅ ΠΏΠΎΠ»ΠΎΠΆΠΈΡ‚Π΅Π»ΡŒΠ½ΠΎ).

1. ΠšΠΎΡ€Π΅Π½ΡŒ ΠΈΠ· произвСдСния Π½Π΅ΡΠΊΠΎΠ»ΡŒΠΊΠΈΡ… сомноТитСлСй Ρ€Π°Π²Π΅Π½ ΠΏΡ€ΠΎΠΈΠ·Π²Π΅Π΄Π΅Π½ΠΈΡŽ ΠΊΠΎΡ€Π½Π΅ΠΉ ΠΈΠ· этих сомноТитСлСй:

2. ΠšΠΎΡ€Π΅Π½ΡŒ ΠΈΠ· ΠΎΡ‚Π½ΠΎΡˆΠ΅Π½ΠΈΡ Ρ€Π°Π²Π΅Π½ ΠΎΡ‚Π½ΠΎΡˆΠ΅Π½ΠΈΡŽ ΠΊΠΎΡ€Π½Π΅ΠΉ Π΄Π΅Π»ΠΈΠΌΠΎΠ³ΠΎ ΠΈ дСлитСля:

3. ΠŸΡ€ΠΈ Π²ΠΎΠ·Π²Π΅Π΄Π΅Π½ΠΈΠΈ корня Π² ΡΡ‚Π΅ΠΏΠ΅Π½ΡŒ достаточно возвСсти Π² эту ΡΡ‚Π΅ΠΏΠ΅Π½ΡŒ ΠΏΠΎΠ΄ΠΊΠΎΡ€Π΅Π½Π½ΠΎΠ΅ число:

4. Если ΡƒΠ²Π΅Π»ΠΈΡ‡ΠΈΡ‚ΡŒ ΡΡ‚Π΅ΠΏΠ΅Π½ΡŒ корня Π² m Ρ€Π°Π· ΠΈ ΠΎΠ΄Π½ΠΎΠ²Ρ€Π΅ΠΌΠ΅Π½Π½ΠΎ возвСсти Π² m -ΡƒΡŽ ΡΡ‚Π΅ΠΏΠ΅Π½ΡŒ ΠΏΠΎΠ΄ΠΊΠΎΡ€Π΅Π½Π½ΠΎΠ΅ число, Ρ‚ΠΎ Π·Π½Π°Ρ‡Π΅Π½ΠΈΠ΅ корня Π½Π΅ измСнится:

5. Если ΡƒΠΌΠ΅Π½ΡŒΡˆΠΈΡ‚ΡŒ ΡΡ‚Π΅ΠΏΠ΅Π½ΡŒ корня Π² m Ρ€Π°Π· ΠΈ ΠΎΠ΄Π½ΠΎΠ²Ρ€Π΅ΠΌΠ΅Π½Π½ΠΎ ΠΈΠ·Π²Π»Π΅Ρ‡ΡŒ ΠΊΠΎΡ€Π΅Π½ΡŒ m -ΠΎΠΉ стСпСни ΠΈΠ· ΠΏΠΎΠ΄ΠΊΠΎΡ€Π΅Π½Π½ΠΎΠ³ΠΎ числа, Ρ‚ΠΎ Π·Π½Π°Ρ‡Π΅Π½ΠΈΠ΅ корня Π½Π΅ измСнится:


Π Π°ΡΡˆΠΈΡ€Π΅Π½ΠΈΠ΅ понятия стСпСни. Π”ΠΎ сих ΠΏΠΎΡ€ ΠΌΡ‹ рассматривали стСпСни Ρ‚ΠΎΠ»ΡŒΠΊΠΎ с Π½Π°Ρ‚ΡƒΡ€Π°Π»ΡŒΠ½Ρ‹ΠΌ ΠΏΠΎΠΊΠ°Π·Π°Ρ‚Π΅Π»Π΅ΠΌ; Π½ΠΎ дСйствия со стСпСнями ΠΈ корнями ΠΌΠΎΠ³ΡƒΡ‚ ΠΏΡ€ΠΈΠ²ΠΎΠ΄ΠΈΡ‚ΡŒ Ρ‚Π°ΠΊΠΆΠ΅ ΠΊ ΠΎΡ‚Ρ€ΠΈΡ†Π°Ρ‚Π΅Π»ΡŒΠ½Ρ‹ΠΌ , Π½ΡƒΠ»Π΅Π²Ρ‹ΠΌ ΠΈ Π΄Ρ€ΠΎΠ±Π½Ρ‹ΠΌ показатСлям. ВсС эти ΠΏΠΎΠΊΠ°Π·Π°Ρ‚Π΅Π»ΠΈ стСпСнСй Ρ‚Ρ€Π΅Π±ΡƒΡŽΡ‚ Π΄ΠΎΠΏΠΎΠ»Π½ΠΈΡ‚Π΅Π»ΡŒΠ½ΠΎΠ³ΠΎ опрСдСлСния.

Π‘Ρ‚Π΅ΠΏΠ΅Π½ΡŒ с ΠΎΡ‚Ρ€ΠΈΡ†Π°Ρ‚Π΅Π»ΡŒΠ½Ρ‹ΠΌ ΠΏΠΎΠΊΠ°Π·Π°Ρ‚Π΅Π»Π΅ΠΌ. Π‘Ρ‚Π΅ΠΏΠ΅Π½ΡŒ Π½Π΅ΠΊΠΎΡ‚ΠΎΡ€ΠΎΠ³ΠΎ числа с ΠΎΡ‚Ρ€ΠΈΡ†Π°Ρ‚Π΅Π»ΡŒΠ½Ρ‹ΠΌ (Ρ†Π΅Π»Ρ‹ΠΌ) ΠΏΠΎΠΊΠ°Π·Π°Ρ‚Π΅Π»Π΅ΠΌ опрСдСляСтся ΠΊΠ°ΠΊ Π΅Π΄ΠΈΠ½ΠΈΡ†Π°, дСлённая Π½Π° ΡΡ‚Π΅ΠΏΠ΅Π½ΡŒ Ρ‚ΠΎΠ³ΠΎ ΠΆΠ΅ числа с ΠΏΠΎΠΊΠ°Π·Π°Ρ‚Π΅Π»Π΅ΠΌ, Ρ€Π°Π²Π½Ρ‹ΠΌ Π°Π±ΡΠΎΠ»ΡŽΡ‚Π½ΠΎΠΉ Π²Π΅Π»Π΅Ρ‡ΠΈΠ½Π΅ ΠΎΡ‚Ρ€ΠΈΡ†Π°Ρ‚Π΅Π»ΡŒΠ½ΠΎΠ³ΠΎ показатСля:

Π’ Π΅ΠΏΠ΅Ρ€ΡŒ Ρ„ΠΎΡ€ΠΌΡƒΠ»Π° a m : a n = a m — n ΠΌΠΎΠΆΠ΅Ρ‚ Π±Ρ‹Ρ‚ΡŒ использована Π½Π΅ Ρ‚ΠΎΠ»ΡŒΠΊΠΎ ΠΏΡ€ΠΈ m , большСм, Ρ‡Π΅ΠΌ n , Π½ΠΎ ΠΈ ΠΏΡ€ΠΈ m , мСньшСм, Ρ‡Π΅ΠΌ n .

П Ρ€ ΠΈ ΠΌ Π΅ Ρ€ . a 4 : a 7 = a 4 — 7 = a — 3 .

Если ΠΌΡ‹ Ρ…ΠΎΡ‚ΠΈΠΌ, Ρ‡Ρ‚ΠΎΠ±Ρ‹ Ρ„ΠΎΡ€ΠΌΡƒΠ»Π° a m : a n = a m n Π±Ρ‹Π»Π° справСдлива ΠΏΡ€ΠΈ m = n , Π½Π°ΠΌ Π½Π΅ΠΎΠ±Ρ…ΠΎΠ΄ΠΈΠΌΠΎ ΠΎΠΏΡ€Π΅Π΄Π΅Π»Π΅Π½ΠΈΠ΅ Π½ΡƒΠ»Π΅Π²ΠΎΠΉ стСпСни.

Π‘Ρ‚Π΅ΠΏΠ΅Π½ΡŒ с Π½ΡƒΠ»Π΅Π²Ρ‹ΠΌ ΠΏΠΎΠΊΠ°Π·Π°Ρ‚Π΅Π»Π΅ΠΌ. Π‘Ρ‚Π΅ΠΏΠ΅Π½ΡŒ любого Π½Π΅Π½ΡƒΠ»Π΅Π²ΠΎΠ³ΠΎ числа с Π½ΡƒΠ»Π΅Π²Ρ‹ΠΌ ΠΏΠΎΠΊΠ°Π·Π°Ρ‚Π΅Π»Π΅ΠΌ Ρ€Π°Π²Π½Π° 1.

П Ρ€ ΠΈ ΠΌ Π΅ Ρ€ Ρ‹. 2 0 = 1, (– 5) 0 = 1, (– 3 / 5) 0 = 1.

Π‘Ρ‚Π΅ΠΏΠ΅Π½ΡŒ с Π΄Ρ€ΠΎΠ±Π½Ρ‹ΠΌ ΠΏΠΎΠΊΠ°Π·Π°Ρ‚Π΅Π»Π΅ΠΌ. Для Ρ‚ΠΎΠ³ΠΎ, Ρ‡Ρ‚ΠΎΠ±Ρ‹ возвСсти Π΄Π΅ΠΉΡΡ‚Π²ΠΈΡ‚Π΅Π»ΡŒΠ½ΠΎΠ΅ число Π° Π² ΡΡ‚Π΅ΠΏΠ΅Π½ΡŒ m / n , Π½ΡƒΠΆΠ½ΠΎ ΠΈΠ·Π²Π»Π΅Ρ‡ΡŒ ΠΊΠΎΡ€Π΅Π½ΡŒ n –ой стСпСни ΠΈΠ· m -ΠΎΠΉ стСпСни этого числа Π° :

О выраТСниях, Π½Π΅ ΠΈΠΌΠ΅ΡŽΡ‰ΠΈΡ… смысла. Π•ΡΡ‚ΡŒ нСсколько Ρ‚Π°ΠΊΠΈΡ… Π²Ρ‹Ρ€Π°ΠΆΠ΅Π½ΠΈΠΉ. любоС число.

Π’ самом Π΄Π΅Π»Π΅, Ссли ΠΏΡ€Π΅Π΄ΠΏΠΎΠ»ΠΎΠΆΠΈΡ‚ΡŒ, Ρ‡Ρ‚ΠΎ это Π²Ρ‹Ρ€Π°ΠΆΠ΅Π½ΠΈΠ΅ Ρ€Π°Π²Π½ΠΎ Π½Π΅ΠΊΠΎΡ‚ΠΎΡ€ΠΎΠΌΡƒ числу

x , Ρ‚ΠΎ согласно ΠΎΠΏΡ€Π΅Π΄Π΅Π»Π΅Π½ΠΈΡŽ ΠΎΠΏΠ΅Ρ€Π°Ρ†ΠΈΠΈ дСлСния ΠΈΠΌΠ΅Π΅ΠΌ: 0 = 0 Β· x . Но это равСнство ΠΈΠΌΠ΅Π΅Ρ‚ мСсто ΠΏΡ€ΠΈ любом числС x , Ρ‡Ρ‚ΠΎ ΠΈ Ρ‚Ρ€Π΅Π±ΠΎΠ²Π°Π»ΠΎΡΡŒ Π΄ΠΎΠΊΠ°Π·Π°Ρ‚ΡŒ.

Π‘Π»ΡƒΡ‡Π°ΠΉ 3.

0 0 — любоС число.

Π”Π΅ΠΉΡΡ‚Π²ΠΈΡ‚Π΅Π»ΡŒΠ½ΠΎ,


Π  Π΅ ш Π΅ Π½ ΠΈ Π΅. Рассмотрим Ρ‚Ρ€ΠΈ основных случая:

1) x = 0 – это Π·Π½Π°Ρ‡Π΅Π½ΠΈΠ΅ Π½Π΅ удовлСтворяСт Π΄Π°Π½Π½ΠΎΠΌΡƒ ΡƒΡ€Π°Π²Π½Π΅Π½ΠΈΡŽ

(ΠŸΠΎΡ‡Π΅ΠΌΡƒ?).

2) ΠΏΡ€ΠΈ x > 0 ΠΏΠΎΠ»ΡƒΡ‡Π°Π΅ΠΌ: x / x = 1, Ρ‚.e. 1 = 1, ΠΎΡ‚ΠΊΡƒΠ΄Π° слСдуСт,

Ρ‡Ρ‚ΠΎ x – любоС число; Π½ΠΎ принимая Π²ΠΎ Π²Π½ΠΈΠΌΠ°Π½ΠΈΠ΅, Ρ‡Ρ‚ΠΎ Π²

НашСм случаС x > 0 , ΠΎΡ‚Π²Π΅Ρ‚ΠΎΠΌ являСтся x > 0 ;

3) ΠΏΡ€ΠΈ x x / x = 1, Ρ‚. e . –1 = 1, ΡΠ»Π΅Π΄ΠΎΠ²Π°Ρ‚Π΅Π»ΡŒΠ½ΠΎ,

Π’ этом случаС Π½Π΅Ρ‚ Ρ€Π΅ΡˆΠ΅Π½ΠΈΡ.

Π’Π°ΠΊΠΈΠΌ ΠΎΠ±Ρ€Π°Π·ΠΎΠΌ, x > 0.

Как ΡƒΠΌΠ½ΠΎΠΆΠ°Ρ‚ΡŒ стСпСни? КакиС стСпСни ΠΌΠΎΠΆΠ½ΠΎ ΠΏΠ΅Ρ€Π΅ΠΌΠ½ΠΎΠΆΠΈΡ‚ΡŒ, Π° ΠΊΠ°ΠΊΠΈΠ΅ — Π½Π΅Ρ‚? Как число ΡƒΠΌΠ½ΠΎΠΆΠΈΡ‚ΡŒ Π½Π° ΡΡ‚Π΅ΠΏΠ΅Π½ΡŒ?

Π’ Π°Π»Π³Π΅Π±Ρ€Π΅ Π½Π°ΠΉΡ‚ΠΈ ΠΏΡ€ΠΎΠΈΠ·Π²Π΅Π΄Π΅Π½ΠΈΠ΅ стСпСнСй ΠΌΠΎΠΆΠ½ΠΎ Π² Π΄Π²ΡƒΡ… случаях:

1) Ссли стСпСни ΠΈΠΌΠ΅ΡŽΡ‚ ΠΎΠ΄ΠΈΠ½Π°ΠΊΠΎΠ²Ρ‹Π΅ основания;

2) Ссли стСпСни ΠΈΠΌΠ΅ΡŽΡ‚ ΠΎΠ΄ΠΈΠ½Π°ΠΊΠΎΠ²Ρ‹Π΅ ΠΏΠΎΠΊΠ°Π·Π°Ρ‚Π΅Π»ΠΈ.

ΠŸΡ€ΠΈ ΡƒΠΌΠ½ΠΎΠΆΠ΅Π½ΠΈΠΈ стСпСнСй с ΠΎΠ΄ΠΈΠ½Π°ΠΊΠΎΠ²Ρ‹ΠΌΠΈ основаниями Π½Π°Π΄ΠΎ основаниС ΠΎΡΡ‚Π°Π²ΠΈΡ‚ΡŒ ΠΏΡ€Π΅ΠΆΠ½ΠΈΠΌ, Π° ΠΏΠΎΠΊΠ°Π·Π°Ρ‚Π΅Π»ΠΈ — ΡΠ»ΠΎΠΆΠΈΡ‚ΡŒ:

ΠŸΡ€ΠΈ ΡƒΠΌΠ½ΠΎΠΆΠ΅Π½ΠΈΠΈ стСпСнСй с ΠΎΠ΄ΠΈΠ½Π°ΠΊΠΎΠ²Ρ‹ΠΌΠΈ показатСлями ΠΎΠ±Ρ‰ΠΈΠΉ ΠΏΠΎΠΊΠ°Π·Π°Ρ‚Π΅Π»ΡŒ ΠΌΠΎΠΆΠ½ΠΎ вынСсти Π·Π° скобки:

Рассмотрим, ΠΊΠ°ΠΊ ΡƒΠΌΠ½ΠΎΠΆΠ°Ρ‚ΡŒ стСпСни, Π½Π° ΠΊΠΎΠ½ΠΊΡ€Π΅Ρ‚Π½Ρ‹Ρ… ΠΏΡ€ΠΈΠΌΠ΅Ρ€Π°Ρ….

Π•Π΄ΠΈΠ½ΠΈΡ†Ρƒ Π² ΠΏΠΎΠΊΠ°Π·Π°Ρ‚Π΅Π»Π΅ стСпСни Π½Π΅ ΠΏΠΈΡˆΡƒΡ‚, Π½ΠΎ ΠΏΡ€ΠΈ ΡƒΠΌΠ½ΠΎΠΆΠ΅Π½ΠΈΠΈ стСпСнСй — ΡƒΡ‡ΠΈΡ‚Ρ‹Π²Π°ΡŽΡ‚:

ΠŸΡ€ΠΈ ΡƒΠΌΠ½ΠΎΠΆΠ΅Π½ΠΈΠΈ количСство стСпСнСй ΠΌΠΎΠΆΠ΅Ρ‚ Π±Ρ‹Ρ‚ΡŒ любоС. Π‘Π»Π΅Π΄ΡƒΠ΅Ρ‚ ΠΏΠΎΠΌΠ½ΠΈΡ‚ΡŒ, Ρ‡Ρ‚ΠΎ ΠΏΠ΅Ρ€Π΅Π΄ Π±ΡƒΠΊΠ²ΠΎΠΉ Π·Π½Π°ΠΊ умноТСния ΠΌΠΎΠΆΠ½ΠΎ Π½Π΅ ΠΏΠΈΡΠ°Ρ‚ΡŒ:

Π’ выраТСниях Π²ΠΎΠ·Π²Π΅Π΄Π΅Π½ΠΈΠ΅ Π² ΡΡ‚Π΅ΠΏΠ΅Π½ΡŒ выполняСтся Π² ΠΏΠ΅Ρ€Π²ΡƒΡŽ ΠΎΡ‡Π΅Ρ€Π΅Π΄ΡŒ.

Если Π½ΡƒΠΆΠ½ΠΎ число ΡƒΠΌΠ½ΠΎΠΆΠΈΡ‚ΡŒ Π½Π° ΡΡ‚Π΅ΠΏΠ΅Π½ΡŒ, сначала слСдуСт Π²Ρ‹ΠΏΠΎΠ»Π½ΠΈΡ‚ΡŒ Π²ΠΎΠ·Π²Π΅Π΄Π΅Π½ΠΈΠ΅ Π² ΡΡ‚Π΅ΠΏΠ΅Π½ΡŒ, Π° ΡƒΠΆΠ΅ ΠΏΠΎΡ‚ΠΎΠΌ — ΡƒΠΌΠ½ΠΎΠΆΠ΅Π½ΠΈΠ΅:

www.algebraclass.ru

Π‘Π»ΠΎΠΆΠ΅Π½ΠΈΠ΅, Π²Ρ‹Ρ‡ΠΈΡ‚Π°Π½ΠΈΠ΅, ΡƒΠΌΠ½ΠΎΠΆΠ΅Π½ΠΈΠ΅, ΠΈ Π΄Π΅Π»Π΅Π½ΠΈΠ΅ стСпСнСй

Π‘Π»ΠΎΠΆΠ΅Π½ΠΈΠ΅ ΠΈ Π²Ρ‹Ρ‡ΠΈΡ‚Π°Π½ΠΈΠ΅ стСпСнСй

ΠžΡ‡Π΅Π²ΠΈΠ΄Π½ΠΎ, Ρ‡Ρ‚ΠΎ числа со стСпСнями ΠΌΠΎΠ³ΡƒΡ‚ ΡΠ»Π°Π³Π°Ρ‚ΡŒΡΡ, ΠΊΠ°ΠΊ Π΄Ρ€ΡƒΠ³ΠΈΠ΅ Π²Π΅Π»ΠΈΡ‡ΠΈΠ½Ρ‹ , ΠΏΡƒΡ‚Π΅ΠΌ ΠΈΡ… слоТСния ΠΎΠ΄Π½ΠΎ Π·Π° Π΄Ρ€ΡƒΠ³ΠΈΠΌ со своими Π·Π½Π°ΠΊΠ°ΠΌΠΈ .

Π’Π°ΠΊ, сумма a 3 ΠΈ b 2 Π΅ΡΡ‚ΡŒ a 3 + b 2 .
Π‘ΡƒΠΌΠΌΠ° a 3 β€” b n ΠΈ h 5 -d 4 Π΅ΡΡ‚ΡŒ a 3 β€” b n + h 5 β€” d 4 .

ΠšΠΎΡΡ„Ρ„ΠΈΡ†ΠΈΠ΅Π½Ρ‚Ρ‹ ΠΎΠ΄ΠΈΠ½Π°ΠΊΠΎΠ²Ρ‹Ρ… стСпСнСй ΠΎΠ΄ΠΈΠ½Π°ΠΊΠΎΠ²Ρ‹Ρ… ΠΏΠ΅Ρ€Π΅ΠΌΠ΅Π½Π½Ρ‹Ρ… ΠΌΠΎΠ³ΡƒΡ‚ ΡΠ»Π°Π³Π°Ρ‚ΡŒΡΡ ΠΈΠ»ΠΈ Π²Ρ‹Ρ‡ΠΈΡ‚Π°Ρ‚ΡŒΡΡ.

Π’Π°ΠΊ, сумма 2a 2 ΠΈ 3a 2 Ρ€Π°Π²Π½Π° 5a 2 .

Π­Ρ‚ΠΎ Ρ‚Π°ΠΊ ΠΆΠ΅ ΠΎΡ‡Π΅Π²ΠΈΠ΄Π½ΠΎ, Ρ‡Ρ‚ΠΎ Ссли Π²Π·ΡΡ‚ΡŒ Π΄Π²Π° ΠΊΠ²Π°Π΄Ρ€Π°Ρ‚Π° Π°, ΠΈΠ»ΠΈ Ρ‚Ρ€ΠΈ ΠΊΠ²Π°Π΄Ρ€Π°Ρ‚Π° Π°, ΠΈΠ»ΠΈ ΠΏΡΡ‚ΡŒ ΠΊΠ²Π°Π΄Ρ€Π°Ρ‚ΠΎΠ² Π°.

Но стСпСни Ρ€Π°Π·Π»ΠΈΡ‡Π½Ρ‹Ρ… ΠΏΠ΅Ρ€Π΅ΠΌΠ΅Π½Π½Ρ‹Ρ… ΠΈ Ρ€Π°Π·Π»ΠΈΡ‡Π½Ρ‹Π΅ стСпСни ΠΎΠ΄ΠΈΠ½Π°ΠΊΠΎΠ²Ρ‹Ρ… ΠΏΠ΅Ρ€Π΅ΠΌΠ΅Π½Π½Ρ‹Ρ… , Π΄ΠΎΠ»ΠΆΠ½Ρ‹ ΡΠ»Π°Π³Π°Ρ‚ΡŒΡΡ ΠΈΡ… слоТСниСм с ΠΈΡ… Π·Π½Π°ΠΊΠ°ΠΌΠΈ.

Π’Π°ΠΊ, сумма a 2 ΠΈ a 3 Π΅ΡΡ‚ΡŒ сумма a 2 + a 3 .

Π­Ρ‚ΠΎ ΠΎΡ‡Π΅Π²ΠΈΠ΄Π½ΠΎ, Ρ‡Ρ‚ΠΎ ΠΊΠ²Π°Π΄Ρ€Π°Ρ‚ числа a, ΠΈ ΠΊΡƒΠ± числа a, Π½Π΅ Ρ€Π°Π²Π½ΠΎ Π½ΠΈ ΡƒΠ΄Π²ΠΎΠ΅Π½Π½ΠΎΠΌΡƒ ΠΊΠ²Π°Π΄Ρ€Π°Ρ‚Ρƒ a, Π½ΠΎ ΡƒΠ΄Π²ΠΎΠ΅Π½Π½ΠΎΠΌΡƒ ΠΊΡƒΠ±Ρƒ a.

Π‘ΡƒΠΌΠΌΠ° a 3 b n ΠΈ 3a 5 b 6 Π΅ΡΡ‚ΡŒ a 3 b n + 3a 5 b 6 .

Π’Ρ‹Ρ‡ΠΈΡ‚Π°Π½ΠΈΠ΅ стСпСнСй проводится Ρ‚Π°ΠΊΠΈΠΌ ΠΆΠ΅ ΠΎΠ±Ρ€Π°Π·ΠΎΠΌ, Ρ‡Ρ‚ΠΎ ΠΈ слоТСниС, Π·Π° ΠΈΡΠΊΠ»ΡŽΡ‡Π΅Π½ΠΈΠ΅ΠΌ Ρ‚ΠΎΠ³ΠΎ, Ρ‡Ρ‚ΠΎ Π·Π½Π°ΠΊΠΈ Π²Ρ‹Ρ‡ΠΈΡ‚Π°Π΅ΠΌΡ‹Ρ… Π΄ΠΎΠ»ΠΆΠ½Ρ‹ соотвСтствСнно Π±Ρ‹Ρ‚ΡŒ ΠΈΠ·ΠΌΠ΅Π½Π΅Π½Ρ‹.

Или:
2a 4 β€” (-6a 4) = 8a 4
3h 2 b 6 β€” 4h 2 b 6 = -h 2 b 6
5(a β€” h) 6 β€” 2(a β€” h) 6 = 3(a β€” h) 6

Π£ΠΌΠ½ΠΎΠΆΠ΅Π½ΠΈΠ΅ стСпСнСй

Числа со стСпСнями ΠΌΠΎΠ³ΡƒΡ‚ Π±Ρ‹Ρ‚ΡŒ ΡƒΠΌΠ½ΠΎΠΆΠ΅Π½Ρ‹, ΠΊΠ°ΠΊ ΠΈ Π΄Ρ€ΡƒΠ³ΠΈΠ΅ Π²Π΅Π»ΠΈΡ‡ΠΈΠ½Ρ‹, ΠΏΡƒΡ‚Π΅ΠΌ написания ΠΈΡ… ΠΎΠ΄Π½ΠΎ Π·Π° Π΄Ρ€ΡƒΠ³ΠΈΠΌ, со Π·Π½Π°ΠΊΠΎΠΌ умноТСния ΠΈΠ»ΠΈ Π±Π΅Π· Π½Π΅Π³ΠΎ ΠΌΠ΅ΠΆΠ΄Ρƒ Π½ΠΈΠΌΠΈ.

Π’Π°ΠΊ, Ρ€Π΅Π·ΡƒΠ»ΡŒΡ‚Π°Ρ‚ умноТСния a 3 Π½Π° b 2 Ρ€Π°Π²Π΅Π½ a 3 b 2 ΠΈΠ»ΠΈ aaabb.

Или:
x -3 β‹… a m = a m x -3
3a 6 y 2 β‹… (-2x) = -6a 6 xy 2
a 2 b 3 y 2 β‹… a 3 b 2 y = a 2 b 3 y 2 a 3 b 2 y

Π Π΅Π·ΡƒΠ»ΡŒΡ‚Π°Ρ‚ Π² послСднСм ΠΏΡ€ΠΈΠΌΠ΅Ρ€Π΅ ΠΌΠΎΠΆΠ΅Ρ‚ Π±Ρ‹Ρ‚ΡŒ упорядочСн ΠΏΡƒΡ‚Ρ‘ΠΌ слоТСния ΠΎΠ΄ΠΈΠ½Π°ΠΊΠΎΠ²Ρ‹Ρ… ΠΏΠ΅Ρ€Π΅ΠΌΠ΅Π½Π½Ρ‹Ρ….
Π’Ρ‹Ρ€Π°ΠΆΠ΅Π½ΠΈΠ΅ ΠΏΡ€ΠΈΠΌΠ΅Ρ‚ Π²ΠΈΠ΄: a 5 b 5 y 3 .

Бравнивая нСсколько чисСл(ΠΏΠ΅Ρ€Π΅ΠΌΠ΅Π½Π½Ρ‹Ρ…) со стСпСнями, ΠΌΡ‹ ΠΌΠΎΠΆΠ΅ΠΌ ΡƒΠ²ΠΈΠ΄Π΅Ρ‚ΡŒ, Ρ‡Ρ‚ΠΎ Ссли Π»ΡŽΠ±Ρ‹Π΅ Π΄Π²Π° ΠΈΠ· Π½ΠΈΡ… ΡƒΠΌΠ½ΠΎΠΆΠ°ΡŽΡ‚ΡΡ, Ρ‚ΠΎ Ρ€Π΅Π·ΡƒΠ»ΡŒΡ‚Π°Ρ‚ β€” это число (пСрСмСнная) со ΡΡ‚Π΅ΠΏΠ΅Π½ΡŒΡŽ, Ρ€Π°Π²Π½ΠΎΠΉ суммС стСпСнСй слагаСмых.

Π’Π°ΠΊ, a 2 .a 3 = aa.aaa = aaaaa = a 5 .

Π—Π΄Π΅ΡΡŒ 5 β€” это ΡΡ‚Π΅ΠΏΠ΅Π½ΡŒ Ρ€Π΅Π·ΡƒΠ»ΡŒΡ‚Π°Ρ‚Π° умноТСния, равная 2 + 3, суммС стСпСнСй слагаСмых.

Π’Π°ΠΊ, a n .a m = a m+n .

Для a n , a бСрётся ΠΊΠ°ΠΊ ΠΌΠ½ΠΎΠΆΠΈΡ‚Π΅Π»ΡŒ ΡΡ‚ΠΎΠ»ΡŒΠΊΠΎ Ρ€Π°Π·, сколько Ρ€Π°Π²Π½Π° ΡΡ‚Π΅ΠΏΠ΅Π½ΡŒ n;

И a m , бСрётся ΠΊΠ°ΠΊ ΠΌΠ½ΠΎΠΆΠΈΡ‚Π΅Π»ΡŒ ΡΡ‚ΠΎΠ»ΡŒΠΊΠΎ Ρ€Π°Π·, сколько Ρ€Π°Π²Π½Π° ΡΡ‚Π΅ΠΏΠ΅Π½ΡŒ m;

ΠŸΠΎΡΡ‚ΠΎΠΌΡƒ, стСпСни с ΠΎΠ΄ΠΈΠ½Π°ΠΊΠΎΠ²Ρ‹ΠΌΠΈ основами ΠΌΠΎΠ³ΡƒΡ‚ Π±Ρ‹Ρ‚ΡŒ ΡƒΠΌΠ½ΠΎΠΆΠ΅Π½Ρ‹ ΠΏΡƒΡ‚Ρ‘ΠΌ слоТСния ΠΏΠΎΠΊΠ°Π·Π°Ρ‚Π΅Π»Π΅ΠΉ стСпСнСй.

Вак, a 2 .a 6 = a 2+6 = a 8 . И x 3 .x 2 .x = x 3+2+1 = x 6 .

Или:
4a n β‹… 2a n = 8a 2n
b 2 y 3 β‹… b 4 y = b 6 y 4
(b + h β€” y) n β‹… (b + h β€” y) = (b + h β€” y) n+1

Π£ΠΌΠ½ΠΎΠΆΡŒΡ‚Π΅ (x 3 + x 2 y + xy 2 + y 3) β‹… (x β€” y).
ΠžΡ‚Π²Π΅Ρ‚: x 4 β€” y 4 .
Π£ΠΌΠ½ΠΎΠΆΡŒΡ‚Π΅ (x 3 + x β€” 5) β‹… (2x 3 + x + 1).

Π­Ρ‚ΠΎ ΠΏΡ€Π°Π²ΠΈΠ»ΠΎ справСдливо ΠΈ для чисСл, ΠΏΠΎΠΊΠ°Π·Π°Ρ‚Π΅Π»ΠΈ стСпСни ΠΊΠΎΡ‚ΠΎΡ€Ρ‹Ρ… β€” ΠΎΡ‚Ρ€ΠΈΡ†Π°Ρ‚Π΅Π»ΡŒΠ½Ρ‹Π΅ .

1. Π’Π°ΠΊ, a -2 .a -3 = a -5 . Π­Ρ‚ΠΎ ΠΌΠΎΠΆΠ½ΠΎ Π·Π°ΠΏΠΈΡΠ°Ρ‚ΡŒ Π² Π²ΠΈΠ΄Π΅ (1/aa).(1/aaa) = 1/aaaaa.

2. y -n .y -m = y -n-m .

3. a -n .a m = a m-n .

Если a + b ΡƒΠΌΠ½ΠΎΠΆΠ°ΡŽΡ‚ΡΡ Π½Π° a β€” b, Ρ€Π΅Π·ΡƒΠ»ΡŒΡ‚Π°Ρ‚ Π±ΡƒΠ΄Π΅Ρ‚ Ρ€Π°Π²Π΅Π½ a 2 β€” b 2: Ρ‚ΠΎ Π΅ΡΡ‚ΡŒ

Π Π΅Π·ΡƒΠ»ΡŒΡ‚Π°Ρ‚ умноТСния суммы ΠΈΠ»ΠΈ Ρ€Π°Π·Π½ΠΈΡ†Ρ‹ Π΄Π²ΡƒΡ… чисСл Ρ€Π°Π²Π΅Π½ суммС ΠΈΠ»ΠΈ Ρ€Π°Π·Π½ΠΈΡ†Π΅ ΠΈΡ… ΠΊΠ²Π°Π΄Ρ€Π°Ρ‚ΠΎΠ².

Если умноТаСтся сумма ΠΈ Ρ€Π°Π·Π½ΠΈΡ†Π° Π΄Π²ΡƒΡ… чисСл, Π²ΠΎΠ·Π²Π΅Π΄Ρ‘Π½Π½Ρ‹Ρ… Π² ΠΊΠ²Π°Π΄Ρ€Π°Ρ‚ , Ρ€Π΅Π·ΡƒΠ»ΡŒΡ‚Π°Ρ‚ Π±ΡƒΠ΄Π΅Ρ‚ Ρ€Π°Π²Π΅Π½ суммС ΠΈΠ»ΠΈ Ρ€Π°Π·Π½ΠΈΡ†Π΅ этих чисСл Π² Ρ‡Π΅Ρ‚Π²Ρ‘Ρ€Ρ‚ΠΎΠΉ стСпСни.

Π’Π°ΠΊ, (a β€” y).(a + y) = a 2 β€” y 2 .
(a 2 β€” y 2)β‹…(a 2 + y 2) = a 4 β€” y 4 .
(a 4 β€” y 4)β‹…(a 4 + y 4) = a 8 β€” y 8 .

Π”Π΅Π»Π΅Π½ΠΈΠ΅ стСпСнСй

Числа со стСпСнями ΠΌΠΎΠ³ΡƒΡ‚ Π±Ρ‹Ρ‚ΡŒ ΠΏΠΎΠ΄Π΅Π»Π΅Π½Ρ‹, ΠΊΠ°ΠΊ ΠΈ Π΄Ρ€ΡƒΠ³ΠΈΠ΅ числа, ΠΏΡƒΡ‚Π΅ΠΌ отнимая ΠΎΡ‚ Π΄Π΅Π»ΠΈΠΌΠΎΠ³ΠΎ дСлитСля, ΠΈΠ»ΠΈ Ρ€Π°Π·ΠΌΠ΅Ρ‰Π΅Π½ΠΈΠ΅ΠΌ ΠΈΡ… Π² Ρ„ΠΎΡ€ΠΌΠ΅ Π΄Ρ€ΠΎΠ±ΠΈ.

Π’Π°ΠΊΠΈΠΌ ΠΎΠ±Ρ€Π°Π·ΠΎΠΌ a 3 b 2 Π΄Π΅Π»Ρ‘Π½Π½ΠΎΠ΅ Π½Π° b 2 , Ρ€Π°Π²Π½ΠΎ a 3 .

Π—Π°ΠΏΠΈΡΡŒ a 5 , Π΄Π΅Π»Ρ‘Π½Π½ΠΎΠ³ΠΎ Π½Π° a 3 , выглядит ΠΊΠ°ΠΊ $\frac $. Но это Ρ€Π°Π²Π½ΠΎ a 2 . Π’ рядС чисСл
a +4 , a +3 , a +2 , a +1 , a 0 , a -1 , a -2 , a -3 , a -4 .
любоС число ΠΌΠΎΠΆΠ΅Ρ‚ Π±Ρ‹Ρ‚ΡŒ ΠΏΠΎΠ΄Π΅Π»Π΅Π½ΠΎ Π½Π° Π΄Ρ€ΡƒΠ³ΠΎΠ΅, Π° ΠΏΠΎΠΊΠ°Π·Π°Ρ‚Π΅Π»ΡŒ стСпСни Π±ΡƒΠ΄Π΅Ρ‚ Ρ€Π°Π²Π΅Π½ Ρ€Π°Π·Π½ΠΈΡ†Π΅ ΠΏΠΎΠΊΠ°Π·Π°Ρ‚Π΅Π»Π΅ΠΉ Π΄Π΅Π»ΠΈΠΌΡ‹Ρ… чисСл. 3$

НСобходимо ΠΎΡ‡Π΅Π½ΡŒ Ρ…ΠΎΡ€ΠΎΡˆΠΎ ΡƒΡΠ²ΠΎΠΈΡ‚ΡŒ ΡƒΠΌΠ½ΠΎΠΆΠ΅Π½ΠΈΠ΅ ΠΈ Π΄Π΅Π»Π΅Π½ΠΈΠ΅ стСпСнСй, Ρ‚Π°ΠΊ ΠΊΠ°ΠΊ Ρ‚Π°ΠΊΠΈΠ΅ ΠΎΠΏΠ΅Ρ€Π°Ρ†ΠΈΠΈ ΠΎΡ‡Π΅Π½ΡŒ ΡˆΠΈΡ€ΠΎΠΊΠΎ ΠΏΡ€ΠΈΠΌΠ΅Π½ΡΡŽΡ‚ΡΡ Π² Π°Π»Π³Π΅Π±Ρ€Π΅.

ΠŸΡ€ΠΈΠΌΠ΅Ρ€Ρ‹ Ρ€Π΅ΡˆΠ΅Π½ΠΈΡ ΠΏΡ€ΠΈΠΌΠ΅Ρ€ΠΎΠ² с дробями, содСрТащими числа со стСпСнями

1. Π£ΠΌΠ΅Π½ΡŒΡˆΠΈΡ‚Π΅ ΠΏΠΎΠΊΠ°Π·Π°Ρ‚Π΅Π»ΠΈ стСпСнСй Π² $\frac $ ΠžΡ‚Π²Π΅Ρ‚: $\frac $.

2. Π£ΠΌΠ΅Π½ΡŒΡˆΠΈΡ‚Π΅ ΠΏΠΎΠΊΠ°Π·Π°Ρ‚Π΅Π»ΠΈ стСпСнСй Π² $\frac $. ΠžΡ‚Π²Π΅Ρ‚: $\frac $ ΠΈΠ»ΠΈ 2x.

3. Π£ΠΌΠ΅Π½ΡŒΡˆΠΈΡ‚Π΅ ΠΏΠΎΠΊΠ°Π·Π°Ρ‚Π΅Π»ΠΈ стСпСнСй a 2 /a 3 ΠΈ a -3 /a -4 ΠΈ ΠΏΡ€ΠΈΠ²Π΅Π΄ΠΈΡ‚Π΅ ΠΊ ΠΎΠ±Ρ‰Π΅ΠΌΡƒ Π·Π½Π°ΠΌΠ΅Π½Π°Ρ‚Π΅Π»ΡŽ.
a 2 .a -4 Π΅ΡΡ‚ΡŒ a -2 ΠΏΠ΅Ρ€Π²Ρ‹ΠΉ Ρ‡ΠΈΡΠ»ΠΈΡ‚Π΅Π»ΡŒ.
a 3 .a -3 Π΅ΡΡ‚ΡŒ a 0 = 1, Π²Ρ‚ΠΎΡ€ΠΎΠΉ Ρ‡ΠΈΡΠ»ΠΈΡ‚Π΅Π»ΡŒ.
a 3 .a -4 Π΅ΡΡ‚ΡŒ a -1 , ΠΎΠ±Ρ‰ΠΈΠΉ Ρ‡ΠΈΡΠ»ΠΈΡ‚Π΅Π»ΡŒ.
ПослС упрощСния: a -2 /a -1 ΠΈ 1/a -1 .

4. Π£ΠΌΠ΅Π½ΡŒΡˆΠΈΡ‚Π΅ ΠΏΠΎΠΊΠ°Π·Π°Ρ‚Π΅Π»ΠΈ стСпСнСй 2a 4 /5a 3 ΠΈ 2 /a 4 ΠΈ ΠΏΡ€ΠΈΠ²Π΅Π΄ΠΈΡ‚Π΅ ΠΊ ΠΎΠ±Ρ‰Π΅ΠΌΡƒ Π·Π½Π°ΠΌΠ΅Π½Π°Ρ‚Π΅Π»ΡŽ.
ΠžΡ‚Π²Π΅Ρ‚: 2a 3 /5a 7 ΠΈ 5a 5 /5a 7 ΠΈΠ»ΠΈ 2a 3 /5a 2 ΠΈ 5/5a 2 .

5. Π£ΠΌΠ½ΠΎΠΆΡŒΡ‚Π΅ (a 3 + b)/b 4 Π½Π° (a β€” b)/3.

6. Π£ΠΌΠ½ΠΎΠΆΡŒΡ‚Π΅ (a 5 + 1)/x 2 Π½Π° (b 2 β€” 1)/(x + a).

7. Π£ΠΌΠ½ΠΎΠΆΡŒΡ‚Π΅ b 4 /a -2 Π½Π° h -3 /x ΠΈ a n /y -3 .

8. Π Π°Π·Π΄Π΅Π»ΠΈΡ‚Π΅ a 4 /y 3 Π½Π° a 3 /y 2 . ΠžΡ‚Π²Π΅Ρ‚: a/y.

Бвойства стСпСни

НапоминаСм, Ρ‡Ρ‚ΠΎ Π² Π΄Π°Π½Π½ΠΎΠΌ ΡƒΡ€ΠΎΠΊΠ΅ Ρ€Π°Π·Π±ΠΈΡ€Π°ΡŽΡ‚ΡΡ свойства стСпСнСй с Π½Π°Ρ‚ΡƒΡ€Π°Π»ΡŒΠ½Ρ‹ΠΌΠΈ показатСлями ΠΈ Π½ΡƒΠ»Ρ‘ΠΌ. Π‘Ρ‚Π΅ΠΏΠ΅Π½ΠΈ с Ρ€Π°Ρ†ΠΈΠΎΠ½Π°Π»ΡŒΠ½Ρ‹ΠΌΠΈ показатСлями ΠΈ ΠΈΡ… свойства Π±ΡƒΠ΄ΡƒΡ‚ рассмотрСны Π² ΡƒΡ€ΠΎΠΊΠ°Ρ… для 8 классов.

Π‘Ρ‚Π΅ΠΏΠ΅Π½ΡŒ с Π½Π°Ρ‚ΡƒΡ€Π°Π»ΡŒΠ½Ρ‹ΠΌ ΠΏΠΎΠΊΠ°Π·Π°Ρ‚Π΅Π»Π΅ΠΌ ΠΎΠ±Π»Π°Π΄Π°Π΅Ρ‚ нСсколькими Π²Π°ΠΆΠ½Ρ‹ΠΌΠΈ свойствами, ΠΊΠΎΡ‚ΠΎΡ€Ρ‹Π΅ ΠΏΠΎΠ·Π²ΠΎΠ»ΡΡŽΡ‚ ΡƒΠΏΡ€ΠΎΡ‰Π°Ρ‚ΡŒ вычислСния Π² ΠΏΡ€ΠΈΠΌΠ΅Ρ€Π°Ρ… со стСпСнями.

Бвойство β„– 1


ΠŸΡ€ΠΎΠΈΠ·Π²Π΅Π΄Π΅Π½ΠΈΠ΅ стСпСнСй

ΠŸΡ€ΠΈ ΡƒΠΌΠ½ΠΎΠΆΠ΅Π½ΠΈΠΈ стСпСнСй с ΠΎΠ΄ΠΈΠ½Π°ΠΊΠΎΠ²Ρ‹ΠΌΠΈ основаниями основаниС остаётся Π±Π΅Π· ΠΈΠ·ΠΌΠ΅Π½Π΅Π½ΠΈΠΉ, Π° ΠΏΠΎΠΊΠ°Π·Π°Ρ‚Π΅Π»ΠΈ стСпСнСй ΡΠΊΠ»Π°Π΄Ρ‹Π²Π°ΡŽΡ‚ΡΡ.

a m Β· a n = a m + n , Π³Π΄Π΅ Β« a Β» — любоС число, Π° Β« m Β», Β« n Β» — Π»ΡŽΠ±Ρ‹Π΅ Π½Π°Ρ‚ΡƒΡ€Π°Π»ΡŒΠ½Ρ‹Π΅ числа.

Π”Π°Π½Π½ΠΎΠ΅ свойство стСпСнСй Ρ‚Π°ΠΊΠΆΠ΅ дСйствуСт Π½Π° ΠΏΡ€ΠΎΠΈΠ·Π²Π΅Π΄Π΅Π½ΠΈΠ΅ Ρ‚Ρ€Ρ‘Ρ… ΠΈ Π±ΠΎΠ»Π΅Π΅ стСпСнСй.

  • Π£ΠΏΡ€ΠΎΡΡ‚ΠΈΡ‚ΡŒ Π²Ρ‹Ρ€Π°ΠΆΠ΅Π½ΠΈΠ΅.
    b Β· b 2 Β· b 3 Β· b 4 Β· b 5 = b 1 + 2 + 3 + 4 + 5 = b 15
  • ΠŸΡ€Π΅Π΄ΡΡ‚Π°Π²ΠΈΡ‚ΡŒ Π² Π²ΠΈΠ΄Π΅ стСпСни.
    6 15 Β· 36 = 6 15 Β· 6 2 = 6 15 Β· 6 2 = 6 17
  • ΠŸΡ€Π΅Π΄ΡΡ‚Π°Π²ΠΈΡ‚ΡŒ Π² Π²ΠΈΠ΄Π΅ стСпСни.
    (0,8) 3 Β· (0,8) 12 = (0,8) 3 + 12 = (0,8) 15
  • ΠžΠ±Ρ€Π°Ρ‚ΠΈΡ‚Π΅ Π²Π½ΠΈΠΌΠ°Π½ΠΈΠ΅, Ρ‡Ρ‚ΠΎ Π² ΡƒΠΊΠ°Π·Π°Π½Π½ΠΎΠΌ свойствС Ρ€Π΅Ρ‡ΡŒ шла Ρ‚ΠΎΠ»ΡŒΠΊΠΎ ΠΎΠ± ΡƒΠΌΠ½ΠΎΠΆΠ΅Π½ΠΈΠΈ стСпСнСй с ΠΎΠ΄ΠΈΠ½Π°ΠΊΠΎΠ²Ρ‹ΠΌΠΈ основаниями . Оно Π½Π΅ относится ΠΊ ΠΈΡ… слоТСнию.

    НСльзя Π·Π°ΠΌΠ΅Π½ΡΡ‚ΡŒ сумму (3 3 + 3 2) Π½Π° 3 5 . Π­Ρ‚ΠΎ понятно, Ссли
    ΠΏΠΎΡΡ‡ΠΈΡ‚Π°Ρ‚ΡŒ (3 3 + 3 2) = (27 + 9) = 36 , Π° 3 5 = 243

    Бвойство β„– 2


    ЧастноС стСпСнСй

    ΠŸΡ€ΠΈ Π΄Π΅Π»Π΅Π½ΠΈΠΈ стСпСнСй с ΠΎΠ΄ΠΈΠ½Π°ΠΊΠΎΠ²Ρ‹ΠΌΠΈ основаниями основаниС остаётся Π±Π΅Π· ΠΈΠ·ΠΌΠ΅Π½Π΅Π½ΠΈΠΉ, Π° ΠΈΠ· показатСля стСпСни Π΄Π΅Π»ΠΈΠΌΠΎΠ³ΠΎ Π²Ρ‹Ρ‡ΠΈΡ‚Π°ΡŽΡ‚ ΠΏΠΎΠΊΠ°Π·Π°Ρ‚Π΅Π»ΡŒ стСпСни дСлитСля.

  • Π—Π°ΠΏΠΈΡΠ°Ρ‚ΡŒ частноС Π² Π²ΠΈΠ΄Π΅ стСпСни
    (2b) 5: (2b) 3 = (2b) 5 βˆ’ 3 = (2b) 2
  • Π’Ρ‹Ρ‡ΠΈΡΠ»ΠΈΡ‚ΡŒ.

    11 3 βˆ’ 2 Β· 4 2 βˆ’ 1 = 11 Β· 4 = 44
    ΠŸΡ€ΠΈΠΌΠ΅Ρ€. Π Π΅ΡˆΠΈΡ‚ΡŒ ΡƒΡ€Π°Π²Π½Π΅Π½ΠΈΠ΅. Π˜ΡΠΏΠΎΠ»ΡŒΠ·ΡƒΠ΅ΠΌ свойство частного стСпСнСй.
    3 8: t = 3 4

    ΠžΡ‚Π²Π΅Ρ‚: t = 3 4 = 81

    ΠŸΠΎΠ»ΡŒΠ·ΡƒΡΡΡŒ свойствами β„– 1 ΠΈ β„– 2, ΠΌΠΎΠΆΠ½ΠΎ Π»Π΅Π³ΠΊΠΎ ΡƒΠΏΡ€ΠΎΡ‰Π°Ρ‚ΡŒ выраТСния ΠΈ ΠΏΡ€ΠΎΠΈΠ·Π²ΠΎΠ΄ΠΈΡ‚ΡŒ вычислСния.

      ΠŸΡ€ΠΈΠΌΠ΅Ρ€. Π£ΠΏΡ€ΠΎΡΡ‚ΠΈΡ‚ΡŒ Π²Ρ‹Ρ€Π°ΠΆΠ΅Π½ΠΈΠ΅.
      4 5m + 6 Β· 4 m + 2: 4 4m + 3 = 4 5m + 6 + m + 2: 4 4m + 3 = 4 6m + 8 βˆ’ 4m βˆ’ 3 = 4 2m + 5

    ΠŸΡ€ΠΈΠΌΠ΅Ρ€. Найти Π·Π½Π°Ρ‡Π΅Π½ΠΈΠ΅ выраТСния, ΠΈΡΠΏΠΎΠ»ΡŒΠ·ΡƒΡ свойства стСпСни.

    2 11 βˆ’ 5 = 2 6 = 64

    ΠžΠ±Ρ€Π°Ρ‚ΠΈΡ‚Π΅ Π²Π½ΠΈΠΌΠ°Π½ΠΈΠ΅, Ρ‡Ρ‚ΠΎ Π² свойствС 2 Ρ€Π΅Ρ‡ΡŒ шла Ρ‚ΠΎΠ»ΡŒΠΊΠΎ ΠΎ Π΄Π΅Π»Π΅Π½ΠΈΠΈ стСпСнСй с ΠΎΠ΄ΠΈΠ½Π°ΠΊΠΎΠ²Ρ‹ΠΌΠΈ основаниями.

    НСльзя Π·Π°ΠΌΠ΅Π½ΡΡ‚ΡŒ Ρ€Π°Π·Π½ΠΎΡΡ‚ΡŒ (4 3 βˆ’4 2) Π½Π° 4 1 . Π­Ρ‚ΠΎ понятно, Ссли ΠΏΠΎΡΡ‡ΠΈΡ‚Π°Ρ‚ΡŒ (4 3 βˆ’4 2) = (64 βˆ’ 16) = 48 , Π° 4 1 = 4

    Бвойство β„– 3


    Π’ΠΎΠ·Π²Π΅Π΄Π΅Π½ΠΈΠ΅ стСпСни Π² ΡΡ‚Π΅ΠΏΠ΅Π½ΡŒ

    ΠŸΡ€ΠΈ Π²ΠΎΠ·Π²Π΅Π΄Π΅Π½ΠΈΠΈ стСпСни Π² ΡΡ‚Π΅ΠΏΠ΅Π½ΡŒ основаниС стСпСни остаётся Π±Π΅Π· измСнСния, Π° ΠΏΠΎΠΊΠ°Π·Π°Ρ‚Π΅Π»ΠΈ стСпСнСй ΠΏΠ΅Ρ€Π΅ΠΌΠ½ΠΎΠΆΠ°ΡŽΡ‚ΡΡ.

    (a n) m = a n Β· m , Π³Π΄Π΅ Β« a Β» — любоС число, Π° Β« m Β», Β« n Β» — Π»ΡŽΠ±Ρ‹Π΅ Π½Π°Ρ‚ΡƒΡ€Π°Π»ΡŒΠ½Ρ‹Π΅ числа.


    ΠžΠ±Ρ€Π°Ρ‚ΠΈΡ‚Π΅ Π²Π½ΠΈΠΌΠ°Π½ΠΈΠ΅, Ρ‡Ρ‚ΠΎ свойство β„– 4, ΠΊΠ°ΠΊ ΠΈ Π΄Ρ€ΡƒΠ³ΠΈΠ΅ свойства стСпСнСй, ΠΏΡ€ΠΈΠΌΠ΅Π½ΡΡŽΡ‚ ΠΈ Π² ΠΎΠ±Ρ€Π°Ρ‚Π½ΠΎΠΌ порядкС.

    (a n Β· b n)= (a Β· b) n

    Π’ΠΎ Π΅ΡΡ‚ΡŒ, Ρ‡Ρ‚ΠΎΠ±Ρ‹ ΠΏΠ΅Ρ€Π΅ΠΌΠ½ΠΎΠΆΠΈΡ‚ΡŒ стСпСни с ΠΎΠ΄ΠΈΠ½Π°ΠΊΠΎΠ²Ρ‹ΠΌΠΈ показатСлями ΠΌΠΎΠΆΠ½ΠΎ ΠΏΠ΅Ρ€Π΅ΠΌΠ½ΠΎΠΆΠΈΡ‚ΡŒ основания, Π° ΠΏΠΎΠΊΠ°Π·Π°Ρ‚Π΅Π»ΡŒ стСпСни ΠΎΡΡ‚Π°Π²ΠΈΡ‚ΡŒ Π½Π΅ΠΈΠ·ΠΌΠ΅Π½Π½Ρ‹ΠΌ.

  • ΠŸΡ€ΠΈΠΌΠ΅Ρ€. Π’Ρ‹Ρ‡ΠΈΡΠ»ΠΈΡ‚ΡŒ.
    2 4 Β· 5 4 = (2 Β· 5) 4 = 10 4 = 10 000
  • ΠŸΡ€ΠΈΠΌΠ΅Ρ€. Π’Ρ‹Ρ‡ΠΈΡΠ»ΠΈΡ‚ΡŒ.
    0,5 16 Β· 2 16 = (0,5 Β· 2) 16 = 1
  • Π’ Π±ΠΎΠ»Π΅Π΅ слоТных ΠΏΡ€ΠΈΠΌΠ΅Ρ€Π°Ρ… ΠΌΠΎΠ³ΡƒΡ‚ Π²ΡΡ‚Ρ€Π΅Ρ‚ΠΈΡ‚ΡŒΡΡ случаи, ΠΊΠΎΠ³Π΄Π° ΡƒΠΌΠ½ΠΎΠΆΠ΅Π½ΠΈΠ΅ ΠΈ Π΄Π΅Π»Π΅Π½ΠΈΠ΅ Π½Π°Π΄ΠΎ Π²Ρ‹ΠΏΠΎΠ»Π½ΠΈΡ‚ΡŒ Π½Π°Π΄ стСпСнями с Ρ€Π°Π·Π½Ρ‹ΠΌΠΈ основаниями ΠΈ Ρ€Π°Π·Π½Ρ‹ΠΌΠΈ показатСлями. Π’ этом случаС совСтуСм ΠΏΠΎΡΡ‚ΡƒΠΏΠ°Ρ‚ΡŒ ΡΠ»Π΅Π΄ΡƒΡŽΡ‰ΠΈΠΌ ΠΎΠ±Ρ€Π°Π·ΠΎΠΌ.

    НапримСр, 4 5 Β· 3 2 = 4 3 Β· 4 2 Β· 3 2 = 4 3 Β· (4 Β· 3) 2 = 64 Β· 12 2 = 64 Β· 144 = 9216

    ΠŸΡ€ΠΈΠΌΠ΅Ρ€ возвСдСния Π² ΡΡ‚Π΅ΠΏΠ΅Π½ΡŒ дСсятичной Π΄Ρ€ΠΎΠ±ΠΈ.

    4 21 Β· (βˆ’0,25) 20 = 4 Β· 4 20 Β· (βˆ’0,25) 20 = 4 Β· (4 Β· (βˆ’0,25)) 20 = 4 Β· (βˆ’1) 20 = 4 Β· 1 = 4

    Бвойства 5


    Π‘Ρ‚Π΅ΠΏΠ΅Π½ΡŒ частного (Π΄Ρ€ΠΎΠ±ΠΈ)

    Π§Ρ‚ΠΎΠ±Ρ‹ возвСсти Π² ΡΡ‚Π΅ΠΏΠ΅Π½ΡŒ частноС, ΠΌΠΎΠΆΠ½ΠΎ возвСсти Π² эту ΡΡ‚Π΅ΠΏΠ΅Π½ΡŒ ΠΎΡ‚Π΄Π΅Π»ΡŒΠ½ΠΎ Π΄Π΅Π»ΠΈΠΌΠΎΠ΅ ΠΈ Π΄Π΅Π»ΠΈΡ‚Π΅Π»ΡŒ, ΠΈ ΠΏΠ΅Ρ€Π²Ρ‹ΠΉ Ρ€Π΅Π·ΡƒΠ»ΡŒΡ‚Π°Ρ‚ Ρ€Π°Π·Π΄Π΅Π»ΠΈΡ‚ΡŒ Π½Π° Π²Ρ‚ΠΎΡ€ΠΎΠΉ.

    (a: b) n = a n: b n , Π³Π΄Π΅ Β« a Β», Β« b Β» — Π»ΡŽΠ±Ρ‹Π΅ Ρ€Π°Ρ†ΠΈΠΎΠ½Π°Π»ΡŒΠ½Ρ‹Π΅ числа, b β‰  0, n — любоС Π½Π°Ρ‚ΡƒΡ€Π°Π»ΡŒΠ½ΠΎΠ΅ число.

  • ΠŸΡ€ΠΈΠΌΠ΅Ρ€. ΠŸΡ€Π΅Π΄ΡΡ‚Π°Π²ΠΈΡ‚ΡŒ Π²Ρ‹Ρ€Π°ΠΆΠ΅Π½ΠΈΠ΅ Π² Π²ΠΈΠ΄Π΅ частного стСпСнСй.
    (5: 3) 12 = 5 12: 3 12
  • НапоминаСм, Ρ‡Ρ‚ΠΎ частноС ΠΌΠΎΠΆΠ½ΠΎ ΠΏΡ€Π΅Π΄ΡΡ‚Π°Π²ΠΈΡ‚ΡŒ Π² Π²ΠΈΠ΄Π΅ Π΄Ρ€ΠΎΠ±ΠΈ. ΠŸΠΎΡΡ‚ΠΎΠΌΡƒ Π½Π° Ρ‚Π΅ΠΌΠ΅ Π²ΠΎΠ·Π²Π΅Π΄Π΅Π½ΠΈΠ΅ Π΄Ρ€ΠΎΠ±ΠΈ Π² ΡΡ‚Π΅ΠΏΠ΅Π½ΡŒ ΠΌΡ‹ остановимся Π±ΠΎΠ»Π΅Π΅ ΠΏΠΎΠ΄Ρ€ΠΎΠ±Π½ΠΎ Π½Π° ΡΠ»Π΅Π΄ΡƒΡŽΡ‰Π΅ΠΉ страницС.

    Π‘Ρ‚Π΅ΠΏΠ΅Π½ΠΈ ΠΈ ΠΊΠΎΡ€Π½ΠΈ

    ΠžΠΏΠ΅Ρ€Π°Ρ†ΠΈΠΈ со стСпСнями ΠΈ корнями. Π‘Ρ‚Π΅ΠΏΠ΅Π½ΡŒ с ΠΎΡ‚Ρ€ΠΈΡ†Π°Ρ‚Π΅Π»ΡŒΠ½Ρ‹ΠΌ ,

    Π½ΡƒΠ»Π΅Π²Ρ‹ΠΌ ΠΈ Π΄Ρ€ΠΎΠ±Π½Ρ‹ΠΌ ΠΏΠΎΠΊΠ°Π·Π°Ρ‚Π΅Π»Π΅ΠΌ. О выраТСниях, Π½Π΅ ΠΈΠΌΠ΅ΡŽΡ‰ΠΈΡ… смысла.

    ΠžΠΏΠ΅Ρ€Π°Ρ†ΠΈΠΈ со стСпСнями.

    1. ΠŸΡ€ΠΈ ΡƒΠΌΠ½ΠΎΠΆΠ΅Π½ΠΈΠΈ стСпСнСй с ΠΎΠ΄ΠΈΠ½Π°ΠΊΠΎΠ²Ρ‹ΠΌ основаниСм ΠΈΡ… ΠΏΠΎΠΊΠ°Π·Π°Ρ‚Π΅Π»ΠΈ ΡΠΊΠ»Π°Π΄Ρ‹Π²Π°ΡŽΡ‚ΡΡ:

    a m Β· a n = a m + n .

    2. ΠŸΡ€ΠΈ Π΄Π΅Π»Π΅Π½ΠΈΠΈ стСпСнСй с ΠΎΠ΄ΠΈΠ½Π°ΠΊΠΎΠ²Ρ‹ΠΌ основаниСм ΠΈΡ… ΠΏΠΎΠΊΠ°Π·Π°Ρ‚Π΅Π»ΠΈ Π²Ρ‹Ρ‡ΠΈΡ‚Π°ΡŽΡ‚ΡΡ .

    3. Π‘Ρ‚Π΅ΠΏΠ΅Π½ΡŒ произвСдСния Π΄Π²ΡƒΡ… ΠΈΠ»ΠΈ Π½Π΅ΡΠΊΠΎΠ»ΡŒΠΊΠΈΡ… сомноТитСлСй Ρ€Π°Π²Π½Π° ΠΏΡ€ΠΎΠΈΠ·Π²Π΅Π΄Π΅Π½ΠΈΡŽ стСпСнСй этих сомноТитСлСй.

    4. Π‘Ρ‚Π΅ΠΏΠ΅Π½ΡŒ ΠΎΡ‚Π½ΠΎΡˆΠ΅Π½ΠΈΡ (Π΄Ρ€ΠΎΠ±ΠΈ) Ρ€Π°Π²Π½Π° ΠΎΡ‚Π½ΠΎΡˆΠ΅Π½ΠΈΡŽ стСпСнСй Π΄Π΅Π»ΠΈΠΌΠΎΠ³ΠΎ (числитСля) ΠΈ дСлитСля (знамСнатСля):

    (a / b ) n = a n / b n .

    5. ΠŸΡ€ΠΈ Π²ΠΎΠ·Π²Π΅Π΄Π΅Π½ΠΈΠΈ стСпСни Π² ΡΡ‚Π΅ΠΏΠ΅Π½ΡŒ ΠΈΡ… ΠΏΠΎΠΊΠ°Π·Π°Ρ‚Π΅Π»ΠΈ ΠΏΠ΅Ρ€Π΅ΠΌΠ½ΠΎΠΆΠ°ΡŽΡ‚ΡΡ:

    ВсС Π²Ρ‹ΡˆΠ΅ΠΏΡ€ΠΈΠ²Π΅Π΄Π΅Π½Π½Ρ‹Π΅ Ρ„ΠΎΡ€ΠΌΡƒΠ»Ρ‹ Ρ‡ΠΈΡ‚Π°ΡŽΡ‚ΡΡ ΠΈ Π²Ρ‹ΠΏΠΎΠ»Π½ΡΡŽΡ‚ΡΡ Π² ΠΎΠ±ΠΎΠΈΡ… направлСниях слСва Π½Π°ΠΏΡ€Π°Π²ΠΎ ΠΈ Π½Π°ΠΎΠ±ΠΎΡ€ΠΎΡ‚.

    П Ρ€ ΠΈ ΠΌ Π΅ Ρ€. (2 Β· 3 Β· 5 / 15) Β² = 2 Β² Β· 3 Β² Β· 5 Β² / 15 Β² = 900 / 225 = 4 .

    ΠžΠΏΠ΅Ρ€Π°Ρ†ΠΈΠΈ с корнями. Π’ΠΎ всСх Π½ΠΈΠΆΠ΅ΠΏΡ€ΠΈΠ²Π΅Π΄Π΅Π½Π½Ρ‹Ρ… Ρ„ΠΎΡ€ΠΌΡƒΠ»Π°Ρ… символ ΠΎΠ·Π½Π°Ρ‡Π°Π΅Ρ‚ арифмСтичСский ΠΊΠΎΡ€Π΅Π½ΡŒ (ΠΏΠΎΠ΄ΠΊΠΎΡ€Π΅Π½Π½ΠΎΠ΅ Π²Ρ‹Ρ€Π°ΠΆΠ΅Π½ΠΈΠ΅ ΠΏΠΎΠ»ΠΎΠΆΠΈΡ‚Π΅Π»ΡŒΠ½ΠΎ).

    1. ΠšΠΎΡ€Π΅Π½ΡŒ ΠΈΠ· произвСдСния Π½Π΅ΡΠΊΠΎΠ»ΡŒΠΊΠΈΡ… сомноТитСлСй Ρ€Π°Π²Π΅Π½ ΠΏΡ€ΠΎΠΈΠ·Π²Π΅Π΄Π΅Π½ΠΈΡŽ ΠΊΠΎΡ€Π½Π΅ΠΉ ΠΈΠ· этих сомноТитСлСй:

    2. ΠšΠΎΡ€Π΅Π½ΡŒ ΠΈΠ· ΠΎΡ‚Π½ΠΎΡˆΠ΅Π½ΠΈΡ Ρ€Π°Π²Π΅Π½ ΠΎΡ‚Π½ΠΎΡˆΠ΅Π½ΠΈΡŽ ΠΊΠΎΡ€Π½Π΅ΠΉ Π΄Π΅Π»ΠΈΠΌΠΎΠ³ΠΎ ΠΈ дСлитСля:

    3. ΠŸΡ€ΠΈ Π²ΠΎΠ·Π²Π΅Π΄Π΅Π½ΠΈΠΈ корня Π² ΡΡ‚Π΅ΠΏΠ΅Π½ΡŒ достаточно возвСсти Π² эту ΡΡ‚Π΅ΠΏΠ΅Π½ΡŒ ΠΏΠΎΠ΄ΠΊΠΎΡ€Π΅Π½Π½ΠΎΠ΅ число:

    4. Если ΡƒΠ²Π΅Π»ΠΈΡ‡ΠΈΡ‚ΡŒ ΡΡ‚Π΅ΠΏΠ΅Π½ΡŒ корня Π² m Ρ€Π°Π· ΠΈ ΠΎΠ΄Π½ΠΎΠ²Ρ€Π΅ΠΌΠ΅Π½Π½ΠΎ возвСсти Π² m -ΡƒΡŽ ΡΡ‚Π΅ΠΏΠ΅Π½ΡŒ ΠΏΠΎΠ΄ΠΊΠΎΡ€Π΅Π½Π½ΠΎΠ΅ число, Ρ‚ΠΎ Π·Π½Π°Ρ‡Π΅Π½ΠΈΠ΅ корня Π½Π΅ измСнится:

    5. Если ΡƒΠΌΠ΅Π½ΡŒΡˆΠΈΡ‚ΡŒ ΡΡ‚Π΅ΠΏΠ΅Π½ΡŒ корня Π² m Ρ€Π°Π· ΠΈ ΠΎΠ΄Π½ΠΎΠ²Ρ€Π΅ΠΌΠ΅Π½Π½ΠΎ ΠΈΠ·Π²Π»Π΅Ρ‡ΡŒ ΠΊΠΎΡ€Π΅Π½ΡŒ m -ΠΎΠΉ стСпСни ΠΈΠ· ΠΏΠΎΠ΄ΠΊΠΎΡ€Π΅Π½Π½ΠΎΠ³ΠΎ числа, Ρ‚ΠΎ Π·Π½Π°Ρ‡Π΅Π½ΠΈΠ΅ корня Π½Π΅ измСнится:


    Π Π°ΡΡˆΠΈΡ€Π΅Π½ΠΈΠ΅ понятия стСпСни. Π”ΠΎ сих ΠΏΠΎΡ€ ΠΌΡ‹ рассматривали стСпСни Ρ‚ΠΎΠ»ΡŒΠΊΠΎ с Π½Π°Ρ‚ΡƒΡ€Π°Π»ΡŒΠ½Ρ‹ΠΌ ΠΏΠΎΠΊΠ°Π·Π°Ρ‚Π΅Π»Π΅ΠΌ; Π½ΠΎ дСйствия со стСпСнями ΠΈ корнями ΠΌΠΎΠ³ΡƒΡ‚ ΠΏΡ€ΠΈΠ²ΠΎΠ΄ΠΈΡ‚ΡŒ Ρ‚Π°ΠΊΠΆΠ΅ ΠΊ ΠΎΡ‚Ρ€ΠΈΡ†Π°Ρ‚Π΅Π»ΡŒΠ½Ρ‹ΠΌ , Π½ΡƒΠ»Π΅Π²Ρ‹ΠΌ ΠΈ Π΄Ρ€ΠΎΠ±Π½Ρ‹ΠΌ показатСлям. ВсС эти ΠΏΠΎΠΊΠ°Π·Π°Ρ‚Π΅Π»ΠΈ стСпСнСй Ρ‚Ρ€Π΅Π±ΡƒΡŽΡ‚ Π΄ΠΎΠΏΠΎΠ»Π½ΠΈΡ‚Π΅Π»ΡŒΠ½ΠΎΠ³ΠΎ опрСдСлСния.

    Π‘Ρ‚Π΅ΠΏΠ΅Π½ΡŒ с ΠΎΡ‚Ρ€ΠΈΡ†Π°Ρ‚Π΅Π»ΡŒΠ½Ρ‹ΠΌ ΠΏΠΎΠΊΠ°Π·Π°Ρ‚Π΅Π»Π΅ΠΌ. Π‘Ρ‚Π΅ΠΏΠ΅Π½ΡŒ Π½Π΅ΠΊΠΎΡ‚ΠΎΡ€ΠΎΠ³ΠΎ числа с ΠΎΡ‚Ρ€ΠΈΡ†Π°Ρ‚Π΅Π»ΡŒΠ½Ρ‹ΠΌ (Ρ†Π΅Π»Ρ‹ΠΌ) ΠΏΠΎΠΊΠ°Π·Π°Ρ‚Π΅Π»Π΅ΠΌ опрСдСляСтся ΠΊΠ°ΠΊ Π΅Π΄ΠΈΠ½ΠΈΡ†Π°, дСлённая Π½Π° ΡΡ‚Π΅ΠΏΠ΅Π½ΡŒ Ρ‚ΠΎΠ³ΠΎ ΠΆΠ΅ числа с ΠΏΠΎΠΊΠ°Π·Π°Ρ‚Π΅Π»Π΅ΠΌ, Ρ€Π°Π²Π½Ρ‹ΠΌ Π°Π±ΡΠΎΠ»ΡŽΡ‚Π½ΠΎΠΉ Π²Π΅Π»Π΅Ρ‡ΠΈΠ½Π΅ ΠΎΡ‚Ρ€ΠΈΡ†Π°Ρ‚Π΅Π»ΡŒΠ½ΠΎΠ³ΠΎ показатСля:

    Π’ Π΅ΠΏΠ΅Ρ€ΡŒ Ρ„ΠΎΡ€ΠΌΡƒΠ»Π° a m : a n = a m β€” n ΠΌΠΎΠΆΠ΅Ρ‚ Π±Ρ‹Ρ‚ΡŒ использована Π½Π΅ Ρ‚ΠΎΠ»ΡŒΠΊΠΎ ΠΏΡ€ΠΈ m , большСм, Ρ‡Π΅ΠΌ n , Π½ΠΎ ΠΈ ΠΏΡ€ΠΈ m , мСньшСм, Ρ‡Π΅ΠΌ n .

    П Ρ€ ΠΈ ΠΌ Π΅ Ρ€. a 4: a 7 = a 4 β€” 7 = a β€” 3 .

    Если ΠΌΡ‹ Ρ…ΠΎΡ‚ΠΈΠΌ, Ρ‡Ρ‚ΠΎΠ±Ρ‹ Ρ„ΠΎΡ€ΠΌΡƒΠ»Π° a m : a n = a m β€” n Π±Ρ‹Π»Π° справСдлива ΠΏΡ€ΠΈ m = n , Π½Π°ΠΌ Π½Π΅ΠΎΠ±Ρ…ΠΎΠ΄ΠΈΠΌΠΎ ΠΎΠΏΡ€Π΅Π΄Π΅Π»Π΅Π½ΠΈΠ΅ Π½ΡƒΠ»Π΅Π²ΠΎΠΉ стСпСни.

    Π‘Ρ‚Π΅ΠΏΠ΅Π½ΡŒ с Π½ΡƒΠ»Π΅Π²Ρ‹ΠΌ ΠΏΠΎΠΊΠ°Π·Π°Ρ‚Π΅Π»Π΅ΠΌ. Π‘Ρ‚Π΅ΠΏΠ΅Π½ΡŒ любого Π½Π΅Π½ΡƒΠ»Π΅Π²ΠΎΠ³ΠΎ числа с Π½ΡƒΠ»Π΅Π²Ρ‹ΠΌ ΠΏΠΎΠΊΠ°Π·Π°Ρ‚Π΅Π»Π΅ΠΌ Ρ€Π°Π²Π½Π° 1.

    П Ρ€ ΠΈ ΠΌ Π΅ Ρ€ Ρ‹. 2 0 = 1, (– 5) 0 = 1, (– 3 / 5) 0 = 1.

    Π‘Ρ‚Π΅ΠΏΠ΅Π½ΡŒ с Π΄Ρ€ΠΎΠ±Π½Ρ‹ΠΌ ΠΏΠΎΠΊΠ°Π·Π°Ρ‚Π΅Π»Π΅ΠΌ. Для Ρ‚ΠΎΠ³ΠΎ, Ρ‡Ρ‚ΠΎΠ±Ρ‹ возвСсти Π΄Π΅ΠΉΡΡ‚Π²ΠΈΡ‚Π΅Π»ΡŒΠ½ΠΎΠ΅ число Π° Π² ΡΡ‚Π΅ΠΏΠ΅Π½ΡŒ m / n , Π½ΡƒΠΆΠ½ΠΎ ΠΈΠ·Π²Π»Π΅Ρ‡ΡŒ ΠΊΠΎΡ€Π΅Π½ΡŒ n –ой стСпСни ΠΈΠ· m -ΠΎΠΉ стСпСни этого числа Π°:

    О выраТСниях, Π½Π΅ ΠΈΠΌΠ΅ΡŽΡ‰ΠΈΡ… смысла. Π•ΡΡ‚ΡŒ нСсколько Ρ‚Π°ΠΊΠΈΡ… Π²Ρ‹Ρ€Π°ΠΆΠ΅Π½ΠΈΠΉ.

    Π³Π΄Π΅ a β‰  0 , Π½Π΅ сущСствуСт.

    Π’ самом Π΄Π΅Π»Π΅, Ссли ΠΏΡ€Π΅Π΄ΠΏΠΎΠ»ΠΎΠΆΠΈΡ‚ΡŒ, Ρ‡Ρ‚ΠΎ x – Π½Π΅ΠΊΠΎΡ‚ΠΎΡ€ΠΎΠ΅ число, Ρ‚ΠΎ Π² соотвСтствии с ΠΎΠΏΡ€Π΅Π΄Π΅Π»Π΅Π½ΠΈΠ΅ΠΌ ΠΎΠΏΠ΅Ρ€Π°Ρ†ΠΈΠΈ дСлСния ΠΈΠΌΠ΅Π΅ΠΌ: a = 0Β· x , Ρ‚.e. a = 0, Ρ‡Ρ‚ΠΎ ΠΏΡ€ΠΎΡ‚ΠΈΠ²ΠΎΡ€Π΅Ρ‡ΠΈΡ‚ ΡƒΡΠ»ΠΎΠ²ΠΈΡŽ: a β‰  0

    β€” любоС число.

    Π’ самом Π΄Π΅Π»Π΅, Ссли ΠΏΡ€Π΅Π΄ΠΏΠΎΠ»ΠΎΠΆΠΈΡ‚ΡŒ, Ρ‡Ρ‚ΠΎ это Π²Ρ‹Ρ€Π°ΠΆΠ΅Π½ΠΈΠ΅ Ρ€Π°Π²Π½ΠΎ Π½Π΅ΠΊΠΎΡ‚ΠΎΡ€ΠΎΠΌΡƒ числу x , Ρ‚ΠΎ согласно ΠΎΠΏΡ€Π΅Π΄Π΅Π»Π΅Π½ΠΈΡŽ ΠΎΠΏΠ΅Ρ€Π°Ρ†ΠΈΠΈ дСлСния ΠΈΠΌΠ΅Π΅ΠΌ: 0 = 0 Β· x . Но это равСнство ΠΈΠΌΠ΅Π΅Ρ‚ мСсто ΠΏΡ€ΠΈ любом числС x , Ρ‡Ρ‚ΠΎ ΠΈ Ρ‚Ρ€Π΅Π±ΠΎΠ²Π°Π»ΠΎΡΡŒ Π΄ΠΎΠΊΠ°Π·Π°Ρ‚ΡŒ.

    0 0 β€” любоС число.

    Π  Π΅ ш Π΅ Π½ ΠΈ Π΅. Рассмотрим Ρ‚Ρ€ΠΈ основных случая:

    1) x = 0 – это Π·Π½Π°Ρ‡Π΅Π½ΠΈΠ΅ Π½Π΅ удовлСтворяСт Π΄Π°Π½Π½ΠΎΠΌΡƒ ΡƒΡ€Π°Π²Π½Π΅Π½ΠΈΡŽ

    2) ΠΏΡ€ΠΈ x > 0 ΠΏΠΎΠ»ΡƒΡ‡Π°Π΅ΠΌ: x / x = 1, Ρ‚.e. 1 = 1, ΠΎΡ‚ΠΊΡƒΠ΄Π° слСдуСт,

    Ρ‡Ρ‚ΠΎ x – любоС число; Π½ΠΎ принимая Π²ΠΎ Π²Π½ΠΈΠΌΠ°Π½ΠΈΠ΅, Ρ‡Ρ‚ΠΎ Π²

    нашСм случаС x > 0 , ΠΎΡ‚Π²Π΅Ρ‚ΠΎΠΌ являСтся x > 0 ;

    ΠŸΡ€Π°Π²ΠΈΠ»Π° умноТСния стСпСнСй с Ρ€Π°Π·Π½Ρ‹ΠΌ основаниСм

    Π‘Π’Π•ΠŸΠ•ΠΠ¬ Π‘ Π ΠΠ¦Π˜ΠžΠΠΠ›Π¬ΠΠ«Πœ ΠŸΠžΠšΠΠ—ΠΠ’Π•Π›Π•Πœ,

    Π‘Π’Π•ΠŸΠ•ΠΠΠΠ― ЀУНКЦИЯ IV

    Β§ 69. Π£ΠΌΠ½ΠΎΠΆΠ΅Π½ΠΈΠ΅ ΠΈ Π΄Π΅Π»Π΅Π½ΠΈΠ΅ стСпСнСй с ΠΎΠ΄ΠΈΠ½Π°ΠΊΠΎΠ²Ρ‹ΠΌΠΈ основаниями

    Π’Π΅ΠΎΡ€Π΅ΠΌΠ° 1. Π§Ρ‚ΠΎΠ±Ρ‹ ΠΏΠ΅Ρ€Π΅ΠΌΠ½ΠΎΠΆΠΈΡ‚ΡŒ стСпСни с ΠΎΠ΄ΠΈΠ½Π°ΠΊΠΎΠ²Ρ‹ΠΌΠΈ основаниями, достаточно ΠΏΠΎΠΊΠ°Π·Π°Ρ‚Π΅Π»ΠΈ стСпСнСй ΡΠ»ΠΎΠΆΠΈΡ‚ΡŒ, Π° основаниС ΠΎΡΡ‚Π°Π²ΠΈΡ‚ΡŒ ΠΏΡ€Π΅ΠΆΠ½ΠΈΠΌ , Ρ‚ΠΎ Π΅ΡΡ‚ΡŒ

    Π”ΠΎΠΊΠ°Π·Π°Ρ‚Π΅Π»ΡŒΡΡ‚Π²ΠΎ. По ΠΎΠΏΡ€Π΅Π΄Π΅Π»Π΅Π½ΠΈΡŽ стСпСни

    2 2 2 3 = 2 5 = 32; (-3) (-3) 3 = (-3) 4 = 81.

    ΠœΡ‹ рассмотрСли ΠΏΡ€ΠΎΠΈΠ·Π²Π΅Π΄Π΅Π½ΠΈΠ΅ Π΄Π²ΡƒΡ… стСпСнСй. На самом ΠΆΠ΅ Π΄Π΅Π»Π΅ Π΄ΠΎΠΊΠ°Π·Π°Π½Π½ΠΎΠ΅ свойство Π²Π΅Ρ€Π½ΠΎ для любого числа стСпСнСй с ΠΎΠ΄ΠΈΠ½Π°ΠΊΠΎΠ²Ρ‹ΠΌΠΈ основаниями.

    Π’Π΅ΠΎΡ€Π΅ΠΌΠ° 2. Π§Ρ‚ΠΎΠ±Ρ‹ Ρ€Π°Π·Π΄Π΅Π»ΠΈΡ‚ΡŒ стСпСни с ΠΎΠ΄ΠΈΠ½Π°ΠΊΠΎΠ²Ρ‹ΠΌΠΈ основаниями, ΠΊΠΎΠ³Π΄Π° ΠΏΠΎΠΊΠ°Π·Π°Ρ‚Π΅Π»ΡŒ Π΄Π΅Π»ΠΈΠΌΠΎΠ³ΠΎ большС показатСля дСлитСля, достаточно ΠΈΠ· показатСля Π΄Π΅Π»ΠΈΠΌΠΎΠ³ΠΎ Π²Ρ‹Ρ‡Π΅ΡΡ‚ΡŒ ΠΏΠΎΠΊΠ°Π·Π°Ρ‚Π΅Π»ΡŒ дСлитСля, Π° основаниС ΠΎΡΡ‚Π°Π²ΠΈΡ‚ΡŒ ΠΏΡ€Π΅ΠΆΠ½ΠΈΠΌ, Ρ‚ΠΎ Π΅ΡΡ‚ΡŒ ΠΏΡ€ΠΈ Ρ‚ > ΠΏ

    (a =/= 0)

    Π”ΠΎΠΊΠ°Π·Π°Ρ‚Π΅Π»ΡŒΡΡ‚Π²ΠΎ. Напомним, Ρ‡Ρ‚ΠΎ частным ΠΎΡ‚ дСлСния ΠΎΠ΄Π½ΠΎΠ³ΠΎ числа Π½Π° Π΄Ρ€ΡƒΠ³ΠΎΠ΅ называСтся число, ΠΊΠΎΡ‚ΠΎΡ€ΠΎΠ΅ ΠΏΡ€ΠΈ ΡƒΠΌΠ½ΠΎΠΆΠ΅Π½ΠΈΠΈ Π½Π° Π΄Π΅Π»ΠΈΡ‚Π΅Π»ΡŒ Π΄Π°Π΅Ρ‚ Π΄Π΅Π»ΠΈΠΌΠΎΠ΅. ΠŸΠΎΡΡ‚ΠΎΠΌΡƒ Π΄ΠΎΠΊΠ°Π·Π°Ρ‚ΡŒ Ρ„ΠΎΡ€ΠΌΡƒΠ»Ρƒ , Π³Π΄Π΅ a =/= 0, это всС Ρ€Π°Π²Π½ΠΎ, Ρ‡Ρ‚ΠΎ Π΄ΠΎΠΊΠ°Π·Π°Ρ‚ΡŒ Ρ„ΠΎΡ€ΠΌΡƒΠ»Ρƒ

    Если Ρ‚ > ΠΏ , Ρ‚ΠΎ число Ρ‚ — ΠΏ Π±ΡƒΠ΄Π΅Ρ‚ Π½Π°Ρ‚ΡƒΡ€Π°Π»ΡŒΠ½Ρ‹ΠΌ; ΡΠ»Π΅Π΄ΠΎΠ²Π°Ρ‚Π΅Π»ΡŒΠ½ΠΎ, ΠΏΠΎ Ρ‚Π΅ΠΎΡ€Π΅ΠΌΠ΅ 1

    Π’Π΅ΠΎΡ€Π΅ΠΌΠ° 2 Π΄ΠΎΠΊΠ°Π·Π°Π½Π°.

    Π‘Π»Π΅Π΄ΡƒΠ΅Ρ‚ ΠΎΠ±Ρ€Π°Ρ‚ΠΈΡ‚ΡŒ Π²Π½ΠΈΠΌΠ°Π½ΠΈΠ΅ Π½Π° Ρ‚ΠΎ, Ρ‡Ρ‚ΠΎ Ρ„ΠΎΡ€ΠΌΡƒΠ»Π°

    Π΄ΠΎΠΊΠ°Π·Π°Π½Π° Π½Π°ΠΌΠΈ лишь Π² ΠΏΡ€Π΅Π΄ΠΏΠΎΠ»ΠΎΠΆΠ΅Π½ΠΈΠΈ, Ρ‡Ρ‚ΠΎ Ρ‚ > ΠΏ . ΠŸΠΎΡΡ‚ΠΎΠΌΡƒ ΠΈΠ· Π΄ΠΎΠΊΠ°Π·Π°Π½Π½ΠΎΠ³ΠΎ ΠΏΠΎΠΊΠ° нСльзя Π΄Π΅Π»Π°Ρ‚ΡŒ, Π½Π°ΠΏΡ€ΠΈΠΌΠ΅Ρ€, Ρ‚Π°ΠΊΠΈΡ… Π²Ρ‹Π²ΠΎΠ΄ΠΎΠ²:

    К Ρ‚ΠΎΠΌΡƒ ΠΆΠ΅ стСпСни с ΠΎΡ‚Ρ€ΠΈΡ†Π°Ρ‚Π΅Π»ΡŒΠ½Ρ‹ΠΌΠΈ показатСлями Π½Π°ΠΌΠΈ Π΅Ρ‰Π΅ Π½Π΅ Ρ€Π°ΡΡΠΌΠ°Ρ‚Ρ€ΠΈΠ²Π°Π»ΠΈΡΡŒ ΠΈ ΠΌΡ‹ ΠΏΠΎΠΊΠ° Ρ‡Ρ‚ΠΎ Π½Π΅ Π·Π½Π°Π΅ΠΌ, ΠΊΠ°ΠΊΠΎΠΉ смысл ΠΌΠΎΠΆΠ½ΠΎ ΠΏΡ€ΠΈΠ΄Π°Ρ‚ΡŒ Π²Ρ‹Ρ€Π°ΠΆΠ΅Π½ΠΈΡŽ 3 2 .

    Π’Π΅ΠΎΡ€Π΅ΠΌΠ° 3. Π§Ρ‚ΠΎΠ±Ρ‹ возвСсти ΡΡ‚Π΅ΠΏΠ΅Π½ΡŒ Π² ΡΡ‚Π΅ΠΏΠ΅Π½ΡŒ, достаточно ΠΏΠ΅Ρ€Π΅ΠΌΠ½ΠΎΠΆΠΈΡ‚ΡŒ ΠΏΠΎΠΊΠ°Π·Π°Ρ‚Π΅Π»ΠΈ, оставив основаниС стСпСни ΠΏΡ€Π΅ΠΆΠ½ΠΈΠΌ , Ρ‚ΠΎ Π΅ΡΡ‚ΡŒ

    Π”ΠΎΠΊΠ°Π·Π°Ρ‚Π΅Π»ΡŒΡΡ‚Π²ΠΎ. Π˜ΡΠΏΠΎΠ»ΡŒΠ·ΡƒΡ ΠΎΠΏΡ€Π΅Π΄Π΅Π»Π΅Π½ΠΈΠ΅ стСпСни ΠΈ Ρ‚Π΅ΠΎΡ€Π΅ΠΌΡƒ 1 этого ΠΏΠ°Ρ€Π°Π³Ρ€Π°Ρ„Π°, ΠΏΠΎΠ»ΡƒΡ‡Π°Π΅ΠΌ:

    Ρ‡Ρ‚ΠΎ ΠΈ Ρ‚Ρ€Π΅Π±ΠΎΠ²Π°Π»ΠΎΡΡŒ Π΄ΠΎΠΊΠ°Π·Π°Ρ‚ΡŒ.

    НапримСр, (2 3) 2 = 2 6 = 64;

    518 (Устно.) ΠžΠΏΡ€Π΅Π΄Π΅Π»ΠΈΡ‚ΡŒ Ρ… ΠΈΠ· ΡƒΡ€Π°Π²Π½Π΅Π½ΠΈΠΉ:

    1) 2 2 2 2 3 2 4 2 5 2 6 = 2 x ; 3) 4 2 4 4 4 6 4 8 4 10 = 2 x ;

    2) 3 3 3 3 5 3 7 3 9 = 3 x ; 4) 1 / 5 1 / 25 1 / 125 1 / 625 = 1 / 5 x .

    519. (Π£ с Ρ‚ Π½ ΠΎ.) Π£ΠΏΡ€ΠΎΡΡ‚ΠΈΡ‚ΡŒ:

    520. (Π£ с Ρ‚ Π½ ΠΎ.) Π£ΠΏΡ€ΠΎΡΡ‚ΠΈΡ‚ΡŒ:

    521. Π”Π°Π½Π½Ρ‹Π΅ выраТСния ΠΏΡ€Π΅Π΄ΡΡ‚Π°Π²ΠΈΡ‚ΡŒ Π² Π²ΠΈΠ΄Π΅ стСпСнСй с ΠΎΠ΄ΠΈΠ½Π°ΠΊΠΎΠ²Ρ‹ΠΌΠΈ основаниями:

    1) 32 ΠΈ 64; 3) 8 5 ΠΈ 16 3 ; 5) 4 100 ΠΈ 32 50 ;

    2) -1000 ΠΈ 100; 4) -27 ΠΈ -243; 6) 81 75 8 200 ΠΈ 3 600 4 150 .

    Π‘Π»ΠΎΠΆΠ΅Π½ΠΈΠ΅ ΠΏΠ΅Ρ€Π΅ΠΌΠ΅Π½Π½Ρ‹Ρ… с Ρ€Π°Π·Π½Ρ‹ΠΌΠΈ стСпСнями. ДСйствия с ΠΎΠ΄Π½ΠΎΡ‡Π»Π΅Π½Π°ΠΌΠΈ. ΠŸΡ€ΠΈΠΌΠ΅Π½Π΅Π½ΠΈΠ΅ стСпСнСй ΠΈ ΠΈΡ… свойств

    Одной ΠΈΠ· Π³Π»Π°Π²Π½Ρ‹Ρ… характСристик Π² Π°Π»Π³Π΅Π±Ρ€Π΅, Π΄Π° ΠΈ Π²ΠΎ всСй ΠΌΠ°Ρ‚Π΅ΠΌΠ°Ρ‚ΠΈΠΊΠ΅ являСтся ΡΡ‚Π΅ΠΏΠ΅Π½ΡŒ. ΠšΠΎΠ½Π΅Ρ‡Π½ΠΎ, Π² 21 Π²Π΅ΠΊΠ΅ всС расчСты ΠΌΠΎΠΆΠ½ΠΎ ΠΏΡ€ΠΎΠ²ΠΎΠ΄ΠΈΡ‚ΡŒ Π½Π° ΠΎΠ½Π»Π°ΠΉΠ½-ΠΊΠ°Π»ΡŒΠΊΡƒΠ»ΡΡ‚ΠΎΡ€Π΅, Π½ΠΎ Π»ΡƒΡ‡ΡˆΠ΅ для развития ΠΌΠΎΠ·Π³ΠΎΠ² Π½Π°ΡƒΡ‡ΠΈΡ‚ΡŒΡΡ Π΄Π΅Π»Π°Ρ‚ΡŒ это самому.

    Π’ Π΄Π°Π½Π½ΠΎΠΉ ΡΡ‚Π°Ρ‚ΡŒΠ΅ рассмотрим самыС Π²Π°ΠΆΠ½Ρ‹Π΅ вопросы, ΠΊΠ°ΡΠ°ΡŽΡ‰ΠΈΠ΅ΡΡ этого опрСдСлСния. А ΠΈΠΌΠ΅Π½Π½ΠΎ, ΠΏΠΎΠΉΠΌΠ΅ΠΌ Ρ‡Ρ‚ΠΎ это Π²ΠΎΠΎΠ±Ρ‰Π΅ Ρ‚Π°ΠΊΠΎΠ΅ ΠΈ ΠΊΠ°ΠΊΠΎΠ²Ρ‹ основныС Π΅Π³ΠΎ Ρ„ΡƒΠ½ΠΊΡ†ΠΈΠΈ, ΠΊΠ°ΠΊΠΈΠ΅ ΠΈΠΌΠ΅ΡŽΡ‚ΡΡ свойства Π² ΠΌΠ°Ρ‚Π΅ΠΌΠ°Ρ‚ΠΈΠΊΠ΅.

    Рассмотрим Π½Π° ΠΏΡ€ΠΈΠΌΠ΅Ρ€Π°Ρ… Ρ‚ΠΎ, ΠΊΠ°ΠΊ выглядит расчСт, ΠΊΠ°ΠΊΠΎΠ²Ρ‹ основныС Ρ„ΠΎΡ€ΠΌΡƒΠ»Ρ‹. Π Π°Π·Π±Π΅Ρ€Π΅ΠΌ основныС Π²ΠΈΠ΄Ρ‹ Π²Π΅Π»ΠΈΡ‡ΠΈΠ½Ρ‹ ΠΈ Ρ‚ΠΎ, Ρ‡Π΅ΠΌ ΠΎΠ½ΠΈ ΠΎΡ‚Π»ΠΈΡ‡Π°ΡŽΡ‚ΡΡ ΠΎΡ‚ Π΄Ρ€ΡƒΠ³ΠΈΡ… Ρ„ΡƒΠ½ΠΊΡ†ΠΈΠΉ.

    ПоймСм, ΠΊΠ°ΠΊ Ρ€Π΅ΡˆΠ°Ρ‚ΡŒ с ΠΏΠΎΠΌΠΎΡ‰ΡŒΡŽ этой Π²Π΅Π»ΠΈΡ‡ΠΈΠ½Ρ‹ Ρ€Π°Π·Π»ΠΈΡ‡Π½Ρ‹Π΅ Π·Π°Π΄Π°Ρ‡ΠΈ. ПокаТСм Π½Π° ΠΏΡ€ΠΈΠΌΠ΅Ρ€Π°Ρ…, ΠΊΠ°ΠΊ Π²ΠΎΠ·Π²ΠΎΠ΄ΠΈΡ‚ΡŒ Π² Π½ΡƒΠ»Π΅Π²ΡƒΡŽ ΡΡ‚Π΅ΠΏΠ΅Π½ΡŒ, ΠΈΡ€Ρ€Π°Ρ†ΠΈΠΎΠ½Π°Π»ΡŒΠ½ΡƒΡŽ, ΠΎΡ‚Ρ€ΠΈΡ†Π°Ρ‚Π΅Π»ΡŒΠ½ΡƒΡŽ ΠΈ Π΄Ρ€.

    Онлайн-ΠΊΠ°Π»ΡŒΠΊΡƒΠ»ΡΡ‚ΠΎΡ€ возвСдСния Π² ΡΡ‚Π΅ΠΏΠ΅Π½ΡŒ

    Π§Ρ‚ΠΎ Ρ‚Π°ΠΊΠΎΠ΅ ΡΡ‚Π΅ΠΏΠ΅Π½ΡŒ числа

    Π§Ρ‚ΠΎ ΠΆΠ΅ ΠΏΠΎΠ΄Ρ€Π°Π·ΡƒΠΌΠ΅Π²Π°ΡŽΡ‚ ΠΏΠΎΠ΄ Π²Ρ‹Ρ€Π°ΠΆΠ΅Π½ΠΈΠ΅ΠΌ «возвСсти число Π² ΡΡ‚Π΅ΠΏΠ΅Π½ΡŒΒ»?

    Π‘Ρ‚Π΅ΠΏΠ΅Π½ΡŒΡŽ n числа Π° являСтся ΠΏΡ€ΠΎΠΈΠ·Π²Π΅Π΄Π΅Π½ΠΈΠ΅ ΠΌΠ½ΠΎΠΆΠΈΡ‚Π΅Π»Π΅ΠΉ Π²Π΅Π»ΠΈΡ‡ΠΈΠ½ΠΎΠΉ Π° n-Ρ€Π°Π· подряд.

    ΠœΠ°Ρ‚Π΅ΠΌΠ°Ρ‚ΠΈΡ‡Π΅ΡΠΊΠΈ это выглядит ΡΠ»Π΅Π΄ΡƒΡŽΡ‰ΠΈΠΌ ΠΎΠ±Ρ€Π°Π·ΠΎΠΌ:

    a n = a * a * a * …a n .

    НапримСр:

    • 2 3 = 2 Π² Ρ‚Ρ€Π΅Ρ‚ΡŒΠ΅ΠΉ стСп. = 2 * 2 * 2 = 8;
    • 4 2 = 4 Π² стСп. Π΄Π²Π° = 4 * 4 = 16;
    • 5 4 = 5 Π² стСп. Ρ‡Π΅Ρ‚Ρ‹Ρ€Π΅ = 5 * 5 * 5 * 5 = 625;
    • 10 5 = 10 Π² 5 стСп. = 10 * 10 * 10 * 10 * 10 = 100000;
    • 10 4 = 10 Π² 4 стСп. = 10 * 10 * 10 * 10 = 10000.

    НиТС Π±ΡƒΠ΄Π΅Ρ‚ прСдставлСна Ρ‚Π°Π±Π»ΠΈΡ†Π° ΠΊΠ²Π°Π΄Ρ€Π°Ρ‚ΠΎΠ² ΠΈ ΠΊΡƒΠ±ΠΎΠ² ΠΎΡ‚ 1 Π΄ΠΎ 10.

    Π’Π°Π±Π»ΠΈΡ†Π° стСпСнСй ΠΎΡ‚ 1 Π΄ΠΎ 10

    НиТС Π±ΡƒΠ΄ΡƒΡ‚ ΠΏΡ€ΠΈΠ²Π΅Π΄Π΅Π½Ρ‹ Ρ€Π΅Π·ΡƒΠ»ΡŒΡ‚Π°Ρ‚Ρ‹ возвСдСния Π½Π°Ρ‚ΡƒΡ€Π°Π»ΡŒΠ½Ρ‹Ρ… чисСл Π² ΠΏΠΎΠ»ΠΎΠΆΠΈΡ‚Π΅Π»ΡŒΠ½Ρ‹Π΅ стСпСни – Β«ΠΎΡ‚ 1 Π΄ΠΎ 100Β».

    Π§-Π»ΠΎ 2-ая ст-нь 3-я ст-нь
    1 1 1
    2 4 8
    3 9 27
    4 16 64
    5 25 125
    6 36 216
    7 49 343
    8 64 512
    9 81 279
    10 100 1000

    Бвойства стСпСнСй

    Π§Ρ‚ΠΎ ΠΆΠ΅ Ρ…Π°Ρ€Π°ΠΊΡ‚Π΅Ρ€Π½ΠΎ для Ρ‚Π°ΠΊΠΎΠΉ матСматичСской Ρ„ΡƒΠ½ΠΊΡ†ΠΈΠΈ? Рассмотрим Π±Π°Π·ΠΎΠ²Ρ‹Π΅ свойства.

    Π£Ρ‡Π΅Π½Ρ‹ΠΌΠΈ установлСно ΡΠ»Π΅Π΄ΡƒΡŽΡ‰ΠΈΠ΅ ΠΏΡ€ΠΈΠ·Π½Π°ΠΊΠΈ, Ρ…Π°Ρ€Π°ΠΊΡ‚Π΅Ρ€Π½Ρ‹Π΅ для всСх стСпСнСй:

    • a n * a m = (a) (n+m) ;
    • a n: a m = (a) (n-m) ;
    • (a b) m =(a) (b*m) .

    ΠŸΡ€ΠΎΠ²Π΅Ρ€ΠΈΠΌ Π½Π° ΠΏΡ€ΠΈΠΌΠ΅Ρ€Π°Ρ…:

    2 3 * 2 2 = 8 * 4 = 32. Π‘ Π΄Ρ€ΡƒΠ³ΠΎΠΉ стороны 2 5 = 2 * 2 * 2 * 2 * 2 =32.

    Аналогично: 2 3: 2 2 = 8 / 4 =2. Π˜Π½Π°Ρ‡Π΅ 2 3-2 = 2 1 =2.

    (2 3) 2 = 8 2 = 64. А Ссли ΠΏΠΎ-Π΄Ρ€ΡƒΠ³ΠΎΠΌΡƒ? 2 6 = 2 * 2 * 2 * 2 * 2 * 2 = 32 * 2 = 64.

    Как Π²ΠΈΠ΄ΠΈΠΌ, ΠΏΡ€Π°Π²ΠΈΠ»Π° Ρ€Π°Π±ΠΎΡ‚Π°ΡŽΡ‚.

    А ΠΊΠ°ΠΊ ΠΆΠ΅ Π±Ρ‹Ρ‚ΡŒ со слоТСниСм ΠΈ Π²Ρ‹Ρ‡ΠΈΡ‚Π°Π½ΠΈΠ΅ΠΌ ? Всё просто. ВыполняСтся сначала Π²ΠΎΠ·Π²Π΅Π΄Π΅Π½ΠΈΠ΅ Π² ΡΡ‚Π΅ΠΏΠ΅Π½ΡŒ, Π° ΡƒΠΆ ΠΏΠΎΡ‚ΠΎΠΌ слоТСниС ΠΈ Π²Ρ‹Ρ‡ΠΈΡ‚Π°Π½ΠΈΠ΅.

    ΠŸΠΎΡΠΌΠΎΡ‚Ρ€ΠΈΠΌ Π½Π° ΠΏΡ€ΠΈΠΌΠ΅Ρ€Π°Ρ…:

    • 3 3 + 2 4 = 27 + 16 = 43;
    • 5 2 – 3 2 = 25 – 9 = 16. ΠžΠ±Ρ€Π°Ρ‚ΠΈΡ‚Π΅ Π²Π½ΠΈΠΌΠ°Π½ΠΈΠ΅: ΠΏΡ€Π°Π²ΠΈΠ»ΠΎ Π½Π΅ Π±ΡƒΠ΄Π΅Ρ‚ Π²Ρ‹ΠΏΠΎΠ»Π½ΡΡ‚ΡŒΡΡ, Ссли сначала произвСсти Π²Ρ‹Ρ‡ΠΈΡ‚Π°Π½ΠΈΠ΅: (5 β€” 3) 2 = 2 2 = 4.

    А Π²ΠΎΡ‚ Π² этом случаС Π½Π°Π΄ΠΎ Π²Ρ‹Ρ‡ΠΈΡΠ»ΡΡ‚ΡŒ сначала слоТСниС, ΠΏΠΎΡΠΊΠΎΠ»ΡŒΠΊΡƒ ΠΏΡ€ΠΈΡΡƒΡ‚ΡΡ‚Π²ΡƒΡŽΡ‚ дСйствия Π² скобках: (5 + 3) 3 = 8 3 = 512.

    Как ΠΏΡ€ΠΎΠΈΠ·Π²ΠΎΠ΄ΠΈΡ‚ΡŒ вычислСния Π² Π±ΠΎΠ»Π΅Π΅ слоТных случаях ? ΠŸΠΎΡ€ΡΠ΄ΠΎΠΊ Ρ‚ΠΎΡ‚ ΠΆΠ΅:

    • ΠΏΡ€ΠΈ Π½Π°Π»ΠΈΡ‡ΠΈΠΈ скобок – Π½Π°Ρ‡ΠΈΠ½Π°Ρ‚ΡŒ Π½ΡƒΠΆΠ½ΠΎ с Π½ΠΈΡ…;
    • Π·Π°Ρ‚Π΅ΠΌ Π²ΠΎΠ·Π²Π΅Π΄Π΅Π½ΠΈΠ΅ Π² ΡΡ‚Π΅ΠΏΠ΅Π½ΡŒ;
    • ΠΏΠΎΡ‚ΠΎΠΌ Π²Ρ‹ΠΏΠΎΠ»Π½ΡΡ‚ΡŒ дСйствия умноТСния, дСлСния;
    • послС слоТСниС, Π²Ρ‹Ρ‡ΠΈΡ‚Π°Π½ΠΈΠ΅.

    Π•ΡΡ‚ΡŒ спСцифичСскиС свойства, Ρ…Π°Ρ€Π°ΠΊΡ‚Π΅Ρ€Π½Ρ‹Π΅ Π½Π΅ для всСх стСпСнСй:

    1. ΠšΠΎΡ€Π΅Π½ΡŒ n-ΠΎΠΉ стСпСни ΠΈΠ· числа a Π² стСпСни m Π·Π°ΠΏΠΈΡˆΠ΅Ρ‚ΡΡ Π² Π²ΠΈΠ΄Π΅: a m / n .
    2. ΠŸΡ€ΠΈ Π²ΠΎΠ·Π²Π΅Π΄Π΅Π½ΠΈΠΈ Π΄Ρ€ΠΎΠ±ΠΈ Π² ΡΡ‚Π΅ΠΏΠ΅Π½ΡŒ: этой ΠΏΡ€ΠΎΡ†Π΅Π΄ΡƒΡ€Π΅ ΠΏΠΎΠ΄Π²Π΅Ρ€ΠΆΠ΅Π½Ρ‹ ΠΊΠ°ΠΊ Ρ‡ΠΈΡΠ»ΠΈΡ‚Π΅Π»ΡŒ, Ρ‚Π°ΠΊ ΠΈ Π΅Π΅ Π·Π½Π°ΠΌΠ΅Π½Π°Ρ‚Π΅Π»ΡŒ.
    3. ΠŸΡ€ΠΈ Π²ΠΎΠ·Π²Π΅Π΄Π΅Π½ΠΈΠΈ произвСдСния Ρ€Π°Π·Π½Ρ‹Ρ… чисСл Π² ΡΡ‚Π΅ΠΏΠ΅Π½ΡŒ, Π²Ρ‹Ρ€Π°ΠΆΠ΅Π½ΠΈΠ΅ Π±ΡƒΠ΄Π΅Ρ‚ ΡΠΎΠΎΡ‚Π²Π΅Ρ‚ΡΡ‚Π²ΠΎΠ²Π°Ρ‚ΡŒ ΠΏΡ€ΠΎΠΈΠ·Π²Π΅Π΄Π΅Π½ΠΈΡŽ этих чисСл Π² Π·Π°Π΄Π°Π½Π½ΠΎΠΉ стСпСни. Π’ΠΎ Π΅ΡΡ‚ΡŒ: (a * b) n = a n * b n .
    4. ΠŸΡ€ΠΈ Π²ΠΎΠ·Π²Π΅Π΄Π΅Π½ΠΈΠΈ числа Π² ΠΎΡ‚Ρ€ΠΈΡ†Π°Ρ‚Π΅Π»ΡŒΠ½ΡƒΡŽ стСп., Π½ΡƒΠΆΠ½ΠΎ Ρ€Π°Π·Π΄Π΅Π»ΠΈΡ‚ΡŒ 1 Π½Π° число Π² Ρ‚ΠΎΠΉ ΠΆΠ΅ ст-Π½ΠΈ, Π½ΠΎ со Π·Π½Π°ΠΊΠΎΠΌ Β«+Β».
    5. Если Π·Π½Π°ΠΌΠ΅Π½Π°Ρ‚Π΅Π»ΡŒ Π΄Ρ€ΠΎΠ±ΠΈ находится Π² ΠΎΡ‚Ρ€ΠΈΡ†Π°Ρ‚Π΅Π»ΡŒΠ½ΠΎΠΉ стСпСни, Ρ‚ΠΎ это Π²Ρ‹Ρ€Π°ΠΆΠ΅Π½ΠΈΠ΅ Π±ΡƒΠ΄Π΅Ρ‚ Ρ€Π°Π²Π½ΠΎ ΠΏΡ€ΠΎΠΈΠ·Π²Π΅Π΄Π΅Π½ΠΈΡŽ числитСля Π½Π° Π·Π½Π°ΠΌΠ΅Π½Π°Ρ‚Π΅Π»ΡŒ Π² ΠΏΠΎΠ»ΠΎΠΆΠΈΡ‚Π΅Π»ΡŒΠ½ΠΎΠΉ стСпСни.
    6. Π›ΡŽΠ±ΠΎΠ΅ число Π² стСпСни 0 = 1, Π° Π² стСп. 1 = самому сСбС.

    Π­Ρ‚ΠΈ ΠΏΡ€Π°Π²ΠΈΠ»Π° Π²Π°ΠΆΠ½Ρ‹ Π² ΠΎΡ‚Π΄Π΅Π»ΡŒΠ½Ρ‹Ρ… случаях, ΠΈΡ… рассмотрим ΠΏΠΎΠ΄Ρ€ΠΎΠ±Π½Π΅ΠΉ Π½ΠΈΠΆΠ΅.

    Π‘Ρ‚Π΅ΠΏΠ΅Π½ΡŒ с ΠΎΡ‚Ρ€ΠΈΡ†Π°Ρ‚Π΅Π»ΡŒΠ½Ρ‹ΠΌ ΠΏΠΎΠΊΠ°Π·Π°Ρ‚Π΅Π»Π΅ΠΌ

    Π§Ρ‚ΠΎ Π΄Π΅Π»Π°Ρ‚ΡŒ ΠΏΡ€ΠΈ минусовой стСпСни, Ρ‚. Π΅. ΠΊΠΎΠ³Π΄Π° ΠΏΠΎΠΊΠ°Π·Π°Ρ‚Π΅Π»ΡŒ ΠΎΡ‚Ρ€ΠΈΡ†Π°Ρ‚Π΅Π»ΡŒΠ½Ρ‹ΠΉ?

    Π˜ΡΡ…ΠΎΠ΄Ρ ΠΈΠ· свойств 4 ΠΈ 5 (смотри ΠΏΡƒΠ½ΠΊΡ‚ Π²Ρ‹ΡˆΠ΅), получаСтся :

    A (- n) = 1 / A n , 5 (-2) = 1 / 5 2 = 1 / 25.

    И Π½Π°ΠΎΠ±ΠΎΡ€ΠΎΡ‚:

    1 / A (- n) = A n , 1 / 2 (-3) = 2 3 = 8.

    А Ссли Π΄Ρ€ΠΎΠ±ΡŒ?

    (A / B) (- n) = (B / A) n , (3 / 5) (-2) = (5 / 3) 2 = 25 / 9.

    Π‘Ρ‚Π΅ΠΏΠ΅Π½ΡŒ с Π½Π°Ρ‚ΡƒΡ€Π°Π»ΡŒΠ½Ρ‹ΠΌ ΠΏΠΎΠΊΠ°Π·Π°Ρ‚Π΅Π»Π΅ΠΌ

    Под Π½Π΅ΠΉ ΠΏΠΎΠ½ΠΈΠΌΠ°ΡŽΡ‚ ΡΡ‚Π΅ΠΏΠ΅Π½ΡŒ с показатСлями, Ρ€Π°Π²Π½Ρ‹ΠΌΠΈ Ρ†Π΅Π»Ρ‹ΠΌ числам.

    Π§Ρ‚ΠΎ Π½ΡƒΠΆΠ½ΠΎ Π·Π°ΠΏΠΎΠΌΠ½ΠΈΡ‚ΡŒ:

    A 0 = 1, 1 0 = 1; 2 0 = 1; 3.15 0 = 1; (-4) 0 = 1…и Ρ‚. Π΄.

    A 1 = A, 1 1 = 1; 2 1 = 2; 3 1 = 3…и Ρ‚. Π΄.

    ΠšΡ€ΠΎΠΌΠ΅ Ρ‚ΠΎΠ³ΠΎ, Ссли (-a) 2 n +2 , n=0, 1, 2…то Ρ€Π΅Π·ΡƒΠ»ΡŒΡ‚Π°Ρ‚ Π±ΡƒΠ΄Π΅Ρ‚ со Π·Π½Π°ΠΊΠΎΠΌ Β«+Β». Если ΠΎΡ‚Ρ€ΠΈΡ†Π°Ρ‚Π΅Π»ΡŒΠ½ΠΎΠ΅ число возводится Π² Π½Π΅Ρ‡Π΅Ρ‚Π½ΡƒΡŽ ΡΡ‚Π΅ΠΏΠ΅Π½ΡŒ, Ρ‚ΠΎ Π½Π°ΠΎΠ±ΠΎΡ€ΠΎΡ‚.

    ΠžΠ±Ρ‰ΠΈΠ΅ свойства, Π΄Π° ΠΈ всС спСцифичСскиС ΠΏΡ€ΠΈΠ·Π½Π°ΠΊΠΈ, описанныС Π²Ρ‹ΡˆΠ΅, Ρ‚Π°ΠΊΠΆΠ΅ Ρ…Π°Ρ€Π°ΠΊΡ‚Π΅Ρ€Π½Ρ‹ для Π½ΠΈΡ….

    Дробная ΡΡ‚Π΅ΠΏΠ΅Π½ΡŒ

    Π­Ρ‚ΠΎΡ‚ Π²ΠΈΠ΄ ΠΌΠΎΠΆΠ½ΠΎ Π·Π°ΠΏΠΈΡΠ°Ρ‚ΡŒ схСмой: A m / n . ЧитаСтся ΠΊΠ°ΠΊ: ΠΊΠΎΡ€Π΅Π½ΡŒ n-ΠΎΠΉ стСпСни ΠΈΠ· числа A Π² стСпСни m.

    Π‘ Π΄Ρ€ΠΎΠ±Π½Ρ‹ΠΌ ΠΏΠΎΠΊΠ°Π·Π°Ρ‚Π΅Π»Π΅ΠΌ ΠΌΠΎΠΆΠ½ΠΎ Π΄Π΅Π»Π°Ρ‚ΡŒ, Ρ‡Ρ‚ΠΎ ΡƒΠ³ΠΎΠ΄Π½ΠΎ: ΡΠΎΠΊΡ€Π°Ρ‰Π°Ρ‚ΡŒ, Ρ€Π°ΡΠΊΠ»Π°Π΄Ρ‹Π²Π°Ρ‚ΡŒ Π½Π° части, Π²ΠΎΠ·Π²ΠΎΠ΄ΠΈΡ‚ΡŒ Π² Π΄Ρ€ΡƒΠ³ΡƒΡŽ ΡΡ‚Π΅ΠΏΠ΅Π½ΡŒ ΠΈ Ρ‚. Π΄.

    Π‘Ρ‚Π΅ΠΏΠ΅Π½ΡŒ с ΠΈΡ€Ρ€Π°Ρ†ΠΈΠΎΠ½Π°Π»ΡŒΠ½Ρ‹ΠΌ ΠΏΠΎΠΊΠ°Π·Π°Ρ‚Π΅Π»Π΅ΠΌ

    ΠŸΡƒΡΡ‚ΡŒ Ξ± – ΠΈΡ€Ρ€Π°Ρ†ΠΈΠΎΠ½Π°Π»ΡŒΠ½ΠΎΠ΅ число, Π° А Λƒ 0.

    Π§Ρ‚ΠΎΠ±Ρ‹ ΠΏΠΎΠ½ΡΡ‚ΡŒ ΡΡƒΡ‚ΡŒ стСпСни с Ρ‚Π°ΠΊΠΈΠΌ ΠΏΠΎΠΊΠ°Π·Π°Ρ‚Π΅Π»Π΅ΠΌ, рассмотрим Ρ€Π°Π·Π½Ρ‹Π΅ Π²ΠΎΠ·ΠΌΠΎΠΆΠ½Ρ‹Π΅ случаи:

    • А = 1. Π Π΅Π·ΡƒΠ»ΡŒΡ‚Π°Ρ‚ Π±ΡƒΠ΄Π΅Ρ‚ Ρ€Π°Π²Π΅Π½ 1. ΠŸΠΎΡΠΊΠΎΠ»ΡŒΠΊΡƒ сущСствуСт аксиома – 1 Π²ΠΎ всСх стСпСнях Ρ€Π°Π²Π½Π° Π΅Π΄ΠΈΠ½ΠΈΡ†Π΅;

    А r 1 Λ‚ А Ξ± Λ‚ А r 2 , r 1 Λ‚ r 2 – Ρ€Π°Ρ†ΠΈΠΎΠ½Π°Π»ΡŒΠ½Ρ‹Π΅ числа;

    • 0˂А˂1.

    Π’ этом случаС Π½Π°ΠΎΠ±ΠΎΡ€ΠΎΡ‚: А r 2 Λ‚ А Ξ± Λ‚ А r 1 ΠΏΡ€ΠΈ Ρ‚Π΅Ρ… ΠΆΠ΅ условиях, Ρ‡Ρ‚ΠΎ ΠΈ Π²ΠΎ Π²Ρ‚ΠΎΡ€ΠΎΠΌ ΠΏΡƒΠ½ΠΊΡ‚Π΅.

    НапримСр, ΠΏΠΎΠΊΠ°Π·Π°Ρ‚Π΅Π»ΡŒ стСпСни число Ο€. Оно Ρ€Π°Ρ†ΠΈΠΎΠ½Π°Π»ΡŒΠ½ΠΎΠ΅.

    r 1 – Π² этом случаС Ρ€Π°Π²Π½ΠΎ 3;

    r 2 – Π±ΡƒΠ΄Π΅Ρ‚ Ρ€Π°Π²Π½ΠΎ 4.

    Π’ΠΎΠ³Π΄Π°, ΠΏΡ€ΠΈ А = 1, 1 Ο€ = 1.

    А = 2, Ρ‚ΠΎ 2 3 Λ‚ 2 Ο€ Λ‚ 2 4 , 8 Λ‚ 2 Ο€ Λ‚ 16.

    А = 1/2, Ρ‚ΠΎ (Β½) 4 Λ‚ (Β½) Ο€ Λ‚ (Β½) 3 , 1/16 Λ‚ (Β½) Ο€ Λ‚ 1/8.

    Для Ρ‚Π°ΠΊΠΈΡ… стСпСнСй Ρ…Π°Ρ€Π°ΠΊΡ‚Π΅Ρ€Π½Ρ‹ всС матСматичСскиС ΠΎΠΏΠ΅Ρ€Π°Ρ†ΠΈΠΈ ΠΈ спСцифичСскиС свойства, описанныС Π²Ρ‹ΡˆΠ΅.

    Π—Π°ΠΊΠ»ΡŽΡ‡Π΅Π½ΠΈΠ΅

    ΠŸΠΎΠ΄Π²Π΅Π΄Ρ‘ΠΌ ΠΈΡ‚ΠΎΠ³ΠΈ β€” для Ρ‡Π΅Π³ΠΎ ΠΆΠ΅ Π½ΡƒΠΆΠ½Ρ‹ эти Π²Π΅Π»ΠΈΡ‡ΠΈΠ½Ρ‹, Π² Ρ‡Π΅ΠΌ прСимущСство Ρ‚Π°ΠΊΠΈΡ… Ρ„ΡƒΠ½ΠΊΡ†ΠΈΠΉ? ΠšΠΎΠ½Π΅Ρ‡Π½ΠΎ, Π² ΠΏΠ΅Ρ€Π²ΡƒΡŽ ΠΎΡ‡Π΅Ρ€Π΅Π΄ΡŒ ΠΎΠ½ΠΈ ΡƒΠΏΡ€ΠΎΡ‰Π°ΡŽΡ‚ Тизнь ΠΌΠ°Ρ‚Π΅ΠΌΠ°Ρ‚ΠΈΠΊΠΎΠ² ΠΈ программистов ΠΏΡ€ΠΈ Ρ€Π΅ΡˆΠ΅Π½ΠΈΠΈ ΠΏΡ€ΠΈΠΌΠ΅Ρ€ΠΎΠ², ΠΏΠΎΡΠΊΠΎΠ»ΡŒΠΊΡƒ ΠΏΠΎΠ·Π²ΠΎΠ»ΡΡŽΡ‚ ΠΌΠΈΠ½ΠΈΠΌΠΈΠ·ΠΈΡ€ΠΎΠ²Π°Ρ‚ΡŒ расчСты, ΡΠΎΠΊΡ€Π°Ρ‚ΠΈΡ‚ΡŒ Π°Π»Π³ΠΎΡ€ΠΈΡ‚ΠΌΡ‹, ΡΠΈΡΡ‚Π΅ΠΌΠ°Ρ‚ΠΈΠ·ΠΈΡ€ΠΎΠ²Π°Ρ‚ΡŒ Π΄Π°Π½Π½Ρ‹Π΅ ΠΈ ΠΌΠ½ΠΎΠ³ΠΎΠ΅ Π΄Ρ€ΡƒΠ³ΠΎΠ΅.

    Π“Π΄Π΅ Π΅Ρ‰Π΅ ΠΌΠΎΠ³ΡƒΡ‚ ΠΏΡ€ΠΈΠ³ΠΎΠ΄ΠΈΡ‚ΡŒΡΡ эти знания? Π’ любой Ρ€Π°Π±ΠΎΡ‡Π΅ΠΉ ΡΠΏΠ΅Ρ†ΠΈΠ°Π»ΡŒΠ½ΠΎΡΡ‚ΠΈ: ΠΌΠ΅Π΄ΠΈΡ†ΠΈΠ½Π΅, Ρ„Π°Ρ€ΠΌΠ°ΠΊΠΎΠ»ΠΎΠ³ΠΈΠΈ, стоматологии, ΡΡ‚Ρ€ΠΎΠΈΡ‚Π΅Π»ΡŒΡΡ‚Π²Π΅, Ρ‚Π΅Ρ…Π½ΠΈΠΊΠ΅, ΠΈΠ½ΠΆΠ΅Π½Π΅Ρ€ΠΈΠΈ, конструировании ΠΈ Ρ‚. Π΄.

    Π‘Ρ‚Π°Ρ‚ΡŒΠΈ ΠΏΠΎ СстСствСнным Π½Π°ΡƒΠΊΠ°ΠΌ ΠΈ ΠΌΠ°Ρ‚Π΅ΠΌΠ°Ρ‚ΠΈΠΊΠ΅

    Бвойства стСпСнСй с ΠΎΠ΄ΠΈΠ½Π°ΠΊΠΎΠ²Ρ‹ΠΌΠΈ основаниями

    БущСствуСт Ρ‚Ρ€ΠΈ свойства стСпСнСй с ΠΎΠ΄ΠΈΠ½Π°ΠΊΠΎΠ²Ρ‹ΠΌΠΈ основаниями ΠΈ Π½Π°Ρ‚ΡƒΡ€Π°Π»ΡŒΠ½Ρ‹ΠΌΠΈ показатСлями. Π­Ρ‚ΠΎ

    • ΠŸΡ€ΠΎΠΈΠ·Π²Π΅Π΄Π΅Π½ΠΈΠ΅ сумма
    • ЧастноС Π΄Π²ΡƒΡ… стСпСнСй с ΠΎΠ΄ΠΈΠ½Π°ΠΊΠΎΠ²Ρ‹ΠΌΠΈ основаниями Ρ€Π°Π²Π½ΠΎ Π²Ρ‹Ρ€Π°ΠΆΠ΅Π½ΠΈΡŽ, Π³Π΄Π΅ основаниС Ρ‚ΠΎ ΠΆΠ΅ самоС, Π° ΠΏΠΎΠΊΠ°Π·Π°Ρ‚Π΅Π»ΡŒ Π΅ΡΡ‚ΡŒ Ρ€Π°Π·Π½ΠΎΡΡ‚ΡŒ ΠΏΠΎΠΊΠ°Π·Π°Ρ‚Π΅Π»Π΅ΠΉ исходных ΠΌΠ½ΠΎΠΆΠΈΡ‚Π΅Π»Π΅ΠΉ.
    • Π’ΠΎΠ·Π²Π΅Π΄Π΅Π½ΠΈΠ΅ стСпСни числа Π² ΡΡ‚Π΅ΠΏΠ΅Π½ΡŒ Ρ€Π°Π²Π½ΠΎ Π²Ρ‹Ρ€Π°ΠΆΠ΅Π½ΠΈΡŽ, Π² ΠΊΠΎΡ‚ΠΎΡ€ΠΎΠΌ основаниС — это Ρ‚ΠΎ ΠΆΠ΅ самоС число, Π° ΠΏΠΎΠΊΠ°Π·Π°Ρ‚Π΅Π»ΡŒ — это ΠΏΡ€ΠΎΠΈΠ·Π²Π΅Π΄Π΅Π½ΠΈΠ΅ Π΄Π²ΡƒΡ… стСпСнСй.

    Π‘ΡƒΠ΄ΡŒΡ‚Π΅ Π²Π½ΠΈΠΌΠ°Ρ‚Π΅Π»ΡŒΠ½Ρ‹! ΠŸΡ€Π°Π²ΠΈΠ» ΠΎΡ‚Π½ΠΎΡΠΈΡ‚Π΅Π»ΡŒΠ½ΠΎ слоТСния ΠΈ вычитания стСпСнСй с ΠΎΠ΄ΠΈΠ½Π°ΠΊΠΎΠ²Ρ‹ΠΌΠΈ основаниями Π½Π΅ сущСствуСт .

    Π—Π°ΠΏΠΈΡˆΠ΅ΠΌ эти свойства-ΠΏΡ€Π°Π²ΠΈΠ»Π° Π² Π²ΠΈΠ΄Π΅ Ρ„ΠΎΡ€ΠΌΡƒΠ»:

    • a m ? a n = a m+n
    • a m ? a n = a m–n
    • (a m) n = a mn

    Π’Π΅ΠΏΠ΅Ρ€ΡŒ рассмотрим ΠΈΡ… Π½Π° ΠΊΠΎΠ½ΠΊΡ€Π΅Ρ‚Π½Ρ‹Ρ… ΠΏΡ€ΠΈΠΌΠ΅Ρ€Π°Ρ… ΠΈ ΠΏΠΎΠΏΡ€ΠΎΠ±ΡƒΠ΅ΠΌ Π΄ΠΎΠΊΠ°Π·Π°Ρ‚ΡŒ.

    5 2 ? 5 3 = 5 5 — здСсь ΠΌΡ‹ ΠΏΡ€ΠΈΠΌΠ΅Π½ΠΈΠ»ΠΈ ΠΏΡ€Π°Π²ΠΈΠ»ΠΎ; Π° Ρ‚Π΅ΠΏΠ΅Ρ€ΡŒ прСдставим ΠΊΠ°ΠΊ Π±Ρ‹ ΠΌΡ‹ Ρ€Π΅ΡˆΠ°Π»ΠΈ этот ΠΏΡ€ΠΈΠΌΠ΅Ρ€, Ссли Π±Ρ‹ Π½Π΅ Π·Π½Π°Π»ΠΈ ΠΏΡ€Π°Π²ΠΈΠ»Π°:

    5 2 ? 5 3 = 5 ? 5 ? 5 ? 5 ? 5 = 5 5 — ΠΏΡΡ‚ΡŒ Π² ΠΊΠ²Π°Π΄Ρ€Π°Ρ‚Π΅ — это ΠΏΡΡ‚ΡŒ ΡƒΠΌΠ½ΠΎΠΆΠ΅Π½Π½ΠΎΠ΅ Π½Π° ΠΏΡΡ‚ΡŒ, Π° Π² ΠΊΡƒΠ±Π΅ — ΠΏΡ€ΠΎΠΈΠ·Π²Π΅Π΄Π΅Π½ΠΈΠ΅ Ρ‚Ρ€Π΅Ρ… пятСрок. Π’ Ρ€Π΅Π·ΡƒΠ»ΡŒΡ‚Π°Ρ‚Π΅ ΠΏΠΎΠ»ΡƒΡ‡ΠΈΠ»ΠΎΡΡŒ ΠΏΡ€ΠΎΠΈΠ·Π²Π΅Π΄Π΅Π½ΠΈΠ΅ пяти пятСрок, Π½ΠΎ это Π½Π΅Ρ‡Ρ‚ΠΎ ΠΈΠ½ΠΎΠ΅ ΠΊΠ°ΠΊ ΠΏΡΡ‚ΡŒ Π² пятой стСпСни: 5 5 .

    3 9 ? 3 5 = 3 9–5 = 3 4 . Π—Π°ΠΏΠΈΡˆΠ΅ΠΌ Π΄Π΅Π»Π΅Π½ΠΈΠ΅ Π² Π²ΠΈΠ΄Π΅ Π΄Ρ€ΠΎΠ±ΠΈ:

    Π•Π΅ ΠΌΠΎΠΆΠ½ΠΎ ΡΠΎΠΊΡ€Π°Ρ‚ΠΈΡ‚ΡŒ:

    Π’ Ρ€Π΅Π·ΡƒΠ»ΡŒΡ‚Π°Ρ‚Π΅ ΠΏΠΎΠ»ΡƒΡ‡ΠΈΠΌ:

    Π’Π°ΠΊΠΈΠΌ ΠΎΠ±Ρ€Π°Π·ΠΎΠΌ ΠΌΡ‹ Π΄ΠΎΠΊΠ°Π·Π°Π»ΠΈ, Ρ‡Ρ‚ΠΎ ΠΏΡ€ΠΈ Π΄Π΅Π»Π΅Π½ΠΈΠΈ Π΄Π²ΡƒΡ… стСпСнСй с ΠΎΠ΄ΠΈΠ½Π°ΠΊΠΎΠ²Ρ‹ΠΌΠΈ основаниями, ΠΈΡ… ΠΏΠΎΠΊΠ°Π·Π°Ρ‚Π΅Π»ΠΈ Π½Π°Π΄ΠΎ Π²Ρ‹Ρ‡ΠΈΡ‚Π°Ρ‚ΡŒ.

    Однако ΠΏΡ€ΠΈ Π΄Π΅Π»Π΅Π½ΠΈΠΈ нСльзя, Ρ‡Ρ‚ΠΎΠ±Ρ‹ Π΄Π΅Π»ΠΈΡ‚Π΅Π»ΡŒ Π±Ρ‹Π» Ρ€Π°Π²Π΅Π½ Π½ΡƒΠ»ΡŽ (Ρ‚Π°ΠΊ ΠΊΠ°ΠΊ Π½Π° ноль Π΄Π΅Π»ΠΈΡ‚ΡŒ нСльзя). ΠšΡ€ΠΎΠΌΠ΅ Ρ‚ΠΎΠ³ΠΎ, ΠΏΠΎΡΠΊΠΎΠ»ΡŒΠΊΡƒ ΠΌΡ‹ рассматриваСм стСпСни Ρ‚ΠΎΠ»ΡŒΠΊΠΎ с Π½Π°Ρ‚ΡƒΡ€Π°Π»ΡŒΠ½Ρ‹ΠΌΠΈ показатСлями, Ρ‚ΠΎ Π½Π΅ ΠΌΠΎΠΆΠ΅ΠΌ Π² Ρ€Π΅Π·ΡƒΠ»ΡŒΡ‚Π°Ρ‚Π΅ вычитания ΠΏΠΎΠΊΠ°Π·Π°Ρ‚Π΅Π»Π΅ΠΉ ΠΏΠΎΠ»ΡƒΡ‡ΠΈΡ‚ΡŒ число мСньшС, Ρ‡Π΅ΠΌ 1. ΠŸΠΎΡΡ‚ΠΎΠΌΡƒ Π½Π° Ρ„ΠΎΡ€ΠΌΡƒΠ»Ρƒ a m ? a n = a m–n Π½Π°ΠΊΠ»Π°Π΄Ρ‹Π²Π°ΡŽΡ‚ΡΡ ограничСния: a ? 0 ΠΈ m > n.

    ΠŸΠ΅Ρ€Π΅ΠΉΠ΄Π΅ΠΌ ΠΊ Ρ‚Ρ€Π΅Ρ‚ΡŒΠ΅ΠΌΡƒ свойству:
    (2 2) 4 = 2 2?4 = 2 8

    Π—Π°ΠΏΠΈΡˆΠ΅ΠΌ Π² Ρ€Π°Π·Π²Π΅Ρ€Π½ΡƒΡ‚ΠΎΠΌ Π²ΠΈΠ΄Π΅:
    (2 2) 4 = (2 ? 2) 4 = (2 ? 2) ? (2 ? 2) ? (2 ? 2) ? (2 ? 2) = 2 ? 2 ? 2 ? 2 ? 2 ? 2 ? 2 ? 2 = 2 8

    МоТно ΠΏΡ€ΠΈΠΉΡ‚ΠΈ ΠΊ Ρ‚Π°ΠΊΠΎΠΌΡƒ Π²Ρ‹Π²ΠΎΠ΄Ρƒ ΠΈ логичСски рассуТдая. НуТно ΠΏΠ΅Ρ€Π΅ΠΌΠ½ΠΎΠΆΠΈΡ‚ΡŒ Π΄Π²Π° Π² ΠΊΠ²Π°Π΄Ρ€Π°Ρ‚Π΅ Ρ‡Π΅Ρ‚Ρ‹Ρ€Π΅ Ρ€Π°Π·Π°. Но Π² ΠΊΠ°ΠΆΠ΄ΠΎΠΌ ΠΊΠ²Π°Π΄Ρ€Π°Ρ‚Π΅ Π΄Π²Π΅ Π΄Π²ΠΎΠΉΠΊΠΈ, Π·Π½Π°Ρ‡ΠΈΡ‚ всСго Π΄Π²ΠΎΠ΅ΠΊ Π±ΡƒΠ΄Π΅Ρ‚ восСмь.

    scienceland.info

    ΠŸΡ€Π°Π²ΠΈΠ»Π° слоТСния ΠΈ вычитания.

    1. ΠžΡ‚ ΠΏΠ΅Ρ€Π΅ΠΌΠ΅Π½Ρ‹ мСст слагаСмых сумма Π½Π΅ измСнится (ΠΊΠΎΠΌΠΌΡƒΡ‚Π°Ρ‚ΠΈΠ²Π½ΠΎΠ΅ свойство слоТСния)

    13+25=38, ΠΌΠΎΠΆΠ½ΠΎ Π·Π°ΠΏΠΈΡΠ°Ρ‚ΡŒ ΠΊΠ°ΠΊ: 25+13=38

    2. Π Π΅Π·ΡƒΠ»ΡŒΡ‚Π°Ρ‚ слоТСния Π½Π΅ измСнится, Ссли сосСдниС слагаСмыС Π·Π°ΠΌΠ΅Π½ΠΈΡ‚ΡŒ ΠΈΡ… суммой (ассоциативноС свойство слоТСния).

    10+13+3+5=31 ΠΌΠΎΠΆΠ½ΠΎ Π·Π°ΠΏΠΈΡΠ°Ρ‚ΡŒ ΠΊΠ°ΠΊ: 23+3+5=31; 26+5=31; 23+8=31 ΠΈ Ρ‚.Π΄.

    3. Π•Π΄ΠΈΠ½ΠΈΡ†Ρ‹ ΡΠΊΠ»Π°Π΄Ρ‹Π²Π°ΡŽΡ‚ΡΡ с Π΅Π΄ΠΈΠ½ΠΈΡ†Π°ΠΌΠΈ, дСсятки с дСсятками ΠΈ Ρ‚.Π΄.

    34+11=45 (3 дСсяка плюс Π΅Ρ‰Π΅ 1 дСсяток; 4 Π΅Π΄ΠΈΠ½ΠΈΡ†Ρ‹ плюс 1 Π΅Π΄ΠΈΠ½ΠΈΡ†Π°).

    4. Π•Π΄ΠΈΠ½ΠΈΡ†Ρ‹ Π²Ρ‹Ρ‡ΠΈΡ‚Π°ΡŽΡ‚ΡΡ ΠΈΠ· Π΅Π΄ΠΈΠ½ΠΈΡ†, дСсятки ΠΈΠ· дСсятков ΠΈ Ρ‚.Π΄.

    53-12=41 (3 Π΅Π΄ΠΈΠ½ΠΈΡ†Ρ‹ минус 2 Π΅Π΄ΠΈΠ½ΠΈΡ†Ρ‹; 5 дСсятков минус 1 дСсяток)

    ΠΏΡ€ΠΈΠΌΠ΅Ρ‡Π°Π½ΠΈΠ΅: 10 Π΅Π΄ΠΈΠ½ΠΈΡ† ΡΠΎΡΡ‚Π°Π²Π»ΡΡŽΡ‚ ΠΎΠ΄ΠΈΠ½ дСсяток. Π­Ρ‚ΠΎ Π½Π°Π΄ΠΎ ΠΏΠΎΠΌΠ½ΠΈΡ‚ΡŒ ΠΏΡ€ΠΈ Π²Ρ‹Ρ‡ΠΈΡ‚Π°Π½ΠΈΠΈ, Ρ‚.ΠΊ. Ссли количСство Π΅Π΄ΠΈΠ½ΠΈΡ† Ρƒ Π²Ρ‹Ρ‡ΠΈΡ‚Π°Π΅ΠΌΠΎΠ³ΠΎ большС, Ρ‡Π΅ΠΌ Ρƒ ΡƒΠΌΠ΅Π½ΡŒΡˆΠ°Π΅ΠΌΠΎΠ³ΠΎ, Ρ‚ΠΎ ΠΌΡ‹ ΠΌΠΎΠΆΠ΅ΠΌ Β«Π·Π°Π½ΡΡ‚ΡŒΒ» ΠΎΠ΄ΠΈΠ½ дСсяток Ρƒ ΡƒΠΌΠ΅Π½ΡŒΡˆΠ°Π΅ΠΌΠΎΠ³ΠΎ.

    41-12=29 (Для Ρ‚ΠΎΠ³ΠΎ Ρ‡Ρ‚ΠΎΠ±Ρ‹ ΠΈ 1 Π²Ρ‹Ρ‡Π΅ΡΡ‚ΡŒ 2, ΠΌΡ‹ сначала Π΄ΠΎΠ»ΠΆΠ½Ρ‹ Β«Π·Π°Π½ΡΡ‚ΡŒΒ» Π΅Π΄ΠΈΠ½ΠΈΡ†Ρƒ Ρƒ дСсятков, ΠΏΠΎΠ»ΡƒΡ‡Π°Π΅ΠΌ 11-2=9; ΠΏΠΎΠΌΠ½ΠΈΠΌ, Ρ‡Ρ‚ΠΎ Ρƒ ΡƒΠΌΠ΅Π½ΡŒΡˆΠ°Π΅ΠΌΠΎΠ³ΠΎ остаСтся Π½Π° 1 дСсяток мСньшС, ΡΠ»Π΅Π΄ΠΎΠ²Π°Ρ‚Π΅Π»ΡŒΠ½ΠΎ, остаСтся 3 дСсятка ΠΈ ΠΎΡ‚ Π½Π΅Π³ΠΎ отнимаСтся 1 дСсяток. ΠžΡ‚Π²Π΅Ρ‚ 29).

    5. Если ΠΈΠ· суммы Π΄Π²ΡƒΡ… слагаСмых Π²Ρ‹Ρ‡Π΅ΡΡ‚ΡŒ ΠΎΠ΄Π½ΠΎ ΠΈΠ· Π½ΠΈΡ…, Ρ‚ΠΎ получится Π²Ρ‚ΠΎΡ€ΠΎΠ΅ слагаСмоС.

    Π­Ρ‚ΠΎ Π·Π½Π°Ρ‡ΠΈΡ‚, Ρ‡Ρ‚ΠΎ слоТСниС ΠΌΠΎΠΆΠ½ΠΎ ΠΏΡ€ΠΎΠ²Π΅Ρ€ΠΈΡ‚ΡŒ с ΠΏΠΎΠΌΠΎΡ‰ΡŒΡŽ вычитания.

    Для ΠΏΡ€ΠΎΠ²Π΅Ρ€ΠΊΠΈ ΠΈΠ· суммы Π²Ρ‹Ρ‡ΠΈΡ‚Π°ΡŽΡ‚ ΠΎΠ΄Π½ΠΎ ΠΈΠ· слагаСмых: 49-7=42 ΠΈΠ»ΠΈ 49-42=7

    Если Π² Ρ€Π΅Π·ΡƒΠ»ΡŒΡ‚Π°Ρ‚Π΅ вычитания Π²Ρ‹ Π½Π΅ ΠΏΠΎΠ»ΡƒΡ‡ΠΈΠ»ΠΈ ΠΎΠ΄Π½ΠΎ ΠΈΠ· слагаСмых, Π·Π½Π°Ρ‡ΠΈΡ‚ Π² вашСм слоТСнии Π±Ρ‹Π»Π° Π΄ΠΎΠΏΡƒΡ‰Π΅Π½Π° ошибка.

    6. Если ΠΊ разности ΠΏΡ€ΠΈΠ±Π°Π²ΠΈΡ‚ΡŒ Π²Ρ‹Ρ‡ΠΈΡ‚Π°Π΅ΠΌΠΎΠ΅, Ρ‚ΠΎ получится ΡƒΠΌΠ΅Π½ΡŒΡˆΠ°Π΅ΠΌΠΎΠ΅.

    Π­Ρ‚ΠΎ Π·Π½Π°Ρ‡ΠΈΡ‚, Ρ‡Ρ‚ΠΎ Π²Ρ‹Ρ‡ΠΈΡ‚Π°Π½ΠΈΠ΅ ΠΌΠΎΠΆΠ½ΠΎ ΠΏΡ€ΠΎΠ²Π΅Ρ€ΠΈΡ‚ΡŒ слоТСниСм.

    Для ΠΏΡ€ΠΎΠ²Π΅Ρ€ΠΊΠΈ ΠΊ разности ΠΏΡ€ΠΈΠ±Π°Π²ΠΈΠΌ Π²Ρ‹Ρ‡ΠΈΡ‚Π°Π΅ΠΌΠΎΠ΅: 19+50=69.

    Если Π² Ρ€Π΅Π·ΡƒΠ»ΡŒΡ‚Π°Ρ‚Π΅ описанной Π²Ρ‹ΡˆΠ΅ ΠΏΡ€ΠΎΡ†Π΅Π΄ΡƒΡ€Ρ‹ Π²Ρ‹ Π½Π΅ ΠΏΠΎΠ»ΡƒΡ‡ΠΈΠ»ΠΈ ΡƒΠΌΠ΅Π½ΡˆΡŒΡˆΠ°Π΅ΠΌΠΎΠ΅, Π·Π½Π°Ρ‡ΠΈΡ‚ Π² вашСм Π²Ρ‹Ρ‡ΠΈΡ‚Π°Π½ΠΈΠΈ Π±Ρ‹Π»Π° Π΄ΠΎΠΏΡƒΡ‰Π΅Π½Π° ошибка.

    Π‘Π»ΠΎΠΆΠ΅Π½ΠΈΠ΅ ΠΈ Π²Ρ‹Ρ‡ΠΈΡ‚Π°Π½ΠΈΠ΅ Ρ€Π°Ρ†ΠΈΠΎΠ½Π°Π»ΡŒΠ½Ρ‹Ρ… чисСл

    Π’ Π΄Π°Π½Π½ΠΎΠΌ ΡƒΡ€ΠΎΠΊΠ΅ рассматриваСтся слоТСниС ΠΈ Π²Ρ‹Ρ‡ΠΈΡ‚Π°Π½ΠΈΠ΅ Ρ€Π°Ρ†ΠΈΠΎΠ½Π°Π»ΡŒΠ½Ρ‹Ρ… чисСл. Π’Π΅ΠΌΠ° относится ΠΊ ΠΊΠ°Ρ‚Π΅Π³ΠΎΡ€ΠΈΠΈ слоТных. Π—Π΄Π΅ΡΡŒ Π½Π΅ΠΎΠ±Ρ…ΠΎΠ΄ΠΈΠΌΠΎ ΠΈΡΠΏΠΎΠ»ΡŒΠ·ΠΎΠ²Π°Ρ‚ΡŒ вСсь арсСнал ΠΏΠΎΠ»ΡƒΡ‡Π΅Π½Π½Ρ‹Ρ… Ρ€Π°Π½Π΅Π΅ Π·Π½Π°Π½ΠΈΠΉ.

    ΠŸΡ€Π°Π²ΠΈΠ»Π° слоТСния ΠΈ вычитания Ρ†Π΅Π»Ρ‹Ρ… чисСл справСдливы ΠΈ для Ρ€Π°Ρ†ΠΈΠΎΠ½Π°Π»ΡŒΠ½Ρ‹Ρ… чисСл. Напомним, Ρ‡Ρ‚ΠΎ Ρ€Π°Ρ†ΠΈΠΎΠ½Π°Π»ΡŒΠ½Ρ‹ΠΌΠΈ Π½Π°Π·Ρ‹Π²Π°ΡŽΡ‚ числа, ΠΊΠΎΡ‚ΠΎΡ€Ρ‹Π΅ ΠΌΠΎΠ³ΡƒΡ‚ Π±Ρ‹Ρ‚ΡŒ прСдставлСны Π² Π²ΠΈΠ΄Π΅ Π΄Ρ€ΠΎΠ±ΠΈ, Π³Π΄Π΅ a – это Ρ‡ΠΈΡΠ»ΠΈΡ‚Π΅Π»ΡŒ Π΄Ρ€ΠΎΠ±ΠΈ, b – Π·Π½Π°ΠΌΠ΅Π½Π°Ρ‚Π΅Π»ΡŒ Π΄Ρ€ΠΎΠ±ΠΈ. ΠŸΡ€ΠΈΡ‡Π΅ΠΌ b Π½Π΅ Π΄ΠΎΠ»ΠΆΠ½ΠΎ Π±Ρ‹Ρ‚ΡŒ Π½ΡƒΠ»Ρ‘ΠΌ.

    Π’ Π΄Π°Π½Π½ΠΎΠΌ ΡƒΡ€ΠΎΠΊΠ΅ Π΄Ρ€ΠΎΠ±ΠΈ ΠΈ ΡΠΌΠ΅ΡˆΠ°Π½Π½Ρ‹Π΅ числа ΠΌΡ‹ всё Ρ‡Π°Ρ‰Π΅ Π±ΡƒΠ΄Π΅ΠΌ Π½Π°Π·Ρ‹Π²Π°Ρ‚ΡŒ ΠΎΠ΄Π½ΠΈΠΌ ΠΎΠ±Ρ‰ΠΈΠΌ словосочСтаниСм — Ρ€Π°Ρ†ΠΈΠΎΠ½Π°Π»ΡŒΠ½Ρ‹Π΅ числа .

    Навигация ΠΏΠΎ ΡƒΡ€ΠΎΠΊΡƒ:

    ΠŸΡ€ΠΈΠΌΠ΅Ρ€ 1. Найти Π·Π½Π°Ρ‡Π΅Π½ΠΈΠ΅ выраТСния

    Π—Π°ΠΊΠ»ΡŽΡ‡ΠΈΠΌ ΠΊΠ°ΠΆΠ΄ΠΎΠ΅ Ρ€Π°Ρ†ΠΈΠΎΠ½Π°Π»ΡŒΠ½ΠΎΠ΅ число Π² скобки вмСстС со своими Π·Π½Π°ΠΊΠ°ΠΌΠΈ. Π£Ρ‡ΠΈΡ‚Ρ‹Π²Π°Π΅ΠΌ, Ρ‡Ρ‚ΠΎ плюс ΠΊΠΎΡ‚ΠΎΡ€Ρ‹ΠΉ Π΄Π°Π½ Π² Π²Ρ‹Ρ€Π°ΠΆΠ΅Π½ΠΈΠΈ являСтся Π·Π½Π°ΠΊΠΎΠΌ ΠΎΠΏΠ΅Ρ€Π°Ρ†ΠΈΠΈ ΠΈ Π½Π΅ относится ΠΊ Π΄Ρ€ΠΎΠ±ΠΈ. Π£ этой Π΄Ρ€ΠΎΠ±ΠΈ свой Π·Π½Π°ΠΊ плюса, ΠΊΠΎΡ‚ΠΎΡ€Ρ‹ΠΉ Π½Π΅Π²ΠΈΠ΄ΠΈΠΌ ΠΏΠΎ ΠΏΡ€ΠΈΡ‡ΠΈΠ½Π΅ Ρ‚ΠΎΠ³ΠΎ, Ρ‡Ρ‚ΠΎ Π΅Π³ΠΎ Π½Π΅ Π·Π°ΠΏΠΈΡΡ‹Π²Π°ΡŽΡ‚. Но ΠΌΡ‹ запишСм Π΅Π³ΠΎ для наглядности:

    Π­Ρ‚ΠΎ слоТСниС Ρ€Π°Ρ†ΠΈΠΎΠ½Π°Π»ΡŒΠ½Ρ‹Ρ… чисСл с Ρ€Π°Π·Π½Ρ‹ΠΌΠΈ Π·Π½Π°ΠΊΠ°ΠΌΠΈ. Π§Ρ‚ΠΎΠ±Ρ‹ ΡΠ»ΠΎΠΆΠΈΡ‚ΡŒ Ρ€Π°Ρ†ΠΈΠΎΠ½Π°Π»ΡŒΠ½Ρ‹Π΅ числа с Ρ€Π°Π·Π½Ρ‹ΠΌΠΈ Π·Π½Π°ΠΊΠ°ΠΌΠΈ, Π½ΡƒΠΆΠ½ΠΎ ΠΈΠ· большСго модуля Π²Ρ‹Ρ‡Π΅ΡΡ‚ΡŒ мСньший, ΠΈ ΠΏΠ΅Ρ€Π΅Π΄ ΠΏΠΎΠ»ΡƒΡ‡Π΅Π½Π½Ρ‹ΠΌ ΠΎΡ‚Π²Π΅Ρ‚ΠΎΠΌ ΠΏΠΎΡΡ‚Π°Π²ΠΈΡ‚ΡŒ Ρ‚ΠΎΡ‚ Π·Π½Π°ΠΊ, ΠΌΠΎΠ΄ΡƒΠ»ΡŒ ΠΊΠΎΡ‚ΠΎΡ€ΠΎΠ³ΠΎ большС. А Ρ‡Ρ‚ΠΎΠ±Ρ‹ ΠΏΠΎΠ½ΡΡ‚ΡŒ ΠΊΠ°ΠΊΠΎΠΉ ΠΌΠΎΠ΄ΡƒΠ»ΡŒ большС, Π° ΠΊΠ°ΠΊΠΎΠΉ мСньшС, Π½ΡƒΠΆΠ½ΠΎ ΡΡƒΠΌΠ΅Ρ‚ΡŒ ΡΡ€Π°Π²Π½ΠΈΡ‚ΡŒ ΠΌΠΎΠ΄ΡƒΠ»ΠΈ этих Π΄Ρ€ΠΎΠ±Π΅ΠΉ Π΄ΠΎ ΠΈΡ… вычислСния:

    ΠœΠΎΠ΄ΡƒΠ»ΡŒ Ρ€Π°Ρ†ΠΈΠΎΠ½Π°Π»ΡŒΠ½ΠΎΠ³ΠΎ числа большС, Ρ‡Π΅ΠΌ ΠΌΠΎΠ΄ΡƒΠ»ΡŒ Ρ€Π°Ρ†ΠΈΠΎΠ½Π°Π»ΡŒΠ½ΠΎΠ³ΠΎ числа. ΠŸΠΎΡΡ‚ΠΎΠΌΡƒ ΠΌΡ‹ ΠΈΠ· Π²Ρ‹Ρ‡Π»ΠΈ . ΠŸΠΎΠ»ΡƒΡ‡ΠΈΠ»ΠΈ ΠΎΡ‚Π²Π΅Ρ‚. Π—Π°Ρ‚Π΅ΠΌ сократив эту Π΄Ρ€ΠΎΠ±ΡŒ Π½Π° 2, ΠΏΠΎΠ»ΡƒΡ‡ΠΈΠ»ΠΈ ΠΎΠΊΠΎΠ½Ρ‡Π°Ρ‚Π΅Π»ΡŒΠ½Ρ‹ΠΉ ΠΎΡ‚Π²Π΅Ρ‚ .

    ΠŸΡ€ΠΈ ΠΆΠ΅Π»Π°Π½ΠΈΠΈ Π½Π΅ΠΊΠΎΡ‚ΠΎΡ€Ρ‹Π΅ ΠΏΡ€ΠΈΠΌΠΈΡ‚ΠΈΠ²Π½Ρ‹Π΅ дСйствия, Ρ‚Π°ΠΊΠΈΠ΅ ΠΊΠ°ΠΊ Π·Π°ΠΊΠ»ΡŽΡ‡Π΅Π½ΠΈΠ΅ чисСл Π² скобки ΠΈ проставлСниС ΠΌΠΎΠ΄ΡƒΠ»Π΅ΠΉ, ΠΌΠΎΠΆΠ½ΠΎ ΠΏΡ€ΠΎΠΏΡƒΡΡ‚ΠΈΡ‚ΡŒ. Π”Π°Π½Π½Ρ‹ΠΉ ΠΏΡ€ΠΈΠΌΠ΅Ρ€ Π²ΠΏΠΎΠ»Π½Π΅ ΠΌΠΎΠΆΠ½ΠΎ Π·Π°ΠΏΠΈΡΠ°Ρ‚ΡŒ ΠΏΠΎΠΊΠΎΡ€ΠΎΡ‡Π΅:

    ΠŸΡ€ΠΈΠΌΠ΅Ρ€ 2. Найти Π·Π½Π°Ρ‡Π΅Π½ΠΈΠ΅ выраТСния

    Π—Π°ΠΊΠ»ΡŽΡ‡ΠΈΠΌ ΠΊΠ°ΠΆΠ΄ΠΎΠ΅ Ρ€Π°Ρ†ΠΈΠΎΠ½Π°Π»ΡŒΠ½ΠΎΠ΅ число Π² скобки вмСстС со своими Π·Π½Π°ΠΊΠ°ΠΌΠΈ. Π£Ρ‡ΠΈΡ‚Ρ‹Π²Π°Π΅ΠΌ, Ρ‡Ρ‚ΠΎ минус ΠΊΠΎΡ‚ΠΎΡ€Ρ‹ΠΉ Π΄Π°Π½ Π² Π²Ρ‹Ρ€Π°ΠΆΠ΅Π½ΠΈΠΈ являСтся Π·Π½Π°ΠΊΠΎΠΌ ΠΎΠΏΠ΅Ρ€Π°Ρ†ΠΈΠΈ ΠΈ Π½Π΅ относится ΠΊ Π΄Ρ€ΠΎΠ±ΠΈ.

    Π”Ρ€ΠΎΠ±ΡŒ Π² Π΄Π°Π½Π½ΠΎΠΌ случаС являСтся ΠΏΠΎΠ»ΠΎΠΆΠΈΡ‚Π΅Π»ΡŒΠ½Ρ‹ΠΌ Ρ€Π°Ρ†ΠΈΠΎΠ½Π°Π»ΡŒΠ½Ρ‹ΠΌ числом, ΠΈΠΌΠ΅ΡŽΡ‰ΠΈΠΌ Π·Π½Π°ΠΊ плюса, ΠΊΠΎΡ‚ΠΎΡ€Ρ‹ΠΉ Π½Π΅Π²ΠΈΠ΄ΠΈΠΌ. Но ΠΌΡ‹ запишСм Π΅Π³ΠΎ для наглядности:

    Π—Π°ΠΌΠ΅Π½ΠΈΠΌ Π²Ρ‹Ρ‡ΠΈΡ‚Π°Π½ΠΈΠ΅ слоТСниСм. Напомним, Ρ‡Ρ‚ΠΎ для этого Π½ΡƒΠΆΠ½ΠΎ ΠΊ ΡƒΠΌΠ΅Π½ΡŒΡˆΠ°Π΅ΠΌΠΎΠΌΡƒ ΠΏΡ€ΠΈΠ±Π°Π²ΠΈΡ‚ΡŒ число ΠΏΡ€ΠΎΡ‚ΠΈΠ²ΠΎΠΏΠΎΠ»ΠΎΠΆΠ½ΠΎΠ΅ Π²Ρ‹Ρ‡ΠΈΡ‚Π°Π΅ΠΌΠΎΠΌΡƒ:

    ΠŸΠΎΠ»ΡƒΡ‡ΠΈΠ»ΠΈ слоТСниС ΠΎΡ‚Ρ€ΠΈΡ†Π°Ρ‚Π΅Π»ΡŒΠ½Ρ‹Ρ… Ρ€Π°Ρ†ΠΈΠΎΠ½Π°Π»ΡŒΠ½Ρ‹Ρ… чисСл. Π§Ρ‚ΠΎΠ±Ρ‹ ΡΠ»ΠΎΠΆΠΈΡ‚ΡŒ ΠΎΡ‚Ρ€ΠΈΡ†Π°Ρ‚Π΅Π»ΡŒΠ½Ρ‹Π΅ Ρ€Π°Ρ†ΠΈΠΎΠ½Π°Π»ΡŒΠ½Ρ‹Π΅ числа, Π½ΡƒΠΆΠ½ΠΎ ΡΠ»ΠΎΠΆΠΈΡ‚ΡŒ ΠΈΡ… ΠΌΠΎΠ΄ΡƒΠ»ΠΈ ΠΈ ΠΏΠ΅Ρ€Π΅Π΄ ΠΏΠΎΠ»ΡƒΡ‡Π΅Π½Π½Ρ‹ΠΌ ΠΎΡ‚Π²Π΅Ρ‚ΠΎΠΌ ΠΏΠΎΡΡ‚Π°Π²ΠΈΡ‚ΡŒ минус:

    ΠŸΡ€ΠΈΠΌΠ΅Ρ€ 3. Найти Π·Π½Π°Ρ‡Π΅Π½ΠΈΠ΅ выраТСния

    Π’ этом Π²Ρ‹Ρ€Π°ΠΆΠ΅Π½ΠΈΠΈ Ρƒ Π΄Ρ€ΠΎΠ±Π΅ΠΉ Ρ€Π°Π·Π½Ρ‹Π΅ Π·Π½Π°ΠΌΠ΅Π½Π°Ρ‚Π΅Π»ΠΈ. Π§Ρ‚ΠΎΠ±Ρ‹ ΠΎΠ±Π»Π΅Π³Ρ‡ΠΈΡ‚ΡŒ сСбС Π·Π°Π΄Π°Ρ‡Ρƒ, ΠΏΡ€ΠΈΠ²Π΅Π΄Ρ‘ΠΌ эти Π΄Ρ€ΠΎΠ±ΠΈ ΠΊ ΠΎΠ΄ΠΈΠ½Π°ΠΊΠΎΠ²ΠΎΠΌΡƒ (ΠΎΠ±Ρ‰Π΅ΠΌΡƒ) Π·Π½Π°ΠΌΠ΅Π½Π°Ρ‚Π΅Π»ΡŽ. НС Π±ΡƒΠ΄Π΅ΠΌ ΠΏΠΎΠ΄Ρ€ΠΎΠ±Π½ΠΎ ΠΎΡΡ‚Π°Π½Π°Π²Π»ΠΈΠ²Π°Ρ‚ΡŒΡΡ Π½Π° этом. Если испытываСтС трудности, ΠΎΠ±ΡΠ·Π°Ρ‚Π΅Π»ΡŒΠ½ΠΎ Π²Π΅Ρ€Π½ΠΈΡ‚Π΅ΡΡŒ ΠΊ ΡƒΡ€ΠΎΠΊΡƒ дСйствия с дробями ΠΈ ΠΏΠΎΠ²Ρ‚ΠΎΡ€ΠΈΡ‚Π΅ Π΅Π³ΠΎ.

    ПослС привСдСния Π΄Ρ€ΠΎΠ±Π΅ΠΉ ΠΊ ΠΎΠ±Ρ‰Π΅ΠΌΡƒ Π·Π½Π°ΠΌΠ΅Π½Π°Ρ‚Π΅Π»ΡŽ Π²Ρ‹Ρ€Π°ΠΆΠ΅Π½ΠΈΠ΅ ΠΏΡ€ΠΈΠΌΠ΅Ρ‚ ΡΠ»Π΅Π΄ΡƒΡŽΡ‰ΠΈΠΉ Π²ΠΈΠ΄:

    Π­Ρ‚ΠΎ слоТСниС Ρ€Π°Ρ†ΠΈΠΎΠ½Π°Π»ΡŒΠ½Ρ‹Ρ… чисСл с Ρ€Π°Π·Π½Ρ‹ΠΌΠΈ Π·Π½Π°ΠΊΠ°ΠΌΠΈ. Π’Ρ‹Ρ‡ΠΈΡ‚Π°Π΅ΠΌ ΠΈΠ· большСго модуля мСньший ΠΈ ΠΏΠ΅Ρ€Π΅Π΄ ΠΏΠΎΠ»ΡƒΡ‡Π΅Π½Π½Ρ‹ΠΌ ΠΎΡ‚Π²Π΅Ρ‚ΠΎΠΌ ставим Ρ‚ΠΎΡ‚ Π·Π½Π°ΠΊ, ΠΌΠΎΠ΄ΡƒΠ»ΡŒ ΠΊΠΎΡ‚ΠΎΡ€ΠΎΠ³ΠΎ большС:

    ΠŸΡ€ΠΈΠΌΠ΅Ρ€ 4. Найти Π·Π½Π°Ρ‡Π΅Π½ΠΈΠ΅ выраТСния

    ΠŸΠΎΠ»ΡƒΡ‡ΠΈΠ»ΠΈ сумму ΠΈΠ· Ρ‚Ρ€Ρ‘Ρ… слагаСмых. Π‘Π½Π°Ρ‡Π°Π»Π° Π½Π°ΠΉΠ΄Ρ‘ΠΌ Π·Π½Π°Ρ‡Π΅Π½ΠΈΠ΅ выраТСния, Π·Π°Ρ‚Π΅ΠΌ ΠΊ ΠΏΠΎΠ»ΡƒΡ‡Π΅Π½Π½ΠΎΠΌΡƒ ΠΎΡ‚Π²Π΅Ρ‚Ρƒ ΠΏΡ€ΠΈΠ±Π°Π²ΠΈΠΌ

    ΠŸΠ΅Ρ€Π²ΠΎΠ΅ дСйствиС:

    Π’Ρ‚ΠΎΡ€ΠΎΠ΅ дСйствиС:

    Π’Π°ΠΊΠΈΠΌ ΠΎΠ±Ρ€Π°Π·ΠΎΠΌ, Π·Π½Π°Ρ‡Π΅Π½ΠΈΠ΅ выраТСния Ρ€Π°Π²Π½ΠΎ.

    РСшСниС для Π΄Π°Π½Π½ΠΎΠ³ΠΎ ΠΏΡ€ΠΈΠΌΠ΅Ρ€Π° ΠΌΠΎΠΆΠ½ΠΎ Π·Π°ΠΏΠΈΡΠ°Ρ‚ΡŒ ΠΏΠΎΠΊΠΎΡ€ΠΎΡ‡Π΅

    ΠŸΡ€ΠΈΠΌΠ΅Ρ€ 5 . Найти Π·Π½Π°Ρ‡Π΅Π½ΠΈΠ΅ выраТСния

    Π—Π°ΠΊΠ»ΡŽΡ‡ΠΈΠΌ ΠΊΠ°ΠΆΠ΄ΠΎΠ΅ число Π² скобки вмСстС со своими Π·Π½Π°ΠΊΠ°ΠΌΠΈ. Для этого смСшанноС число Π²Ρ€Π΅ΠΌΠ΅Π½Π½ΠΎ Ρ€Π°Π·Π²Π΅Ρ€Π½Ρ‘ΠΌ

    Вычислим Ρ†Π΅Π»Ρ‹Π΅ части:

    Π’ Π³Π»Π°Π²Π½ΠΎΠΌ Π²Ρ‹Ρ€Π°ΠΆΠ΅Π½ΠΈΠΈ вмСсто запишСм ΠΏΠΎΠ»ΡƒΡ‡Π΅Π½Π½ΡƒΡŽ Π΅Π΄ΠΈΠ½ΠΈΡ†Ρƒ:

    ΠŸΠΎΠ»ΡƒΡ‡Π΅Π½Π½ΠΎΠ΅ Π²Ρ‹Ρ€Π°ΠΆΠ΅Π½ΠΈΠ΅ свСрнём. Для этого опустим скобки ΠΈ запишСм Π΅Π΄ΠΈΠ½ΠΈΡ†Ρƒ ΠΈ Π΄Ρ€ΠΎΠ±ΡŒ вмСстС

    РСшСниС для Π΄Π°Π½Π½ΠΎΠ³ΠΎ ΠΏΡ€ΠΈΠΌΠ΅Ρ€Π° ΠΌΠΎΠΆΠ½ΠΎ Π·Π°ΠΏΠΈΡΠ°Ρ‚ΡŒ ΠΏΠΎΠΊΠΎΡ€ΠΎΡ‡Π΅:

    ΠŸΡ€ΠΈΠΌΠ΅Ρ€ 6. Найти Π·Π½Π°Ρ‡Π΅Π½ΠΈΠ΅ выраТСния

    ΠŸΠ΅Ρ€Π΅Π²Π΅Π΄Ρ‘ΠΌ смСшанноС число Π² Π½Π΅ΠΏΡ€Π°Π²ΠΈΠ»ΡŒΠ½ΡƒΡŽ Π΄Ρ€ΠΎΠ±ΡŒ. ΠžΡΡ‚Π°Π»ΡŒΠ½ΡƒΡŽ Ρ‡Π°ΡΡ‚ΡŒ ΠΏΠ΅Ρ€Π΅ΠΏΠΈΡˆΠ΅ΠΌ ΠΊΠ°ΠΊ Π΅ΡΡ‚ΡŒ:

    Π—Π°ΠΊΠ»ΡŽΡ‡ΠΈΠΌ ΠΊΠ°ΠΆΠ΄ΠΎΠ΅ Ρ€Π°Ρ†ΠΈΠΎΠ½Π°Π»ΡŒΠ½ΠΎΠ΅ число Π² скобки вмСстС со своими Π·Π½Π°ΠΊΠ°ΠΌΠΈ:

    Π—Π°ΠΌΠ΅Π½ΠΈΠΌ Π²Ρ‹Ρ‡ΠΈΡ‚Π°Π½ΠΈΠ΅ слоТСниСм:

    ΠŸΠΎΠ»ΡƒΡ‡ΠΈΠ»ΠΈ слоТСниС ΠΎΡ‚Ρ€ΠΈΡ†Π°Ρ‚Π΅Π»ΡŒΠ½Ρ‹Ρ… Ρ€Π°Ρ†ΠΈΠΎΠ½Π°Π»ΡŒΠ½Ρ‹Ρ… чисСл. Π‘Π»ΠΎΠΆΠΈΠΌ ΠΌΠΎΠ΄ΡƒΠ»ΠΈ этих чисСл ΠΈ ΠΏΠ΅Ρ€Π΅Π΄ ΠΏΠΎΠ»ΡƒΡ‡Π΅Π½Π½Ρ‹ΠΌ ΠΎΡ‚Π²Π΅Ρ‚ΠΎΠΌ поставим минус:

    Π’Π°ΠΊΠΈΠΌ ΠΎΠ±Ρ€Π°Π·ΠΎΠΌ, Π·Π½Π°Ρ‡Π΅Π½ΠΈΠ΅ выраТСния Ρ€Π°Π²Π½ΠΎ .

    РСшСниС для Π΄Π°Π½Π½ΠΎΠ³ΠΎ ΠΏΡ€ΠΈΠΌΠ΅Ρ€Π° ΠΌΠΎΠΆΠ½ΠΎ Π·Π°ΠΏΠΈΡΠ°Ρ‚ΡŒ ΠΏΠΎΠΊΠΎΡ€ΠΎΡ‡Π΅:

    ΠŸΡ€ΠΈΠΌΠ΅Ρ€ 7. Найти Π·Π½Π°Ρ‡Π΅Π½ΠΈΠ΅ Π²Ρ‹Ρ€Π°ΠΆΠ΅Π½ΠΈΠ΅

    Π—Π°ΠΏΠΈΡˆΠ΅ΠΌ смСшанноС число Π² Ρ€Π°Π·Π²Ρ‘Ρ€Π½ΡƒΡ‚ΠΎΠΌ Π²ΠΈΠ΄Π΅. ΠžΡΡ‚Π°Π»ΡŒΠ½ΠΎΠ΅ ΠΏΠ΅Ρ€Π΅ΠΏΠΈΡˆΠ΅ΠΌ ΠΊΠ°ΠΊ Π΅ΡΡ‚ΡŒ:

    Π—Π°ΠΊΠ»ΡŽΡ‡ΠΈΠΌ ΠΊΠ°ΠΆΠ΄ΠΎΠ΅ Ρ€Π°Ρ†ΠΈΠΎΠ½Π°Π»ΡŒΠ½ΠΎΠ΅ число Π² скобки вмСстС своими Π·Π½Π°ΠΊΠ°ΠΌΠΈ

    Π—Π°ΠΌΠ΅Π½ΠΈΠΌ Π²Ρ‹Ρ‡ΠΈΡ‚Π°Π½ΠΈΠ΅ слоТСниСм Ρ‚Π°ΠΌ, Π³Π΄Π΅ это ΠΌΠΎΠΆΠ½ΠΎ:

    Вычислим Ρ†Π΅Π»Ρ‹Π΅ части:

    Π’ Π³Π»Π°Π²Π½ΠΎΠΌ Π²Ρ‹Ρ€Π°ΠΆΠ΅Π½ΠΈΠΈ вмСсто запишСм ΠΏΠΎΠ»ΡƒΡ‡Π΅Π½Π½ΠΎΠ΅ число?7

    Π’Ρ‹Ρ€Π°ΠΆΠ΅Π½ΠΈΠ΅ являСтся Ρ€Π°Π·Π²Ρ‘Ρ€Π½ΡƒΡ‚ΠΎΠΉ Ρ„ΠΎΡ€ΠΌΠΎΠΉ записи смСшанного числа . МоТно сразу Π·Π°ΠΏΠΈΡΠ°Ρ‚ΡŒ ΠΎΡ‚Π²Π΅Ρ‚, записав вмСстС числа?7 ΠΈ Π΄Ρ€ΠΎΠ±ΡŒ (спрятав минус этой Π΄Ρ€ΠΎΠ±ΠΈ)

    Π’Π°ΠΊΠΈΠΌ ΠΎΠ±Ρ€Π°Π·ΠΎΠΌ, Π·Π½Π°Ρ‡Π΅Π½ΠΈΠ΅ выраТСния Ρ€Π°Π²Π½ΠΎ

    РСшСниС для Π΄Π°Π½Π½ΠΎΠ³ΠΎ ΠΏΡ€ΠΈΠΌΠ΅Ρ€Π° ΠΌΠΎΠΆΠ½ΠΎ Π·Π°ΠΏΠΈΡΠ°Ρ‚ΡŒ Π·Π½Π°Ρ‡ΠΈΡ‚Π΅Π»ΡŒΠ½ΠΎ ΠΊΠΎΡ€ΠΎΡ‡Π΅. Если ΠΏΡ€ΠΎΠΏΡƒΡΡ‚ΠΈΡ‚ΡŒ Π½Π΅ΠΊΠΎΡ‚ΠΎΡ€Ρ‹Π΅ подробности, Ρ‚ΠΎ Π΅Π³ΠΎ ΠΌΠΎΠΆΠ½ΠΎ Π·Π°ΠΏΠΈΡΠ°Ρ‚ΡŒ ΡΠ»Π΅Π΄ΡƒΡŽΡ‰ΠΈΠΌ ΠΎΠ±Ρ€Π°Π·ΠΎΠΌ:

    ΠŸΡ€ΠΈΠΌΠ΅Ρ€ 8. Найти Π·Π½Π°Ρ‡Π΅Π½ΠΈΠ΅ выраТСния

    Π”Π°Π½Π½ΠΎΠ΅ Π²Ρ‹Ρ€Π°ΠΆΠ΅Π½ΠΈΠ΅ ΠΌΠΎΠΆΠ½ΠΎ Π²Ρ‹Ρ‡ΠΈΡΠ»ΠΈΡ‚ΡŒ двумя способами. Рассмотрим ΠΊΠ°ΠΆΠ΄Ρ‹ΠΉ ΠΈΠ· Π½ΠΈΡ….

    ΠŸΠ΅Ρ€Π²Ρ‹ΠΉ способ. Π¦Π΅Π»Ρ‹Π΅ ΠΈ Π΄Ρ€ΠΎΠ±Π½Ρ‹Π΅ части выраТСния Π²Ρ‹Ρ‡ΠΈΡΠ»ΡΡŽΡ‚ΡΡ ΠΏΠΎ ΠΎΡ‚Π΄Π΅Π»ΡŒΠ½ΠΎΡΡ‚ΠΈ.

    Для Π½Π°Ρ‡Π°Π»Π° запишСм ΡΠΌΠ΅ΡˆΠ°Π½Π½Ρ‹Π΅ числа Π² Ρ€Π°Π·Π²Ρ‘Ρ€Π½ΡƒΡ‚ΠΎΠΌ Π²ΠΈΠ΄Π΅:

    Π—Π°ΠΊΠ»ΡŽΡ‡ΠΈΠΌ ΠΊΠ°ΠΆΠ΄ΠΎΠ΅ число Π² скобки вмСстС со своими Π·Π½Π°ΠΊΠ°ΠΌΠΈ:

    Π—Π°ΠΌΠ΅Π½ΠΈΠΌ Π²Ρ‹Ρ‡ΠΈΡ‚Π°Π½ΠΈΠ΅ слоТСниСм Ρ‚Π°ΠΌ, Π³Π΄Π΅ это ΠΌΠΎΠΆΠ½ΠΎ:

    ΠŸΠΎΠ»ΡƒΡ‡ΠΈΠ»ΠΈ сумму ΠΈΠ· Π½Π΅ΡΠΊΠΎΠ»ΡŒΠΊΠΈΡ… слагаСмых. Богласно ΡΠΎΡ‡Π΅Ρ‚Π°Ρ‚Π΅Π»ΡŒΠ½ΠΎΠΌΡƒ Π·Π°ΠΊΠΎΠ½Ρƒ слоТСния, Ссли Π²Ρ‹Ρ€Π°ΠΆΠ΅Π½ΠΈΠ΅ содСрТит нСсколько слагаСмых, Ρ‚ΠΎ сумма Π½Π΅ Π±ΡƒΠ΄Π΅Ρ‚ Π·Π°Π²ΠΈΡΠ΅Ρ‚ΡŒ ΠΎΡ‚ порядка дСйствий. Π­Ρ‚ΠΎ ΠΏΠΎΠ·Π²ΠΎΠ»ΠΈΡ‚ Π½Π°ΠΌ ΡΠ³Ρ€ΡƒΠΏΠΏΠΈΡ€ΠΎΠ²Π°Ρ‚ΡŒ Ρ†Π΅Π»Ρ‹Π΅ ΠΈ Π΄Ρ€ΠΎΠ±Π½Ρ‹Π΅ части ΠΏΠΎ ΠΎΡ‚Π΄Π΅Π»ΡŒΠ½ΠΎΡΡ‚ΠΈ:

    Вычислим Ρ†Π΅Π»Ρ‹Π΅ части:

    Π’ Π³Π»Π°Π²Π½ΠΎΠΌ Π²Ρ‹Ρ€Π°ΠΆΠ΅Π½ΠΈΠΈ вмСсто запишСм ΠΏΠΎΠ»ΡƒΡ‡Π΅Π½Π½ΠΎΠ΅ число?3

    Вычислим Π΄Ρ€ΠΎΠ±Π½Ρ‹Π΅ части:

    Π’ Π³Π»Π°Π²Π½ΠΎΠΌ Π²Ρ‹Ρ€Π°ΠΆΠ΅Π½ΠΈΠΈ вмСсто запишСм ΠΏΠΎΠ»ΡƒΡ‡Π΅Π½Π½ΠΎΠ΅ смСшанноС число

    Π§Ρ‚ΠΎΠ±Ρ‹ Π²Ρ‹Ρ‡ΠΈΡΠ»ΠΈΡ‚ΡŒ ΠΏΠΎΠ»ΡƒΡ‡ΠΈΠ²ΡˆΠ΅Π΅ΡΡ Π²Ρ‹Ρ€Π°ΠΆΠ΅Π½ΠΈΠ΅, смСшанноС число Π½ΡƒΠΆΠ½ΠΎ Π²Ρ€Π΅ΠΌΠ΅Π½Π½ΠΎ Ρ€Π°Π·Π²Π΅Ρ€Π½ΡƒΡ‚ΡŒ, Π·Π°Ρ‚Π΅ΠΌ Π·Π°ΠΊΠ»ΡŽΡ‡ΠΈΡ‚ΡŒ Π² скобки ΠΊΠ°ΠΆΠ΄ΠΎΠ΅ число ΠΈ Π·Π°ΠΌΠ΅Π½ΠΈΡ‚ΡŒ Π²Ρ‹Ρ‡ΠΈΡ‚Π°Π½ΠΈΠ΅ слоТСниСм. Π”Π΅Π»Π°Ρ‚ΡŒ это Π½ΡƒΠΆΠ½ΠΎ ΠΎΡ‡Π΅Π½ΡŒ Π°ΠΊΠΊΡƒΡ€Π°Ρ‚Π½ΠΎ, Ρ‡Ρ‚ΠΎΠ±Ρ‹ Π½Π΅ ΠΏΠ΅Ρ€Π΅ΠΏΡƒΡ‚Π°Ρ‚ΡŒ Π·Π½Π°ΠΊΠΈ слагаСмых.

    ПослС прСобразования выраТСния ΠΌΡ‹ ΠΏΠΎΠ»ΡƒΡ‡ΠΈΠ»ΠΈ Π½ΠΎΠ²ΠΎΠ΅ Π²Ρ‹Ρ€Π°ΠΆΠ΅Π½ΠΈΠ΅ , ΠΊΠΎΡ‚ΠΎΡ€ΠΎΠ΅ Π»Π΅Π³ΠΊΠΎ вычисляСтся. ΠŸΠΎΡ…ΠΎΠΆΠ΅Π΅ Π²Ρ‹Ρ€Π°ΠΆΠ΅Π½ΠΈΠ΅ Π±Ρ‹Π»ΠΎ Π² ΠΏΡ€ΠΈΠΌΠ΅Ρ€Π΅ 7. Напомним, Ρ‡Ρ‚ΠΎ ΠΌΡ‹ ΠΎΡ‚Π΄Π΅Π»ΡŒΠ½ΠΎ слоТили Ρ†Π΅Π»Ρ‹Π΅ части, Π° Π΄Ρ€ΠΎΠ±Π½ΡƒΡŽ оставили ΠΊΠ°ΠΊ Π΅ΡΡ‚ΡŒ:

    Π—Π½Π°Ρ‡ΠΈΡ‚ Π·Π½Π°Ρ‡Π΅Π½ΠΈΠ΅ выраТСния Ρ€Π°Π²Π½ΠΎ

    РСшСниС для Π΄Π°Π½Π½ΠΎΠ³ΠΎ ΠΏΡ€ΠΈΠΌΠ΅Ρ€Π° ΠΌΠΎΠΆΠ½ΠΎ Π·Π°ΠΏΠΈΡΠ°Ρ‚ΡŒ ΠΏΠΎΠΊΠΎΡ€ΠΎΡ‡Π΅

    Π’ ΠΊΠΎΡ€ΠΎΡ‚ΠΊΠΎΠΌ Ρ€Π΅ΡˆΠ΅Π½ΠΈΠΈ ΠΏΡ€ΠΎΠΏΡƒΡΠΊΠ°ΡŽΡ‚ΡΡ этапы Π·Π°ΠΊΠ»ΡŽΡ‡Π΅Π½ΠΈΡ чисСл Π² скобки, Π·Π°ΠΌΠ΅Π½Π° вычитания слоТСниСм, проставлСниС ΠΌΠΎΠ΄ΡƒΠ»Π΅ΠΉ. Если Π²Ρ‹ ΡƒΡ‡ΠΈΡ‚Π΅ΡΡŒ Π² школС ΠΈΠ»ΠΈ Π² Π΄Ρ€ΡƒΠ³ΠΎΠΌ ΡƒΡ‡Π΅Π±Π½ΠΎΠΌ Π·Π°Π²Π΅Π΄Π΅Π½ΠΈΠΈ, Ρ‚ΠΎ ΠΎΡ‚ вас Π±ΡƒΠ΄ΡƒΡ‚ Ρ‚Ρ€Π΅Π±ΠΎΠ²Π°Ρ‚ΡŒ ΠΏΡ€ΠΎΠΏΡƒΡΠΊΠ°Ρ‚ΡŒ эти ΠΏΡ€ΠΈΠΌΠΈΡ‚ΠΈΠ²Π½Ρ‹Π΅ дСйствия, Ρ‡Ρ‚ΠΎΠ±Ρ‹ ΡΡΠΊΠΎΠ½ΠΎΠΌΠΈΡ‚ΡŒ врСмя ΠΈ мСсто. ΠŸΡ€ΠΈΠ²Π΅Π΄Ρ‘Π½Π½ΠΎΠ΅ Π²Ρ‹ΡˆΠ΅ ΠΊΠΎΡ€ΠΎΡ‚ΠΊΠΎΠ΅ Ρ€Π΅ΡˆΠ΅Π½ΠΈΠ΅ ΠΌΠΎΠΆΠ½ΠΎ Π·Π°ΠΏΠΈΡΠ°Ρ‚ΡŒ Π΅Ρ‰Ρ‘ ΠΊΠΎΡ€ΠΎΡ‡Π΅. Π’Ρ‹Π³Π»ΡΠ΄Π΅Ρ‚ΡŒ ΠΎΠ½ΠΎ Π±ΡƒΠ΄Π΅Ρ‚ Ρ‚Π°ΠΊ:

    ΠŸΠΎΡΡ‚ΠΎΠΌΡƒ, Π½Π°Ρ…ΠΎΠ΄ΡΡΡŒ Π² школС ΠΈΠ»ΠΈ Π² ΠΈΠ½ΠΎΠΌ ΡƒΡ‡Π΅Π±Π½ΠΎΠΌ Π·Π°Π²Π΅Π΄Π΅Π½ΠΈΠΈ, Π±ΡƒΠ΄ΡŒΡ‚Π΅ Π³ΠΎΡ‚ΠΎΠ²Ρ‹ ΠΊ Ρ‚ΠΎΠΌΡƒ, Ρ‡Ρ‚ΠΎ Π½Π΅ΠΊΠΎΡ‚ΠΎΡ€Ρ‹Π΅ дСйствия придётся Π²Ρ‹ΠΏΠΎΠ»Π½ΡΡ‚ΡŒ Π² ΡƒΠΌΠ΅.

    Π’Ρ‚ΠΎΡ€ΠΎΠΉ способ. Π‘ΠΌΠ΅ΡˆΠ°Π½Π½Ρ‹Π΅ числа выраТСния пСрСводят Π² Π½Π΅ΠΏΡ€Π°Π²ΠΈΠ»ΡŒΠ½Ρ‹Π΅ Π΄Ρ€ΠΎΠ±ΠΈ ΠΈ Π²Ρ‹Ρ‡ΠΈΡΠ»ΡΡŽΡ‚, ΠΊΠ°ΠΊ ΠΎΠ±Ρ‹Ρ‡Π½Ρ‹Π΅ Π΄Ρ€ΠΎΠ±ΠΈ.

    Π—Π°ΠΊΠ»ΡŽΡ‡ΠΈΠΌ Π² скобки ΠΊΠ°ΠΆΠ΄ΠΎΠ΅ Ρ€Π°Ρ†ΠΈΠΎΠ½Π°Π»ΡŒΠ½ΠΎΠ΅ число вмСстС со своими Π·Π½Π°ΠΊΠ°ΠΌΠΈ

    Π—Π°ΠΌΠ΅Π½ΠΈΠΌ Π²Ρ‹Ρ‡ΠΈΡ‚Π°Π½ΠΈΠ΅ слоТСниСм:

    Π’Π΅ΠΏΠ΅Ρ€ΡŒ ΡΠΌΠ΅ΡˆΠ°Π½Π½Ρ‹Π΅ числа ΠΈ ΠΏΠ΅Ρ€Π΅Π²Π΅Π΄Ρ‘ΠΌ Π² Π½Π΅ΠΏΡ€Π°Π²ΠΈΠ»ΡŒΠ½Ρ‹Π΅ Π΄Ρ€ΠΎΠ±ΠΈ:

    ΠŸΠΎΠ»ΡƒΡ‡ΠΈΠ»ΠΈ слоТСниС ΠΎΡ‚Ρ€ΠΈΡ†Π°Ρ‚Π΅Π»ΡŒΠ½Ρ‹Ρ… Ρ€Π°Ρ†ΠΈΠΎΠ½Π°Π»ΡŒΠ½Ρ‹Ρ… чисСл. Π‘Π»ΠΎΠΆΠΈΠΌ ΠΈΡ… ΠΌΠΎΠ΄ΡƒΠ»ΠΈ ΠΈ ΠΏΠ΅Ρ€Π΅Π΄ ΠΏΠΎΠ»ΡƒΡ‡Π΅Π½Π½Ρ‹ΠΌ ΠΎΡ‚Π²Π΅Ρ‚ΠΎΠΌ поставим минус:

    ΠŸΠΎΠ»ΡƒΡ‡ΠΈΠ»ΠΈ ΠΎΡ‚Π²Π΅Ρ‚ ΠΊΠ°ΠΊ ΠΈ Π² ΠΏΡ€ΠΎΡˆΠ»Ρ‹ΠΉ Ρ€Π°Π·.

    ΠŸΠΎΠ΄Ρ€ΠΎΠ±Π½ΠΎΠ΅ Ρ€Π΅ΡˆΠ΅Π½ΠΈΠ΅ Π²Ρ‚ΠΎΡ€Ρ‹ΠΌ способом выглядит ΡΠ»Π΅Π΄ΡƒΡŽΡ‰ΠΈΠΌ ΠΎΠ±Ρ€Π°Π·ΠΎΠΌ:

    ΠŸΡ€ΠΈΠΌΠ΅Ρ€ 9. Найти выраТСния выраТСния

    ΠŸΠ΅Ρ€Π²Ρ‹ΠΉ способ. Π‘Π»ΠΎΠΆΠΈΠΌ Ρ†Π΅Π»Ρ‹Π΅ ΠΈ Π΄Ρ€ΠΎΠ±Π½Ρ‹Π΅ части ΠΏΠΎ ΠΎΡ‚Π΄Π΅Π»ΡŒΠ½ΠΎΡΡ‚ΠΈ.

    Π’ этот Ρ€Π°Π· ΠΏΠΎΠΏΡ€ΠΎΠ±ΡƒΠ΅ΠΌ ΠΏΡ€ΠΎΠΏΡƒΡΡ‚ΠΈΡ‚ΡŒ Π½Π΅ΠΊΠΎΡ‚ΠΎΡ€Ρ‹Π΅ ΠΏΡ€ΠΈΠΌΠΈΡ‚ΠΈΠ²Π½Ρ‹Π΅ дСйствия, Ρ‚Π°ΠΊΠΈΠ΅ ΠΊΠ°ΠΊ запись выраТСния Π² Ρ€Π°Π·Π²Ρ‘Ρ€Π½ΡƒΡ‚ΠΎΠΌ Π²ΠΈΠ΄Π΅, Π·Π°ΠΊΠ»ΡŽΡ‡Π΅Π½ΠΈΠ΅ чисСл Π² скобки, Π·Π°ΠΌΠ΅Π½Π° вычитания слоТСниСм, проставлСниС ΠΌΠΎΠ΄ΡƒΠ»Π΅ΠΉ:

    ΠžΠ±Ρ€Π°Ρ‚ΠΈΡ‚Π΅ Π²Π½ΠΈΠΌΠ°Π½ΠΈΠ΅, Ρ‡Ρ‚ΠΎ Π΄Ρ€ΠΎΠ±Π½Ρ‹Π΅ части Π±Ρ‹Π»ΠΈ ΠΏΡ€ΠΈΠ²Π΅Π΄Π΅Π½Ρ‹ ΠΊ ΠΎΠ±Ρ‰Π΅ΠΌΡƒ Π·Π½Π°ΠΌΠ΅Π½Π°Ρ‚Π΅Π»ΡŽ.

    Π’Ρ‚ΠΎΡ€ΠΎΠΉ способ. ΠŸΠ΅Ρ€Π΅Π²Π΅Π΄Ρ‘ΠΌ ΡΠΌΠ΅ΡˆΠ°Π½Π½Ρ‹Π΅ числа Π² Π½Π΅ΠΏΡ€Π°Π²ΠΈΠ»ΡŒΠ½Ρ‹Π΅ Π΄Ρ€ΠΎΠ±ΠΈ ΠΈ вычислим, ΠΊΠ°ΠΊ ΠΎΠ±Ρ‹Ρ‡Π½Ρ‹Π΅ Π΄Ρ€ΠΎΠ±ΠΈ.

    ΠŸΡ€ΠΈΠΌΠ΅Ρ€ 10. Найти Π·Π½Π°Ρ‡Π΅Π½ΠΈΠ΅ выраТСния

    Π—Π°ΠΌΠ΅Π½ΠΈΠΌ Π²Ρ‹Ρ‡ΠΈΡ‚Π°Π½ΠΈΠ΅ слоТСниСм:

    Π’ ΠΏΠΎΠ»ΡƒΡ‡ΠΈΠ²ΡˆΠ΅ΠΌΡΡ Π²Ρ‹Ρ€Π°ΠΆΠ΅Π½ΠΈΠΈ Π½Π΅Ρ‚ ΠΎΡ‚Ρ€ΠΈΡ†Π°Ρ‚Π΅Π»ΡŒΠ½Ρ‹Ρ… чисСл, ΠΊΠΎΡ‚ΠΎΡ€Ρ‹Π΅ ΡΠ²Π»ΡΡŽΡ‚ΡΡ основной ΠΏΡ€ΠΈΡ‡ΠΈΠ½ΠΎΠΉ допущСния ошибок. А ΠΏΠΎΡΠΊΠΎΠ»ΡŒΠΊΡƒ Π½Π΅Ρ‚ ΠΎΡ‚Ρ€ΠΈΡ†Π°Ρ‚Π΅Π»ΡŒΠ½Ρ‹Ρ… чисСл, ΠΌΡ‹ ΠΌΠΎΠΆΠ΅ΠΌ ΡƒΠ±Ρ€Π°Ρ‚ΡŒ плюс ΠΏΠ΅Ρ€Π΅Π΄ Π²Ρ‹Ρ‡ΠΈΡ‚Π°Π΅ΠΌΡ‹ΠΌ, Π° Ρ‚Π°ΠΊΠΆΠ΅ ΡƒΠ±Ρ€Π°Ρ‚ΡŒ скобки. Π’ΠΎΠ³Π΄Π° ΠΏΠΎΠ»ΡƒΡ‡ΠΈΠΌ ΠΏΡ€ΠΎΡΡ‚Π΅ΠΉΡˆΠ΅Π΅ Π²Ρ‹Ρ€Π°ΠΆΠ΅Π½ΠΈΠ΅, ΠΊΠΎΡ‚ΠΎΡ€ΠΎΠ΅ вычисляСтся Π»Π΅Π³ΠΊΠΎ:

    Π’ Π΄Π°Π½Π½ΠΎΠΌ ΠΏΡ€ΠΈΠΌΠ΅Ρ€Π΅ Ρ†Π΅Π»Ρ‹Π΅ ΠΈ Π΄Ρ€ΠΎΠ±Π½Ρ‹Π΅ части Π±Ρ‹Π»ΠΈ вычислСны ΠΏΠΎ ΠΎΡ‚Π΄Π΅Π»ΡŒΠ½ΠΎΡΡ‚ΠΈ.

    ΠŸΡ€ΠΈΠΌΠ΅Ρ€ 11. Найти Π·Π½Π°Ρ‡Π΅Π½ΠΈΠ΅ выраТСния

    Π­Ρ‚ΠΎ слоТСниС Ρ€Π°Ρ†ΠΈΠΎΠ½Π°Π»ΡŒΠ½Ρ‹Ρ… чисСл с Ρ€Π°Π·Π½Ρ‹ΠΌΠΈ Π·Π½Π°ΠΊΠ°ΠΌΠΈ. Π’Ρ‹Ρ‡Ρ‚Π΅ΠΌ ΠΈΠ· большСго модуля мСньший ΠΈ ΠΏΠ΅Ρ€Π΅Π΄ ΠΏΠΎΠ»ΡƒΡ‡Π΅Π½Π½Ρ‹ΠΌΠΈ числом поставим Ρ‚ΠΎΡ‚ Π·Π½Π°ΠΊ, ΠΌΠΎΠ΄ΡƒΠ»ΡŒ ΠΊΠΎΡ‚ΠΎΡ€ΠΎΠ³ΠΎ большС:

    ΠŸΡ€ΠΈΠΌΠ΅Ρ€ 12. Найти Π·Π½Π°Ρ‡Π΅Π½ΠΈΠ΅ выраТСния

    Π’Ρ‹Ρ€Π°ΠΆΠ΅Π½ΠΈΠ΅ состоит ΠΈΠ· Π½Π΅ΡΠΊΠΎΠ»ΡŒΠΊΠΈΡ… ΠΏΠ°Ρ€Π°ΠΌΠ΅Ρ‚Ρ€ΠΎΠ². Богласно порядку дСйствий, Π² ΠΏΠ΅Ρ€Π²ΡƒΡŽ ΠΎΡ‡Π΅Ρ€Π΅Π΄ΡŒ Π½Π΅ΠΎΠ±Ρ…ΠΎΠ΄ΠΈΠΌΠΎ Π²Ρ‹ΠΏΠΎΠ»Π½ΠΈΡ‚ΡŒ дСйствия Π² скобках.

    Π‘Π½Π°Ρ‡Π°Π»Π° вычислим Π²Ρ‹Ρ€Π°ΠΆΠ΅Π½ΠΈΠ΅ , Π·Π°Ρ‚Π΅ΠΌ Π²Ρ‹Ρ€Π°ΠΆΠ΅Π½ΠΈΠ΅ ΠŸΠΎΠ»ΡƒΡ‡Π΅Π½Π½Ρ‹Π΅ ΠΎΡ‚Π²Π΅Ρ‚Ρ‹ слоТим.

    ΠŸΠ΅Ρ€Π²ΠΎΠ΅ дСйствиС:

    Π’Ρ‚ΠΎΡ€ΠΎΠ΅ дСйствиС:

    Π’Ρ€Π΅Ρ‚ΡŒΠ΅ дСйствиС:

    ΠžΡ‚Π²Π΅Ρ‚: Π·Π½Π°Ρ‡Π΅Π½ΠΈΠ΅ выраТСния Ρ€Π°Π²Π½ΠΎ

    ΠŸΡ€ΠΈΠΌΠ΅Ρ€ 13. Найти Π·Π½Π°Ρ‡Π΅Π½ΠΈΠ΅ выраТСния

    Π—Π°ΠΌΠ΅Π½ΠΈΠΌ Π²Ρ‹Ρ‡ΠΈΡ‚Π°Π½ΠΈΠ΅ слоТСниСм:

    ΠŸΠΎΠ»ΡƒΡ‡ΠΈΠ»ΠΈ слоТСниСм Ρ€Π°Ρ†ΠΈΠΎΠ½Π°Π»ΡŒΠ½Ρ‹Ρ… чисСл с Ρ€Π°Π·Π½Ρ‹ΠΌΠΈ Π·Π½Π°ΠΊΠ°ΠΌΠΈ. Π’Ρ‹Ρ‡Ρ‚Π΅ΠΌ ΠΈΠ· большСго модуля мСньший ΠΈ ΠΏΠ΅Ρ€Π΅Π΄ ΠΎΡ‚Π²Π΅Ρ‚ΠΎΠΌ поставим Ρ‚ΠΎΡ‚ Π·Π½Π°ΠΊ, ΠΌΠΎΠ΄ΡƒΠ»ΡŒ ΠΊΠΎΡ‚ΠΎΡ€ΠΎΠ³ΠΎ большС. Но ΠΌΡ‹ ΠΈΠΌΠ΅Π΅ΠΌ Π΄Π΅Π»ΠΎ со ΡΠΌΠ΅ΡˆΠ°Π½Π½Ρ‹ΠΌΠΈ числами. Π§Ρ‚ΠΎΠ±Ρ‹ ΠΏΠΎΠ½ΡΡ‚ΡŒ ΠΊΠ°ΠΊΠΎΠΉ ΠΌΠΎΠ΄ΡƒΠ»ΡŒ большС, Π° ΠΊΠ°ΠΊΠΎΠΉ мСньшС, Π½ΡƒΠΆΠ½ΠΎ ΡΡ€Π°Π²Π½ΠΈΡ‚ΡŒ ΠΌΠΎΠ΄ΡƒΠ»ΠΈ этих ΡΠΌΠ΅ΡˆΠ°Π½Π½Ρ‹Ρ… чисСл. А Ρ‡Ρ‚ΠΎΠ±Ρ‹ ΡΡ€Π°Π²Π½ΠΈΡ‚ΡŒ ΠΌΠΎΠ΄ΡƒΠ»ΠΈ ΡΠΌΠ΅ΡˆΠ°Π½Π½Ρ‹Ρ… чисСл, Π½ΡƒΠΆΠ½ΠΎ пСрСвСсти ΠΈΡ… Π² Π½Π΅ΠΏΡ€Π°Π²ΠΈΠ»ΡŒΠ½Ρ‹Π΅ Π΄Ρ€ΠΎΠ±ΠΈ ΠΈ ΡΡ€Π°Π²Π½ΠΈΡ‚ΡŒ, ΠΊΠ°ΠΊ ΠΎΠ±Ρ‹Ρ‡Π½Ρ‹Π΅ Π΄Ρ€ΠΎΠ±ΠΈ.

    На ΡΠ»Π΅Π΄ΡƒΡŽΡ‰Π΅ΠΌ рисункС ΠΏΠΎΠΊΠ°Π·Π°Π½Ρ‹ всС этапы сравнСния ΠΌΠΎΠ΄ΡƒΠ»Π΅ΠΉ ΡΠΌΠ΅ΡˆΠ°Π½Π½Ρ‹Ρ… чисСл

    Π£Π·Π½Π°Π² ΠΊΠ°ΠΊΠΎΠΉ ΠΌΠΎΠ΄ΡƒΠ»ΡŒ большС, Π° ΠΊΠ°ΠΊΠΎΠΉ мСньшС, ΠΌΡ‹ ΠΌΠΎΠΆΠ΅ΠΌ ΠΏΡ€ΠΎΠ΄ΠΎΠ»ΠΆΠΈΡ‚ΡŒ вычислСниС нашСго ΠΏΡ€ΠΈΠΌΠ΅Ρ€Π°:

    Π’Π°ΠΊΠΈΠΌ ΠΎΠ±Ρ€Π°Π·ΠΎΠΌ, Π·Π½Π°Ρ‡Π΅Π½ΠΈΠ΅ выраТСния Ρ€Π°Π²Π½ΠΎ

    Рассмотрим слоТСниС ΠΈ Π²Ρ‹Ρ‡ΠΈΡ‚Π°Π½ΠΈΠ΅ дСсятичных Π΄Ρ€ΠΎΠ±Π΅ΠΉ, ΠΊΠΎΡ‚ΠΎΡ€Ρ‹Π΅ Ρ‚ΠΎΠΆΠ΅ относятся ΠΊ Ρ€Π°Ρ†ΠΈΠΎΠ½Π°Π»ΡŒΠ½Ρ‹ΠΌ числам ΠΈ ΠΊΠΎΡ‚ΠΎΡ€Ρ‹Π΅ ΠΌΠΎΠ³ΡƒΡ‚ Π±Ρ‹Ρ‚ΡŒ, ΠΊΠ°ΠΊ ΠΏΠΎΠ»ΠΎΠΆΠΈΡ‚Π΅Π»ΡŒΠ½Ρ‹ΠΌΠΈ, Ρ‚Π°ΠΊ ΠΈ ΠΎΡ‚Ρ€ΠΈΡ†Π°Ρ‚Π΅Π»ΡŒΠ½Ρ‹ΠΌΠΈ.

    ΠŸΡ€ΠΈΠΌΠ΅Ρ€ 14. Найти Π·Π½Π°Ρ‡Π΅Π½ΠΈΠ΅ выраТСния?3,2 + 4,3

    Π—Π°ΠΊΠ»ΡŽΡ‡ΠΈΠΌ ΠΊΠ°ΠΆΠ΄ΠΎΠ΅ Ρ€Π°Ρ†ΠΈΠΎΠ½Π°Π»ΡŒΠ½ΠΎΠ΅ число Π² скобки вмСстС со своими Π·Π½Π°ΠΊΠ°ΠΌΠΈ. Π£Ρ‡ΠΈΡ‚Ρ‹Π²Π°Π΅ΠΌ, Ρ‡Ρ‚ΠΎ плюс ΠΊΠΎΡ‚ΠΎΡ€Ρ‹ΠΉ Π΄Π°Π½ Π² Π²Ρ‹Ρ€Π°ΠΆΠ΅Π½ΠΈΠΈ являСтся Π·Π½Π°ΠΊΠΎΠΌ ΠΎΠΏΠ΅Ρ€Π°Ρ†ΠΈΠΈ ΠΈ Π½Π΅ относится ΠΊ дСсятичной Π΄Ρ€ΠΎΠ±ΠΈ 4,3. Π£ этой дСсятичной Π΄Ρ€ΠΎΠ±ΠΈ свой Π·Π½Π°ΠΊ плюса, ΠΊΠΎΡ‚ΠΎΡ€Ρ‹ΠΉ Π½Π΅Π²ΠΈΠ΄ΠΈΠΌ ΠΏΠΎ ΠΏΡ€ΠΈΡ‡ΠΈΠ½Π΅ Ρ‚ΠΎΠ³ΠΎ, Ρ‡Ρ‚ΠΎ Π΅Π³ΠΎ Π½Π΅ Π·Π°ΠΏΠΈΡΡ‹Π²Π°ΡŽΡ‚. Но ΠΌΡ‹ Π΅Π³ΠΎ запишСм для наглядности:

    Π­Ρ‚ΠΎ слоТСниС Ρ€Π°Ρ†ΠΈΠΎΠ½Π°Π»ΡŒΠ½Ρ‹Ρ… чисСл с Ρ€Π°Π·Π½Ρ‹ΠΌΠΈ Π·Π½Π°ΠΊΠ°ΠΌΠΈ. Π§Ρ‚ΠΎΠ±Ρ‹ ΡΠ»ΠΎΠΆΠΈΡ‚ΡŒ Ρ€Π°Ρ†ΠΈΠΎΠ½Π°Π»ΡŒΠ½Ρ‹Π΅ числа с Ρ€Π°Π·Π½Ρ‹ΠΌΠΈ Π·Π½Π°ΠΊΠ°ΠΌΠΈ, Π½ΡƒΠΆΠ½ΠΎ ΠΈΠ· большСго модуля Π²Ρ‹Ρ‡Π΅ΡΡ‚ΡŒ мСньший, ΠΈ ΠΏΠ΅Ρ€Π΅Π΄ ΠΏΠΎΠ»ΡƒΡ‡Π΅Π½Π½Ρ‹ΠΌ ΠΎΡ‚Π²Π΅Ρ‚ΠΎΠΌ ΠΏΠΎΡΡ‚Π°Π²ΠΈΡ‚ΡŒ Ρ‚ΠΎΡ‚ Π·Π½Π°ΠΊ, ΠΌΠΎΠ΄ΡƒΠ»ΡŒ ΠΊΠΎΡ‚ΠΎΡ€ΠΎΠ³ΠΎ большС. А Ρ‡Ρ‚ΠΎΠ±Ρ‹ ΠΏΠΎΠ½ΡΡ‚ΡŒ ΠΊΠ°ΠΊΠΎΠΉ ΠΌΠΎΠ΄ΡƒΠ»ΡŒ большС, Π° ΠΊΠ°ΠΊΠΎΠΉ мСньшС, Π½ΡƒΠΆΠ½ΠΎ ΡΡƒΠΌΠ΅Ρ‚ΡŒ ΡΡ€Π°Π²Π½ΠΈΡ‚ΡŒ ΠΌΠΎΠ΄ΡƒΠ»ΠΈ этих дСсятичных Π΄Ρ€ΠΎΠ±Π΅ΠΉ Π΄ΠΎ ΠΈΡ… вычислСния:

    ΠœΠΎΠ΄ΡƒΠ»ΡŒ числа 4,3 большС, Ρ‡Π΅ΠΌ ΠΌΠΎΠ΄ΡƒΠ»ΡŒ числа?3,2 поэтому ΠΌΡ‹ ΠΈΠ· 4,3 Π²Ρ‹Ρ‡Π»ΠΈ 3,2. ΠŸΠΎΠ»ΡƒΡ‡ΠΈΠ»ΠΈ ΠΎΡ‚Π²Π΅Ρ‚ 1,1. ΠžΡ‚Π²Π΅Ρ‚ ΠΏΠΎΠ»ΠΎΠΆΠΈΡ‚Π΅Π»Π΅Π½, ΠΏΠΎΡΠΊΠΎΠ»ΡŒΠΊΡƒ Π² ΠΎΡ‚Π²Π΅Ρ‚Π΅ Π΄ΠΎΠ»ΠΆΠ΅Π½ ΡΡ‚ΠΎΡΡ‚ΡŒ Π·Π½Π°ΠΊ большСго модуля, Ρ‚ΠΎ Π΅ΡΡ‚ΡŒ модуля |+4,3|.

    Π’Π°ΠΊΠΈΠΌ ΠΎΠ±Ρ€Π°Π·ΠΎΠΌ, Π·Π½Π°Ρ‡Π΅Π½ΠΈΠ΅ выраТСния?3,2 + (+4,3) Ρ€Π°Π²Π½ΠΎ 1,1

    ΠŸΡ€ΠΈΠΌΠ΅Ρ€ 15. Найти Π·Π½Π°Ρ‡Π΅Π½ΠΈΠ΅ выраТСния 3,5 + (?8,3)

    Π­Ρ‚ΠΎ слоТСниС Ρ€Π°Ρ†ΠΈΠΎΠ½Π°Π»ΡŒΠ½Ρ‹Ρ… чисСл с Ρ€Π°Π·Π½Ρ‹ΠΌΠΈ Π·Π½Π°ΠΊΠ°ΠΌΠΈ. Как ΠΈ Π² ΠΏΡ€ΠΎΡˆΠ»ΠΎΠΌ ΠΏΡ€ΠΈΠΌΠ΅Ρ€Π΅ ΠΈΠ· большСго модуля Π²Ρ‹Ρ‡ΠΈΡ‚Π°Π΅ΠΌ мСньший ΠΈ ΠΏΠ΅Ρ€Π΅Π΄ ΠΎΡ‚Π²Π΅Ρ‚ΠΎΠΌ ставим Ρ‚ΠΎΡ‚ Π·Π½Π°ΠΊ, ΠΌΠΎΠ΄ΡƒΠ»ΡŒ ΠΊΠΎΡ‚ΠΎΡ€ΠΎΠ³ΠΎ большС

    3,5 + (?8,3) = ?(|?8,3| ? |3,5|) = ?(8,3 ? 3,5) = ?(4,8) = ?4,8

    Π’Π°ΠΊΠΈΠΌ ΠΎΠ±Ρ€Π°Π·ΠΎΠΌ, Π·Π½Π°Ρ‡Π΅Π½ΠΈΠ΅ выраТСния 3,5 + (?8,3) Ρ€Π°Π²Π½ΠΎ?4,8

    Π­Ρ‚ΠΎΡ‚ ΠΏΡ€ΠΈΠΌΠ΅Ρ€ ΠΌΠΎΠΆΠ½ΠΎ Π·Π°ΠΏΠΈΡΠ°Ρ‚ΡŒ ΠΏΠΎΠΊΠΎΡ€ΠΎΡ‡Π΅:

    ΠŸΡ€ΠΈΠΌΠ΅Ρ€ 16. Найти Π·Π½Π°Ρ‡Π΅Π½ΠΈΠ΅ выраТСния?7,2 + (?3,11)

    Π­Ρ‚ΠΎ слоТСниС ΠΎΡ‚Ρ€ΠΈΡ†Π°Ρ‚Π΅Π»ΡŒΠ½Ρ‹Ρ… Ρ€Π°Ρ†ΠΈΠΎΠ½Π°Π»ΡŒΠ½Ρ‹Ρ… чисСл. Π§Ρ‚ΠΎΠ±Ρ‹ ΡΠ»ΠΎΠΆΠΈΡ‚ΡŒ ΠΎΡ‚Ρ€ΠΈΡ†Π°Ρ‚Π΅Π»ΡŒΠ½Ρ‹Π΅ Ρ€Π°Ρ†ΠΈΠΎΠ½Π°Π»ΡŒΠ½Ρ‹Π΅ числа, Π½ΡƒΠΆΠ½ΠΎ ΡΠ»ΠΎΠΆΠΈΡ‚ΡŒ ΠΈΡ… ΠΌΠΎΠ΄ΡƒΠ»ΠΈ ΠΈ ΠΏΠ΅Ρ€Π΅Π΄ ΠΏΠΎΠ»ΡƒΡ‡Π΅Π½Π½Ρ‹ΠΌ ΠΎΡ‚Π²Π΅Ρ‚ΠΎΠΌ ΠΏΠΎΡΡ‚Π°Π²ΠΈΡ‚ΡŒ минус. Π—Π°ΠΏΠΈΡΡŒ с модулями ΠΌΠΎΠΆΠ½ΠΎ ΠΏΡ€ΠΎΠΏΡƒΡΡ‚ΠΈΡ‚ΡŒ, Ρ‡Ρ‚ΠΎΠ±Ρ‹ Π½Π΅ Π·Π°Π³Ρ€ΠΎΠΌΠΎΠΆΠ΄Π°Ρ‚ΡŒ Π²Ρ‹Ρ€Π°ΠΆΠ΅Π½ΠΈΠ΅:

    7,2 + (?3,11) = ?7,20 + (?3,11) = ?(7,20 + 3,11) = ?(10,31) = ?10,31

    Π’Π°ΠΊΠΈΠΌ ΠΎΠ±Ρ€Π°Π·ΠΎΠΌ, Π·Π½Π°Ρ‡Π΅Π½ΠΈΠ΅ выраТСния?7,2 + (?3,11) Ρ€Π°Π²Π½ΠΎ?10,31

    Π­Ρ‚ΠΎΡ‚ ΠΏΡ€ΠΈΠΌΠ΅Ρ€ ΠΌΠΎΠΆΠ½ΠΎ Π·Π°ΠΏΠΈΡΠ°Ρ‚ΡŒ ΠΏΠΎΠΊΠΎΡ€ΠΎΡ‡Π΅:

    ΠŸΡ€ΠΈΠΌΠ΅Ρ€ 17. Найти Π·Π½Π°Ρ‡Π΅Π½ΠΈΠ΅ выраТСния?0,48 + (?2,7)

    Π­Ρ‚ΠΎ слоТСниС ΠΎΡ‚Ρ€ΠΈΡ†Π°Ρ‚Π΅Π»ΡŒΠ½Ρ‹Ρ… Ρ€Π°Ρ†ΠΈΠΎΠ½Π°Π»ΡŒΠ½Ρ‹Ρ… чисСл. Π‘Π»ΠΎΠΆΠΈΠΌ ΠΈΡ… ΠΌΠΎΠ΄ΡƒΠ»ΠΈ ΠΈ ΠΏΠ΅Ρ€Π΅Π΄ ΠΏΠΎΠ»ΡƒΡ‡Π΅Π½Π½Ρ‹ΠΌ ΠΎΡ‚Π²Π΅Ρ‚ΠΎΠΌ поставим Π·Π½Π°ΠΊ минус. Π—Π°ΠΏΠΈΡΡŒ с модулями ΠΌΠΎΠΆΠ½ΠΎ ΠΏΡ€ΠΎΠΏΡƒΡΡ‚ΠΈΡ‚ΡŒ, Ρ‡Ρ‚ΠΎΠ±Ρ‹ Π½Π΅ Π·Π°Π³Ρ€ΠΎΠΌΠΎΠΆΠ΄Π°Ρ‚ΡŒ Π²Ρ‹Ρ€Π°ΠΆΠ΅Π½ΠΈΠ΅:

    0,48 + (?2,7) = (?0,48) + (?2,70) = ?(0,48 + 2,70) = ?(3,18) = ?3,18

    ΠŸΡ€ΠΈΠΌΠ΅Ρ€ 18. Найти Π·Π½Π°Ρ‡Π΅Π½ΠΈΠ΅ выраТСния?4,9 ? 5,9

    Π—Π°ΠΊΠ»ΡŽΡ‡ΠΈΠΌ ΠΊΠ°ΠΆΠ΄ΠΎΠ΅ Ρ€Π°Ρ†ΠΈΠΎΠ½Π°Π»ΡŒΠ½ΠΎΠ΅ число Π² скобки вмСстС со своими Π·Π½Π°ΠΊΠ°ΠΌΠΈ. Π£Ρ‡ΠΈΡ‚Ρ‹Π²Π°Π΅ΠΌ, Ρ‡Ρ‚ΠΎ минус ΠΊΠΎΡ‚ΠΎΡ€Ρ‹ΠΉ Π΄Π°Π½ Π² Π²Ρ‹Ρ€Π°ΠΆΠ΅Π½ΠΈΠΈ, являСтся Π·Π½Π°ΠΊΠΎΠΌ ΠΎΠΏΠ΅Ρ€Π°Ρ†ΠΈΠΈ ΠΈ Π½Π΅ относится ΠΊ дСсятичной Π΄Ρ€ΠΎΠ±ΠΈ 5,9. Π£ этой дСсятичной Π΄Ρ€ΠΎΠ±ΠΈ свой Π·Π½Π°ΠΊ плюса, ΠΊΠΎΡ‚ΠΎΡ€Ρ‹ΠΉ Π½Π΅Π²ΠΈΠ΄ΠΈΠΌ ΠΏΠΎ ΠΏΡ€ΠΈΡ‡ΠΈΠ½Π΅ Ρ‚ΠΎΠ³ΠΎ, Ρ‡Ρ‚ΠΎ ΠΎΠ½ Π½Π΅ записываСтся. Но ΠΌΡ‹ запишСм Π΅Π³ΠΎ для наглядности:

    Π—Π°ΠΌΠ΅Π½ΠΈΠΌ Π²Ρ‹Ρ‡ΠΈΡ‚Π°Π½ΠΈΠ΅ слоТСниСм:

    ΠŸΠΎΠ»ΡƒΡ‡ΠΈΠ»ΠΈ слоТСниС ΠΎΡ‚Ρ€ΠΈΡ†Π°Ρ‚Π΅Π»ΡŒΠ½Ρ‹Ρ… Ρ€Π°Ρ†ΠΈΠΎΠ½Π°Π»ΡŒΠ½Ρ‹Ρ… чисСл. Π‘Π»ΠΎΠΆΠΈΡ‚ΡŒ ΠΈΡ… ΠΌΠΎΠ΄ΡƒΠ»ΠΈ ΠΈ ΠΏΠ΅Ρ€Π΅Π΄ ΠΏΠΎΠ»ΡƒΡ‡Π΅Π½Π½Ρ‹ΠΌ ΠΎΡ‚Π²Π΅Ρ‚ΠΎΠΌ поставим минус. Π—Π°ΠΏΠΈΡΡŒ с модулями ΠΌΠΎΠΆΠ½ΠΎ ΠΏΡ€ΠΎΠΏΡƒΡΡ‚ΠΈΡ‚ΡŒ, Ρ‡Ρ‚ΠΎΠ±Ρ‹ Π½Π΅ Π·Π°Π³Ρ€ΠΎΠΌΠΎΠΆΠ΄Π°Ρ‚ΡŒ Π²Ρ‹Ρ€Π°ΠΆΠ΅Π½ΠΈΠ΅:

    (?4,9) + (?5,9) = ?(4,9 + 5,9) = ?(10,8) = ?10,8

    Π’Π°ΠΊΠΈΠΌ ΠΎΠ±Ρ€Π°Π·ΠΎΠΌ, Π·Π½Π°Ρ‡Π΅Π½ΠΈΠ΅ выраТСния?4,9 ? 5,9 Ρ€Π°Π²Π½ΠΎ?10,8

    = ?(4,9 + 5,9) = ?(10,8) = ?10,8

    ΠŸΡ€ΠΈΠΌΠ΅Ρ€ 19. Найти Π·Π½Π°Ρ‡Π΅Π½ΠΈΠ΅ выраТСния 7 ? 9,3

    Π—Π°ΠΊΠ»ΡŽΡ‡ΠΈΠΌ Π² скобки ΠΊΠ°ΠΆΠ΄ΠΎΠ΅ число вмСстС со своим Π·Π½Π°ΠΊΠ°ΠΌΠΈ

    Π—Π°ΠΌΠ΅Π½ΠΈΠΌ Π²Ρ‹Ρ‡ΠΈΡ‚Π°Π½ΠΈΠ΅ слоТСниСм

    ΠŸΠΎΠ»ΡƒΡ‡ΠΈΠ»ΠΈ слоТСниС Ρ€Π°Ρ†ΠΈΠΎΠ½Π°Π»ΡŒΠ½Ρ‹Ρ… чисСл с Ρ€Π°Π·Π½Ρ‹ΠΌΠΈ Π·Π½Π°ΠΊΠ°ΠΌΠΈ. Π’Ρ‹Ρ‡Ρ‚Π΅ΠΌ ΠΈΠ· большСго модуля мСньший ΠΈ ΠΏΠ΅Ρ€Π΅Π΄ ΠΎΡ‚Π²Π΅Ρ‚ΠΎΠΌ поставим Ρ‚ΠΎΡ‚ Π·Π½Π°ΠΊ, ΠΌΠΎΠ΄ΡƒΠ»ΡŒ ΠΊΠΎΡ‚ΠΎΡ€ΠΎΠ³ΠΎ большС. Π—Π°ΠΏΠΈΡΡŒ с модулями ΠΌΠΎΠΆΠ½ΠΎ ΠΏΡ€ΠΎΠΏΡƒΡΡ‚ΠΈΡ‚ΡŒ, Ρ‡Ρ‚ΠΎΠ±Ρ‹ Π½Π΅ Π·Π°Π³Ρ€ΠΎΠΌΠΎΠΆΠ΄Π°Ρ‚ΡŒ Π²Ρ‹Ρ€Π°ΠΆΠ΅Π½ΠΈΠ΅:

    (+7) + (?9,3) = ?(9,3 ? 7) = ?(2,3) = ?2,3

    Π’Π°ΠΊΠΈΠΌ ΠΎΠ±Ρ€Π°Π·ΠΎΠΌ, Π·Π½Π°Ρ‡Π΅Π½ΠΈΠ΅ выраТСния 7 ? 9,3 Ρ€Π°Π²Π½ΠΎ?2,3

    ΠŸΠΎΠ΄Ρ€ΠΎΠ±Π½ΠΎΠ΅ Ρ€Π΅ΡˆΠ΅Π½ΠΈΠ΅ Π΄Π°Π½Π½ΠΎΠ³ΠΎ ΠΏΡ€ΠΈΠΌΠ΅Ρ€Π° записываСтся ΡΠ»Π΅Π΄ΡƒΡŽΡ‰ΠΈΠΌ ΠΎΠ±Ρ€Π°Π·ΠΎΠΌ:

    7 ? 9,3 = (+7) ? (+9,3) = (+7) + (?9,3) = ?(|?9,3| ? |+7|) =

    ΠšΠΎΡ€ΠΎΡ‚ΠΊΠΎΠ΅ Ρ€Π΅ΡˆΠ΅Π½ΠΈΠ΅ Π±ΡƒΠ΄Π΅Ρ‚ Π²Ρ‹Π³Π»ΡΠ΄Π΅Ρ‚ΡŒ Ρ‚Π°ΠΊ:

    ΠŸΡ€ΠΈΠΌΠ΅Ρ€ 20. Найти Π·Π½Π°Ρ‡Π΅Π½ΠΈΠ΅ выраТСния?0,25 ? (?1,2)

    Π—Π°ΠΌΠ΅Π½ΠΈΠΌ Π²Ρ‹Ρ‡ΠΈΡ‚Π°Π½ΠΈΠ΅ слоТСниСм:

    ΠŸΠΎΠ»ΡƒΡ‡ΠΈΠ»ΠΈ слоТСниС Ρ€Π°Ρ†ΠΈΠΎΠ½Π°Π»ΡŒΠ½Ρ‹Ρ… чисСл с Ρ€Π°Π·Π½Ρ‹ΠΌΠΈ Π·Π½Π°ΠΊΠ°ΠΌΠΈ. Π’Ρ‹Ρ‡Ρ‚Π΅ΠΌ ΠΈΠ· большСго модуля мСньший ΠΈ ΠΏΠ΅Ρ€Π΅Π΄ ΠΎΡ‚Π²Π΅Ρ‚ΠΎΠΌ поставим Ρ‚ΠΎΡ‚ Π·Π½Π°ΠΊ, ΠΌΠΎΠ΄ΡƒΠ»ΡŒ ΠΊΠΎΡ‚ΠΎΡ€ΠΎΠ³ΠΎ большС:

    0,25 + (+1,2) = |+1,2| ? |?0,25| = 1,2 ? 0,25 = 0,95

    ΠŸΠΎΠ΄Ρ€ΠΎΠ±Π½ΠΎΠ΅ Ρ€Π΅ΡˆΠ΅Π½ΠΈΠ΅ Π΄Π°Π½Π½ΠΎΠ³ΠΎ ΠΏΡ€ΠΈΠΌΠ΅Ρ€Π° записываСтся ΡΠ»Π΅Π΄ΡƒΡŽΡ‰ΠΈΠΌ ΠΎΠ±Ρ€Π°Π·ΠΎΠΌ:

    0,25 ? (?1,2) = (?0,25) + (+1,2) = |+1,2| ? |?0,25| = 1,2 ? 0,25 = 0,95

    ΠšΠΎΡ€ΠΎΡ‚ΠΊΠΎΠ΅ Ρ€Π΅ΡˆΠ΅Π½ΠΈΠ΅ Π±ΡƒΠ΄Π΅Ρ‚ Π²Ρ‹Π³Π»ΡΠ΄Π΅Ρ‚ΡŒ Ρ‚Π°ΠΊ:

    ΠŸΡ€ΠΈΠΌΠ΅Ρ€ 21. Найти Π·Π½Π°Ρ‡Π΅Π½ΠΈΠ΅ выраТСния?3,5 + (4,1 ? 7,1)

    Π’ ΠΏΠ΅Ρ€Π²ΡƒΡŽ ΠΎΡ‡Π΅Ρ€Π΅Π΄ΡŒ Π²Ρ‹ΠΏΠΎΠ»Π½ΠΈΠΌ дСйствия Π² скобках, Π·Π°Ρ‚Π΅ΠΌ слоТим ΠΏΠΎΠ»ΡƒΡ‡Π΅Π½Π½Ρ‹ΠΉ ΠΎΡ‚Π²Π΅Ρ‚ с числом?3,5. Π—Π°ΠΏΠΈΡΡŒ с модулями пропустим, Ρ‡Ρ‚ΠΎΠ±Ρ‹ Π½Π΅ Π·Π°Π³Ρ€ΠΎΠΌΠΎΠΆΠ΄Π°Ρ‚ΡŒ выраТСния.

    ΠŸΠ΅Ρ€Π²ΠΎΠ΅ дСйствиС:

    4,1 ? 7,1 = (+4,1) ? (+7,1) = (+4,1) + (?7,1) = ?(7,1 ? 4,1) = ?(3,0) = ?3,0

    Π’Ρ‚ΠΎΡ€ΠΎΠ΅ дСйствиС:

    3,5 + (?3,0) = ?(3,5 + 3,0) = ?(6,5) = ?6,5

    ΠžΡ‚Π²Π΅Ρ‚: Π·Π½Π°Ρ‡Π΅Π½ΠΈΠ΅ выраТСния?3,5 + (4,1 ? 7,1) Ρ€Π°Π²Π½ΠΎ?6,5.

    3,5 + (4,1 ? 7,1) = ?3,5 + (?3,0) = ?6,5

    ΠŸΡ€ΠΈΠΌΠ΅Ρ€ 22. Найти Π·Π½Π°Ρ‡Π΅Π½ΠΈΠ΅ выраТСния (3,5 ? 2,9) ? (3,7 ? 9,1)

    Π’Ρ‹ΠΏΠΎΠ»Π½ΠΈΠΌ дСйствия Π² скобках, Π·Π°Ρ‚Π΅ΠΌ ΠΈΠ· числа ΠΊΠΎΡ‚ΠΎΡ€ΠΎΠ΅ ΠΏΠΎΠ»ΡƒΡ‡ΠΈΠ»ΠΎΡΡŒ Π² Ρ€Π΅Π·ΡƒΠ»ΡŒΡ‚Π°Ρ‚Π΅ выполнСния ΠΏΠ΅Ρ€Π²Ρ‹Ρ… скобок Π²Ρ‹Ρ‡Ρ‚Π΅ΠΌ число, ΠΊΠΎΡ‚ΠΎΡ€ΠΎΠ΅ ΠΏΠΎΠ»ΡƒΡ‡ΠΈΠ»ΠΎΡΡŒ Π² Ρ€Π΅Π·ΡƒΠ»ΡŒΡ‚Π°Ρ‚Π΅ выполнСния Π²Ρ‚ΠΎΡ€Ρ‹Ρ… скобок. Π—Π°ΠΏΠΈΡΡŒ с модулями пропустим, Ρ‡Ρ‚ΠΎΠ±Ρ‹ Π½Π΅ Π·Π°Π³Ρ€ΠΎΠΌΠΎΠΆΠ΄Π°Ρ‚ΡŒ выраТСния.

    ΠŸΠ΅Ρ€Π²ΠΎΠ΅ дСйствиС:

    3,5 ? 2,9 = (+3,5) ? (+2,9) = (+3,5) + (?2,9) = 3,5 ? 2,9 = 0,6

    Π’Ρ‚ΠΎΡ€ΠΎΠ΅ дСйствиС:

    3,7 ? 9,1 = (+3,7) ? (+9,1) = (+3,7) + (?9,1) = ?(9,1 ? 3,7) = ?(5,4) = ?5,4

    Π’Ρ€Π΅Ρ‚ΡŒΠ΅ дСйствиС

    0,6 ? (?5,4) = (+0,6) + (+5,4) = 0,6 + 5,4 = 6,0 = 6

    ΠžΡ‚Π²Π΅Ρ‚: Π·Π½Π°Ρ‡Π΅Π½ΠΈΠ΅ выраТСния (3,5 ? 2,9) ? (3,7 ? 9,1) Ρ€Π°Π²Π½ΠΎ 6.

    ΠšΠΎΡ€ΠΎΡ‚ΠΊΠΎΠ΅ Ρ€Π΅ΡˆΠ΅Π½ΠΈΠ΅ Π΄Π°Π½Π½ΠΎΠ³ΠΎ ΠΏΡ€ΠΈΠΌΠ΅Ρ€Π° ΠΌΠΎΠΆΠ½ΠΎ Π·Π°ΠΏΠΈΡΠ°Ρ‚ΡŒ ΡΠ»Π΅Π΄ΡƒΡŽΡ‰ΠΈΠΌ ΠΎΠ±Ρ€Π°Π·ΠΎΠΌ:

    (3,5 ? 2,9) ? (3,7 ? 9,1) = 0,6 ? (?5,4) = 6,0 = 6

    ΠŸΡ€ΠΈΠΌΠ΅Ρ€ 23. Найти Π·Π½Π°Ρ‡Π΅Π½ΠΈΠ΅ выраТСния?3,8 + 17,15 ? 6,2 ? 6,15

    Π—Π°ΠΊΠ»ΡŽΡ‡ΠΈΠΌ Π² скобки ΠΊΠ°ΠΆΠ΄ΠΎΠ΅ Ρ€Π°Ρ†ΠΈΠΎΠ½Π°Π»ΡŒΠ½ΠΎΠ΅ число вмСстС со своими Π·Π½Π°ΠΊΠ°ΠΌΠΈ

    Π—Π°ΠΌΠ΅Π½ΠΈΠΌ Π²Ρ‹Ρ‡ΠΈΡ‚Π°Π½ΠΈΠ΅ слоТСниСм Ρ‚Π°ΠΌ, Π³Π΄Π΅ это ΠΌΠΎΠΆΠ½ΠΎ

    Π’Ρ‹Ρ€Π°ΠΆΠ΅Π½ΠΈΠ΅ состоит ΠΈΠ· Π½Π΅ΡΠΊΠΎΠ»ΡŒΠΊΠΈΡ… слагаСмых. Богласно ΡΠΎΡ‡Π΅Ρ‚Π°Ρ‚Π΅Π»ΡŒΠ½ΠΎΠΌΡƒ Π·Π°ΠΊΠΎΠ½Ρƒ слоТСния, Ссли Π²Ρ‹Ρ€Π°ΠΆΠ΅Π½ΠΈΠ΅ состоит ΠΈΠ· Π½Π΅ΡΠΊΠΎΠ»ΡŒΠΊΠΈΡ… слагаСмых, Ρ‚ΠΎ сумма Π½Π΅ Π±ΡƒΠ΄Π΅Ρ‚ Π·Π°Π²ΠΈΡΠ΅Ρ‚ΡŒ ΠΎΡ‚ порядка дСйствий. Π­Ρ‚ΠΎ Π·Π½Π°Ρ‡ΠΈΡ‚, Ρ‡Ρ‚ΠΎ слагаСмыС ΠΌΠΎΠΆΠ½ΠΎ ΡΠΊΠ»Π°Π΄Ρ‹Π²Π°Ρ‚ΡŒ Π² любом порядкС.

    НС Π±ΡƒΠ΄Π΅ΠΌ ΠΈΠ·ΠΎΠ±Ρ€Π΅Ρ‚Π°Ρ‚ΡŒ вСлосипСд, Π° слоТим всС слагаСмыС слСва Π½Π°ΠΏΡ€Π°Π²ΠΎ Π² порядкС ΠΈΡ… слСдования:

    ΠŸΠ΅Ρ€Π²ΠΎΠ΅ дСйствиС:

    (?3,8) + (+17,15) = 17,15 ? 3,80 = 13,35

    Π’Ρ‚ΠΎΡ€ΠΎΠ΅ дСйствиС:

    13,35 + (?6,2) = 13,35 ? ?6,20 = 7,15

    Π’Ρ€Π΅Ρ‚ΡŒΠ΅ дСйствиС:

    7,15 + (?6,15) = 7,15 ? 6,15 = 1,00 = 1

    ΠžΡ‚Π²Π΅Ρ‚: Π·Π½Π°Ρ‡Π΅Π½ΠΈΠ΅ выраТСния?3,8 + 17,15 ? 6,2 ? 6,15 Ρ€Π°Π²Π½ΠΎ 1.

    ΠšΠΎΡ€ΠΎΡ‚ΠΊΠΎΠ΅ Ρ€Π΅ΡˆΠ΅Π½ΠΈΠ΅ Π΄Π°Π½Π½ΠΎΠ³ΠΎ ΠΏΡ€ΠΈΠΌΠ΅Ρ€Π° ΠΌΠΎΠΆΠ½ΠΎ Π·Π°ΠΏΠΈΡΠ°Ρ‚ΡŒ ΡΠ»Π΅Π΄ΡƒΡŽΡ‰ΠΈΠΌ ΠΎΠ±Ρ€Π°Π·ΠΎΠΌ:

    3,8 + 17,15 ? 6,2 ? 6,15 = 13,35 + (?6,2) ? 6,15 = 7,15 ? 6,15 = 1,00 = 1

    ΠšΠΎΡ€ΠΎΡ‚ΠΊΠΈΠ΅ Ρ€Π΅ΡˆΠ΅Π½ΠΈΡ ΡΠΎΠ·Π΄Π°ΡŽΡ‚ мСньшС ΠΏΡ€ΠΎΠ±Π»Π΅ΠΌ ΠΈ ΠΏΡƒΡ‚Π°Π½ΠΈΡ†, поэтому ΠΆΠ΅Π»Π°Ρ‚Π΅Π»ΡŒΠ½ΠΎ ΠΏΡ€ΠΈΠ²Ρ‹ΠΊΠ½ΡƒΡ‚ΡŒ ΠΊ Π½ΠΈΠΌ.

    ΠŸΡ€ΠΈΠΌΠ΅Ρ€ 24. Найти Π·Π½Π°Ρ‡Π΅Π½ΠΈΠ΅ выраТСния

    ΠŸΠ΅Ρ€Π΅Π²Π΅Π΄Ρ‘ΠΌ Π΄Π΅ΡΡΡ‚ΠΈΡ‡Π½ΡƒΡŽ Π΄Ρ€ΠΎΠ±ΡŒ?1,8 Π² смСшанноС число. ΠžΡΡ‚Π°Π»ΡŒΠ½ΠΎΠ΅ ΠΏΠ΅Ρ€Π΅ΠΏΠΈΡˆΠ΅ΠΌ, ΠΊΠ°ΠΊ Π΅ΡΡ‚ΡŒ. Если испытываСтС затруднСния с ΠΏΠ΅Ρ€Π΅Π²ΠΎΠ΄ΠΎΠΌ дСсятичной Π΄Ρ€ΠΎΠ±ΠΈ Π² смСшанноС число, ΠΎΠ±ΡΠ·Π°Ρ‚Π΅Π»ΡŒΠ½ΠΎ ΠΏΠΎΠ²Ρ‚ΠΎΡ€ΠΈΡ‚Π΅ ΡƒΡ€ΠΎΠΊ дСсятичныС Π΄Ρ€ΠΎΠ±ΠΈ.

    ΠŸΡ€ΠΈΠΌΠ΅Ρ€ 25. Найти Π·Π½Π°Ρ‡Π΅Π½ΠΈΠ΅ выраТСния

    Π—Π°ΠΌΠ΅Π½ΠΈΠΌ Π²Ρ‹Ρ‡ΠΈΡ‚Π°Π½ΠΈΠ΅ слоТСниСм. ΠŸΠΎΠΏΡƒΡ‚Π½ΠΎ ΠΏΠ΅Ρ€Π΅Π²Π΅Π΄Ρ‘ΠΌ Π΄Π΅ΡΡΡ‚ΠΈΡ‡Π½ΡƒΡŽ Π΄Ρ€ΠΎΠ±ΡŒ (?4,4) Π² Π½Π΅ΠΏΡ€Π°Π²ΠΈΠ»ΡŒΠ½ΡƒΡŽ Π΄Ρ€ΠΎΠ±ΡŒ

    Π’ ΠΏΠΎΠ»ΡƒΡ‡ΠΈΠ²ΡˆΠ΅ΠΌΡΡ Π²Ρ‹Ρ€Π°ΠΆΠ΅Π½ΠΈΠΈ Π½Π΅Ρ‚ ΠΎΡ‚Ρ€ΠΈΡ†Π°Ρ‚Π΅Π»ΡŒΠ½Ρ‹Ρ… чисСл. А ΠΏΠΎΡΠΊΠΎΠ»ΡŒΠΊΡƒ Π½Π΅Ρ‚ ΠΎΡ‚Ρ€ΠΈΡ†Π°Ρ‚Π΅Π»ΡŒΠ½Ρ‹Ρ… чисСл, ΠΌΡ‹ ΠΌΠΎΠΆΠ΅ΠΌ ΡƒΠ±Ρ€Π°Ρ‚ΡŒ плюс ΠΏΠ΅Ρ€Π΅Π΄ Π²Ρ‚ΠΎΡ€Ρ‹ΠΌ числом, ΠΈ ΠΎΠΏΡƒΡΡ‚ΠΈΡ‚ΡŒ скобки. Π’ΠΎΠ³Π΄Π° ΠΏΠΎΠ»ΡƒΡ‡ΠΈΠΌ простоС Π²Ρ‹Ρ€Π°ΠΆΠ΅Π½ΠΈΠ΅ Π½Π° слоТСниС, ΠΊΠΎΡ‚ΠΎΡ€ΠΎΠ΅ Ρ€Π΅ΡˆΠ°Π΅Ρ‚ΡΡ Π»Π΅Π³ΠΊΠΎ

    ΠŸΡ€ΠΈΠΌΠ΅Ρ€ 26. Найти Π·Π½Π°Ρ‡Π΅Π½ΠΈΠ΅ выраТСния

    ΠŸΠ΅Ρ€Π΅Π²Π΅Π΄Ρ‘ΠΌ смСшанноС число Π² Π½Π΅ΠΏΡ€Π°Π²ΠΈΠ»ΡŒΠ½ΡƒΡŽ Π΄Ρ€ΠΎΠ±ΡŒ, Π° Π΄Π΅ΡΡΡ‚ΠΈΡ‡Π½ΡƒΡŽ Π΄Ρ€ΠΎΠ±ΡŒ?0,85 Π² ΠΎΠ±Ρ‹ΠΊΠ½ΠΎΠ²Π΅Π½Π½ΡƒΡŽ Π΄Ρ€ΠΎΠ±ΡŒ. ΠŸΠΎΠ»ΡƒΡ‡ΠΈΠΌ ΡΠ»Π΅Π΄ΡƒΡŽΡ‰Π΅Π΅ Π²Ρ‹Ρ€Π°ΠΆΠ΅Π½ΠΈΠ΅:

    ΠŸΠΎΠ»ΡƒΡ‡ΠΈΠ»ΠΈ слоТСниС ΠΎΡ‚Ρ€ΠΈΡ†Π°Ρ‚Π΅Π»ΡŒΠ½Ρ‹Ρ… Ρ€Π°Ρ†ΠΈΠΎΠ½Π°Π»ΡŒΠ½Ρ‹Ρ… чисСл. Π‘Π»ΠΎΠΆΠΈΠΌ ΠΈΡ… ΠΌΠΎΠ΄ΡƒΠ»ΠΈ ΠΈ ΠΏΠ΅Ρ€Π΅Π΄ ΠΏΠΎΠ»ΡƒΡ‡Π΅Π½Π½Ρ‹ΠΌ ΠΎΡ‚Π²Π΅Ρ‚ΠΎΠΌ поставим Π·Π½Π°ΠΊ минус. Π—Π°ΠΏΠΈΡΡŒ с модулями ΠΌΠΎΠΆΠ½ΠΎ ΠΏΡ€ΠΎΠΏΡƒΡΡ‚ΠΈΡ‚ΡŒ, Ρ‡Ρ‚ΠΎΠ±Ρ‹ Π½Π΅ Π·Π°Π³Ρ€ΠΎΠΌΠΎΠΆΠ΄Π°Ρ‚ΡŒ Π²Ρ‹Ρ€Π°ΠΆΠ΅Π½ΠΈΠ΅:

    ΠŸΡ€ΠΈΠΌΠ΅Ρ€ 27. Найти Π·Π½Π°Ρ‡Π΅Π½ΠΈΠ΅ выраТСния

    ΠŸΠ΅Ρ€Π΅Π²Π΅Π΄Ρ‘ΠΌ ΠΎΠ±Π΅ Π΄Ρ€ΠΎΠ±ΠΈ Π² Π½Π΅ΠΏΡ€Π°Π²ΠΈΠ»ΡŒΠ½Ρ‹Π΅ Π΄Ρ€ΠΎΠ±ΠΈ. Π§Ρ‚ΠΎΠ±Ρ‹ пСрСвСсти Π΄Π΅ΡΡΡ‚ΠΈΡ‡Π½ΡƒΡŽ Π΄Ρ€ΠΎΠ±ΡŒ 2,05 Π² Π½Π΅ΠΏΡ€Π°Π²ΠΈΠ»ΡŒΠ½ΡƒΡŽ Π΄Ρ€ΠΎΠ±ΡŒ, ΠΌΠΎΠΆΠ½ΠΎ пСрСвСсти Π΅Π΅ сначала Π² смСшанноС число, Π° Π·Π°Ρ‚Π΅ΠΌ Π² Π½Π΅ΠΏΡ€Π°Π²ΠΈΠ»ΡŒΠ½ΡƒΡŽ Π΄Ρ€ΠΎΠ±ΡŒ:

    ПослС ΠΏΠ΅Ρ€Π΅Π²ΠΎΠ΄Π° ΠΎΠ±Π΅ΠΈΡ… Π΄Ρ€ΠΎΠ±Π΅ΠΉ Π² Π½Π΅ΠΏΡ€Π°Π²ΠΈΠ»ΡŒΠ½Ρ‹Π΅ Π΄Ρ€ΠΎΠ±ΠΈ, ΠΏΠΎΠ»ΡƒΡ‡ΠΈΠΌ ΡΠ»Π΅Π΄ΡƒΡŽΡ‰Π΅Π΅ Π²Ρ‹Ρ€Π°ΠΆΠ΅Π½ΠΈΠ΅:

    ΠŸΠΎΠ»ΡƒΡ‡ΠΈΠ»ΠΈ слоТСниС Ρ€Π°Ρ†ΠΈΠΎΠ½Π°Π»ΡŒΠ½Ρ‹Ρ… чисСл с Ρ€Π°Π·Π½Ρ‹ΠΌΠΈ Π·Π½Π°ΠΊΠ°ΠΌΠΈ. Π’Ρ‹Ρ‡Ρ‚Π΅ΠΌ ΠΈΠ· большСго модуля мСньший ΠΈ ΠΏΠ΅Ρ€Π΅Π΄ ΠΏΠΎΠ»ΡƒΡ‡Π΅Π½Π½Ρ‹ΠΌ ΠΎΡ‚Π²Π΅Ρ‚ΠΎΠΌ поставим Ρ‚ΠΎΡ‚ Π·Π½Π°ΠΊ, ΠΌΠΎΠ΄ΡƒΠ»ΡŒ ΠΊΠΎΡ‚ΠΎΡ€ΠΎΠ³ΠΎ большС:

    ΠŸΡ€ΠΈΠΌΠ΅Ρ€ 28. Найти Π·Π½Π°Ρ‡Π΅Π½ΠΈΠ΅ выраТСния

    Π—Π°ΠΌΠ΅Π½ΠΈΠΌ Π²Ρ‹Ρ‡ΠΈΡ‚Π°Π½ΠΈΠ΅ слоТСниСм. ΠŸΠΎΠΏΡƒΡ‚Π½ΠΎ ΠΏΠ΅Ρ€Π΅Π²Π΅Π΄Ρ‘ΠΌ Π΄Π΅ΡΡΡ‚ΠΈΡ‡Π½ΡƒΡŽ Π΄Ρ€ΠΎΠ±ΡŒ Π² ΠΎΠ±Ρ‹ΠΊΠ½ΠΎΠ²Π΅Π½Π½ΡƒΡŽ Π΄Ρ€ΠΎΠ±ΡŒ

    ΠŸΡ€ΠΈΠΌΠ΅Ρ€ 29. Найти Π·Π½Π°Ρ‡Π΅Π½ΠΈΠ΅ выраТСния

    ΠŸΠ΅Ρ€Π΅Π²Π΅Π΄Ρ‘ΠΌ дСсятичныС Π΄Ρ€ΠΎΠ±ΠΈ?0,25 ΠΈ?1,25 Π² ΠΎΠ±Ρ‹ΠΊΠ½ΠΎΠ²Π΅Π½Π½Ρ‹Π΅ Π΄Ρ€ΠΎΠ±ΠΈ, ΠΎΡΡ‚Π°Π»ΡŒΠ½ΠΎΠ΅ оставим ΠΊΠ°ΠΊ Π΅ΡΡ‚ΡŒ. ΠŸΠΎΠ»ΡƒΡ‡ΠΈΠΌ ΡΠ»Π΅Π΄ΡƒΡŽΡ‰Π΅Π΅ Π²Ρ‹Ρ€Π°ΠΆΠ΅Π½ΠΈΠ΅:

    МоТно сначала Π·Π°ΠΌΠ΅Π½ΠΈΡ‚ΡŒ Π²Ρ‹Ρ‡ΠΈΡ‚Π°Π½ΠΈΠ΅ слоТСниСм Ρ‚Π°ΠΌ, Π³Π΄Π΅ это ΠΌΠΎΠΆΠ½ΠΎ ΠΈ ΡΠ»ΠΎΠΆΠΈΡ‚ΡŒ Ρ€Π°Ρ†ΠΈΠΎΠ½Π°Π»ΡŒΠ½Ρ‹Π΅ числа ΠΎΠ΄Π½ΠΎ Π·Π° Π΄Ρ€ΡƒΠ³ΠΈΠΌ. Π•ΡΡ‚ΡŒ ΠΈ Π²Ρ‚ΠΎΡ€ΠΎΠΉ Π²Π°Ρ€ΠΈΠ°Π½Ρ‚: сначала ΡΠ»ΠΎΠΆΠΈΡ‚ΡŒ Ρ€Π°Ρ†ΠΈΠΎΠ½Π°Π»ΡŒΠ½Ρ‹Π΅ числа ΠΈ , Π° Π·Π°Ρ‚Π΅ΠΌ ΠΈΠ· ΠΏΠΎΠ»ΡƒΡ‡Π΅Π½Π½ΠΎΠ³ΠΎ числа Π²Ρ‹Ρ‡Π΅ΡΡ‚ΡŒ Ρ€Π°Ρ†ΠΈΠΎΠ½Π°Π»ΡŒΠ½ΠΎΠ΅ число . Π­Ρ‚ΠΈΠΌ Π²Π°Ρ€ΠΈΠ°Π½Ρ‚ΠΎΠΌ ΠΈ Π²ΠΎΡΠΏΠΎΠ»ΡŒΠ·ΡƒΠ΅ΠΌΡΡ.

    ΠŸΠ΅Ρ€Π²ΠΎΠ΅ дСйствиС:

    Π’Ρ‚ΠΎΡ€ΠΎΠ΅ дСйствиС:

    ΠžΡ‚Π²Π΅Ρ‚: Π·Π½Π°Ρ‡Π΅Π½ΠΈΠ΅ выраТСния Ρ€Π°Π²Π½ΠΎ?2.

    ΠŸΡ€ΠΈΠΌΠ΅Ρ€ 30. Найти Π·Π½Π°Ρ‡Π΅Π½ΠΈΠ΅ выраТСния

    ΠŸΠ΅Ρ€Π΅Π²Π΅Π΄Ρ‘ΠΌ дСсятичныС Π΄Ρ€ΠΎΠ±ΠΈ Π² ΠΎΠ±Ρ‹ΠΊΠ½ΠΎΠ²Π΅Π½Π½Ρ‹Π΅. ΠžΡΡ‚Π°Π»ΡŒΠ½ΠΎΠ΅ оставим ΠΊΠ°ΠΊ Π΅ΡΡ‚ΡŒ

    ΠŸΠΎΠ»ΡƒΡ‡ΠΈΠ»ΠΈ сумму ΠΈΠ· Π½Π΅ΡΠΊΠΎΠ»ΡŒΠΊΠΈΡ… слагаСмых. Если сумма состоит ΠΈΠ· Π½Π΅ΡΠΊΠΎΠ»ΡŒΠΊΠΈΡ… слагаСмых, Ρ‚ΠΎ Π²Ρ‹Ρ€Π°ΠΆΠ΅Π½ΠΈΠ΅ ΠΌΠΎΠΆΠ½ΠΎ Π²Ρ‹Ρ‡ΠΈΡΠ»ΡΡ‚ΡŒ Π² любом порядкС. Π­Ρ‚ΠΎ слСдуСт ΠΈΠ· ΡΠΎΡ‡Π΅Ρ‚Π°Ρ‚Π΅Π»ΡŒΠ½ΠΎΠ³ΠΎ Π·Π°ΠΊΠΎΠ½Π° слоТСния.

    ΠŸΠΎΡΡ‚ΠΎΠΌΡƒ ΠΌΡ‹ ΠΌΠΎΠΆΠ΅ΠΌ ΠΎΡ€Π³Π°Π½ΠΈΠ·ΠΎΠ²Π°Ρ‚ΡŒ Π½Π°ΠΈΠ±ΠΎΠ»Π΅Π΅ ΡƒΠ΄ΠΎΠ±Π½Ρ‹ΠΉ для нас Π²Π°Ρ€ΠΈΠ°Π½Ρ‚. Π’ ΠΏΠ΅Ρ€Π²ΡƒΡŽ ΠΎΡ‡Π΅Ρ€Π΅Π΄ΡŒ ΠΌΠΎΠΆΠ½ΠΎ ΡΠ»ΠΎΠΆΠΈΡ‚ΡŒ ΠΏΠ΅Ρ€Π²ΠΎΠ΅ ΠΈ послСднСС слагаСмоС, Π° ΠΈΠΌΠ΅Π½Π½ΠΎ Ρ€Π°Ρ†ΠΈΠΎΠ½Π°Π»ΡŒΠ½Ρ‹Π΅ числа ΠΈ . Π£ этих чисСл ΠΎΠ΄ΠΈΠ½Π°ΠΊΠΎΠ²Ρ‹Π΅ Π·Π½Π°ΠΌΠ΅Π½Π°Ρ‚Π΅Π»ΠΈ, Π° Π·Π½Π°Ρ‡ΠΈΡ‚ это освободит нас ΠΎΡ‚ нСобходимости ΠΏΡ€ΠΈΠ²ΠΎΠ΄ΠΈΡ‚ΡŒ ΠΈΡ… ΠΊ Π½Π΅ΠΌΡƒ.

    ΠŸΠ΅Ρ€Π²ΠΎΠ΅ дСйствиС:

    ΠŸΠΎΠ»ΡƒΡ‡Π΅Π½Π½ΠΎΠ΅ число ΠΌΠΎΠΆΠ½ΠΎ ΡΠ»ΠΎΠΆΠΈΡ‚ΡŒ со Π²Ρ‚ΠΎΡ€Ρ‹ΠΌ слагаСмым, Π° ΠΈΠΌΠ΅Π½Π½ΠΎ с Ρ€Π°Ρ†ΠΈΠΎΠ½Π°Π»ΡŒΠ½Ρ‹ΠΌ числом . Π£ Ρ€Π°Ρ†ΠΈΠΎΠ½Π°Π»ΡŒΠ½Ρ‹Ρ… чисСл ΠΈ ΠΎΠ΄ΠΈΠ½Π°ΠΊΠΎΠ²Ρ‹Π΅ Π·Π½Π°ΠΌΠ΅Π½Π°Ρ‚Π΅Π»ΠΈ Π² Π΄Ρ€ΠΎΠ±Π½Ρ‹Ρ… частях, Ρ‡Ρ‚ΠΎ ΠΎΠΏΡΡ‚ΡŒ ΠΆΠ΅ являСтся прСимущСством для нас

    Π’Ρ‚ΠΎΡ€ΠΎΠ΅ дСйствиС:

    Ну ΠΈ слоТим ΠΏΠΎΠ»ΡƒΡ‡Π΅Π½Π½ΠΎΠ΅ число?7 с послСдним слагаСмым, Π° ΠΈΠΌΠ΅Π½Π½ΠΎ с Ρ€Π°Ρ†ΠΈΠΎΠ½Π°Π»ΡŒΠ½Ρ‹ΠΌ числом . Π£Π΄ΠΎΠ±Π½ΠΎ Ρ‚ΠΎ, Ρ‡Ρ‚ΠΎ ΠΏΡ€ΠΈ вычислСнии Π΄Π°Π½Π½ΠΎΠ³ΠΎ выраТСния, сСмёрки исчСзнут, Ρ‚ΠΎ Π΅ΡΡ‚ΡŒ ΠΈΡ… сумма Π±ΡƒΠ΄Π΅Ρ‚ Ρ€Π°Π²Π½Π° Π½ΡƒΠ»ΡŽ, ΠΏΠΎΡΠΊΠΎΠ»ΡŒΠΊΡƒ сумма ΠΏΡ€ΠΎΡ‚ΠΈΠ²ΠΎΠΏΠΎΠ»ΠΎΠΆΠ½Ρ‹Ρ… чисСл Ρ€Π°Π²Π½Π° Π½ΡƒΠ»ΡŽ

    Π’Ρ€Π΅Ρ‚ΡŒΠ΅ дСйствиС:

    ΠžΡ‚Π²Π΅Ρ‚: Π·Π½Π°Ρ‡Π΅Π½ΠΈΠ΅ выраТСния Ρ€Π°Π²Π½ΠΎ

    ΠŸΠΎΠ½Ρ€Π°Π²ΠΈΠ»ΡΡ ΡƒΡ€ΠΎΠΊ?
    Вступай Π² Π½Π°ΡˆΡƒ Π½ΠΎΠ²ΡƒΡŽ Π³Ρ€ΡƒΠΏΠΏΡƒ Π’ΠΊΠΎΠ½Ρ‚Π°ΠΊΡ‚Π΅ ΠΈ Π½Π°Ρ‡Π½ΠΈ ΠΏΠΎΠ»ΡƒΡ‡Π°Ρ‚ΡŒ увСдомлСния ΠΎ Π½ΠΎΠ²Ρ‹Ρ… ΡƒΡ€ΠΎΠΊΠ°Ρ…

    Π‘Π»ΠΎΠΆΠ΅Π½ΠΈΠ΅ ΠΈ Π²Ρ‹Ρ‡ΠΈΡ‚Π°Π½ΠΈΠ΅ Ρ†Π΅Π»Ρ‹Ρ… чисСл

    Π’ этом ΡƒΡ€ΠΎΠΊΠ΅ ΠΌΡ‹ ΠΈΠ·ΡƒΡ‡ΠΈΠΌ слоТСниС ΠΈ Π²Ρ‹Ρ‡ΠΈΡ‚Π°Π½ΠΈΠ΅ Ρ†Π΅Π»Ρ‹Ρ… чисСл , Π° Ρ‚Π°ΠΊΠΆΠ΅ ΠΏΡ€Π°Π²ΠΈΠ»Π° для ΠΈΡ… слоТСния ΠΈ вычитания.

    Напомним, Ρ‡Ρ‚ΠΎ Ρ†Π΅Π»Ρ‹Π΅ числа — это всС ΠΏΠΎΠ»ΠΎΠΆΠΈΡ‚Π΅Π»ΡŒΠ½Ρ‹Π΅ ΠΈ ΠΎΡ‚Ρ€ΠΈΡ†Π°Ρ‚Π΅Π»ΡŒΠ½Ρ‹Π΅ числа, Π° Ρ‚Π°ΠΊΠΆΠ΅ число 0. НапримСр, ΡΠ»Π΅Π΄ΡƒΡŽΡ‰ΠΈΠ΅ числа ΡΠ²Π»ΡΡŽΡ‚ΡΡ Ρ†Π΅Π»Ρ‹ΠΌΠΈ:

    ΠŸΠΎΠ»ΠΎΠΆΠΈΡ‚Π΅Π»ΡŒΠ½Ρ‹Π΅ числа Π»Π΅Π³ΠΊΠΎ ΡΠΊΠ»Π°Π΄Ρ‹Π²Π°ΡŽΡ‚ΡΡ ΠΈ Π²Ρ‹Ρ‡ΠΈΡ‚Π°ΡŽΡ‚ΡΡ, ΡƒΠΌΠ½ΠΎΠΆΠ°ΡŽΡ‚ΡΡ ΠΈ дСлятся. К соТалСнию, этого нСльзя ΡΠΊΠ°Π·Π°Ρ‚ΡŒ ΠΎΠ± ΠΎΡ‚Ρ€ΠΈΡ†Π°Ρ‚Π΅Π»ΡŒΠ½Ρ‹Ρ… числах, ΠΊΠΎΡ‚ΠΎΡ€Ρ‹Π΅ ΡΠΌΡƒΡ‰Π°ΡŽΡ‚ ΠΌΠ½ΠΎΠ³ΠΈΡ… Π½ΠΎΠ²ΠΈΡ‡ΠΊΠΎΠ² своими минусами ΠΏΠ΅Ρ€Π΅Π΄ ΠΊΠ°ΠΆΠ΄ΠΎΠΉ Ρ†ΠΈΡ„Ρ€ΠΎΠΉ. Как ΠΏΠΎΠΊΠ°Π·Ρ‹Π²Π°Π΅Ρ‚ ΠΏΡ€Π°ΠΊΡ‚ΠΈΠΊΠ°, ошибки сдСланныС ΠΈΠ·-Π·Π° ΠΎΡ‚Ρ€ΠΈΡ†Π°Ρ‚Π΅Π»ΡŒΠ½Ρ‹Ρ… чисСл, Ρ€Π°ΡΡΡ‚Ρ€Π°ΠΈΠ²Π°ΡŽΡ‚ ΠΎΠ±ΡƒΡ‡Π°ΡŽΡ‰ΠΈΡ…ΡΡ большС всСго.

    ΠŸΡ€ΠΈΠΌΠ΅Ρ€Ρ‹ слоТСния ΠΈ вычитания Ρ†Π΅Π»Ρ‹Ρ… чисСл

    ΠŸΠ΅Ρ€Π²ΠΎΠ΅ Ρ‡Π΅ΠΌΡƒ слСдуСт Π½Π°ΡƒΡ‡ΠΈΡ‚ΡŒΡΡ, это ΡΠΊΠ»Π°Π΄Ρ‹Π²Π°Ρ‚ΡŒ ΠΈ Π²Ρ‹Ρ‡ΠΈΡ‚Π°Ρ‚ΡŒ Ρ†Π΅Π»Ρ‹Π΅ числа с ΠΏΠΎΠΌΠΎΡ‰ΡŒΡŽ ΠΊΠΎΠΎΡ€Π΄ΠΈΠ½Π°Ρ‚Π½ΠΎΠΉ прямой. БовсСм Π½Π΅ΠΎΠ±ΡΠ·Π°Ρ‚Π΅Π»ΡŒΠ½ΠΎ Ρ€ΠΈΡΠΎΠ²Π°Ρ‚ΡŒ ΠΊΠΎΠΎΡ€Π΄ΠΈΠ½Π°Ρ‚Π½ΡƒΡŽ ΠΏΡ€ΡΠΌΡƒΡŽ. Достаточно Π²ΠΎΠΎΠ±Ρ€Π°ΠΆΠ°Ρ‚ΡŒ Π΅Ρ‘ Π² своих мыслях ΠΈ Π²ΠΈΠ΄Π΅Ρ‚ΡŒ, Π³Π΄Π΅ Ρ€Π°ΡΠΏΠΎΠ»Π°Π³Π°ΡŽΡ‚ΡΡ ΠΎΡ‚Ρ€ΠΈΡ†Π°Ρ‚Π΅Π»ΡŒΠ½Ρ‹Π΅ числа, Π° Π³Π΄Π΅ ΠΏΠΎΠ»ΠΎΠΆΠΈΡ‚Π΅Π»ΡŒΠ½Ρ‹Π΅.

    Рассмотрим ΠΏΡ€ΠΎΡΡ‚Π΅ΠΉΡˆΠ΅Π΅ Π²Ρ‹Ρ€Π°ΠΆΠ΅Π½ΠΈΠ΅: 1 + 3. Π—Π½Π°Ρ‡Π΅Π½ΠΈΠ΅ Π΄Π°Π½Π½ΠΎΠ³ΠΎ выраТСния Ρ€Π°Π²Π½ΠΎ 4:

    Π­Ρ‚ΠΎΡ‚ ΠΏΡ€ΠΈΠΌΠ΅Ρ€ ΠΌΠΎΠΆΠ½ΠΎ ΠΏΠΎΠ½ΡΡ‚ΡŒ с ΠΏΠΎΠΌΠΎΡ‰ΡŒΡŽ ΠΊΠΎΠΎΡ€Π΄ΠΈΠ½Π°Ρ‚Π½ΠΎΠΉ прямой. Для этого ΠΈΠ· Ρ‚ΠΎΡ‡ΠΊΠΈ, Π³Π΄Π΅ располагаСтся число 1, Π½ΡƒΠΆΠ½ΠΎ ΡΠ΄Π²ΠΈΠ½ΡƒΡ‚ΡŒΡΡ Π²ΠΏΡ€Π°Π²ΠΎ Π½Π° Ρ‚Ρ€ΠΈ шага. Π’ Ρ€Π΅Π·ΡƒΠ»ΡŒΡ‚Π°Ρ‚Π΅, ΠΌΡ‹ окаТСмся Π² Ρ‚ΠΎΡ‡ΠΊΠ΅, Π³Π΄Π΅ располагаСтся число 4. На рисункС ΠΌΠΎΠΆΠ½ΠΎ ΡƒΠ²ΠΈΠ΄Π΅Ρ‚ΡŒ ΠΊΠ°ΠΊ это происходит:

    Π—Π½Π°ΠΊ плюса Π² Π²Ρ‹Ρ€Π°ΠΆΠ΅Π½ΠΈΠΈ 1 + 3 ΡƒΠΊΠ°Π·Ρ‹Π²Π°Π΅Ρ‚ Π½Π°ΠΌ, Ρ‡Ρ‚ΠΎ ΠΌΡ‹ Π΄ΠΎΠ»ΠΆΠ½Ρ‹ Π΄Π²ΠΈΠ³Π°Ρ‚ΡŒΡΡ Π²ΠΏΡ€Π°Π²ΠΎ Π² сторону увСличСния чисСл.

    ΠŸΡ€ΠΈΠΌΠ΅Ρ€ 2. Найдём Π·Π½Π°Ρ‡Π΅Π½ΠΈΠ΅ выраТСния 1 ? 3.

    Π—Π½Π°Ρ‡Π΅Π½ΠΈΠ΅ Π΄Π°Π½Π½ΠΎΠ³ΠΎ выраТСния Ρ€Π°Π²Π½ΠΎ?2

    Π­Ρ‚ΠΎΡ‚ ΠΏΡ€ΠΈΠΌΠ΅Ρ€ ΠΎΠΏΡΡ‚ΡŒ ΠΆΠ΅ ΠΌΠΎΠΆΠ½ΠΎ ΠΏΠΎΠ½ΡΡ‚ΡŒ с ΠΏΠΎΠΌΠΎΡ‰ΡŒΡŽ ΠΊΠΎΠΎΡ€Π΄ΠΈΠ½Π°Ρ‚Π½ΠΎΠΉ прямой. Для этого ΠΈΠ· Ρ‚ΠΎΡ‡ΠΊΠΈ, Π³Π΄Π΅ располагаСтся число 1 Π½ΡƒΠΆΠ½ΠΎ ΡΠ΄Π²ΠΈΠ½ΡƒΡ‚ΡŒΡΡ Π²Π»Π΅Π²ΠΎ Π½Π° Ρ‚Ρ€ΠΈ шага. Π’ Ρ€Π΅Π·ΡƒΠ»ΡŒΡ‚Π°Ρ‚Π΅ ΠΌΡ‹ окаТСмся Π² Ρ‚ΠΎΡ‡ΠΊΠ΅, Π³Π΄Π΅ располагаСтся ΠΎΡ‚Ρ€ΠΈΡ†Π°Ρ‚Π΅Π»ΡŒΠ½ΠΎΠ΅ число?2. На рисункС ΠΌΠΎΠΆΠ½ΠΎ ΡƒΠ²ΠΈΠ΄Π΅Ρ‚ΡŒ, ΠΊΠ°ΠΊ это происходит:

    Π—Π½Π°ΠΊ минуса Π² Π²Ρ‹Ρ€Π°ΠΆΠ΅Π½ΠΈΠΈ 1 ? 3 ΡƒΠΊΠ°Π·Ρ‹Π²Π°Π΅Ρ‚ Π½Π°ΠΌ, Ρ‡Ρ‚ΠΎ ΠΌΡ‹ Π΄ΠΎΠ»ΠΆΠ½Ρ‹ Π΄Π²ΠΈΠ³Π°Ρ‚ΡŒΡΡ Π²Π»Π΅Π²ΠΎ Π² сторону ΡƒΠΌΠ΅Π½ΡŒΡˆΠ΅Π½ΠΈΡ чисСл.

    Π’ΠΎΠΎΠ±Ρ‰Π΅, Π½Π°Π΄ΠΎ Π·Π°ΠΏΠΎΠΌΠ½ΠΈΡ‚ΡŒ, Ρ‡Ρ‚ΠΎ Ссли осущСствляСтся слоТСниС, Ρ‚ΠΎ Π½ΡƒΠΆΠ½ΠΎ Π΄Π²ΠΈΠ³Π°Ρ‚ΡŒΡΡ Π²ΠΏΡ€Π°Π²ΠΎ Π² сторону увСличСния. Если ΠΆΠ΅ осущСствляСтся Π²Ρ‹Ρ‡ΠΈΡ‚Π°Π½ΠΈΠ΅, Ρ‚ΠΎ Π½ΡƒΠΆΠ½ΠΎ Π΄Π²ΠΈΠ³Π°Ρ‚ΡŒΡΡ Π²Π»Π΅Π²ΠΎ Π² сторону ΡƒΠΌΠ΅Π½ΡŒΡˆΠ΅Π½ΠΈΡ.

    ΠŸΡ€ΠΈΠΌΠ΅Ρ€ 3. Найти Π·Π½Π°Ρ‡Π΅Π½ΠΈΠ΅ выраТСния?2 + 4

    Π—Π½Π°Ρ‡Π΅Π½ΠΈΠ΅ Π΄Π°Π½Π½ΠΎΠ³ΠΎ выраТСния Ρ€Π°Π²Π½ΠΎ 2

    Π­Ρ‚ΠΎΡ‚ ΠΏΡ€ΠΈΠΌΠ΅Ρ€ ΠΎΠΏΡΡ‚ΡŒ ΠΆΠ΅ ΠΌΠΎΠΆΠ½ΠΎ ΠΏΠΎΠ½ΡΡ‚ΡŒ с ΠΏΠΎΠΌΠΎΡ‰ΡŒΡŽ ΠΊΠΎΠΎΡ€Π΄ΠΈΠ½Π°Ρ‚Π½ΠΎΠΉ прямой. Для этого ΠΈΠ· Ρ‚ΠΎΡ‡ΠΊΠΈ, Π³Π΄Π΅ располагаСтся ΠΎΡ‚Ρ€ΠΈΡ†Π°Ρ‚Π΅Π»ΡŒΠ½ΠΎΠ΅ число?2 Π½ΡƒΠΆΠ½ΠΎ ΡΠ΄Π²ΠΈΠ½ΡƒΡ‚ΡŒΡΡ Π²ΠΏΡ€Π°Π²ΠΎ Π½Π° Ρ‡Π΅Ρ‚Ρ‹Ρ€Π΅ шага. Π’ Ρ€Π΅Π·ΡƒΠ»ΡŒΡ‚Π°Ρ‚Π΅ ΠΌΡ‹ окаТСмся Π² Ρ‚ΠΎΡ‡ΠΊΠ΅, Π³Π΄Π΅ располагаСтся ΠΏΠΎΠ»ΠΎΠΆΠΈΡ‚Π΅Π»ΡŒΠ½ΠΎΠ΅ число 2.

    Π’ΠΈΠ΄Π½ΠΎ, Ρ‡Ρ‚ΠΎ ΠΌΡ‹ ΡΠ΄Π²ΠΈΠ½ΡƒΠ»ΠΈΡΡŒ ΠΈΠ· Ρ‚ΠΎΡ‡ΠΊΠΈ Π³Π΄Π΅ располагаСтся ΠΎΡ‚Ρ€ΠΈΡ†Π°Ρ‚Π΅Π»ΡŒΠ½ΠΎΠ΅ число?2 Π² ΠΏΡ€Π°Π²ΡƒΡŽ сторону Π½Π° Ρ‡Π΅Ρ‚Ρ‹Ρ€Π΅ шага ΠΈ оказались Π² Ρ‚ΠΎΡ‡ΠΊΠ΅, Π³Π΄Π΅ располагаСтся ΠΏΠΎΠ»ΠΎΠΆΠΈΡ‚Π΅Π»ΡŒΠ½ΠΎΠ΅ число 2.

    Π—Π½Π°ΠΊ плюса Π² Π²Ρ‹Ρ€Π°ΠΆΠ΅Π½ΠΈΠΈ?2 + 4 ΡƒΠΊΠ°Π·Ρ‹Π²Π°Π΅Ρ‚ Π½Π°ΠΌ, Ρ‡Ρ‚ΠΎ ΠΌΡ‹ Π΄ΠΎΠ»ΠΆΠ½Ρ‹ Π΄Π²ΠΈΠ³Π°Ρ‚ΡŒΡΡ Π²ΠΏΡ€Π°Π²ΠΎ Π² сторону увСличСния чисСл.

    ΠŸΡ€ΠΈΠΌΠ΅Ρ€ 4. Найти Π·Π½Π°Ρ‡Π΅Π½ΠΈΠ΅ выраТСния?1 ? 3

    Π—Π½Π°Ρ‡Π΅Π½ΠΈΠ΅ Π΄Π°Π½Π½ΠΎΠ³ΠΎ выраТСния Ρ€Π°Π²Π½ΠΎ?4

    Π­Ρ‚ΠΎΡ‚ ΠΏΡ€ΠΈΠΌΠ΅Ρ€ ΠΎΠΏΡΡ‚ΡŒ ΠΆΠ΅ ΠΌΠΎΠΆΠ½ΠΎ Ρ€Π΅ΡˆΠΈΡ‚ΡŒ с ΠΏΠΎΠΌΠΎΡ‰ΡŒΡŽ ΠΊΠΎΠΎΡ€Π΄ΠΈΠ½Π°Ρ‚Π½ΠΎΠΉ прямой. Для этого ΠΈΠ· Ρ‚ΠΎΡ‡ΠΊΠΈ, Π³Π΄Π΅ располагаСтся ΠΎΡ‚Ρ€ΠΈΡ†Π°Ρ‚Π΅Π»ΡŒΠ½ΠΎΠ΅ число?1 Π½ΡƒΠΆΠ½ΠΎ ΡΠ΄Π²ΠΈΠ½ΡƒΡ‚ΡŒΡΡ Π²Π»Π΅Π²ΠΎ Π½Π° Ρ‚Ρ€ΠΈ шага. Π’ Ρ€Π΅Π·ΡƒΠ»ΡŒΡ‚Π°Ρ‚Π΅ ΠΌΡ‹ окаТСмся Π² Ρ‚ΠΎΡ‡ΠΊΠ΅, Π³Π΄Π΅ располагаСтся ΠΎΡ‚Ρ€ΠΈΡ†Π°Ρ‚Π΅Π»ΡŒΠ½ΠΎΠ΅ число?4

    Π’ΠΈΠ΄Π½ΠΎ, Ρ‡Ρ‚ΠΎ ΠΌΡ‹ ΡΠ΄Π²ΠΈΠ½ΡƒΠ»ΠΈΡΡŒ ΠΈΠ· Ρ‚ΠΎΡ‡ΠΊΠΈ Π³Π΄Π΅ располагаСтся ΠΎΡ‚Ρ€ΠΈΡ†Π°Ρ‚Π΅Π»ΡŒΠ½ΠΎΠ΅ число?1 Π² Π»Π΅Π²ΡƒΡŽ сторону Π½Π° Ρ‚Ρ€ΠΈ шага ΠΈ оказались Π² Ρ‚ΠΎΡ‡ΠΊΠ΅, Π³Π΄Π΅ располагаСтся ΠΎΡ‚Ρ€ΠΈΡ†Π°Ρ‚Π΅Π»ΡŒΠ½ΠΎΠ΅ число?4.

    Π—Π½Π°ΠΊ минуса Π² Π²Ρ‹Ρ€Π°ΠΆΠ΅Π½ΠΈΠΈ?1 ? 3 ΡƒΠΊΠ°Π·Ρ‹Π²Π°Π΅Ρ‚ Π½Π°ΠΌ, Ρ‡Ρ‚ΠΎ ΠΌΡ‹ Π΄ΠΎΠ»ΠΆΠ½Ρ‹ Π΄Π²ΠΈΠ³Π°Ρ‚ΡŒΡΡ Π²Π»Π΅Π²ΠΎ Π² сторону ΡƒΠΌΠ΅Π½ΡŒΡˆΠ΅Π½ΠΈΡ чисСл.

    ΠŸΡ€ΠΈΠΌΠ΅Ρ€ 5. Найти Π·Π½Π°Ρ‡Π΅Π½ΠΈΠ΅ выраТСния?2 + 2

    Π—Π½Π°Ρ‡Π΅Π½ΠΈΠ΅ Π΄Π°Π½Π½ΠΎΠ³ΠΎ выраТСния Ρ€Π°Π²Π½ΠΎ 0

    Π­Ρ‚ΠΎΡ‚ ΠΏΡ€ΠΈΠΌΠ΅Ρ€ ΠΌΠΎΠΆΠ½ΠΎ Ρ€Π΅ΡˆΠΈΡ‚ΡŒ с ΠΏΠΎΠΌΠΎΡ‰ΡŒΡŽ ΠΊΠΎΠΎΡ€Π΄ΠΈΠ½Π°Ρ‚Π½ΠΎΠΉ прямой. Для этого ΠΈΠ· Ρ‚ΠΎΡ‡ΠΊΠΈ, Π³Π΄Π΅ располагаСтся ΠΎΡ‚Ρ€ΠΈΡ†Π°Ρ‚Π΅Π»ΡŒΠ½ΠΎΠ΅ число?2 Π½ΡƒΠΆΠ½ΠΎ ΡΠ΄Π²ΠΈΠ½ΡƒΡ‚ΡŒΡΡ Π²ΠΏΡ€Π°Π²ΠΎ Π½Π° Π΄Π²Π° шага. Π’ Ρ€Π΅Π·ΡƒΠ»ΡŒΡ‚Π°Ρ‚Π΅ ΠΌΡ‹ окаТСмся Π² Ρ‚ΠΎΡ‡ΠΊΠ΅, Π³Π΄Π΅ располагаСтся число 0

    Π’ΠΈΠ΄Π½ΠΎ, Ρ‡Ρ‚ΠΎ ΠΌΡ‹ ΡΠ΄Π²ΠΈΠ½ΡƒΠ»ΠΈΡΡŒ ΠΈΠ· Ρ‚ΠΎΡ‡ΠΊΠΈ Π³Π΄Π΅ располагаСтся ΠΎΡ‚Ρ€ΠΈΡ†Π°Ρ‚Π΅Π»ΡŒΠ½ΠΎΠ΅ число?2 Π² ΠΏΡ€Π°Π²ΡƒΡŽ сторону Π½Π° Π΄Π²Π° шага ΠΈ оказались Π² Ρ‚ΠΎΡ‡ΠΊΠ΅, Π³Π΄Π΅ располагаСтся число 0.

    Π—Π½Π°ΠΊ плюса Π² Π²Ρ‹Ρ€Π°ΠΆΠ΅Π½ΠΈΠΈ?2 + 2 ΡƒΠΊΠ°Π·Ρ‹Π²Π°Π΅Ρ‚ Π½Π°ΠΌ, Ρ‡Ρ‚ΠΎ ΠΌΡ‹ Π΄ΠΎΠ»ΠΆΠ½Ρ‹ Π΄Π²ΠΈΠ³Π°Ρ‚ΡŒΡΡ Π²ΠΏΡ€Π°Π²ΠΎ Π² сторону увСличСния чисСл.

    ΠŸΡ€Π°Π²ΠΈΠ»Π° слоТСния ΠΈ вычитания Ρ†Π΅Π»Ρ‹Ρ… чисСл

    Π§Ρ‚ΠΎΠ±Ρ‹ Π²Ρ‹Ρ‡ΠΈΡΠ»ΠΈΡ‚ΡŒ Ρ‚ΠΎ ΠΈΠ»ΠΈ ΠΈΠ½ΠΎΠ΅ Π²Ρ‹Ρ€Π°ΠΆΠ΅Π½ΠΈΠ΅, Π½Π΅ΠΎΠ±ΡΠ·Π°Ρ‚Π΅Π»ΡŒΠ½ΠΎ ΠΊΠ°ΠΆΠ΄Ρ‹ΠΉ Ρ€Π°Π· Π²ΠΎΠΎΠ±Ρ€Π°ΠΆΠ°Ρ‚ΡŒ ΠΊΠΎΠΎΡ€Π΄ΠΈΠ½Π°Ρ‚Π½ΡƒΡŽ ΠΏΡ€ΡΠΌΡƒΡŽ, ΠΈ Ρ‚Π΅ΠΌ Π±ΠΎΠ»Π΅Π΅ Ρ€ΠΈΡΠΎΠ²Π°Ρ‚ΡŒ Π΅Ρ‘. Π£Π΄ΠΎΠ±Π½Π΅Π΅ Π²ΠΎΡΠΏΠΎΠ»ΡŒΠ·ΠΎΠ²Π°Ρ‚ΡŒΡΡ Π³ΠΎΡ‚ΠΎΠ²Ρ‹ΠΌΠΈ ΠΏΡ€Π°Π²ΠΈΠ»Π°ΠΌΠΈ.

    ΠŸΡ€ΠΈΠΌΠ΅Π½ΡΡ ΠΏΡ€Π°Π²ΠΈΠ»Π°, Π½ΡƒΠΆΠ½ΠΎ ΠΎΠ±Ρ€Π°Ρ‰Π°Ρ‚ΡŒ внимания Π½Π° Π·Π½Π°ΠΊ ΠΎΠΏΠ΅Ρ€Π°Ρ†ΠΈΠΈ ΠΈ Π·Π½Π°ΠΊΠΈ чисСл, ΠΊΠΎΡ‚ΠΎΡ€Ρ‹Π΅ Π½ΡƒΠΆΠ½ΠΎ ΡΠ»ΠΎΠΆΠΈΡ‚ΡŒ ΠΈΠ»ΠΈ Π²Ρ‹Ρ‡Π΅ΡΡ‚ΡŒ. ΠžΡ‚ этого Π±ΡƒΠ΄Π΅Ρ‚ Π·Π°Π²ΠΈΡΠ΅Ρ‚ΡŒ ΠΊΠ°ΠΊΠΎΠ΅ ΠΏΡ€Π°Π²ΠΈΠ»ΠΎ ΠΏΡ€ΠΈΠΌΠ΅Π½ΡΡ‚ΡŒ.

    ΠŸΡ€ΠΈΠΌΠ΅Ρ€ 1. Найти Π·Π½Π°Ρ‡Π΅Π½ΠΈΠ΅ выраТСния?2 + 5

    Π—Π΄Π΅ΡΡŒ ΠΊ ΠΎΡ‚Ρ€ΠΈΡ†Π°Ρ‚Π΅Π»ΡŒΠ½ΠΎΠΌΡƒ числу прибавляСтся ΠΏΠΎΠ»ΠΎΠΆΠΈΡ‚Π΅Π»ΡŒΠ½ΠΎΠ΅ число. Π”Ρ€ΡƒΠ³ΠΈΠΌΠΈ словами, осущСствляСтся слоТСниС чисСл с Ρ€Π°Π·Π½Ρ‹ΠΌΠΈ Π·Π½Π°ΠΊΠ°ΠΌΠΈ. ?2 это ΠΎΡ‚Ρ€ΠΈΡ†Π°Ρ‚Π΅Π»ΡŒΠ½ΠΎΠ΅ число, Π° 5 — ΠΏΠΎΠ»ΠΎΠΆΠΈΡ‚Π΅Π»ΡŒΠ½ΠΎΠ΅. Для Ρ‚Π°ΠΊΠΈΡ… случаСв прСдусмотрСно ΡΠ»Π΅Π΄ΡƒΡŽΡ‰Π΅Π΅ ΠΏΡ€Π°Π²ΠΈΠ»ΠΎ:

    Π˜Ρ‚Π°ΠΊ, посмотрим ΠΊΠ°ΠΊΠΎΠΉ ΠΌΠΎΠ΄ΡƒΠ»ΡŒ большС:

    ΠœΠΎΠ΄ΡƒΠ»ΡŒ числа 5 большС, Ρ‡Π΅ΠΌ ΠΌΠΎΠ΄ΡƒΠ»ΡŒ числа?2. ΠŸΡ€Π°Π²ΠΈΠ»ΠΎ Ρ‚Ρ€Π΅Π±ΡƒΠ΅Ρ‚ ΠΈΠ· большСго модуля Π²Ρ‹Ρ‡Π΅ΡΡ‚ΡŒ мСньший. ΠŸΠΎΡΡ‚ΠΎΠΌΡƒ, ΠΌΡ‹ Π΄ΠΎΠ»ΠΆΠ½Ρ‹ ΠΈΠ· 5 Π²Ρ‹Ρ‡Π΅ΡΡ‚ΡŒ 2, ΠΈ ΠΏΠ΅Ρ€Π΅Π΄ ΠΏΠΎΠ»ΡƒΡ‡Π΅Π½Π½Ρ‹ΠΌ ΠΎΡ‚Π²Π΅Ρ‚ΠΎΠΌ ΠΏΠΎΡΡ‚Π°Π²ΠΈΡ‚ΡŒ Ρ‚ΠΎΡ‚ Π·Π½Π°ΠΊ, ΠΌΠΎΠ΄ΡƒΠ»ΡŒ ΠΊΠΎΡ‚ΠΎΡ€ΠΎΠ³ΠΎ большС.

    Π£ числа 5 ΠΌΠΎΠ΄ΡƒΠ»ΡŒ большС, поэтому Π·Π½Π°ΠΊ этого числа ΠΈ Π±ΡƒΠ΄Π΅Ρ‚ Π² ΠΎΡ‚Π²Π΅Ρ‚Π΅. Π’ΠΎ Π΅ΡΡ‚ΡŒ, ΠΎΡ‚Π²Π΅Ρ‚ Π±ΡƒΠ΄Π΅Ρ‚ ΠΏΠΎΠ»ΠΎΠΆΠΈΡ‚Π΅Π»ΡŒΠ½Ρ‹ΠΌ:

    ΠžΠ±Ρ‹Ρ‡Π½ΠΎ Π·Π°ΠΏΠΈΡΡ‹Π²Π°ΡŽΡ‚ ΠΏΠΎΠΊΠΎΡ€ΠΎΡ‡Π΅?2 + 5 = 3

    ΠŸΡ€ΠΈΠΌΠ΅Ρ€ 2. Найти Π·Π½Π°Ρ‡Π΅Π½ΠΈΠ΅ выраТСния 3 + (?2)

    Π—Π΄Π΅ΡΡŒ ΠΊΠ°ΠΊ ΠΈ Π² ΠΏΡ€Π΅Π΄Ρ‹Π΄ΡƒΡ‰Π΅ΠΌ ΠΏΡ€ΠΈΠΌΠ΅Ρ€Π΅, осущСствляСтся слоТСниС чисСл с Ρ€Π°Π·Π½Ρ‹ΠΌΠΈ Π·Π½Π°ΠΊΠ°ΠΌΠΈ. 3 — это ΠΏΠΎΠ»ΠΎΠΆΠΈΡ‚Π΅Π»ΡŒΠ½ΠΎΠ΅ число, Π°?2 — ΠΎΡ‚Ρ€ΠΈΡ†Π°Ρ‚Π΅Π»ΡŒΠ½ΠΎΠ΅. ΠžΠ±Ρ€Π°Ρ‚ΠΈΡ‚Π΅ Π²Π½ΠΈΠΌΠ°Π½ΠΈΠ΅, Ρ‡Ρ‚ΠΎ число?2 Π·Π°ΠΊΠ»ΡŽΡ‡Π΅Π½ΠΎ Π² скобки, Ρ‡Ρ‚ΠΎΠ±Ρ‹ ΡΠ΄Π΅Π»Π°Ρ‚ΡŒ Π²Ρ‹Ρ€Π°ΠΆΠ΅Π½ΠΈΠ΅ понятнСС ΠΈ красивСС. Π­Ρ‚ΠΎ Π²Ρ‹Ρ€Π°ΠΆΠ΅Π½ΠΈΠ΅ Π½Π°ΠΌΠ½ΠΎΠ³ΠΎ ΠΏΡ€ΠΎΡ‰Π΅ для восприятия, Ρ‡Π΅ΠΌ Π²Ρ‹Ρ€Π°ΠΆΠ΅Π½ΠΈΠ΅ 3+?2.

    Π˜Ρ‚Π°ΠΊ, ΠΏΡ€ΠΈΠΌΠ΅Π½ΠΈΠΌ ΠΏΡ€Π°Π²ΠΈΠ»ΠΎ слоТСния чисСл с Ρ€Π°Π·Π½Ρ‹ΠΌΠΈ Π·Π½Π°ΠΊΠ°ΠΌΠΈ. Как ΠΈ Π² ΠΏΡ€ΠΎΡˆΠ»ΠΎΠΌ ΠΏΡ€ΠΈΠΌΠ΅Ρ€Π΅, ΠΈΠ· большСго модуля Π²Ρ‹Ρ‡ΠΈΡ‚Π°Π΅ΠΌ мСньший ΠΌΠΎΠ΄ΡƒΠ»ΡŒ ΠΈ ΠΏΠ΅Ρ€Π΅Π΄ ΠΎΡ‚Π²Π΅Ρ‚ΠΎΠΌ ставим Ρ‚ΠΎΡ‚ Π·Π½Π°ΠΊ, ΠΌΠΎΠ΄ΡƒΠ»ΡŒ ΠΊΠΎΡ‚ΠΎΡ€ΠΎΠ³ΠΎ большС:

    3 + (?2) = |3| ? |?2| = 3 ? 2 = 1

    ΠœΠΎΠ΄ΡƒΠ»ΡŒ числа 3 большС, Ρ‡Π΅ΠΌ ΠΌΠΎΠ΄ΡƒΠ»ΡŒ числа?2, поэтому ΠΌΡ‹ ΠΈΠ· 3 Π²Ρ‹Ρ‡Π»ΠΈ 2, ΠΈ ΠΏΠ΅Ρ€Π΅Π΄ ΠΏΠΎΠ»ΡƒΡ‡Π΅Π½Π½Ρ‹ΠΌ ΠΎΡ‚Π²Π΅Ρ‚ΠΎΠΌ поставили Ρ‚ΠΎΡ‚ Π·Π½Π°ΠΊ ΠΌΠΎΠ΄ΡƒΠ»ΡŒ, ΠΊΠΎΡ‚ΠΎΡ€ΠΎΠ³ΠΎ большС. Π£ числа 3 ΠΌΠΎΠ΄ΡƒΠ»ΡŒ большС, поэтому Π·Π½Π°ΠΊ этого числа ΠΈ поставлСн Π² ΠΎΡ‚Π²Π΅Ρ‚Π΅. Π’ΠΎ Π΅ΡΡ‚ΡŒ, ΠΎΡ‚Π²Π΅Ρ‚ ΠΏΠΎΠ»ΠΎΠΆΠΈΡ‚Π΅Π»ΡŒΠ½Ρ‹ΠΉ.

    ΠžΠ±Ρ‹Ρ‡Π½ΠΎ Π·Π°ΠΏΠΈΡΡ‹Π²Π°ΡŽΡ‚ ΠΏΠΎΠΊΠΎΡ€ΠΎΡ‡Π΅ 3 + (?2) = 1

    ΠŸΡ€ΠΈΠΌΠ΅Ρ€ 3. Найти Π·Π½Π°Ρ‡Π΅Π½ΠΈΠ΅ выраТСния 3 ? 7

    Π’ этом Π²Ρ‹Ρ€Π°ΠΆΠ΅Π½ΠΈΠΈ ΠΈΠ· мСньшСго числа вычитаСтся большСС. Для Ρ‚Π°ΠΊΠΎΠ³ΠΎ случая прСдусмотрСно ΡΠ»Π΅Π΄ΡƒΡŽΡ‰Π΅Π΅ ΠΏΡ€Π°Π²ΠΈΠ»ΠΎ:

    Π§Ρ‚ΠΎΠ±Ρ‹ ΠΈΠ· мСньшСго числа Π²Ρ‹Ρ‡Π΅ΡΡ‚ΡŒ большСС, Π½ΡƒΠΆΠ½ΠΎ ΠΈΠ· большСго числа Π²Ρ‹Ρ‡Π΅ΡΡ‚ΡŒ мСньшСС ΠΈ ΠΏΠ΅Ρ€Π΅Π΄ ΠΏΠΎΠ»ΡƒΡ‡Π΅Π½Π½Ρ‹ΠΌ ΠΎΡ‚Π²Π΅Ρ‚ΠΎΠΌ ΠΏΠΎΡΡ‚Π°Π²ΠΈΡ‚ΡŒ минус.

    Π’ этом Π²Ρ‹Ρ€Π°ΠΆΠ΅Π½ΠΈΠΈ Π΅ΡΡ‚ΡŒ нСбольшая Π·Π°Π³Π²ΠΎΠ·Π΄ΠΊΠ°. Вспомним, Ρ‡Ρ‚ΠΎ Π·Π½Π°ΠΊ равСнства (=) ставится ΠΌΠ΅ΠΆΠ΄Ρƒ Π²Π΅Π»ΠΈΡ‡ΠΈΠ½Π°ΠΌΠΈ ΠΈ выраТСниями Ρ‚ΠΎΠ³Π΄Π°, ΠΊΠΎΠ³Π΄Π° ΠΎΠ½ΠΈ Ρ€Π°Π²Π½Ρ‹ ΠΌΠ΅ΠΆΠ΄Ρƒ собой.

    Π—Π½Π°Ρ‡Π΅Π½ΠΈΠ΅ выраТСния 3 ? 7 ΠΊΠ°ΠΊ ΠΌΡ‹ ΡƒΠ·Π½Π°Π»ΠΈ Ρ€Π°Π²Π½ΠΎ?4. Π­Ρ‚ΠΎ ΠΎΠ·Π½Π°Ρ‡Π°Π΅Ρ‚, Ρ‡Ρ‚ΠΎ Π»ΡŽΠ±Ρ‹Π΅ прСобразования ΠΊΠΎΡ‚ΠΎΡ€Ρ‹Π΅ ΠΌΡ‹ Π±ΡƒΠ΄Π΅ΠΌ ΡΠΎΠ²Π΅Ρ€ΡˆΠ°Ρ‚ΡŒ Π² Π΄Π°Π½Π½ΠΎΠΌ Π²Ρ‹Ρ€Π°ΠΆΠ΅Π½ΠΈΠΈ, Π΄ΠΎΠ»ΠΆΠ½Ρ‹ Π±Ρ‹Ρ‚ΡŒ Ρ€Π°Π²Π½Ρ‹?4

    Но ΠΌΡ‹ Π²ΠΈΠ΄ΠΈΠΌ, Ρ‡Ρ‚ΠΎ Π½Π° Π²Ρ‚ΠΎΡ€ΠΎΠΌ этапС располагаСтся Π²Ρ‹Ρ€Π°ΠΆΠ΅Π½ΠΈΠ΅ 7 ? 3, ΠΊΠΎΡ‚ΠΎΡ€ΠΎΠ΅ Π½Π΅ Ρ€Π°Π²Π½ΠΎ?4.

    Π§Ρ‚ΠΎΠ±Ρ‹ ΠΈΡΠΏΡ€Π°Π²ΠΈΡ‚ΡŒ эту ΡΠΈΡ‚ΡƒΠ°Ρ†ΠΈΡŽ, Π²Ρ‹Ρ€Π°ΠΆΠ΅Π½ΠΈΠ΅ 7 ? 3 Π½ΡƒΠΆΠ½ΠΎ Π²Π·ΡΡ‚ΡŒ Π² скобки ΠΈ ΠΏΠ΅Ρ€Π΅Π΄ этой скобкой ΠΏΠΎΡΡ‚Π°Π²ΠΈΡ‚ΡŒ минус:

    3 ? 7 = ? (7 ? 3) = ? (4) = ?4

    Π’ этом случаС равСнство Π±ΡƒΠ΄Π΅Ρ‚ ΡΠΎΠ±Π»ΡŽΠ΄Π°Ρ‚ΡŒΡΡ Π½Π° ΠΊΠ°ΠΆΠ΄ΠΎΠΌ этапС:

    ПослС Ρ‚ΠΎΠ³ΠΎ ΠΊΠ°ΠΊ Π²Ρ‹Ρ€Π°ΠΆΠ΅Π½ΠΈΠ΅ вычислСно, скобки ΠΌΠΎΠΆΠ½ΠΎ ΡƒΠ±Ρ€Π°Ρ‚ΡŒ, Ρ‡Ρ‚ΠΎ ΠΌΡ‹ ΠΈ сдСлали.

    ΠŸΠΎΡΡ‚ΠΎΠΌΡƒ, Ρ‡Ρ‚ΠΎΠ±Ρ‹ Π±Ρ‹Ρ‚ΡŒ Π±ΠΎΠ»Π΅Π΅ Ρ‚ΠΎΡ‡Π½Ρ‹ΠΌ, Ρ€Π΅ΡˆΠ΅Π½ΠΈΠ΅ Π΄ΠΎΠ»ΠΆΠ½ΠΎ Π²Ρ‹Π³Π»ΡΠ΄Π΅Ρ‚ΡŒ Ρ‚Π°ΠΊ:

    3 ? 7 = ? (7 ? 3) = ? (4) = ? 4

    Π”Π°Π½Π½ΠΎΠ΅ ΠΏΡ€Π°Π²ΠΈΠ»ΠΎ ΠΌΠΎΠΆΠ½ΠΎ Π·Π°ΠΏΠΈΡΠ°Ρ‚ΡŒ с ΠΏΠΎΠΌΠΎΡ‰ΡŒΡŽ ΠΏΠ΅Ρ€Π΅ΠΌΠ΅Π½Π½Ρ‹Ρ…. Π’Ρ‹Π³Π»ΡΠ΄Π΅Ρ‚ΡŒ ΠΎΠ½ΠΎ Π±ΡƒΠ΄Π΅Ρ‚ ΡΠ»Π΅Π΄ΡƒΡŽΡ‰ΠΈΠΌ ΠΎΠ±Ρ€Π°Π·ΠΎΠΌ:

    a ? b = ? (b ? a)

    Π‘ΠΎΠ»ΡŒΡˆΠΎΠ΅ количСство скобок ΠΈ Π·Π½Π°ΠΊΠΎΠ² ΠΎΠΏΠ΅Ρ€Π°Ρ†ΠΈΠΉ ΠΌΠΎΠ³ΡƒΡ‚ ΡƒΡΠ»ΠΎΠΆΠ½ΡΡ‚ΡŒ Ρ€Π΅ΡˆΠ΅Π½ΠΈΠ΅, казалось Π±Ρ‹ совсСм простой Π·Π°Π΄Π°Ρ‡ΠΈ, поэтому цСлСсообразнСС Π½Π°ΡƒΡ‡ΠΈΡ‚ΡŒΡΡ Π·Π°ΠΏΠΈΡΡ‹Π²Π°Ρ‚ΡŒ Ρ‚Π°ΠΊΠΈΠ΅ ΠΏΡ€ΠΈΠΌΠ΅Ρ€Ρ‹ ΠΊΠΎΡ€ΠΎΡ‚ΠΊΠΎ, Π½Π°ΠΏΡ€ΠΈΠΌΠ΅Ρ€ 3 ? 7 = ? 4.

    На самом Π΄Π΅Π»Π΅ слоТСниС ΠΈ Π²Ρ‹Ρ‡ΠΈΡ‚Π°Π½ΠΈΠ΅ Ρ†Π΅Π»Ρ‹Ρ… чисСл сводится Ρ‚ΠΎΠ»ΡŒΠΊΠΎ ΠΊ слоТСнию. Π§Ρ‚ΠΎ это ΠΎΠ·Π½Π°Ρ‡Π°Π΅Ρ‚? Π­Ρ‚ΠΎ ΠΎΠ·Π½Π°Ρ‡Π°Π΅Ρ‚, Ρ‡Ρ‚ΠΎ Ссли трСбуСтся ΠΎΡΡƒΡ‰Π΅ΡΡ‚Π²ΠΈΡ‚ΡŒ Π²Ρ‹Ρ‡ΠΈΡ‚Π°Π½ΠΈΠ΅ чисСл, эту ΠΎΠΏΠ΅Ρ€Π°Ρ†ΠΈΡŽ ΠΌΠΎΠΆΠ½ΠΎ Π·Π°ΠΌΠ΅Π½ΠΈΡ‚ΡŒ слоТСниСм.

    Π˜Ρ‚Π°ΠΊ знакомимся с Π½ΠΎΠ²Ρ‹ΠΌ ΠΏΡ€Π°Π²ΠΈΠ»ΠΎΠΌ:

    Π’Ρ‹Ρ‡Π΅ΡΡ‚ΡŒ ΠΎΠ΄Π½ΠΎ число ΠΈΠ· Π΄Ρ€ΡƒΠ³ΠΎΠ³ΠΎ ΠΎΠ·Π½Π°Ρ‡Π°Π΅Ρ‚ ΠΏΡ€ΠΈΠ±Π°Π²ΠΈΡ‚ΡŒ ΠΊ ΡƒΠΌΠ΅Π½ΡŒΡˆΠ°Π΅ΠΌΠΎΠΌΡƒ Ρ‚Π°ΠΊΠΎΠ΅ число, ΠΊΠΎΡ‚ΠΎΡ€ΠΎΠ΅ Π±ΡƒΠ΄Π΅Ρ‚ ΠΏΡ€ΠΎΡ‚ΠΈΠ²ΠΎΠΏΠΎΠ»ΠΎΠΆΠ½ΠΎ Π²Ρ‹Ρ‡ΠΈΡ‚Π°Π΅ΠΌΠΎΠΌΡƒ.

    НапримСр, рассмотрим ΠΏΡ€ΠΎΡΡ‚Π΅ΠΉΡˆΠ΅Π΅ Π²Ρ‹Ρ€Π°ΠΆΠ΅Π½ΠΈΠ΅ 5 ? 3. На Π½Π°Ρ‡Π°Π»ΡŒΠ½Ρ‹Ρ… этапах изучСния ΠΌΠ°Ρ‚Π΅ΠΌΠ°Ρ‚ΠΈΠΊΠΈ ΠΌΡ‹ просто ставили Π·Π½Π°ΠΊ равСнства ΠΈ записывали ΠΎΡ‚Π²Π΅Ρ‚:

    Но сСйчас ΠΌΡ‹ прогрСссируСм Π² ΠΈΠ·ΡƒΡ‡Π΅Π½ΠΈΠΈ, поэтому Π½Π°Π΄ΠΎ ΠΏΡ€ΠΈΡΠΏΠΎΡΠ°Π±Π»ΠΈΠ²Π°Ρ‚ΡŒΡΡ ΠΊ Π½ΠΎΠ²Ρ‹ΠΌ ΠΏΡ€Π°Π²ΠΈΠ»Π°ΠΌ. НовоС ΠΏΡ€Π°Π²ΠΈΠ»ΠΎ Π³ΠΎΠ²ΠΎΡ€ΠΈΡ‚, Ρ‡Ρ‚ΠΎ Π²Ρ‹Ρ‡Π΅ΡΡ‚ΡŒ ΠΎΠ΄Π½ΠΎ число ΠΈΠ· Π΄Ρ€ΡƒΠ³ΠΎΠ³ΠΎ ΠΎΠ·Π½Π°Ρ‡Π°Π΅Ρ‚ ΠΏΡ€ΠΈΠ±Π°Π²ΠΈΡ‚ΡŒ ΠΊ ΡƒΠΌΠ΅Π½ΡŒΡˆΠ°Π΅ΠΌΠΎΠΌΡƒ Ρ‚Π°ΠΊΠΎΠ΅ число, ΠΊΠΎΡ‚ΠΎΡ€ΠΎΠ΅ Π±ΡƒΠ΄Π΅Ρ‚ ΠΏΡ€ΠΎΡ‚ΠΈΠ²ΠΎΠΏΠΎΠ»ΠΎΠΆΠ½ΠΎ Π²Ρ‹Ρ‡ΠΈΡ‚Π°Π΅ΠΌΠΎΠΌΡƒ.

    На ΠΏΡ€ΠΈΠΌΠ΅Ρ€Π΅ выраТСния 5?3 ΠΏΠΎΠΏΡ€ΠΎΠ±ΡƒΠ΅ΠΌ ΠΏΠΎΠ½ΡΡ‚ΡŒ это ΠΏΡ€Π°Π²ΠΈΠ»ΠΎ. УмСньшаСмоС Π² Π΄Π°Π½Π½ΠΎΠΌ Π²Ρ‹Ρ€Π°ΠΆΠ΅Π½ΠΈΠΈ это 5, Π° Π²Ρ‹Ρ‡ΠΈΡ‚Π°Π΅ΠΌΠΎΠ΅ это 3. ΠŸΡ€Π°Π²ΠΈΠ»ΠΎ Π³ΠΎΠ²ΠΎΡ€ΠΈΡ‚, Ρ‡Ρ‚ΠΎ для Ρ‚ΠΎΠ³ΠΎ Ρ‡Ρ‚ΠΎΠ±Ρ‹ ΠΈΠ· 5 Π²Ρ‹Ρ‡Π΅ΡΡ‚ΡŒ 3 , Π½ΡƒΠΆΠ½ΠΎ ΠΊ 5 ΠΏΡ€ΠΈΠ±Π°Π²ΠΈΡ‚ΡŒ Ρ‚Π°ΠΊΠΎΠ΅ число, ΠΊΠΎΡ‚ΠΎΡ€ΠΎΠ΅ Π±ΡƒΠ΄Π΅Ρ‚ ΠΏΡ€ΠΎΡ‚ΠΈΠ²ΠΎΠΏΠΎΠ»ΠΎΠΆΠ½ΠΎ 3. ΠŸΡ€ΠΎΡ‚ΠΈΠ²ΠΎΠΏΠΎΠ»ΠΎΠΆΠ½ΠΎΠ΅ для числа 3 это число?3. ЗаписываСм Π½ΠΎΠ²ΠΎΠ΅ Π²Ρ‹Ρ€Π°ΠΆΠ΅Π½ΠΈΠ΅:

    А ΠΊΠ°ΠΊ Π½Π°Ρ…ΠΎΠ΄ΠΈΡ‚ΡŒ значСния для Ρ‚Π°ΠΊΠΈΡ… Π²Ρ‹Ρ€Π°ΠΆΠ΅Π½ΠΈΠΉ ΠΌΡ‹ ΡƒΠΆΠ΅ Π·Π½Π°Π΅ΠΌ. Π­Ρ‚ΠΎ слоТСниС чисСл с Ρ€Π°Π·Π½Ρ‹ΠΌΠΈ Π·Π½Π°ΠΊΠ°ΠΌΠΈ, ΠΊΠΎΡ‚ΠΎΡ€ΠΎΠ΅ ΠΌΡ‹ рассмотрСли Π²Ρ‹ΡˆΠ΅. Π§Ρ‚ΠΎΠ±Ρ‹ ΡΠ»ΠΎΠΆΠΈΡ‚ΡŒ числа с Ρ€Π°Π·Π½Ρ‹ΠΌΠΈ Π·Π½Π°ΠΊΠ°ΠΌΠΈ, Π½ΡƒΠΆΠ½ΠΎ ΠΈΠ· большСго модуля Π²Ρ‹Ρ‡Π΅ΡΡ‚ΡŒ мСньший, ΠΈ ΠΏΠ΅Ρ€Π΅Π΄ ΠΏΠΎΠ»ΡƒΡ‡Π΅Π½Π½Ρ‹ΠΌ ΠΎΡ‚Π²Π΅Ρ‚ΠΎΠΌ ΠΏΠΎΡΡ‚Π°Π²ΠΈΡ‚ΡŒ Ρ‚ΠΎΡ‚ Π·Π½Π°ΠΊ, ΠΌΠΎΠ΄ΡƒΠ»ΡŒ ΠΊΠΎΡ‚ΠΎΡ€ΠΎΠ³ΠΎ большС:

    5 + (?3) = |5| ? |?3| = 5 ? 3 = 2

    ΠœΠΎΠ΄ΡƒΠ»ΡŒ числа 5 большС, Ρ‡Π΅ΠΌ ΠΌΠΎΠ΄ΡƒΠ»ΡŒ числа?3. ΠŸΠΎΡΡ‚ΠΎΠΌΡƒ ΠΌΡ‹ ΠΈΠ· 5 Π²Ρ‹Ρ‡Π»ΠΈ 3 ΠΈ ΠΏΠΎΠ»ΡƒΡ‡ΠΈΠ»ΠΈ 2. Π£ числа 5 ΠΌΠΎΠ΄ΡƒΠ»ΡŒ большС, поэтому Π·Π½Π°ΠΊ этого числа ΠΈ поставили Π² ΠΎΡ‚Π²Π΅Ρ‚Π΅. Π’ΠΎ Π΅ΡΡ‚ΡŒ ΠΎΡ‚Π²Π΅Ρ‚ ΠΏΠΎΠ»ΠΎΠΆΠΈΡ‚Π΅Π»Π΅Π½.

    ΠŸΠΎΠ½Π°Ρ‡Π°Π»Ρƒ быстро Π·Π°ΠΌΠ΅Π½ΡΡ‚ΡŒ Π²Ρ‹Ρ‡ΠΈΡ‚Π°Π½ΠΈΠ΅ слоТСниСм удаётся Π½Π΅ всСм. Π­Ρ‚ΠΎ связано с Ρ‚Π΅ΠΌ, Ρ‡Ρ‚ΠΎ ΠΏΠΎΠ»ΠΎΠΆΠΈΡ‚Π΅Π»ΡŒΠ½Ρ‹Π΅ числа Π·Π°ΠΏΠΈΡΡ‹Π²Π°ΡŽΡ‚ΡΡ Π±Π΅Π· своСго Π·Π½Π°ΠΊΠ° плюс.

    НапримСр, Π² Π²Ρ‹Ρ€Π°ΠΆΠ΅Π½ΠΈΠΈ 3 ? 1 Π·Π½Π°ΠΊ минуса, ΡƒΠΊΠ°Π·Ρ‹Π²Π°ΡŽΡ‰ΠΈΠΉ Π½Π° Π²Ρ‹Ρ‡ΠΈΡ‚Π°Π½ΠΈΠ΅, являСтся Π·Π½Π°ΠΊΠΎΠΌ ΠΎΠΏΠ΅Ρ€Π°Ρ†ΠΈΠΈ ΠΈ Π½Π΅ относится ΠΊ Π΅Π΄ΠΈΠ½ΠΈΡ†Π΅. Π•Π΄ΠΈΠ½ΠΈΡ†Π° Π² Π΄Π°Π½Π½ΠΎΠΌ случаС являСтся ΠΏΠΎΠ»ΠΎΠΆΠΈΡ‚Π΅Π»ΡŒΠ½Ρ‹ΠΌ числом ΠΈ Ρƒ Π½Π΅Ρ‘ Π΅ΡΡ‚ΡŒ свой Π·Π½Π°ΠΊ плюса, Π½ΠΎ ΠΌΡ‹ Π΅Π³ΠΎ Π½Π΅ Π²ΠΈΠ΄ΠΈΠΌ, ΠΏΠΎΡΠΊΠΎΠ»ΡŒΠΊΡƒ плюс ΠΏΠ΅Ρ€Π΅Π΄ ΠΏΠΎΠ»ΠΎΠΆΠΈΡ‚Π΅Π»ΡŒΠ½Ρ‹ΠΌΠΈ числами ΠΏΠΎ Ρ‚Ρ€Π°Π΄ΠΈΡ†ΠΈΠΈ Π½Π΅ Π·Π°ΠΏΠΈΡΡ‹Π²Π°ΡŽΡ‚.

    А стало Π±Ρ‹Ρ‚ΡŒ для наглядности Π΄Π°Π½Π½ΠΎΠ΅ Π²Ρ‹Ρ€Π°ΠΆΠ΅Π½ΠΈΠ΅ ΠΌΠΎΠΆΠ½ΠΎ Π·Π°ΠΏΠΈΡΠ°Ρ‚ΡŒ ΡΠ»Π΅Π΄ΡƒΡŽΡ‰ΠΈΠΌ ΠΎΠ±Ρ€Π°Π·ΠΎΠΌ:

    Для удобства числа со своим Π·Π½Π°ΠΊΠ°ΠΌΠΈ Π·Π°ΠΊΠ»ΡŽΡ‡Π°ΡŽΡ‚ Π² скобки. Π’ Ρ‚Π°ΠΊΠΎΠΌ случаС Π·Π°ΠΌΠ΅Π½ΠΈΡ‚ΡŒ Π²Ρ‹Ρ‡ΠΈΡ‚Π°Π½ΠΈΠ΅ слоТСниСм Π½Π°ΠΌΠ½ΠΎΠ³ΠΎ ΠΏΡ€ΠΎΡ‰Π΅. Π’Ρ‹Ρ‡ΠΈΡ‚Π°Π΅ΠΌΠΎΠ΅ Π² Π΄Π°Π½Π½ΠΎΠΌ случаС это число (+1), Π° ΠΏΡ€ΠΎΡ‚ΠΈΠ²ΠΎΠΏΠΎΠ»ΠΎΠΆΠ½ΠΎΠ΅ Π΅ΠΌΡƒ число (?1). Π—Π°ΠΌΠ΅Π½ΠΈΠΌ ΠΎΠΏΠ΅Ρ€Π°Ρ†ΠΈΡŽ вычитания слоТСниСм ΠΈ вмСсто Π²Ρ‹Ρ‡ΠΈΡ‚Π°Π΅ΠΌΠΎΠ³ΠΎ (+1) записываСм ΠΏΡ€ΠΎΡ‚ΠΈΠ²ΠΎΠΏΠΎΠ»ΠΎΠΆΠ½ΠΎΠ΅ Π΅ΠΌΡƒ число (?1)

    (+3) ? (+1) = (+3) + (?1) = |+3| ? |?1| = 3 ? 1 = 2

    На ΠΏΠ΅Ρ€Π²Ρ‹ΠΉ взгляд покаТСтся, ΠΊΠ°ΠΊΠΎΠΉ смысл Π² этих Π»ΠΈΡˆΠ½ΠΈΡ… тСлодвиТСниях, Ссли ΠΌΠΎΠΆΠ½ΠΎ старым Π΄ΠΎΠ±Ρ€Ρ‹ΠΌ ΠΌΠ΅Ρ‚ΠΎΠ΄ΠΎΠΌ ΠΏΠΎΡΡ‚Π°Π²ΠΈΡ‚ΡŒ Π·Π½Π°ΠΊ равСнства ΠΈ сразу Π·Π°ΠΏΠΈΡΠ°Ρ‚ΡŒ ΠΎΡ‚Π²Π΅Ρ‚ 2. На самом Π΄Π΅Π»Π΅ это ΠΏΡ€Π°Π²ΠΈΠ»ΠΎ Π΅Ρ‰Ρ‘ Π½Π΅ Ρ€Π°Π· нас Π²Ρ‹Ρ€ΡƒΡ‡ΠΈΡ‚.

    РСшим ΠΏΡ€Π΅Π΄Ρ‹Π΄ΡƒΡ‰ΠΈΠΉ ΠΏΡ€ΠΈΠΌΠ΅Ρ€ 3 ? 7, ΠΈΡΠΏΠΎΠ»ΡŒΠ·ΡƒΡ ΠΏΡ€Π°Π²ΠΈΠ»ΠΎ вычитания. Π‘Π½Π°Ρ‡Π°Π»Π° ΠΏΡ€ΠΈΠ²Π΅Π΄Ρ‘ΠΌ Π²Ρ‹Ρ€Π°ΠΆΠ΅Π½ΠΈΠ΅ ΠΊ Π½ΠΎΡ€ΠΌΠ°Π»ΡŒΠ½ΠΎΠΌΡƒ Π²ΠΈΠ΄Ρƒ, расставив ΠΊΠ°ΠΆΠ΄ΠΎΠΌΡƒ числу свои Π·Π½Π°ΠΊΠΈ. Π£ Ρ‚Ρ€ΠΎΠΉΠΊΠΈ Π·Π½Π°ΠΊ плюса, ΠΏΠΎΡΠΊΠΎΠ»ΡŒΠΊΡƒ ΠΎΠ½Π° являСтся ΠΏΠΎΠ»ΠΎΠΆΠΈΡ‚Π΅Π»ΡŒΠ½Ρ‹ΠΌ числом. ΠœΠΈΠ½ΡƒΡ, ΡƒΠΊΠ°Π·Ρ‹Π²Π°ΡŽΡ‰ΠΈΠΉ Π½Π° Π²Ρ‹Ρ‡ΠΈΡ‚Π°Π½ΠΈΠ΅ Π½Π΅ относится ΠΊ сСмёркС. Π£ сСмёрки Π·Π½Π°ΠΊ плюса, ΠΏΠΎΡΠΊΠΎΠ»ΡŒΠΊΡƒ ΠΎΠ½Π° Ρ‚Π°ΠΊΠΆΠ΅ являСтся ΠΏΠΎΠ»ΠΎΠΆΠΈΡ‚Π΅Π»ΡŒΠ½Ρ‹ΠΌ числом:

    Π—Π°ΠΌΠ΅Π½ΠΈΠΌ Π²Ρ‹Ρ‡ΠΈΡ‚Π°Π½ΠΈΠ΅ слоТСниСм:

    Π”Π°Π»ΡŒΠ½Π΅ΠΉΡˆΠ΅Π΅ вычислСниС Π½Π΅ составляСт Ρ‚Ρ€ΡƒΠ΄Π°:

    ΠŸΡ€ΠΈΠΌΠ΅Ρ€ 7. Найти Π·Π½Π°Ρ‡Π΅Π½ΠΈΠ΅ выраТСния?4 ? 5

    ΠŸΠ΅Ρ€Π΅Π΄ Π½Π°ΠΌΠΈ снова опСрация вычитания. Π­Ρ‚Ρƒ ΠΎΠΏΠ΅Ρ€Π°Ρ†ΠΈΡŽ Π½ΡƒΠΆΠ½ΠΎ Π·Π°ΠΌΠ΅Π½ΠΈΡ‚ΡŒ слоТСниСм. К ΡƒΠΌΠ΅Π½ΡŒΡˆΠ°Π΅ΠΌΠΎΠΌΡƒ (?4) ΠΏΡ€ΠΈΠ±Π°Π²ΠΈΠΌ число, ΠΏΡ€ΠΎΡ‚ΠΈΠ²ΠΎΠΏΠΎΠ»ΠΎΠΆΠ½ΠΎΠ΅ Π²Ρ‹Ρ‡ΠΈΡ‚Π°Π΅ΠΌΠΎΠΌΡƒ (+5). ΠŸΡ€ΠΎΡ‚ΠΈΠ²ΠΎΠΏΠΎΠ»ΠΎΠΆΠ½ΠΎΠ΅ число для Π²Ρ‹Ρ‡ΠΈΡ‚Π°Π΅ΠΌΠΎΠ³ΠΎ (+5) это число (?5).

    ΠœΡ‹ ΠΏΡ€ΠΈΡˆΠ»ΠΈ ΠΊ ситуации, Π³Π΄Π΅ Π½ΡƒΠΆΠ½ΠΎ ΡΠ»ΠΎΠΆΠΈΡ‚ΡŒ ΠΎΡ‚Ρ€ΠΈΡ†Π°Ρ‚Π΅Π»ΡŒΠ½Ρ‹Π΅ числа. Для Ρ‚Π°ΠΊΠΈΡ… случаСв прСдусмотрСно ΡΠ»Π΅Π΄ΡƒΡŽΡ‰Π΅Π΅ ΠΏΡ€Π°Π²ΠΈΠ»ΠΎ:

    Π§Ρ‚ΠΎΠ±Ρ‹ ΡΠ»ΠΎΠΆΠΈΡ‚ΡŒ ΠΎΡ‚Ρ€ΠΈΡ†Π°Ρ‚Π΅Π»ΡŒΠ½Ρ‹Π΅ числа, Π½ΡƒΠΆΠ½ΠΎ ΡΠ»ΠΎΠΆΠΈΡ‚ΡŒ ΠΈΡ… ΠΌΠΎΠ΄ΡƒΠ»ΠΈ, ΠΈ ΠΏΠ΅Ρ€Π΅Π΄ ΠΏΠΎΠ»ΡƒΡ‡Π΅Π½Π½Ρ‹ΠΌ ΠΎΡ‚Π²Π΅Ρ‚ΠΎΠΌ ΠΏΠΎΡΡ‚Π°Π²ΠΈΡ‚ΡŒ минус.

    Π˜Ρ‚Π°ΠΊ, слоТим ΠΌΠΎΠ΄ΡƒΠ»ΠΈ чисСл, ΠΊΠ°ΠΊ ΠΎΡ‚ нас Ρ‚Ρ€Π΅Π±ΡƒΠ΅Ρ‚ ΠΏΡ€Π°Π²ΠΈΠ»ΠΎ ΠΈ поставим ΠΏΠ΅Ρ€Π΅Π΄ ΠΏΠΎΠ»ΡƒΡ‡Π΅Π½Π½Ρ‹ΠΌ ΠΎΡ‚Π²Π΅Ρ‚ΠΎΠΌ минус:

    (?4) ? (+5) = (?4) + (?5) = |?4| + |?5| = 4 + 5 = ?9

    Π—Π°ΠΏΠΈΡΡŒ с модулями Π½Π΅ΠΎΠ±Ρ…ΠΎΠ΄ΠΈΠΌΠΎ Π·Π°ΠΊΠ»ΡŽΡ‡ΠΈΡ‚ΡŒ Π² скобки ΠΈ ΠΏΠ΅Ρ€Π΅Π΄ этими скобками ΠΏΠΎΡΡ‚Π°Π²ΠΈΡ‚ΡŒ минус. Π’Π°ΠΊ ΠΌΡ‹ обСспСчим минус, ΠΊΠΎΡ‚ΠΎΡ€Ρ‹ΠΉ Π΄ΠΎΠ»ΠΆΠ΅Π½ ΡΡ‚ΠΎΡΡ‚ΡŒ ΠΏΠ΅Ρ€Π΅Π΄ ΠΎΡ‚Π²Π΅Ρ‚ΠΎΠΌ:

    (?4) ? (+5) = (?4) + (?5) = ?(|?4| + |?5|) = ?(4 + 5) = ?(9) = ?9

    РСшСниС для Π΄Π°Π½Π½ΠΎΠ³ΠΎ ΠΏΡ€ΠΈΠΌΠ΅Ρ€Π° ΠΌΠΎΠΆΠ½ΠΎ Π·Π°ΠΏΠΈΡΠ°Ρ‚ΡŒ ΠΏΠΎΠΊΠΎΡ€ΠΎΡ‡Π΅:

    ΠŸΡ€ΠΈΠΌΠ΅Ρ€ 8. Найти Π·Π½Π°Ρ‡Π΅Π½ΠΈΠ΅ выраТСния?3 ? 5 ? 7 ? 9

    ΠŸΡ€ΠΈΠ²Π΅Π΄Ρ‘ΠΌ Π²Ρ‹Ρ€Π°ΠΆΠ΅Π½ΠΈΠ΅ ΠΊ понятному Π²ΠΈΠ΄Ρƒ. Π—Π΄Π΅ΡΡŒ всС числа, ΠΊΡ€ΠΎΠΌΠ΅ числа?3 ΡΠ²Π»ΡΡŽΡ‚ΡΡ ΠΏΠΎΠ»ΠΎΠΆΠΈΡ‚Π΅Π»ΡŒΠ½Ρ‹ΠΌΠΈ, поэтому Ρƒ Π½ΠΈΡ… Π±ΡƒΠ΄ΡƒΡ‚ Π·Π½Π°ΠΊΠΈ плюса:

    Π—Π°ΠΌΠ΅Π½ΠΈΠΌ ΠΎΠΏΠ΅Ρ€Π°Ρ†ΠΈΠΈ вычитания опСрациями слоТСния. ВсС минусы (ΠΊΡ€ΠΎΠΌΠ΅ минуса, ΠΊΠΎΡ‚ΠΎΡ€Ρ‹ΠΉ ΠΏΠ΅Ρ€Π΅Π΄ Ρ‚Ρ€ΠΎΠΉΠΊΠΎΠΉ) ΠΏΠΎΠΌΠ΅Π½ΡΡŽΡ‚ΡΡ Π½Π° ΠΏΠ»ΡŽΡΡ‹ ΠΈ всС ΠΏΠΎΠ»ΠΎΠΆΠΈΡ‚Π΅Π»ΡŒΠ½Ρ‹Π΅ числа ΠΏΠΎΠΌΠ΅Π½ΡΡŽΡ‚ΡΡ Π½Π° ΠΏΡ€ΠΎΡ‚ΠΈΠ²ΠΎΠΏΠΎΠ»ΠΎΠΆΠ½Ρ‹Π΅:

    Π’Π΅ΠΏΠ΅Ρ€ΡŒ ΠΏΡ€ΠΈΠΌΠ΅Π½ΠΈΠΌ ΠΏΡ€Π°Π²ΠΈΠ»ΠΎ слоТСния ΠΎΡ‚Ρ€ΠΈΡ†Π°Ρ‚Π΅Π»ΡŒΠ½Ρ‹Ρ… чисСл. Π§Ρ‚ΠΎΠ±Ρ‹ ΡΠ»ΠΎΠΆΠΈΡ‚ΡŒ ΠΎΡ‚Ρ€ΠΈΡ†Π°Ρ‚Π΅Π»ΡŒΠ½Ρ‹Π΅ числа, Π½ΡƒΠΆΠ½ΠΎ ΡΠ»ΠΎΠΆΠΈΡ‚ΡŒ ΠΈΡ… ΠΌΠΎΠ΄ΡƒΠ»ΠΈ ΠΈ ΠΏΠ΅Ρ€Π΅Π΄ ΠΏΠΎΠ»ΡƒΡ‡Π΅Π½Π½Ρ‹ΠΌ ΠΎΡ‚Π²Π΅Ρ‚ΠΎΠΌ ΠΏΠΎΡΡ‚Π°Π²ΠΈΡ‚ΡŒ минус:

    = ?(|?3| + |?5| + |?7| + |?9|) = ?(3 + 5 + 7 + 9) = ?(24) = ?24

    РСшСниС для Π΄Π°Π½Π½ΠΎΠ³ΠΎ ΠΏΡ€ΠΈΠΌΠ΅Ρ€Π° ΠΌΠΎΠΆΠ½ΠΎ Π·Π°ΠΏΠΈΡΠ°Ρ‚ΡŒ ΠΏΠΎΠΊΠΎΡ€ΠΎΡ‡Π΅:

    3 ? 5 ? 7 ? 9 = ?(3 + 5 + 7 + 9) = ?24

    ΠŸΡ€ΠΈΠΌΠ΅Ρ€ 9. Найти Π·Π½Π°Ρ‡Π΅Π½ΠΈΠ΅ выраТСния?10 + 6 ? 15 + 11 ? 7

    ΠŸΡ€ΠΈΠ²Π΅Π΄Ρ‘ΠΌ Π²Ρ‹Ρ€Π°ΠΆΠ΅Π½ΠΈΠ΅ ΠΊ понятному Π²ΠΈΠ΄Ρƒ:

    Π—Π΄Π΅ΡΡŒ сразу Π΄Π²Π΅ ΠΎΠΏΠ΅Ρ€Π°Ρ†ΠΈΠΈ: слоТСниС ΠΈ Π²Ρ‹Ρ‡ΠΈΡ‚Π°Π½ΠΈΠ΅. Π‘Π»ΠΎΠΆΠ΅Π½ΠΈΠ΅ оставляСм ΠΊΠ°ΠΊ Π΅ΡΡ‚ΡŒ, Π° Π²Ρ‹Ρ‡ΠΈΡ‚Π°Π½ΠΈΠ΅ замСняСм слоТСниСм:

    (?10) + (+6) ? (+15) + (+11) ? (+7) = (?10) + (+6) + (?15) + (+11) + (?7)

    Боблюдая порядок дСйствий, Π²Ρ‹ΠΏΠΎΠ»Π½ΠΈΠΌ ΠΏΠΎΠΎΡ‡Π΅Ρ€Ρ‘Π΄Π½ΠΎ ΠΊΠ°ΠΆΠ΄ΠΎΠ΅ дСйствиС, ΠΎΠΏΠΈΡ€Π°ΡΡΡŒ Π½Π° Ρ€Π°Π½Π΅Π΅ ΠΈΠ·ΡƒΡ‡Π΅Π½Π½Ρ‹Π΅ ΠΏΡ€Π°Π²ΠΈΠ»Π°. Записи с модулями ΠΌΠΎΠΆΠ½ΠΎ ΠΏΡ€ΠΎΠΏΡƒΡΡ‚ΠΈΡ‚ΡŒ:

    ΠŸΠ΅Ρ€Π²ΠΎΠ΅ дСйствиС:

    (?10) + (+6) = ? (10 ? 6) = ? (4) = ? 4

    Π’Ρ‚ΠΎΡ€ΠΎΠ΅ дСйствиС:

    (?4) + (?15) = ? (4 + 15) = ? (19) = ? 19

    Π’Ρ€Π΅Ρ‚ΡŒΠ΅ дСйствиС:

    (?19) + (+11) = ? (19 ? 11) = ? (8) = ?8

    Π§Π΅Ρ‚Π²Ρ‘Ρ€Ρ‚ΠΎΠ΅ дСйствиС:

    (?8) + (?7) = ? (8 + 7) = ? (15) = ? 15

    Π’Π°ΠΊΠΈΠΌ ΠΎΠ±Ρ€Π°Π·ΠΎΠΌ, Π·Π½Π°Ρ‡Π΅Π½ΠΈΠ΅ выраТСния?10 + 6 ? 15 + 11 ? 7 Ρ€Π°Π²Π½ΠΎ?15

    ΠŸΡ€ΠΈΠΌΠ΅Ρ‡Π°Π½ΠΈΠ΅ . ΠŸΡ€ΠΈΠ²ΠΎΠ΄ΠΈΡ‚ΡŒ Π²Ρ‹Ρ€Π°ΠΆΠ΅Π½ΠΈΠ΅ ΠΊ понятному Π²ΠΈΠ΄Ρƒ, Π·Π°ΠΊΠ»ΡŽΡ‡Π°Ρ числа Π² скобки, вовсС Π½Π΅ΠΎΠ±ΡΠ·Π°Ρ‚Π΅Π»ΡŒΠ½ΠΎ. Когда происходит ΠΏΡ€ΠΈΠ²Ρ‹ΠΊΠ°Π½ΠΈΠ΅ ΠΊ ΠΎΡ‚Ρ€ΠΈΡ†Π°Ρ‚Π΅Π»ΡŒΠ½Ρ‹ΠΌ числам, это дСйствиС ΠΌΠΎΠΆΠ½ΠΎ ΠΏΡ€ΠΎΠΏΡƒΡΡ‚ΠΈΡ‚ΡŒ, ΠΏΠΎΡΠΊΠΎΠ»ΡŒΠΊΡƒ ΠΎΠ½ΠΎ ΠΎΡ‚Π½ΠΈΠΌΠ°Π΅Ρ‚ врСмя ΠΈ ΠΌΠΎΠΆΠ΅Ρ‚ Π·Π°ΠΏΡƒΡ‚Π°Ρ‚ΡŒ.

    Π˜Ρ‚Π°ΠΊ, для слоТСния ΠΈ вычитания Ρ†Π΅Π»Ρ‹Ρ… чисСл Π½Π΅ΠΎΠ±Ρ…ΠΎΠ΄ΠΈΠΌΠΎ Π·Π°ΠΏΠΎΠΌΠ½ΠΈΡ‚ΡŒ ΡΠ»Π΅Π΄ΡƒΡŽΡ‰ΠΈΠ΅ ΠΏΡ€Π°Π²ΠΈΠ»Π°:

    Π§Ρ‚ΠΎΠ±Ρ‹ ΡΠ»ΠΎΠΆΠΈΡ‚ΡŒ числа с Ρ€Π°Π·Π½Ρ‹ΠΌΠΈ Π·Π½Π°ΠΊΠ°ΠΌΠΈ, Π½ΡƒΠΆΠ½ΠΎ ΠΈΠ· большСго модуля Π²Ρ‹Ρ‡Π΅ΡΡ‚ΡŒ мСньший ΠΌΠΎΠ΄ΡƒΠ»ΡŒ, ΠΈ ΠΏΠ΅Ρ€Π΅Π΄ ΠΏΠΎΠ»ΡƒΡ‡Π΅Π½Π½Ρ‹ΠΌ ΠΎΡ‚Π²Π΅Ρ‚ΠΎΠΌ ΠΏΠΎΡΡ‚Π°Π²ΠΈΡ‚ΡŒ Ρ‚ΠΎΡ‚ Π·Π½Π°ΠΊ, ΠΌΠΎΠ΄ΡƒΠ»ΡŒ ΠΊΠΎΡ‚ΠΎΡ€ΠΎΠ³ΠΎ большС.

    Π§Ρ‚ΠΎΠ±Ρ‹ ΠΈΠ· мСньшСго числа Π²Ρ‹Ρ‡Π΅ΡΡ‚ΡŒ большСС, Π½ΡƒΠΆΠ½ΠΎ ΠΈΠ· большСго числа Π²Ρ‹Ρ‡Π΅ΡΡ‚ΡŒ мСньшСС ΠΈ ΠΏΠ΅Ρ€Π΅Π΄ ΠΏΠΎΠ»ΡƒΡ‡Π΅Π½Π½Ρ‹ΠΌ ΠΎΡ‚Π²Π΅Ρ‚ΠΎΠΌ ΠΏΠΎΡΡ‚Π°Π²ΠΈΡ‚ΡŒ Π·Π½Π°ΠΊ минуса.

    Π’Ρ‹Ρ‡Π΅ΡΡ‚ΡŒ ΠΎΠ΄Π½ΠΎ число ΠΈΠ· Π΄Ρ€ΡƒΠ³ΠΎΠ³ΠΎ ΠΎΠ·Π½Π°Ρ‡Π°Π΅Ρ‚, ΠΏΡ€ΠΈΠ±Π°Π²ΠΈΡ‚ΡŒ ΠΊ ΡƒΠΌΠ΅Π½ΡŒΡˆΠ°Π΅ΠΌΠΎΠΌΡƒ число ΠΏΡ€ΠΎΡ‚ΠΈΠ²ΠΎΠΏΠΎΠ»ΠΎΠΆΠ½ΠΎΠ΅ Π²Ρ‹Ρ‡ΠΈΡ‚Π°Π΅ΠΌΠΎΠΌΡƒ.

    Π§Ρ‚ΠΎΠ±Ρ‹ ΡΠ»ΠΎΠΆΠΈΡ‚ΡŒ ΠΎΡ‚Ρ€ΠΈΡ†Π°Ρ‚Π΅Π»ΡŒΠ½Ρ‹Π΅ числа, Π½ΡƒΠΆΠ½ΠΎ ΡΠ»ΠΎΠΆΠΈΡ‚ΡŒ ΠΈΡ… ΠΌΠΎΠ΄ΡƒΠ»ΠΈ, ΠΈ ΠΏΠ΅Ρ€Π΅Π΄ ΠΏΠΎΠ»ΡƒΡ‡Π΅Π½Π½Ρ‹ΠΌ ΠΎΡ‚Π²Π΅Ρ‚ΠΎΠΌ ΠΏΠΎΡΡ‚Π°Π²ΠΈΡ‚ΡŒ Π·Π½Π°ΠΊ минус.

    • Π₯ΠΎΠΊΠΊΠ΅ΠΉ Π±Π΅Π· ΠΏΡ€Π°Π²ΠΈΠ» Π’ ΠšΠΎΠ½Ρ‚Π°ΠΊΡ‚Π΅ Π˜Π³Ρ€Π° Π±Ρ‹Π»Π° Π²Ρ‹ΠΏΡƒΡ‰Π΅Π½Π° Π² сСнтябрС 2012 Π³ΠΎΠ΄Π°, ΠΈ Π½Π°Π±Ρ€Π°Π»Π° ΡƒΠΆΠ΅ ΠΏΠΎΡ‡Ρ‚ΠΈ 700 000 ΠΏΠΎΠ»ΡŒΠ·ΠΎΠ²Π°Ρ‚Π΅Π»Π΅ΠΉ. ΠŸΡ€Π΅Π΄ΡƒΡΠΌΠΎΡ‚Ρ€Π΅Π½ΠΎ Π΄Π²Π° Ρ€Π΅ΠΆΠΈΠΌΠ° ΠΈΠ³Ρ€Ρ‹ ΠΈ мноТСство возмоТностСй для комплСктования ΠΊΠΎΠΌΠ°Π½Π΄Ρ‹. Π’Π΅Ρ‡Π΅Π½ΠΈΠ΅ ΠΌΠ°Ρ‚Ρ‡Π° Π² Π₯ΠΎΠΊΠΊΠ΅Π΅ Π±Π΅Π· ΠΏΡ€Π°Π²ΠΈΠ» Π’ ΠšΠΎΠ½Ρ‚Π°ΠΊΡ‚Π΅ Π½Π°ΠΏΠΎΠΌΠΈΠ½Π°Π΅Ρ‚ Ρ€Π°Π½Π½ΠΈΠ΅ ΠΈΠ³Ρ€Ρ‹ сСрии NHL ΠΎΡ‚ Electronic Arts. 3 ΠΈΠ³Ρ€ΠΎΠΊΠ° Π½Π° […]
    • ΠŸΡ€Π°Π²ΠΈΠ»Π° ΠΏΠΎΠΊΠ΅Ρ€Π° ΠžΠΌΠ°Ρ…Π° Π₯ΠΎΠ»Π΄Π΅ΠΌ ΠžΠΌΠ°Ρ…Π° Π₯Π°ΠΉ-Π›ΠΎΡƒ ΠΈ пятикарточная ΠžΠΌΠ°Ρ…Π° ΠžΠΌΠ°Ρ…Π° Π₯ΠΎΠ»Π΄Π΅ΠΌ (Omaha Hold»Em) являСтся нСбольшим Π²ΠΈΠ΄ΠΎΠΈΠ·ΠΌΠ΅Π½Π΅Π½ΠΈΠ΅ΠΌ ВСхасского Π₯ΠΎΠ»Π΄Π΅ΠΌΠ°. Если Π²Ρ‹ ΠΏΠ»ΠΎΡ…ΠΎ Π·Π½Π°ΠΊΠΎΠΌΡ‹ с этой самой популярной Ρ€Π°Π·Π½ΠΎΠ²ΠΈΠ΄Π½ΠΎΡΡ‚ΡŒΡŽ ΠΏΠΎΠΊΠ΅Ρ€Π°, ΠΈΠ·ΡƒΡ‡Π°ΠΉΡ‚Π΅ ΠΏΡ€Π°Π²ΠΈΠ»Π° ВСхасского Π₯ΠΎΠ»Π΄Π΅ΠΌΠ° ΠΏΠΎ ссылкС; ΠΈΡ… Π·Π½Π°Π½ΠΈΠ΅ Π½Π΅ΠΎΠ±Ρ…ΠΎΠ΄ΠΈΠΌΠΎ для понимания ΠΏΡ€Π°Π²ΠΈΠ» ΠžΠΌΠ°Ρ…ΠΈ. ВсС […]
    • РСшСниС Π·Π°Π΄Π°Ρ‡ ΠΏΠΎ Π³Π΅Π½Π΅Ρ‚ΠΈΠΊΠ΅ с использованиСм 1 ΠΈ 2 Π·Π°ΠΊΠΎΠ½ΠΎΠ² МСндСля ЛСкция 8 Julia Kjahrenova 1. — прСзСнтация ΠŸΡ€Π΅Π·Π΅Π½Ρ‚Π°Ρ†ΠΈΡ Π±Ρ‹Π»Π° ΠΎΠΏΡƒΠ±Π»ΠΈΠΊΠΎΠ²Π°Π½Π° 3 Π³ΠΎΠ΄Π° Π½Π°Π·Π°Π΄ ΠΏΠΎΠ»ΡŒΠ·ΠΎΠ²Π°Ρ‚Π΅Π»Π΅ΠΌΠΠ»ΠΈΠ½Π° ΠΡ€Ρ‚Π΅ΠΌΡŒΠ΅Π²Π° ΠŸΠΎΡ…ΠΎΠΆΠΈΠ΅ ΠΏΡ€Π΅Π·Π΅Π½Ρ‚Π°Ρ†ΠΈΠΈ ΠŸΡ€Π΅Π·Π΅Π½Ρ‚Π°Ρ†ΠΈΡ Π½Π° Ρ‚Π΅ΠΌΡƒ: » РСшСниС Π·Π°Π΄Π°Ρ‡ ΠΏΠΎ Π³Π΅Π½Π΅Ρ‚ΠΈΠΊΠ΅ с использованиСм 1 ΠΈ 2 Π·Π°ΠΊΠΎΠ½ΠΎΠ² МСндСля ЛСкция 8 Julia Kjahrenova 1.» […]
    • 5-7 Π°Π»Π³Π΅Π±Ρ€Π° ΠΏΡ€Π°Π²ΠΈΠ»Π° Π§ΠΈΡΠ»ΠΎΠ²ΡƒΡŽ ΠΏΠΎΡΠ»Π΅Π΄ΠΎΠ²Π°Ρ‚Π΅Π»ΡŒΠ½ΠΎΡΡ‚ΡŒ, ΠΊΠ°ΠΆΠ΄Ρ‹ΠΉ Ρ‡Π»Π΅Π½ ΠΊΠΎΡ‚ΠΎΡ€ΠΎΠΉ, начиная со Π²Ρ‚ΠΎΡ€ΠΎΠ³ΠΎ, Ρ€Π°Π²Π΅Π½ ΠΏΡ€Π΅Π΄Ρ‹Π΄ΡƒΡ‰Π΅ΠΌΡƒ, слоТСнному с ΠΎΠ΄Π½ΠΈΠΌ ΠΈ Ρ‚Π΅ΠΌ ΠΆΠ΅ для Π΄Π°Π½Π½ΠΎΠΉ ΠΏΠΎΡΠ»Π΅Π΄ΠΎΠ²Π°Ρ‚Π΅Π»ΡŒΠ½ΠΎΡΡ‚ΠΈ числом d, Π½Π°Π·Ρ‹Π²Π°ΡŽΡ‚ арифмСтичСской прогрСссиСй. Число d Π½Π°Π·Ρ‹Π²Π°ΡŽΡ‚ Ρ€Π°Π·Π½ΠΎΡΡ‚ΡŒΡŽ арифмСтичСской прогрСссии. Π’ арифмСтичСской прогрСссии, Ρ‚. Π΅. Π² […]
    • ΠžΠΏΡ€Π΅Π΄Π΅Π»ΡΠ΅ΠΌ ставку транспортного Π½Π°Π»ΠΎΠ³Π° для Ρ„ΡƒΡ€Π³ΠΎΠ½ΠΎΠ² ΠΈ Π΄Ρ€ΡƒΠ³ΠΈΡ… Π½Π΅Ρ‚ΠΈΠΏΠΈΡ‡Π½Ρ‹Ρ… Π°Π²Ρ‚ΠΎΠΌΠΎΠ±ΠΈΠ»Π΅ΠΉ с ΠΊΠ°Ρ‚Π΅Π³ΠΎΡ€ΠΈΠ΅ΠΉ «B» Π’Ρ‹Π»Π°Π²Π»ΠΈΠ²Π°Π΅ΠΌ Π½ΡƒΠΆΠ½ΡƒΡŽ ΠΈΠ½Ρ„ΠΎΡ€ΠΌΠ°Ρ†ΠΈΡŽ ΠΈΠ· ПВБ Π‘Ρ€Π°Π·Ρƒ скаТСм, Ρ‡Ρ‚ΠΎ Π΄Π°Π½Π½Ρ‹Π΅, ΡƒΠΊΠ°Π·Π°Π½Π½Ρ‹Π΅ Π² строкС 4 «ΠšΠ°Ρ‚Сгория Π’Π‘ (A, B, C, D, ΠΏΡ€ΠΈΡ†Π΅ΠΏ)» паспорта транспортного срСдства (ПВБ), ΡƒΡ‡ΠΈΡ‚Ρ‹Π²Π°Ρ‚ΡŒ Π½Π΅ Π½ΡƒΠΆΠ½ΠΎ. Π’Π΅Π΄ΡŒ катСгория «B» вовсС Π½Π΅ ΠΎΠ·Π½Π°Ρ‡Π°Π΅Ρ‚, […]
    • Π Π΅ΠΉΡ‚ΠΈΠ½Π³ страховых ΠΊΠΎΠΌΠΏΠ°Π½ΠΈΠΉ ΠžΠ‘ΠΠ“Πž ΠžΠ‘ΠΠ“Πž относится ΠΊ ΠΎΠ±ΡΠ·Π°Ρ‚Π΅Π»ΡŒΠ½ΠΎΠΌΡƒ ΡΡ‚Ρ€Π°Ρ…ΠΎΠ²Π°Π½ΠΈΡŽ, ΠΎΠ½ΠΎ дСйствуСт Π½Π΅ Ρ‚ΠΎΠ»ΡŒΠΊΠΎ Π½Π° Ρ‚Π΅Ρ€Ρ€ΠΈΡ‚ΠΎΡ€ΠΈΠΈ России, Π½ΠΎ ΠΈ Π² Π΄Ρ€ΡƒΠ³ΠΈΡ… странах Π±Π»ΠΈΠΆΠ½Π΅Π³ΠΎ Π·Π°Ρ€ΡƒΠ±Π΅ΠΆΡŒΡ. ΠžΡ„ΠΎΡ€ΠΌΠ»Π΅Π½ΠΈΠ΅ΠΌ Π΄Π°Π½Π½Ρ‹Ρ… полисов Π·Π°Π½ΠΈΠΌΠ°ΡŽΡ‚ΡΡ ΠΌΠ½ΠΎΠ³ΠΈΠ΅ страховыС ΠΊΠΎΠΌΠΏΠ°Π½ΠΈΠΈ, ΠΊΠΎΡ‚ΠΎΡ€Ρ‹Π΅ ΠΏΠΎΠ»ΡƒΡ‡ΠΈΠ»ΠΈ ΡΠΎΠΎΡ‚Π²Π΅Ρ‚ΡΡ‚Π²ΡƒΡŽΡ‰ΡƒΡŽ Π»ΠΈΡ†Π΅Π½Π·ΠΈΡŽ Π½Π° Π²Π΅Π΄Π΅Π½ΠΈΠ΅ ΠΏΠΎΠ΄ΠΎΠ±Π½ΠΎΠΉ Π΄Π΅ΡΡ‚Π΅Π»ΡŒΠ½ΠΎΡΡ‚ΠΈ. Однако, […]
    • ΠŸΡ€ΠΎΠΆΠΈΠ²Π°Π½ΠΈΠ΅ гостиница ΡƒΡ„Π° Мини-ΠΎΡ‚Π΅Π»ΡŒ Π² Π£Ρ„Π΅ 5 Five Rooms ΠŸΡ€ΠΈΠ³Π»Π°ΡˆΠ°Π΅ΠΌ гостСй столицы Π² ΡƒΡŽΡ‚Π½Ρ‹ΠΉ ΠΊΠΎΠΌΡ„ΠΎΡ€Ρ‚Π°Π±Π΅Π»ΡŒΠ½Ρ‹ΠΉ ΠΎΡ‚Π΅Π»ΡŒ, располоТСнный Π² Ρ†Π΅Π½Ρ‚Ρ€Π΅ Π³ΠΎΡ€ΠΎΠ΄Π° Π£Ρ„Π° ΠΏΠΎ ΡƒΠ»ΠΈΡ†Π΅ Комсомольская 159/1. Π’ нСпосрСдствСнной близости ΠΎΡ‚ отСля располоТСны кинокомплСкс Β«Π˜ΡΠΊΡ€Π° IMAXΒ», Ρ†ΠΈΡ€ΠΊ, рСсторан-ΠΊΠ»ΡƒΠ± А ΠΊΠ°Ρ„Π΅, рСсторан Beer Berry, Π’Π Π¦ […]
    • ΠŸΡ€Π°Π²ΠΈΠ»Π° использования Present Simple Tense Π² английском языкС Present Simple Tense – это грамматичСскоС врСмя, ΠΊΠΎΡ‚ΠΎΡ€ΠΎΠ΅ считаСтся ΠΎΠ΄Π½ΠΈΠΌ ΠΈΠ· самых простых Π² ΠΏΠΎΠ½ΠΈΠΌΠ°Π½ΠΈΠΈ, ΠΏΠΎΡΠΊΠΎΠ»ΡŒΠΊΡƒ, настоящСС простоС врСмя сущСствуСт Π²ΠΎ всСх языках. Π’ славянских языках Ρ‚Π°ΠΊ Ρ‚ΠΎΡ‡Π½ΠΎ. Если Π²Ρ‹ Ρ‡ΠΈΡ‚Π°Π΅Ρ‚Π΅ эту ΡΡ‚Π°Ρ‚ΡŒΡŽ, это Π·Π½Π°Ρ‡ΠΈΡ‚, Ρ‡Ρ‚ΠΎ Π²Ρ‹ Ρ‚ΠΎΠ»ΡŒΠΊΠΎ […]

    Как ΡƒΠΌΠ½ΠΎΠΆΠ°Ρ‚ΡŒ стСпСни? КакиС стСпСни ΠΌΠΎΠΆΠ½ΠΎ ΠΏΠ΅Ρ€Π΅ΠΌΠ½ΠΎΠΆΠΈΡ‚ΡŒ, Π° ΠΊΠ°ΠΊΠΈΠ΅ — Π½Π΅Ρ‚? Как число ΡƒΠΌΠ½ΠΎΠΆΠΈΡ‚ΡŒ Π½Π° ΡΡ‚Π΅ΠΏΠ΅Π½ΡŒ?

    Π’ Π°Π»Π³Π΅Π±Ρ€Π΅ Π½Π°ΠΉΡ‚ΠΈ ΠΏΡ€ΠΎΠΈΠ·Π²Π΅Π΄Π΅Π½ΠΈΠ΅ стСпСнСй ΠΌΠΎΠΆΠ½ΠΎ Π² Π΄Π²ΡƒΡ… случаях:

    1) Ссли стСпСни ΠΈΠΌΠ΅ΡŽΡ‚ ΠΎΠ΄ΠΈΠ½Π°ΠΊΠΎΠ²Ρ‹Π΅ основания;

    2) Ссли стСпСни ΠΈΠΌΠ΅ΡŽΡ‚ ΠΎΠ΄ΠΈΠ½Π°ΠΊΠΎΠ²Ρ‹Π΅ ΠΏΠΎΠΊΠ°Π·Π°Ρ‚Π΅Π»ΠΈ.

    ΠŸΡ€ΠΈ ΡƒΠΌΠ½ΠΎΠΆΠ΅Π½ΠΈΠΈ стСпСнСй с ΠΎΠ΄ΠΈΠ½Π°ΠΊΠΎΠ²Ρ‹ΠΌΠΈ основаниями Π½Π°Π΄ΠΎ основаниС ΠΎΡΡ‚Π°Π²ΠΈΡ‚ΡŒ ΠΏΡ€Π΅ΠΆΠ½ΠΈΠΌ, Π° ΠΏΠΎΠΊΠ°Π·Π°Ρ‚Π΅Π»ΠΈ — ΡΠ»ΠΎΠΆΠΈΡ‚ΡŒ:

    ΠŸΡ€ΠΈ ΡƒΠΌΠ½ΠΎΠΆΠ΅Π½ΠΈΠΈ стСпСнСй с ΠΎΠ΄ΠΈΠ½Π°ΠΊΠΎΠ²Ρ‹ΠΌΠΈ показатСлями ΠΎΠ±Ρ‰ΠΈΠΉ ΠΏΠΎΠΊΠ°Π·Π°Ρ‚Π΅Π»ΡŒ ΠΌΠΎΠΆΠ½ΠΎ вынСсти Π·Π° скобки:

    Рассмотрим, ΠΊΠ°ΠΊ ΡƒΠΌΠ½ΠΎΠΆΠ°Ρ‚ΡŒ стСпСни, Π½Π° ΠΊΠΎΠ½ΠΊΡ€Π΅Ρ‚Π½Ρ‹Ρ… ΠΏΡ€ΠΈΠΌΠ΅Ρ€Π°Ρ….

    Π•Π΄ΠΈΠ½ΠΈΡ†Ρƒ Π² ΠΏΠΎΠΊΠ°Π·Π°Ρ‚Π΅Π»Π΅ стСпСни Π½Π΅ ΠΏΠΈΡˆΡƒΡ‚, Π½ΠΎ ΠΏΡ€ΠΈ ΡƒΠΌΠ½ΠΎΠΆΠ΅Π½ΠΈΠΈ стСпСнСй — ΡƒΡ‡ΠΈΡ‚Ρ‹Π²Π°ΡŽΡ‚:

    ΠŸΡ€ΠΈ ΡƒΠΌΠ½ΠΎΠΆΠ΅Π½ΠΈΠΈ количСство стСпСнСй ΠΌΠΎΠΆΠ΅Ρ‚ Π±Ρ‹Ρ‚ΡŒ любоС. Π‘Π»Π΅Π΄ΡƒΠ΅Ρ‚ ΠΏΠΎΠΌΠ½ΠΈΡ‚ΡŒ, Ρ‡Ρ‚ΠΎ ΠΏΠ΅Ρ€Π΅Π΄ Π±ΡƒΠΊΠ²ΠΎΠΉ Π·Π½Π°ΠΊ умноТСния ΠΌΠΎΠΆΠ½ΠΎ Π½Π΅ ΠΏΠΈΡΠ°Ρ‚ΡŒ:

    Π’ выраТСниях Π²ΠΎΠ·Π²Π΅Π΄Π΅Π½ΠΈΠ΅ Π² ΡΡ‚Π΅ΠΏΠ΅Π½ΡŒ выполняСтся Π² ΠΏΠ΅Ρ€Π²ΡƒΡŽ ΠΎΡ‡Π΅Ρ€Π΅Π΄ΡŒ.

    Если Π½ΡƒΠΆΠ½ΠΎ число ΡƒΠΌΠ½ΠΎΠΆΠΈΡ‚ΡŒ Π½Π° ΡΡ‚Π΅ΠΏΠ΅Π½ΡŒ, сначала слСдуСт Π²Ρ‹ΠΏΠΎΠ»Π½ΠΈΡ‚ΡŒ Π²ΠΎΠ·Π²Π΅Π΄Π΅Π½ΠΈΠ΅ Π² ΡΡ‚Π΅ΠΏΠ΅Π½ΡŒ, Π° ΡƒΠΆΠ΅ ΠΏΠΎΡ‚ΠΎΠΌ — ΡƒΠΌΠ½ΠΎΠΆΠ΅Π½ΠΈΠ΅:

    www.algebraclass.ru

    Π‘Π»ΠΎΠΆΠ΅Π½ΠΈΠ΅, Π²Ρ‹Ρ‡ΠΈΡ‚Π°Π½ΠΈΠ΅, ΡƒΠΌΠ½ΠΎΠΆΠ΅Π½ΠΈΠ΅, ΠΈ Π΄Π΅Π»Π΅Π½ΠΈΠ΅ стСпСнСй

    Π‘Π»ΠΎΠΆΠ΅Π½ΠΈΠ΅ ΠΈ Π²Ρ‹Ρ‡ΠΈΡ‚Π°Π½ΠΈΠ΅ стСпСнСй

    ΠžΡ‡Π΅Π²ΠΈΠ΄Π½ΠΎ, Ρ‡Ρ‚ΠΎ числа со стСпСнями ΠΌΠΎΠ³ΡƒΡ‚ ΡΠ»Π°Π³Π°Ρ‚ΡŒΡΡ, ΠΊΠ°ΠΊ Π΄Ρ€ΡƒΠ³ΠΈΠ΅ Π²Π΅Π»ΠΈΡ‡ΠΈΠ½Ρ‹ , ΠΏΡƒΡ‚Π΅ΠΌ ΠΈΡ… слоТСния ΠΎΠ΄Π½ΠΎ Π·Π° Π΄Ρ€ΡƒΠ³ΠΈΠΌ со своими Π·Π½Π°ΠΊΠ°ΠΌΠΈ .

    Π’Π°ΠΊ, сумма a 3 ΠΈ b 2 Π΅ΡΡ‚ΡŒ a 3 + b 2 .
    Π‘ΡƒΠΌΠΌΠ° a 3 β€” b n ΠΈ h 5 -d 4 Π΅ΡΡ‚ΡŒ a 3 β€” b n + h 5 β€” d 4 .

    ΠšΠΎΡΡ„Ρ„ΠΈΡ†ΠΈΠ΅Π½Ρ‚Ρ‹ ΠΎΠ΄ΠΈΠ½Π°ΠΊΠΎΠ²Ρ‹Ρ… стСпСнСй ΠΎΠ΄ΠΈΠ½Π°ΠΊΠΎΠ²Ρ‹Ρ… ΠΏΠ΅Ρ€Π΅ΠΌΠ΅Π½Π½Ρ‹Ρ… ΠΌΠΎΠ³ΡƒΡ‚ ΡΠ»Π°Π³Π°Ρ‚ΡŒΡΡ ΠΈΠ»ΠΈ Π²Ρ‹Ρ‡ΠΈΡ‚Π°Ρ‚ΡŒΡΡ.

    Π’Π°ΠΊ, сумма 2a 2 ΠΈ 3a 2 Ρ€Π°Π²Π½Π° 5a 2 .

    Π­Ρ‚ΠΎ Ρ‚Π°ΠΊ ΠΆΠ΅ ΠΎΡ‡Π΅Π²ΠΈΠ΄Π½ΠΎ, Ρ‡Ρ‚ΠΎ Ссли Π²Π·ΡΡ‚ΡŒ Π΄Π²Π° ΠΊΠ²Π°Π΄Ρ€Π°Ρ‚Π° Π°, ΠΈΠ»ΠΈ Ρ‚Ρ€ΠΈ ΠΊΠ²Π°Π΄Ρ€Π°Ρ‚Π° Π°, ΠΈΠ»ΠΈ ΠΏΡΡ‚ΡŒ ΠΊΠ²Π°Π΄Ρ€Π°Ρ‚ΠΎΠ² Π°.

    Но стСпСни Ρ€Π°Π·Π»ΠΈΡ‡Π½Ρ‹Ρ… ΠΏΠ΅Ρ€Π΅ΠΌΠ΅Π½Π½Ρ‹Ρ… ΠΈ Ρ€Π°Π·Π»ΠΈΡ‡Π½Ρ‹Π΅ стСпСни ΠΎΠ΄ΠΈΠ½Π°ΠΊΠΎΠ²Ρ‹Ρ… ΠΏΠ΅Ρ€Π΅ΠΌΠ΅Π½Π½Ρ‹Ρ… , Π΄ΠΎΠ»ΠΆΠ½Ρ‹ ΡΠ»Π°Π³Π°Ρ‚ΡŒΡΡ ΠΈΡ… слоТСниСм с ΠΈΡ… Π·Π½Π°ΠΊΠ°ΠΌΠΈ.

    Π’Π°ΠΊ, сумма a 2 ΠΈ a 3 Π΅ΡΡ‚ΡŒ сумма a 2 + a 3 .

    Π­Ρ‚ΠΎ ΠΎΡ‡Π΅Π²ΠΈΠ΄Π½ΠΎ, Ρ‡Ρ‚ΠΎ ΠΊΠ²Π°Π΄Ρ€Π°Ρ‚ числа a, ΠΈ ΠΊΡƒΠ± числа a, Π½Π΅ Ρ€Π°Π²Π½ΠΎ Π½ΠΈ ΡƒΠ΄Π²ΠΎΠ΅Π½Π½ΠΎΠΌΡƒ ΠΊΠ²Π°Π΄Ρ€Π°Ρ‚Ρƒ a, Π½ΠΎ ΡƒΠ΄Π²ΠΎΠ΅Π½Π½ΠΎΠΌΡƒ ΠΊΡƒΠ±Ρƒ a.

    Π‘ΡƒΠΌΠΌΠ° a 3 b n ΠΈ 3a 5 b 6 Π΅ΡΡ‚ΡŒ a 3 b n + 3a 5 b 6 .

    Π’Ρ‹Ρ‡ΠΈΡ‚Π°Π½ΠΈΠ΅ стСпСнСй проводится Ρ‚Π°ΠΊΠΈΠΌ ΠΆΠ΅ ΠΎΠ±Ρ€Π°Π·ΠΎΠΌ, Ρ‡Ρ‚ΠΎ ΠΈ слоТСниС, Π·Π° ΠΈΡΠΊΠ»ΡŽΡ‡Π΅Π½ΠΈΠ΅ΠΌ Ρ‚ΠΎΠ³ΠΎ, Ρ‡Ρ‚ΠΎ Π·Π½Π°ΠΊΠΈ Π²Ρ‹Ρ‡ΠΈΡ‚Π°Π΅ΠΌΡ‹Ρ… Π΄ΠΎΠ»ΠΆΠ½Ρ‹ соотвСтствСнно Π±Ρ‹Ρ‚ΡŒ ΠΈΠ·ΠΌΠ΅Π½Π΅Π½Ρ‹.

    Или:
    2a 4 β€” (-6a 4) = 8a 4
    3h 2 b 6 β€” 4h 2 b 6 = -h 2 b 6
    5(a β€” h) 6 β€” 2(a β€” h) 6 = 3(a β€” h) 6

    Π£ΠΌΠ½ΠΎΠΆΠ΅Π½ΠΈΠ΅ стСпСнСй

    Числа со стСпСнями ΠΌΠΎΠ³ΡƒΡ‚ Π±Ρ‹Ρ‚ΡŒ ΡƒΠΌΠ½ΠΎΠΆΠ΅Π½Ρ‹, ΠΊΠ°ΠΊ ΠΈ Π΄Ρ€ΡƒΠ³ΠΈΠ΅ Π²Π΅Π»ΠΈΡ‡ΠΈΠ½Ρ‹, ΠΏΡƒΡ‚Π΅ΠΌ написания ΠΈΡ… ΠΎΠ΄Π½ΠΎ Π·Π° Π΄Ρ€ΡƒΠ³ΠΈΠΌ, со Π·Π½Π°ΠΊΠΎΠΌ умноТСния ΠΈΠ»ΠΈ Π±Π΅Π· Π½Π΅Π³ΠΎ ΠΌΠ΅ΠΆΠ΄Ρƒ Π½ΠΈΠΌΠΈ.

    Π’Π°ΠΊ, Ρ€Π΅Π·ΡƒΠ»ΡŒΡ‚Π°Ρ‚ умноТСния a 3 Π½Π° b 2 Ρ€Π°Π²Π΅Π½ a 3 b 2 ΠΈΠ»ΠΈ aaabb.

    Или:
    x -3 β‹… a m = a m x -3
    3a 6 y 2 β‹… (-2x) = -6a 6 xy 2
    a 2 b 3 y 2 β‹… a 3 b 2 y = a 2 b 3 y 2 a 3 b 2 y

    Π Π΅Π·ΡƒΠ»ΡŒΡ‚Π°Ρ‚ Π² послСднСм ΠΏΡ€ΠΈΠΌΠ΅Ρ€Π΅ ΠΌΠΎΠΆΠ΅Ρ‚ Π±Ρ‹Ρ‚ΡŒ упорядочСн ΠΏΡƒΡ‚Ρ‘ΠΌ слоТСния ΠΎΠ΄ΠΈΠ½Π°ΠΊΠΎΠ²Ρ‹Ρ… ΠΏΠ΅Ρ€Π΅ΠΌΠ΅Π½Π½Ρ‹Ρ….
    Π’Ρ‹Ρ€Π°ΠΆΠ΅Π½ΠΈΠ΅ ΠΏΡ€ΠΈΠΌΠ΅Ρ‚ Π²ΠΈΠ΄: a 5 b 5 y 3 .

    Бравнивая нСсколько чисСл(ΠΏΠ΅Ρ€Π΅ΠΌΠ΅Π½Π½Ρ‹Ρ…) со стСпСнями, ΠΌΡ‹ ΠΌΠΎΠΆΠ΅ΠΌ ΡƒΠ²ΠΈΠ΄Π΅Ρ‚ΡŒ, Ρ‡Ρ‚ΠΎ Ссли Π»ΡŽΠ±Ρ‹Π΅ Π΄Π²Π° ΠΈΠ· Π½ΠΈΡ… ΡƒΠΌΠ½ΠΎΠΆΠ°ΡŽΡ‚ΡΡ, Ρ‚ΠΎ Ρ€Π΅Π·ΡƒΠ»ΡŒΡ‚Π°Ρ‚ β€” это число (пСрСмСнная) со ΡΡ‚Π΅ΠΏΠ΅Π½ΡŒΡŽ, Ρ€Π°Π²Π½ΠΎΠΉ суммС стСпСнСй слагаСмых.

    Π’Π°ΠΊ, a 2 .a 3 = aa.aaa = aaaaa = a 5 .

    Π—Π΄Π΅ΡΡŒ 5 β€” это ΡΡ‚Π΅ΠΏΠ΅Π½ΡŒ Ρ€Π΅Π·ΡƒΠ»ΡŒΡ‚Π°Ρ‚Π° умноТСния, равная 2 + 3, суммС стСпСнСй слагаСмых.

    Π’Π°ΠΊ, a n .a m = a m+n .

    Для a n , a бСрётся ΠΊΠ°ΠΊ ΠΌΠ½ΠΎΠΆΠΈΡ‚Π΅Π»ΡŒ ΡΡ‚ΠΎΠ»ΡŒΠΊΠΎ Ρ€Π°Π·, сколько Ρ€Π°Π²Π½Π° ΡΡ‚Π΅ΠΏΠ΅Π½ΡŒ n;

    И a m , бСрётся ΠΊΠ°ΠΊ ΠΌΠ½ΠΎΠΆΠΈΡ‚Π΅Π»ΡŒ ΡΡ‚ΠΎΠ»ΡŒΠΊΠΎ Ρ€Π°Π·, сколько Ρ€Π°Π²Π½Π° ΡΡ‚Π΅ΠΏΠ΅Π½ΡŒ m;

    ΠŸΠΎΡΡ‚ΠΎΠΌΡƒ, стСпСни с ΠΎΠ΄ΠΈΠ½Π°ΠΊΠΎΠ²Ρ‹ΠΌΠΈ основами ΠΌΠΎΠ³ΡƒΡ‚ Π±Ρ‹Ρ‚ΡŒ ΡƒΠΌΠ½ΠΎΠΆΠ΅Π½Ρ‹ ΠΏΡƒΡ‚Ρ‘ΠΌ слоТСния ΠΏΠΎΠΊΠ°Π·Π°Ρ‚Π΅Π»Π΅ΠΉ стСпСнСй.

    Вак, a 2 .a 6 = a 2+6 = a 8 . И x 3 .x 2 .x = x 3+2+1 = x 6 .

    Или:
    4a n β‹… 2a n = 8a 2n
    b 2 y 3 β‹… b 4 y = b 6 y 4
    (b + h β€” y) n β‹… (b + h β€” y) = (b + h β€” y) n+1

    Π£ΠΌΠ½ΠΎΠΆΡŒΡ‚Π΅ (x 3 + x 2 y + xy 2 + y 3) β‹… (x β€” y).
    ΠžΡ‚Π²Π΅Ρ‚: x 4 β€” y 4 .
    Π£ΠΌΠ½ΠΎΠΆΡŒΡ‚Π΅ (x 3 + x β€” 5) β‹… (2x 3 + x + 1).

    Π­Ρ‚ΠΎ ΠΏΡ€Π°Π²ΠΈΠ»ΠΎ справСдливо ΠΈ для чисСл, ΠΏΠΎΠΊΠ°Π·Π°Ρ‚Π΅Π»ΠΈ стСпСни ΠΊΠΎΡ‚ΠΎΡ€Ρ‹Ρ… β€” ΠΎΡ‚Ρ€ΠΈΡ†Π°Ρ‚Π΅Π»ΡŒΠ½Ρ‹Π΅ .

    1. Π’Π°ΠΊ, a -2 .a -3 = a -5 . Π­Ρ‚ΠΎ ΠΌΠΎΠΆΠ½ΠΎ Π·Π°ΠΏΠΈΡΠ°Ρ‚ΡŒ Π² Π²ΠΈΠ΄Π΅ (1/aa).(1/aaa) = 1/aaaaa.

    2. y -n .y -m = y -n-m .

    3. a -n .a m = a m-n .

    Если a + b ΡƒΠΌΠ½ΠΎΠΆΠ°ΡŽΡ‚ΡΡ Π½Π° a β€” b, Ρ€Π΅Π·ΡƒΠ»ΡŒΡ‚Π°Ρ‚ Π±ΡƒΠ΄Π΅Ρ‚ Ρ€Π°Π²Π΅Π½ a 2 β€” b 2: Ρ‚ΠΎ Π΅ΡΡ‚ΡŒ

    Π Π΅Π·ΡƒΠ»ΡŒΡ‚Π°Ρ‚ умноТСния суммы ΠΈΠ»ΠΈ Ρ€Π°Π·Π½ΠΈΡ†Ρ‹ Π΄Π²ΡƒΡ… чисСл Ρ€Π°Π²Π΅Π½ суммС ΠΈΠ»ΠΈ Ρ€Π°Π·Π½ΠΈΡ†Π΅ ΠΈΡ… ΠΊΠ²Π°Π΄Ρ€Π°Ρ‚ΠΎΠ².

    Если умноТаСтся сумма ΠΈ Ρ€Π°Π·Π½ΠΈΡ†Π° Π΄Π²ΡƒΡ… чисСл, Π²ΠΎΠ·Π²Π΅Π΄Ρ‘Π½Π½Ρ‹Ρ… Π² ΠΊΠ²Π°Π΄Ρ€Π°Ρ‚ , Ρ€Π΅Π·ΡƒΠ»ΡŒΡ‚Π°Ρ‚ Π±ΡƒΠ΄Π΅Ρ‚ Ρ€Π°Π²Π΅Π½ суммС ΠΈΠ»ΠΈ Ρ€Π°Π·Π½ΠΈΡ†Π΅ этих чисСл Π² Ρ‡Π΅Ρ‚Π²Ρ‘Ρ€Ρ‚ΠΎΠΉ стСпСни.

    Π’Π°ΠΊ, (a β€” y).(a + y) = a 2 β€” y 2 .
    (a 2 β€” y 2)β‹…(a 2 + y 2) = a 4 β€” y 4 .
    (a 4 β€” y 4)β‹…(a 4 + y 4) = a 8 β€” y 8 .

    Π”Π΅Π»Π΅Π½ΠΈΠ΅ стСпСнСй

    Числа со стСпСнями ΠΌΠΎΠ³ΡƒΡ‚ Π±Ρ‹Ρ‚ΡŒ ΠΏΠΎΠ΄Π΅Π»Π΅Π½Ρ‹, ΠΊΠ°ΠΊ ΠΈ Π΄Ρ€ΡƒΠ³ΠΈΠ΅ числа, ΠΏΡƒΡ‚Π΅ΠΌ отнимая ΠΎΡ‚ Π΄Π΅Π»ΠΈΠΌΠΎΠ³ΠΎ дСлитСля, ΠΈΠ»ΠΈ Ρ€Π°Π·ΠΌΠ΅Ρ‰Π΅Π½ΠΈΠ΅ΠΌ ΠΈΡ… Π² Ρ„ΠΎΡ€ΠΌΠ΅ Π΄Ρ€ΠΎΠ±ΠΈ. 3$

    НСобходимо ΠΎΡ‡Π΅Π½ΡŒ Ρ…ΠΎΡ€ΠΎΡˆΠΎ ΡƒΡΠ²ΠΎΠΈΡ‚ΡŒ ΡƒΠΌΠ½ΠΎΠΆΠ΅Π½ΠΈΠ΅ ΠΈ Π΄Π΅Π»Π΅Π½ΠΈΠ΅ стСпСнСй, Ρ‚Π°ΠΊ ΠΊΠ°ΠΊ Ρ‚Π°ΠΊΠΈΠ΅ ΠΎΠΏΠ΅Ρ€Π°Ρ†ΠΈΠΈ ΠΎΡ‡Π΅Π½ΡŒ ΡˆΠΈΡ€ΠΎΠΊΠΎ ΠΏΡ€ΠΈΠΌΠ΅Π½ΡΡŽΡ‚ΡΡ Π² Π°Π»Π³Π΅Π±Ρ€Π΅.

    ΠŸΡ€ΠΈΠΌΠ΅Ρ€Ρ‹ Ρ€Π΅ΡˆΠ΅Π½ΠΈΡ ΠΏΡ€ΠΈΠΌΠ΅Ρ€ΠΎΠ² с дробями, содСрТащими числа со стСпСнями

    1. Π£ΠΌΠ΅Π½ΡŒΡˆΠΈΡ‚Π΅ ΠΏΠΎΠΊΠ°Π·Π°Ρ‚Π΅Π»ΠΈ стСпСнСй Π² $\frac $ ΠžΡ‚Π²Π΅Ρ‚: $\frac $.

    2. Π£ΠΌΠ΅Π½ΡŒΡˆΠΈΡ‚Π΅ ΠΏΠΎΠΊΠ°Π·Π°Ρ‚Π΅Π»ΠΈ стСпСнСй Π² $\frac $. ΠžΡ‚Π²Π΅Ρ‚: $\frac $ ΠΈΠ»ΠΈ 2x.

    3. Π£ΠΌΠ΅Π½ΡŒΡˆΠΈΡ‚Π΅ ΠΏΠΎΠΊΠ°Π·Π°Ρ‚Π΅Π»ΠΈ стСпСнСй a 2 /a 3 ΠΈ a -3 /a -4 ΠΈ ΠΏΡ€ΠΈΠ²Π΅Π΄ΠΈΡ‚Π΅ ΠΊ ΠΎΠ±Ρ‰Π΅ΠΌΡƒ Π·Π½Π°ΠΌΠ΅Π½Π°Ρ‚Π΅Π»ΡŽ.
    a 2 .a -4 Π΅ΡΡ‚ΡŒ a -2 ΠΏΠ΅Ρ€Π²Ρ‹ΠΉ Ρ‡ΠΈΡΠ»ΠΈΡ‚Π΅Π»ΡŒ.
    a 3 .a -3 Π΅ΡΡ‚ΡŒ a 0 = 1, Π²Ρ‚ΠΎΡ€ΠΎΠΉ Ρ‡ΠΈΡΠ»ΠΈΡ‚Π΅Π»ΡŒ.
    a 3 .a -4 Π΅ΡΡ‚ΡŒ a -1 , ΠΎΠ±Ρ‰ΠΈΠΉ Ρ‡ΠΈΡΠ»ΠΈΡ‚Π΅Π»ΡŒ.
    ПослС упрощСния: a -2 /a -1 ΠΈ 1/a -1 .

    4. Π£ΠΌΠ΅Π½ΡŒΡˆΠΈΡ‚Π΅ ΠΏΠΎΠΊΠ°Π·Π°Ρ‚Π΅Π»ΠΈ стСпСнСй 2a 4 /5a 3 ΠΈ 2 /a 4 ΠΈ ΠΏΡ€ΠΈΠ²Π΅Π΄ΠΈΡ‚Π΅ ΠΊ ΠΎΠ±Ρ‰Π΅ΠΌΡƒ Π·Π½Π°ΠΌΠ΅Π½Π°Ρ‚Π΅Π»ΡŽ.
    ΠžΡ‚Π²Π΅Ρ‚: 2a 3 /5a 7 ΠΈ 5a 5 /5a 7 ΠΈΠ»ΠΈ 2a 3 /5a 2 ΠΈ 5/5a 2 .

    5. Π£ΠΌΠ½ΠΎΠΆΡŒΡ‚Π΅ (a 3 + b)/b 4 Π½Π° (a β€” b)/3.

    6. Π£ΠΌΠ½ΠΎΠΆΡŒΡ‚Π΅ (a 5 + 1)/x 2 Π½Π° (b 2 β€” 1)/(x + a).

    7. Π£ΠΌΠ½ΠΎΠΆΡŒΡ‚Π΅ b 4 /a -2 Π½Π° h -3 /x ΠΈ a n /y -3 .

    8. Π Π°Π·Π΄Π΅Π»ΠΈΡ‚Π΅ a 4 /y 3 Π½Π° a 3 /y 2 . ΠžΡ‚Π²Π΅Ρ‚: a/y.

    Бвойства стСпСни

    НапоминаСм, Ρ‡Ρ‚ΠΎ Π² Π΄Π°Π½Π½ΠΎΠΌ ΡƒΡ€ΠΎΠΊΠ΅ Ρ€Π°Π·Π±ΠΈΡ€Π°ΡŽΡ‚ΡΡ свойства стСпСнСй с Π½Π°Ρ‚ΡƒΡ€Π°Π»ΡŒΠ½Ρ‹ΠΌΠΈ показатСлями ΠΈ Π½ΡƒΠ»Ρ‘ΠΌ. Π‘Ρ‚Π΅ΠΏΠ΅Π½ΠΈ с Ρ€Π°Ρ†ΠΈΠΎΠ½Π°Π»ΡŒΠ½Ρ‹ΠΌΠΈ показатСлями ΠΈ ΠΈΡ… свойства Π±ΡƒΠ΄ΡƒΡ‚ рассмотрСны Π² ΡƒΡ€ΠΎΠΊΠ°Ρ… для 8 классов.

    Π‘Ρ‚Π΅ΠΏΠ΅Π½ΡŒ с Π½Π°Ρ‚ΡƒΡ€Π°Π»ΡŒΠ½Ρ‹ΠΌ ΠΏΠΎΠΊΠ°Π·Π°Ρ‚Π΅Π»Π΅ΠΌ ΠΎΠ±Π»Π°Π΄Π°Π΅Ρ‚ нСсколькими Π²Π°ΠΆΠ½Ρ‹ΠΌΠΈ свойствами, ΠΊΠΎΡ‚ΠΎΡ€Ρ‹Π΅ ΠΏΠΎΠ·Π²ΠΎΠ»ΡΡŽΡ‚ ΡƒΠΏΡ€ΠΎΡ‰Π°Ρ‚ΡŒ вычислСния Π² ΠΏΡ€ΠΈΠΌΠ΅Ρ€Π°Ρ… со стСпСнями.

    Бвойство β„– 1


    ΠŸΡ€ΠΎΠΈΠ·Π²Π΅Π΄Π΅Π½ΠΈΠ΅ стСпСнСй

    ΠŸΡ€ΠΈ ΡƒΠΌΠ½ΠΎΠΆΠ΅Π½ΠΈΠΈ стСпСнСй с ΠΎΠ΄ΠΈΠ½Π°ΠΊΠΎΠ²Ρ‹ΠΌΠΈ основаниями основаниС остаётся Π±Π΅Π· ΠΈΠ·ΠΌΠ΅Π½Π΅Π½ΠΈΠΉ, Π° ΠΏΠΎΠΊΠ°Π·Π°Ρ‚Π΅Π»ΠΈ стСпСнСй ΡΠΊΠ»Π°Π΄Ρ‹Π²Π°ΡŽΡ‚ΡΡ.

    a m Β· a n = a m + n , Π³Π΄Π΅ Β« a Β» — любоС число, Π° Β« m Β», Β« n Β» — Π»ΡŽΠ±Ρ‹Π΅ Π½Π°Ρ‚ΡƒΡ€Π°Π»ΡŒΠ½Ρ‹Π΅ числа.

    Π”Π°Π½Π½ΠΎΠ΅ свойство стСпСнСй Ρ‚Π°ΠΊΠΆΠ΅ дСйствуСт Π½Π° ΠΏΡ€ΠΎΠΈΠ·Π²Π΅Π΄Π΅Π½ΠΈΠ΅ Ρ‚Ρ€Ρ‘Ρ… ΠΈ Π±ΠΎΠ»Π΅Π΅ стСпСнСй.

  • Π£ΠΏΡ€ΠΎΡΡ‚ΠΈΡ‚ΡŒ Π²Ρ‹Ρ€Π°ΠΆΠ΅Π½ΠΈΠ΅.
    b Β· b 2 Β· b 3 Β· b 4 Β· b 5 = b 1 + 2 + 3 + 4 + 5 = b 15
  • ΠŸΡ€Π΅Π΄ΡΡ‚Π°Π²ΠΈΡ‚ΡŒ Π² Π²ΠΈΠ΄Π΅ стСпСни.
    6 15 Β· 36 = 6 15 Β· 6 2 = 6 15 Β· 6 2 = 6 17
  • ΠŸΡ€Π΅Π΄ΡΡ‚Π°Π²ΠΈΡ‚ΡŒ Π² Π²ΠΈΠ΄Π΅ стСпСни.
    (0,8) 3 Β· (0,8) 12 = (0,8) 3 + 12 = (0,8) 15
  • ΠžΠ±Ρ€Π°Ρ‚ΠΈΡ‚Π΅ Π²Π½ΠΈΠΌΠ°Π½ΠΈΠ΅, Ρ‡Ρ‚ΠΎ Π² ΡƒΠΊΠ°Π·Π°Π½Π½ΠΎΠΌ свойствС Ρ€Π΅Ρ‡ΡŒ шла Ρ‚ΠΎΠ»ΡŒΠΊΠΎ ΠΎΠ± ΡƒΠΌΠ½ΠΎΠΆΠ΅Π½ΠΈΠΈ стСпСнСй с ΠΎΠ΄ΠΈΠ½Π°ΠΊΠΎΠ²Ρ‹ΠΌΠΈ основаниями . Оно Π½Π΅ относится ΠΊ ΠΈΡ… слоТСнию.

    НСльзя Π·Π°ΠΌΠ΅Π½ΡΡ‚ΡŒ сумму (3 3 + 3 2) Π½Π° 3 5 . Π­Ρ‚ΠΎ понятно, Ссли
    ΠΏΠΎΡΡ‡ΠΈΡ‚Π°Ρ‚ΡŒ (3 3 + 3 2) = (27 + 9) = 36 , Π° 3 5 = 243

    Бвойство β„– 2


    ЧастноС стСпСнСй

    ΠŸΡ€ΠΈ Π΄Π΅Π»Π΅Π½ΠΈΠΈ стСпСнСй с ΠΎΠ΄ΠΈΠ½Π°ΠΊΠΎΠ²Ρ‹ΠΌΠΈ основаниями основаниС остаётся Π±Π΅Π· ΠΈΠ·ΠΌΠ΅Π½Π΅Π½ΠΈΠΉ, Π° ΠΈΠ· показатСля стСпСни Π΄Π΅Π»ΠΈΠΌΠΎΠ³ΠΎ Π²Ρ‹Ρ‡ΠΈΡ‚Π°ΡŽΡ‚ ΠΏΠΎΠΊΠ°Π·Π°Ρ‚Π΅Π»ΡŒ стСпСни дСлитСля.

  • Π—Π°ΠΏΠΈΡΠ°Ρ‚ΡŒ частноС Π² Π²ΠΈΠ΄Π΅ стСпСни
    (2b) 5: (2b) 3 = (2b) 5 βˆ’ 3 = (2b) 2
  • Π’Ρ‹Ρ‡ΠΈΡΠ»ΠΈΡ‚ΡŒ.

    11 3 βˆ’ 2 Β· 4 2 βˆ’ 1 = 11 Β· 4 = 44
    ΠŸΡ€ΠΈΠΌΠ΅Ρ€. Π Π΅ΡˆΠΈΡ‚ΡŒ ΡƒΡ€Π°Π²Π½Π΅Π½ΠΈΠ΅. Π˜ΡΠΏΠΎΠ»ΡŒΠ·ΡƒΠ΅ΠΌ свойство частного стСпСнСй.
    3 8: t = 3 4

    ΠžΡ‚Π²Π΅Ρ‚: t = 3 4 = 81

    ΠŸΠΎΠ»ΡŒΠ·ΡƒΡΡΡŒ свойствами β„– 1 ΠΈ β„– 2, ΠΌΠΎΠΆΠ½ΠΎ Π»Π΅Π³ΠΊΠΎ ΡƒΠΏΡ€ΠΎΡ‰Π°Ρ‚ΡŒ выраТСния ΠΈ ΠΏΡ€ΠΎΠΈΠ·Π²ΠΎΠ΄ΠΈΡ‚ΡŒ вычислСния.

      ΠŸΡ€ΠΈΠΌΠ΅Ρ€. Π£ΠΏΡ€ΠΎΡΡ‚ΠΈΡ‚ΡŒ Π²Ρ‹Ρ€Π°ΠΆΠ΅Π½ΠΈΠ΅.
      4 5m + 6 Β· 4 m + 2: 4 4m + 3 = 4 5m + 6 + m + 2: 4 4m + 3 = 4 6m + 8 βˆ’ 4m βˆ’ 3 = 4 2m + 5

    ΠŸΡ€ΠΈΠΌΠ΅Ρ€. Найти Π·Π½Π°Ρ‡Π΅Π½ΠΈΠ΅ выраТСния, ΠΈΡΠΏΠΎΠ»ΡŒΠ·ΡƒΡ свойства стСпСни.

    2 11 βˆ’ 5 = 2 6 = 64

    ΠžΠ±Ρ€Π°Ρ‚ΠΈΡ‚Π΅ Π²Π½ΠΈΠΌΠ°Π½ΠΈΠ΅, Ρ‡Ρ‚ΠΎ Π² свойствС 2 Ρ€Π΅Ρ‡ΡŒ шла Ρ‚ΠΎΠ»ΡŒΠΊΠΎ ΠΎ Π΄Π΅Π»Π΅Π½ΠΈΠΈ стСпСнСй с ΠΎΠ΄ΠΈΠ½Π°ΠΊΠΎΠ²Ρ‹ΠΌΠΈ основаниями.

    НСльзя Π·Π°ΠΌΠ΅Π½ΡΡ‚ΡŒ Ρ€Π°Π·Π½ΠΎΡΡ‚ΡŒ (4 3 βˆ’4 2) Π½Π° 4 1 . Π­Ρ‚ΠΎ понятно, Ссли ΠΏΠΎΡΡ‡ΠΈΡ‚Π°Ρ‚ΡŒ (4 3 βˆ’4 2) = (64 βˆ’ 16) = 48 , Π° 4 1 = 4

    Бвойство β„– 3


    Π’ΠΎΠ·Π²Π΅Π΄Π΅Π½ΠΈΠ΅ стСпСни Π² ΡΡ‚Π΅ΠΏΠ΅Π½ΡŒ

    ΠŸΡ€ΠΈ Π²ΠΎΠ·Π²Π΅Π΄Π΅Π½ΠΈΠΈ стСпСни Π² ΡΡ‚Π΅ΠΏΠ΅Π½ΡŒ основаниС стСпСни остаётся Π±Π΅Π· измСнСния, Π° ΠΏΠΎΠΊΠ°Π·Π°Ρ‚Π΅Π»ΠΈ стСпСнСй ΠΏΠ΅Ρ€Π΅ΠΌΠ½ΠΎΠΆΠ°ΡŽΡ‚ΡΡ.

    (a n) m = a n Β· m , Π³Π΄Π΅ Β« a Β» — любоС число, Π° Β« m Β», Β« n Β» — Π»ΡŽΠ±Ρ‹Π΅ Π½Π°Ρ‚ΡƒΡ€Π°Π»ΡŒΠ½Ρ‹Π΅ числа.


    ΠžΠ±Ρ€Π°Ρ‚ΠΈΡ‚Π΅ Π²Π½ΠΈΠΌΠ°Π½ΠΈΠ΅, Ρ‡Ρ‚ΠΎ свойство β„– 4, ΠΊΠ°ΠΊ ΠΈ Π΄Ρ€ΡƒΠ³ΠΈΠ΅ свойства стСпСнСй, ΠΏΡ€ΠΈΠΌΠ΅Π½ΡΡŽΡ‚ ΠΈ Π² ΠΎΠ±Ρ€Π°Ρ‚Π½ΠΎΠΌ порядкС.

    (a n Β· b n)= (a Β· b) n

    Π’ΠΎ Π΅ΡΡ‚ΡŒ, Ρ‡Ρ‚ΠΎΠ±Ρ‹ ΠΏΠ΅Ρ€Π΅ΠΌΠ½ΠΎΠΆΠΈΡ‚ΡŒ стСпСни с ΠΎΠ΄ΠΈΠ½Π°ΠΊΠΎΠ²Ρ‹ΠΌΠΈ показатСлями ΠΌΠΎΠΆΠ½ΠΎ ΠΏΠ΅Ρ€Π΅ΠΌΠ½ΠΎΠΆΠΈΡ‚ΡŒ основания, Π° ΠΏΠΎΠΊΠ°Π·Π°Ρ‚Π΅Π»ΡŒ стСпСни ΠΎΡΡ‚Π°Π²ΠΈΡ‚ΡŒ Π½Π΅ΠΈΠ·ΠΌΠ΅Π½Π½Ρ‹ΠΌ.

  • ΠŸΡ€ΠΈΠΌΠ΅Ρ€. Π’Ρ‹Ρ‡ΠΈΡΠ»ΠΈΡ‚ΡŒ.
    2 4 Β· 5 4 = (2 Β· 5) 4 = 10 4 = 10 000
  • ΠŸΡ€ΠΈΠΌΠ΅Ρ€. Π’Ρ‹Ρ‡ΠΈΡΠ»ΠΈΡ‚ΡŒ.
    0,5 16 Β· 2 16 = (0,5 Β· 2) 16 = 1
  • Π’ Π±ΠΎΠ»Π΅Π΅ слоТных ΠΏΡ€ΠΈΠΌΠ΅Ρ€Π°Ρ… ΠΌΠΎΠ³ΡƒΡ‚ Π²ΡΡ‚Ρ€Π΅Ρ‚ΠΈΡ‚ΡŒΡΡ случаи, ΠΊΠΎΠ³Π΄Π° ΡƒΠΌΠ½ΠΎΠΆΠ΅Π½ΠΈΠ΅ ΠΈ Π΄Π΅Π»Π΅Π½ΠΈΠ΅ Π½Π°Π΄ΠΎ Π²Ρ‹ΠΏΠΎΠ»Π½ΠΈΡ‚ΡŒ Π½Π°Π΄ стСпСнями с Ρ€Π°Π·Π½Ρ‹ΠΌΠΈ основаниями ΠΈ Ρ€Π°Π·Π½Ρ‹ΠΌΠΈ показатСлями. Π’ этом случаС совСтуСм ΠΏΠΎΡΡ‚ΡƒΠΏΠ°Ρ‚ΡŒ ΡΠ»Π΅Π΄ΡƒΡŽΡ‰ΠΈΠΌ ΠΎΠ±Ρ€Π°Π·ΠΎΠΌ.

    НапримСр, 4 5 Β· 3 2 = 4 3 Β· 4 2 Β· 3 2 = 4 3 Β· (4 Β· 3) 2 = 64 Β· 12 2 = 64 Β· 144 = 9216

    ΠŸΡ€ΠΈΠΌΠ΅Ρ€ возвСдСния Π² ΡΡ‚Π΅ΠΏΠ΅Π½ΡŒ дСсятичной Π΄Ρ€ΠΎΠ±ΠΈ.

    4 21 Β· (βˆ’0,25) 20 = 4 Β· 4 20 Β· (βˆ’0,25) 20 = 4 Β· (4 Β· (βˆ’0,25)) 20 = 4 Β· (βˆ’1) 20 = 4 Β· 1 = 4

    Бвойства 5


    Π‘Ρ‚Π΅ΠΏΠ΅Π½ΡŒ частного (Π΄Ρ€ΠΎΠ±ΠΈ)

    Π§Ρ‚ΠΎΠ±Ρ‹ возвСсти Π² ΡΡ‚Π΅ΠΏΠ΅Π½ΡŒ частноС, ΠΌΠΎΠΆΠ½ΠΎ возвСсти Π² эту ΡΡ‚Π΅ΠΏΠ΅Π½ΡŒ ΠΎΡ‚Π΄Π΅Π»ΡŒΠ½ΠΎ Π΄Π΅Π»ΠΈΠΌΠΎΠ΅ ΠΈ Π΄Π΅Π»ΠΈΡ‚Π΅Π»ΡŒ, ΠΈ ΠΏΠ΅Ρ€Π²Ρ‹ΠΉ Ρ€Π΅Π·ΡƒΠ»ΡŒΡ‚Π°Ρ‚ Ρ€Π°Π·Π΄Π΅Π»ΠΈΡ‚ΡŒ Π½Π° Π²Ρ‚ΠΎΡ€ΠΎΠΉ.

    (a: b) n = a n: b n , Π³Π΄Π΅ Β« a Β», Β« b Β» — Π»ΡŽΠ±Ρ‹Π΅ Ρ€Π°Ρ†ΠΈΠΎΠ½Π°Π»ΡŒΠ½Ρ‹Π΅ числа, b β‰  0, n — любоС Π½Π°Ρ‚ΡƒΡ€Π°Π»ΡŒΠ½ΠΎΠ΅ число.

  • ΠŸΡ€ΠΈΠΌΠ΅Ρ€. ΠŸΡ€Π΅Π΄ΡΡ‚Π°Π²ΠΈΡ‚ΡŒ Π²Ρ‹Ρ€Π°ΠΆΠ΅Π½ΠΈΠ΅ Π² Π²ΠΈΠ΄Π΅ частного стСпСнСй.
    (5: 3) 12 = 5 12: 3 12
  • НапоминаСм, Ρ‡Ρ‚ΠΎ частноС ΠΌΠΎΠΆΠ½ΠΎ ΠΏΡ€Π΅Π΄ΡΡ‚Π°Π²ΠΈΡ‚ΡŒ Π² Π²ΠΈΠ΄Π΅ Π΄Ρ€ΠΎΠ±ΠΈ. ΠŸΠΎΡΡ‚ΠΎΠΌΡƒ Π½Π° Ρ‚Π΅ΠΌΠ΅ Π²ΠΎΠ·Π²Π΅Π΄Π΅Π½ΠΈΠ΅ Π΄Ρ€ΠΎΠ±ΠΈ Π² ΡΡ‚Π΅ΠΏΠ΅Π½ΡŒ ΠΌΡ‹ остановимся Π±ΠΎΠ»Π΅Π΅ ΠΏΠΎΠ΄Ρ€ΠΎΠ±Π½ΠΎ Π½Π° ΡΠ»Π΅Π΄ΡƒΡŽΡ‰Π΅ΠΉ страницС.

    Π‘Ρ‚Π΅ΠΏΠ΅Π½ΠΈ ΠΈ ΠΊΠΎΡ€Π½ΠΈ

    ΠžΠΏΠ΅Ρ€Π°Ρ†ΠΈΠΈ со стСпСнями ΠΈ корнями. Π‘Ρ‚Π΅ΠΏΠ΅Π½ΡŒ с ΠΎΡ‚Ρ€ΠΈΡ†Π°Ρ‚Π΅Π»ΡŒΠ½Ρ‹ΠΌ ,

    Π½ΡƒΠ»Π΅Π²Ρ‹ΠΌ ΠΈ Π΄Ρ€ΠΎΠ±Π½Ρ‹ΠΌ ΠΏΠΎΠΊΠ°Π·Π°Ρ‚Π΅Π»Π΅ΠΌ. О выраТСниях, Π½Π΅ ΠΈΠΌΠ΅ΡŽΡ‰ΠΈΡ… смысла.

    ΠžΠΏΠ΅Ρ€Π°Ρ†ΠΈΠΈ со стСпСнями.

    1. ΠŸΡ€ΠΈ ΡƒΠΌΠ½ΠΎΠΆΠ΅Π½ΠΈΠΈ стСпСнСй с ΠΎΠ΄ΠΈΠ½Π°ΠΊΠΎΠ²Ρ‹ΠΌ основаниСм ΠΈΡ… ΠΏΠΎΠΊΠ°Π·Π°Ρ‚Π΅Π»ΠΈ ΡΠΊΠ»Π°Π΄Ρ‹Π²Π°ΡŽΡ‚ΡΡ:

    a m Β· a n = a m + n .

    2. ΠŸΡ€ΠΈ Π΄Π΅Π»Π΅Π½ΠΈΠΈ стСпСнСй с ΠΎΠ΄ΠΈΠ½Π°ΠΊΠΎΠ²Ρ‹ΠΌ основаниСм ΠΈΡ… ΠΏΠΎΠΊΠ°Π·Π°Ρ‚Π΅Π»ΠΈ Π²Ρ‹Ρ‡ΠΈΡ‚Π°ΡŽΡ‚ΡΡ .

    3. Π‘Ρ‚Π΅ΠΏΠ΅Π½ΡŒ произвСдСния Π΄Π²ΡƒΡ… ΠΈΠ»ΠΈ Π½Π΅ΡΠΊΠΎΠ»ΡŒΠΊΠΈΡ… сомноТитСлСй Ρ€Π°Π²Π½Π° ΠΏΡ€ΠΎΠΈΠ·Π²Π΅Π΄Π΅Π½ΠΈΡŽ стСпСнСй этих сомноТитСлСй.

    4. Π‘Ρ‚Π΅ΠΏΠ΅Π½ΡŒ ΠΎΡ‚Π½ΠΎΡˆΠ΅Π½ΠΈΡ (Π΄Ρ€ΠΎΠ±ΠΈ) Ρ€Π°Π²Π½Π° ΠΎΡ‚Π½ΠΎΡˆΠ΅Π½ΠΈΡŽ стСпСнСй Π΄Π΅Π»ΠΈΠΌΠΎΠ³ΠΎ (числитСля) ΠΈ дСлитСля (знамСнатСля):

    (a / b ) n = a n / b n .

    5. ΠŸΡ€ΠΈ Π²ΠΎΠ·Π²Π΅Π΄Π΅Π½ΠΈΠΈ стСпСни Π² ΡΡ‚Π΅ΠΏΠ΅Π½ΡŒ ΠΈΡ… ΠΏΠΎΠΊΠ°Π·Π°Ρ‚Π΅Π»ΠΈ ΠΏΠ΅Ρ€Π΅ΠΌΠ½ΠΎΠΆΠ°ΡŽΡ‚ΡΡ:

    ВсС Π²Ρ‹ΡˆΠ΅ΠΏΡ€ΠΈΠ²Π΅Π΄Π΅Π½Π½Ρ‹Π΅ Ρ„ΠΎΡ€ΠΌΡƒΠ»Ρ‹ Ρ‡ΠΈΡ‚Π°ΡŽΡ‚ΡΡ ΠΈ Π²Ρ‹ΠΏΠΎΠ»Π½ΡΡŽΡ‚ΡΡ Π² ΠΎΠ±ΠΎΠΈΡ… направлСниях слСва Π½Π°ΠΏΡ€Π°Π²ΠΎ ΠΈ Π½Π°ΠΎΠ±ΠΎΡ€ΠΎΡ‚.

    П Ρ€ ΠΈ ΠΌ Π΅ Ρ€. (2 Β· 3 Β· 5 / 15) Β² = 2 Β² Β· 3 Β² Β· 5 Β² / 15 Β² = 900 / 225 = 4 .

    ΠžΠΏΠ΅Ρ€Π°Ρ†ΠΈΠΈ с корнями. Π’ΠΎ всСх Π½ΠΈΠΆΠ΅ΠΏΡ€ΠΈΠ²Π΅Π΄Π΅Π½Π½Ρ‹Ρ… Ρ„ΠΎΡ€ΠΌΡƒΠ»Π°Ρ… символ ΠΎΠ·Π½Π°Ρ‡Π°Π΅Ρ‚ арифмСтичСский ΠΊΠΎΡ€Π΅Π½ΡŒ (ΠΏΠΎΠ΄ΠΊΠΎΡ€Π΅Π½Π½ΠΎΠ΅ Π²Ρ‹Ρ€Π°ΠΆΠ΅Π½ΠΈΠ΅ ΠΏΠΎΠ»ΠΎΠΆΠΈΡ‚Π΅Π»ΡŒΠ½ΠΎ).

    1. ΠšΠΎΡ€Π΅Π½ΡŒ ΠΈΠ· произвСдСния Π½Π΅ΡΠΊΠΎΠ»ΡŒΠΊΠΈΡ… сомноТитСлСй Ρ€Π°Π²Π΅Π½ ΠΏΡ€ΠΎΠΈΠ·Π²Π΅Π΄Π΅Π½ΠΈΡŽ ΠΊΠΎΡ€Π½Π΅ΠΉ ΠΈΠ· этих сомноТитСлСй:

    2. ΠšΠΎΡ€Π΅Π½ΡŒ ΠΈΠ· ΠΎΡ‚Π½ΠΎΡˆΠ΅Π½ΠΈΡ Ρ€Π°Π²Π΅Π½ ΠΎΡ‚Π½ΠΎΡˆΠ΅Π½ΠΈΡŽ ΠΊΠΎΡ€Π½Π΅ΠΉ Π΄Π΅Π»ΠΈΠΌΠΎΠ³ΠΎ ΠΈ дСлитСля:

    3. ΠŸΡ€ΠΈ Π²ΠΎΠ·Π²Π΅Π΄Π΅Π½ΠΈΠΈ корня Π² ΡΡ‚Π΅ΠΏΠ΅Π½ΡŒ достаточно возвСсти Π² эту ΡΡ‚Π΅ΠΏΠ΅Π½ΡŒ ΠΏΠΎΠ΄ΠΊΠΎΡ€Π΅Π½Π½ΠΎΠ΅ число:

    4. Если ΡƒΠ²Π΅Π»ΠΈΡ‡ΠΈΡ‚ΡŒ ΡΡ‚Π΅ΠΏΠ΅Π½ΡŒ корня Π² m Ρ€Π°Π· ΠΈ ΠΎΠ΄Π½ΠΎΠ²Ρ€Π΅ΠΌΠ΅Π½Π½ΠΎ возвСсти Π² m -ΡƒΡŽ ΡΡ‚Π΅ΠΏΠ΅Π½ΡŒ ΠΏΠΎΠ΄ΠΊΠΎΡ€Π΅Π½Π½ΠΎΠ΅ число, Ρ‚ΠΎ Π·Π½Π°Ρ‡Π΅Π½ΠΈΠ΅ корня Π½Π΅ измСнится:

    5. Если ΡƒΠΌΠ΅Π½ΡŒΡˆΠΈΡ‚ΡŒ ΡΡ‚Π΅ΠΏΠ΅Π½ΡŒ корня Π² m Ρ€Π°Π· ΠΈ ΠΎΠ΄Π½ΠΎΠ²Ρ€Π΅ΠΌΠ΅Π½Π½ΠΎ ΠΈΠ·Π²Π»Π΅Ρ‡ΡŒ ΠΊΠΎΡ€Π΅Π½ΡŒ m -ΠΎΠΉ стСпСни ΠΈΠ· ΠΏΠΎΠ΄ΠΊΠΎΡ€Π΅Π½Π½ΠΎΠ³ΠΎ числа, Ρ‚ΠΎ Π·Π½Π°Ρ‡Π΅Π½ΠΈΠ΅ корня Π½Π΅ измСнится:


    Π Π°ΡΡˆΠΈΡ€Π΅Π½ΠΈΠ΅ понятия стСпСни. Π”ΠΎ сих ΠΏΠΎΡ€ ΠΌΡ‹ рассматривали стСпСни Ρ‚ΠΎΠ»ΡŒΠΊΠΎ с Π½Π°Ρ‚ΡƒΡ€Π°Π»ΡŒΠ½Ρ‹ΠΌ ΠΏΠΎΠΊΠ°Π·Π°Ρ‚Π΅Π»Π΅ΠΌ; Π½ΠΎ дСйствия со стСпСнями ΠΈ корнями ΠΌΠΎΠ³ΡƒΡ‚ ΠΏΡ€ΠΈΠ²ΠΎΠ΄ΠΈΡ‚ΡŒ Ρ‚Π°ΠΊΠΆΠ΅ ΠΊ ΠΎΡ‚Ρ€ΠΈΡ†Π°Ρ‚Π΅Π»ΡŒΠ½Ρ‹ΠΌ , Π½ΡƒΠ»Π΅Π²Ρ‹ΠΌ ΠΈ Π΄Ρ€ΠΎΠ±Π½Ρ‹ΠΌ показатСлям. ВсС эти ΠΏΠΎΠΊΠ°Π·Π°Ρ‚Π΅Π»ΠΈ стСпСнСй Ρ‚Ρ€Π΅Π±ΡƒΡŽΡ‚ Π΄ΠΎΠΏΠΎΠ»Π½ΠΈΡ‚Π΅Π»ΡŒΠ½ΠΎΠ³ΠΎ опрСдСлСния.

    Π‘Ρ‚Π΅ΠΏΠ΅Π½ΡŒ с ΠΎΡ‚Ρ€ΠΈΡ†Π°Ρ‚Π΅Π»ΡŒΠ½Ρ‹ΠΌ ΠΏΠΎΠΊΠ°Π·Π°Ρ‚Π΅Π»Π΅ΠΌ. Π‘Ρ‚Π΅ΠΏΠ΅Π½ΡŒ Π½Π΅ΠΊΠΎΡ‚ΠΎΡ€ΠΎΠ³ΠΎ числа с ΠΎΡ‚Ρ€ΠΈΡ†Π°Ρ‚Π΅Π»ΡŒΠ½Ρ‹ΠΌ (Ρ†Π΅Π»Ρ‹ΠΌ) ΠΏΠΎΠΊΠ°Π·Π°Ρ‚Π΅Π»Π΅ΠΌ опрСдСляСтся ΠΊΠ°ΠΊ Π΅Π΄ΠΈΠ½ΠΈΡ†Π°, дСлённая Π½Π° ΡΡ‚Π΅ΠΏΠ΅Π½ΡŒ Ρ‚ΠΎΠ³ΠΎ ΠΆΠ΅ числа с ΠΏΠΎΠΊΠ°Π·Π°Ρ‚Π΅Π»Π΅ΠΌ, Ρ€Π°Π²Π½Ρ‹ΠΌ Π°Π±ΡΠΎΠ»ΡŽΡ‚Π½ΠΎΠΉ Π²Π΅Π»Π΅Ρ‡ΠΈΠ½Π΅ ΠΎΡ‚Ρ€ΠΈΡ†Π°Ρ‚Π΅Π»ΡŒΠ½ΠΎΠ³ΠΎ показатСля:

    Π’ Π΅ΠΏΠ΅Ρ€ΡŒ Ρ„ΠΎΡ€ΠΌΡƒΠ»Π° a m : a n = a m β€” n ΠΌΠΎΠΆΠ΅Ρ‚ Π±Ρ‹Ρ‚ΡŒ использована Π½Π΅ Ρ‚ΠΎΠ»ΡŒΠΊΠΎ ΠΏΡ€ΠΈ m , большСм, Ρ‡Π΅ΠΌ n , Π½ΠΎ ΠΈ ΠΏΡ€ΠΈ m , мСньшСм, Ρ‡Π΅ΠΌ n .

    П Ρ€ ΠΈ ΠΌ Π΅ Ρ€. a 4: a 7 = a 4 β€” 7 = a β€” 3 .

    Если ΠΌΡ‹ Ρ…ΠΎΡ‚ΠΈΠΌ, Ρ‡Ρ‚ΠΎΠ±Ρ‹ Ρ„ΠΎΡ€ΠΌΡƒΠ»Π° a m : a n = a m β€” n Π±Ρ‹Π»Π° справСдлива ΠΏΡ€ΠΈ m = n , Π½Π°ΠΌ Π½Π΅ΠΎΠ±Ρ…ΠΎΠ΄ΠΈΠΌΠΎ ΠΎΠΏΡ€Π΅Π΄Π΅Π»Π΅Π½ΠΈΠ΅ Π½ΡƒΠ»Π΅Π²ΠΎΠΉ стСпСни.

    Π‘Ρ‚Π΅ΠΏΠ΅Π½ΡŒ с Π½ΡƒΠ»Π΅Π²Ρ‹ΠΌ ΠΏΠΎΠΊΠ°Π·Π°Ρ‚Π΅Π»Π΅ΠΌ. Π‘Ρ‚Π΅ΠΏΠ΅Π½ΡŒ любого Π½Π΅Π½ΡƒΠ»Π΅Π²ΠΎΠ³ΠΎ числа с Π½ΡƒΠ»Π΅Π²Ρ‹ΠΌ ΠΏΠΎΠΊΠ°Π·Π°Ρ‚Π΅Π»Π΅ΠΌ Ρ€Π°Π²Π½Π° 1.

    П Ρ€ ΠΈ ΠΌ Π΅ Ρ€ Ρ‹. 2 0 = 1, (– 5) 0 = 1, (– 3 / 5) 0 = 1.

    Π‘Ρ‚Π΅ΠΏΠ΅Π½ΡŒ с Π΄Ρ€ΠΎΠ±Π½Ρ‹ΠΌ ΠΏΠΎΠΊΠ°Π·Π°Ρ‚Π΅Π»Π΅ΠΌ. Для Ρ‚ΠΎΠ³ΠΎ, Ρ‡Ρ‚ΠΎΠ±Ρ‹ возвСсти Π΄Π΅ΠΉΡΡ‚Π²ΠΈΡ‚Π΅Π»ΡŒΠ½ΠΎΠ΅ число Π° Π² ΡΡ‚Π΅ΠΏΠ΅Π½ΡŒ m / n , Π½ΡƒΠΆΠ½ΠΎ ΠΈΠ·Π²Π»Π΅Ρ‡ΡŒ ΠΊΠΎΡ€Π΅Π½ΡŒ n –ой стСпСни ΠΈΠ· m -ΠΎΠΉ стСпСни этого числа Π°:

    О выраТСниях, Π½Π΅ ΠΈΠΌΠ΅ΡŽΡ‰ΠΈΡ… смысла. Π•ΡΡ‚ΡŒ нСсколько Ρ‚Π°ΠΊΠΈΡ… Π²Ρ‹Ρ€Π°ΠΆΠ΅Π½ΠΈΠΉ.

    Π³Π΄Π΅ a β‰  0 , Π½Π΅ сущСствуСт.

    Π’ самом Π΄Π΅Π»Π΅, Ссли ΠΏΡ€Π΅Π΄ΠΏΠΎΠ»ΠΎΠΆΠΈΡ‚ΡŒ, Ρ‡Ρ‚ΠΎ x – Π½Π΅ΠΊΠΎΡ‚ΠΎΡ€ΠΎΠ΅ число, Ρ‚ΠΎ Π² соотвСтствии с ΠΎΠΏΡ€Π΅Π΄Π΅Π»Π΅Π½ΠΈΠ΅ΠΌ ΠΎΠΏΠ΅Ρ€Π°Ρ†ΠΈΠΈ дСлСния ΠΈΠΌΠ΅Π΅ΠΌ: a = 0Β· x , Ρ‚.e. a = 0, Ρ‡Ρ‚ΠΎ ΠΏΡ€ΠΎΡ‚ΠΈΠ²ΠΎΡ€Π΅Ρ‡ΠΈΡ‚ ΡƒΡΠ»ΠΎΠ²ΠΈΡŽ: a β‰  0

    β€” любоС число.

    Π’ самом Π΄Π΅Π»Π΅, Ссли ΠΏΡ€Π΅Π΄ΠΏΠΎΠ»ΠΎΠΆΠΈΡ‚ΡŒ, Ρ‡Ρ‚ΠΎ это Π²Ρ‹Ρ€Π°ΠΆΠ΅Π½ΠΈΠ΅ Ρ€Π°Π²Π½ΠΎ Π½Π΅ΠΊΠΎΡ‚ΠΎΡ€ΠΎΠΌΡƒ числу x , Ρ‚ΠΎ согласно ΠΎΠΏΡ€Π΅Π΄Π΅Π»Π΅Π½ΠΈΡŽ ΠΎΠΏΠ΅Ρ€Π°Ρ†ΠΈΠΈ дСлСния ΠΈΠΌΠ΅Π΅ΠΌ: 0 = 0 Β· x . Но это равСнство ΠΈΠΌΠ΅Π΅Ρ‚ мСсто ΠΏΡ€ΠΈ любом числС x , Ρ‡Ρ‚ΠΎ ΠΈ Ρ‚Ρ€Π΅Π±ΠΎΠ²Π°Π»ΠΎΡΡŒ Π΄ΠΎΠΊΠ°Π·Π°Ρ‚ΡŒ.

    0 0 β€” любоС число.

    Π  Π΅ ш Π΅ Π½ ΠΈ Π΅. Рассмотрим Ρ‚Ρ€ΠΈ основных случая:

    1) x = 0 – это Π·Π½Π°Ρ‡Π΅Π½ΠΈΠ΅ Π½Π΅ удовлСтворяСт Π΄Π°Π½Π½ΠΎΠΌΡƒ ΡƒΡ€Π°Π²Π½Π΅Π½ΠΈΡŽ

    2) ΠΏΡ€ΠΈ x > 0 ΠΏΠΎΠ»ΡƒΡ‡Π°Π΅ΠΌ: x / x = 1, Ρ‚.e. 1 = 1, ΠΎΡ‚ΠΊΡƒΠ΄Π° слСдуСт,

    Ρ‡Ρ‚ΠΎ x – любоС число; Π½ΠΎ принимая Π²ΠΎ Π²Π½ΠΈΠΌΠ°Π½ΠΈΠ΅, Ρ‡Ρ‚ΠΎ Π²

    нашСм случаС x > 0 , ΠΎΡ‚Π²Π΅Ρ‚ΠΎΠΌ являСтся x > 0 ;

    ΠŸΡ€Π°Π²ΠΈΠ»Π° умноТСния стСпСнСй с Ρ€Π°Π·Π½Ρ‹ΠΌ основаниСм

    Π‘Π’Π•ΠŸΠ•ΠΠ¬ Π‘ Π ΠΠ¦Π˜ΠžΠΠΠ›Π¬ΠΠ«Πœ ΠŸΠžΠšΠΠ—ΠΠ’Π•Π›Π•Πœ,

    Π‘Π’Π•ΠŸΠ•ΠΠΠΠ― ЀУНКЦИЯ IV

    Β§ 69. Π£ΠΌΠ½ΠΎΠΆΠ΅Π½ΠΈΠ΅ ΠΈ Π΄Π΅Π»Π΅Π½ΠΈΠ΅ стСпСнСй с ΠΎΠ΄ΠΈΠ½Π°ΠΊΠΎΠ²Ρ‹ΠΌΠΈ основаниями

    Π’Π΅ΠΎΡ€Π΅ΠΌΠ° 1. Π§Ρ‚ΠΎΠ±Ρ‹ ΠΏΠ΅Ρ€Π΅ΠΌΠ½ΠΎΠΆΠΈΡ‚ΡŒ стСпСни с ΠΎΠ΄ΠΈΠ½Π°ΠΊΠΎΠ²Ρ‹ΠΌΠΈ основаниями, достаточно ΠΏΠΎΠΊΠ°Π·Π°Ρ‚Π΅Π»ΠΈ стСпСнСй ΡΠ»ΠΎΠΆΠΈΡ‚ΡŒ, Π° основаниС ΠΎΡΡ‚Π°Π²ΠΈΡ‚ΡŒ ΠΏΡ€Π΅ΠΆΠ½ΠΈΠΌ , Ρ‚ΠΎ Π΅ΡΡ‚ΡŒ

    Π”ΠΎΠΊΠ°Π·Π°Ρ‚Π΅Π»ΡŒΡΡ‚Π²ΠΎ. По ΠΎΠΏΡ€Π΅Π΄Π΅Π»Π΅Π½ΠΈΡŽ стСпСни

    2 2 2 3 = 2 5 = 32; (-3) (-3) 3 = (-3) 4 = 81.

    ΠœΡ‹ рассмотрСли ΠΏΡ€ΠΎΠΈΠ·Π²Π΅Π΄Π΅Π½ΠΈΠ΅ Π΄Π²ΡƒΡ… стСпСнСй. На самом ΠΆΠ΅ Π΄Π΅Π»Π΅ Π΄ΠΎΠΊΠ°Π·Π°Π½Π½ΠΎΠ΅ свойство Π²Π΅Ρ€Π½ΠΎ для любого числа стСпСнСй с ΠΎΠ΄ΠΈΠ½Π°ΠΊΠΎΠ²Ρ‹ΠΌΠΈ основаниями.

    Π’Π΅ΠΎΡ€Π΅ΠΌΠ° 2. Π§Ρ‚ΠΎΠ±Ρ‹ Ρ€Π°Π·Π΄Π΅Π»ΠΈΡ‚ΡŒ стСпСни с ΠΎΠ΄ΠΈΠ½Π°ΠΊΠΎΠ²Ρ‹ΠΌΠΈ основаниями, ΠΊΠΎΠ³Π΄Π° ΠΏΠΎΠΊΠ°Π·Π°Ρ‚Π΅Π»ΡŒ Π΄Π΅Π»ΠΈΠΌΠΎΠ³ΠΎ большС показатСля дСлитСля, достаточно ΠΈΠ· показатСля Π΄Π΅Π»ΠΈΠΌΠΎΠ³ΠΎ Π²Ρ‹Ρ‡Π΅ΡΡ‚ΡŒ ΠΏΠΎΠΊΠ°Π·Π°Ρ‚Π΅Π»ΡŒ дСлитСля, Π° основаниС ΠΎΡΡ‚Π°Π²ΠΈΡ‚ΡŒ ΠΏΡ€Π΅ΠΆΠ½ΠΈΠΌ, Ρ‚ΠΎ Π΅ΡΡ‚ΡŒ ΠΏΡ€ΠΈ Ρ‚ > ΠΏ

    (a =/= 0)

    Π”ΠΎΠΊΠ°Π·Π°Ρ‚Π΅Π»ΡŒΡΡ‚Π²ΠΎ. Напомним, Ρ‡Ρ‚ΠΎ частным ΠΎΡ‚ дСлСния ΠΎΠ΄Π½ΠΎΠ³ΠΎ числа Π½Π° Π΄Ρ€ΡƒΠ³ΠΎΠ΅ называСтся число, ΠΊΠΎΡ‚ΠΎΡ€ΠΎΠ΅ ΠΏΡ€ΠΈ ΡƒΠΌΠ½ΠΎΠΆΠ΅Π½ΠΈΠΈ Π½Π° Π΄Π΅Π»ΠΈΡ‚Π΅Π»ΡŒ Π΄Π°Π΅Ρ‚ Π΄Π΅Π»ΠΈΠΌΠΎΠ΅. ΠŸΠΎΡΡ‚ΠΎΠΌΡƒ Π΄ΠΎΠΊΠ°Π·Π°Ρ‚ΡŒ Ρ„ΠΎΡ€ΠΌΡƒΠ»Ρƒ , Π³Π΄Π΅ a =/= 0, это всС Ρ€Π°Π²Π½ΠΎ, Ρ‡Ρ‚ΠΎ Π΄ΠΎΠΊΠ°Π·Π°Ρ‚ΡŒ Ρ„ΠΎΡ€ΠΌΡƒΠ»Ρƒ

    Если Ρ‚ > ΠΏ , Ρ‚ΠΎ число Ρ‚ — ΠΏ Π±ΡƒΠ΄Π΅Ρ‚ Π½Π°Ρ‚ΡƒΡ€Π°Π»ΡŒΠ½Ρ‹ΠΌ; ΡΠ»Π΅Π΄ΠΎΠ²Π°Ρ‚Π΅Π»ΡŒΠ½ΠΎ, ΠΏΠΎ Ρ‚Π΅ΠΎΡ€Π΅ΠΌΠ΅ 1

    Π’Π΅ΠΎΡ€Π΅ΠΌΠ° 2 Π΄ΠΎΠΊΠ°Π·Π°Π½Π°.

    Π‘Π»Π΅Π΄ΡƒΠ΅Ρ‚ ΠΎΠ±Ρ€Π°Ρ‚ΠΈΡ‚ΡŒ Π²Π½ΠΈΠΌΠ°Π½ΠΈΠ΅ Π½Π° Ρ‚ΠΎ, Ρ‡Ρ‚ΠΎ Ρ„ΠΎΡ€ΠΌΡƒΠ»Π°

    Π΄ΠΎΠΊΠ°Π·Π°Π½Π° Π½Π°ΠΌΠΈ лишь Π² ΠΏΡ€Π΅Π΄ΠΏΠΎΠ»ΠΎΠΆΠ΅Π½ΠΈΠΈ, Ρ‡Ρ‚ΠΎ Ρ‚ > ΠΏ . ΠŸΠΎΡΡ‚ΠΎΠΌΡƒ ΠΈΠ· Π΄ΠΎΠΊΠ°Π·Π°Π½Π½ΠΎΠ³ΠΎ ΠΏΠΎΠΊΠ° нСльзя Π΄Π΅Π»Π°Ρ‚ΡŒ, Π½Π°ΠΏΡ€ΠΈΠΌΠ΅Ρ€, Ρ‚Π°ΠΊΠΈΡ… Π²Ρ‹Π²ΠΎΠ΄ΠΎΠ²:

    К Ρ‚ΠΎΠΌΡƒ ΠΆΠ΅ стСпСни с ΠΎΡ‚Ρ€ΠΈΡ†Π°Ρ‚Π΅Π»ΡŒΠ½Ρ‹ΠΌΠΈ показатСлями Π½Π°ΠΌΠΈ Π΅Ρ‰Π΅ Π½Π΅ Ρ€Π°ΡΡΠΌΠ°Ρ‚Ρ€ΠΈΠ²Π°Π»ΠΈΡΡŒ ΠΈ ΠΌΡ‹ ΠΏΠΎΠΊΠ° Ρ‡Ρ‚ΠΎ Π½Π΅ Π·Π½Π°Π΅ΠΌ, ΠΊΠ°ΠΊΠΎΠΉ смысл ΠΌΠΎΠΆΠ½ΠΎ ΠΏΡ€ΠΈΠ΄Π°Ρ‚ΡŒ Π²Ρ‹Ρ€Π°ΠΆΠ΅Π½ΠΈΡŽ 3 2 .

    Π’Π΅ΠΎΡ€Π΅ΠΌΠ° 3. Π§Ρ‚ΠΎΠ±Ρ‹ возвСсти ΡΡ‚Π΅ΠΏΠ΅Π½ΡŒ Π² ΡΡ‚Π΅ΠΏΠ΅Π½ΡŒ, достаточно ΠΏΠ΅Ρ€Π΅ΠΌΠ½ΠΎΠΆΠΈΡ‚ΡŒ ΠΏΠΎΠΊΠ°Π·Π°Ρ‚Π΅Π»ΠΈ, оставив основаниС стСпСни ΠΏΡ€Π΅ΠΆΠ½ΠΈΠΌ , Ρ‚ΠΎ Π΅ΡΡ‚ΡŒ

    Π”ΠΎΠΊΠ°Π·Π°Ρ‚Π΅Π»ΡŒΡΡ‚Π²ΠΎ. Π˜ΡΠΏΠΎΠ»ΡŒΠ·ΡƒΡ ΠΎΠΏΡ€Π΅Π΄Π΅Π»Π΅Π½ΠΈΠ΅ стСпСни ΠΈ Ρ‚Π΅ΠΎΡ€Π΅ΠΌΡƒ 1 этого ΠΏΠ°Ρ€Π°Π³Ρ€Π°Ρ„Π°, ΠΏΠΎΠ»ΡƒΡ‡Π°Π΅ΠΌ:

    Ρ‡Ρ‚ΠΎ ΠΈ Ρ‚Ρ€Π΅Π±ΠΎΠ²Π°Π»ΠΎΡΡŒ Π΄ΠΎΠΊΠ°Π·Π°Ρ‚ΡŒ.

    НапримСр, (2 3) 2 = 2 6 = 64;

    518 (Устно.) ΠžΠΏΡ€Π΅Π΄Π΅Π»ΠΈΡ‚ΡŒ Ρ… ΠΈΠ· ΡƒΡ€Π°Π²Π½Π΅Π½ΠΈΠΉ:

    1) 2 2 2 2 3 2 4 2 5 2 6 = 2 x ; 3) 4 2 4 4 4 6 4 8 4 10 = 2 x ;

    2) 3 3 3 3 5 3 7 3 9 = 3 x ; 4) 1 / 5 1 / 25 1 / 125 1 / 625 = 1 / 5 x .

    519. (Π£ с Ρ‚ Π½ ΠΎ.) Π£ΠΏΡ€ΠΎΡΡ‚ΠΈΡ‚ΡŒ:

    520. (Π£ с Ρ‚ Π½ ΠΎ.) Π£ΠΏΡ€ΠΎΡΡ‚ΠΈΡ‚ΡŒ:

    521. Π”Π°Π½Π½Ρ‹Π΅ выраТСния ΠΏΡ€Π΅Π΄ΡΡ‚Π°Π²ΠΈΡ‚ΡŒ Π² Π²ΠΈΠ΄Π΅ стСпСнСй с ΠΎΠ΄ΠΈΠ½Π°ΠΊΠΎΠ²Ρ‹ΠΌΠΈ основаниями:

    1) 32 ΠΈ 64; 3) 8 5 ΠΈ 16 3 ; 5) 4 100 ΠΈ 32 50 ;

    2) -1000 ΠΈ 100; 4) -27 ΠΈ -243; 6) 81 75 8 200 ΠΈ 3 600 4 150 .

    НапоминаСм, Ρ‡Ρ‚ΠΎ Π² Π΄Π°Π½Π½ΠΎΠΌ ΡƒΡ€ΠΎΠΊΠ΅ Ρ€Π°Π·Π±ΠΈΡ€Π°ΡŽΡ‚ΡΡ свойства стСпСнСй с Π½Π°Ρ‚ΡƒΡ€Π°Π»ΡŒΠ½Ρ‹ΠΌΠΈ показатСлями ΠΈ Π½ΡƒΠ»Ρ‘ΠΌ. Π‘Ρ‚Π΅ΠΏΠ΅Π½ΠΈ с Ρ€Π°Ρ†ΠΈΠΎΠ½Π°Π»ΡŒΠ½Ρ‹ΠΌΠΈ показатСлями ΠΈ ΠΈΡ… свойства Π±ΡƒΠ΄ΡƒΡ‚ рассмотрСны Π² ΡƒΡ€ΠΎΠΊΠ°Ρ… для 8 классов.

    Π‘Ρ‚Π΅ΠΏΠ΅Π½ΡŒ с Π½Π°Ρ‚ΡƒΡ€Π°Π»ΡŒΠ½Ρ‹ΠΌ ΠΏΠΎΠΊΠ°Π·Π°Ρ‚Π΅Π»Π΅ΠΌ ΠΎΠ±Π»Π°Π΄Π°Π΅Ρ‚ нСсколькими Π²Π°ΠΆΠ½Ρ‹ΠΌΠΈ свойствами, ΠΊΠΎΡ‚ΠΎΡ€Ρ‹Π΅ ΠΏΠΎΠ·Π²ΠΎΠ»ΡΡŽΡ‚ ΡƒΠΏΡ€ΠΎΡ‰Π°Ρ‚ΡŒ вычислСния Π² ΠΏΡ€ΠΈΠΌΠ΅Ρ€Π°Ρ… со стСпСнями.

    Бвойство β„– 1


    ΠŸΡ€ΠΎΠΈΠ·Π²Π΅Π΄Π΅Π½ΠΈΠ΅ стСпСнСй

    Π—Π°ΠΏΠΎΠΌΠ½ΠΈΡ‚Π΅!

    ΠŸΡ€ΠΈ ΡƒΠΌΠ½ΠΎΠΆΠ΅Π½ΠΈΠΈ стСпСнСй с ΠΎΠ΄ΠΈΠ½Π°ΠΊΠΎΠ²Ρ‹ΠΌΠΈ основаниями основаниС остаётся Π±Π΅Π· ΠΈΠ·ΠΌΠ΅Π½Π΅Π½ΠΈΠΉ, Π° ΠΏΠΎΠΊΠ°Π·Π°Ρ‚Π΅Π»ΠΈ стСпСнСй ΡΠΊΠ»Π°Π΄Ρ‹Π²Π°ΡŽΡ‚ΡΡ.

    a m Β· a n = a m + n , Π³Π΄Π΅ Β«a Β» β€” любоС число, Π° Β«m Β», Β«n Β» β€” Π»ΡŽΠ±Ρ‹Π΅ Π½Π°Ρ‚ΡƒΡ€Π°Π»ΡŒΠ½Ρ‹Π΅ числа.

    Π”Π°Π½Π½ΠΎΠ΅ свойство стСпСнСй Ρ‚Π°ΠΊΠΆΠ΅ дСйствуСт Π½Π° ΠΏΡ€ΠΎΠΈΠ·Π²Π΅Π΄Π΅Π½ΠΈΠ΅ Ρ‚Ρ€Ρ‘Ρ… ΠΈ Π±ΠΎΠ»Π΅Π΅ стСпСнСй.

    • Π£ΠΏΡ€ΠΎΡΡ‚ΠΈΡ‚ΡŒ Π²Ρ‹Ρ€Π°ΠΆΠ΅Π½ΠΈΠ΅.
      b Β· b 2 Β· b 3 Β· b 4 Β· b 5 = b 1 + 2 + 3 + 4 + 5 = b 15
    • ΠŸΡ€Π΅Π΄ΡΡ‚Π°Π²ΠΈΡ‚ΡŒ Π² Π²ΠΈΠ΄Π΅ стСпСни.
      6 15 Β· 36 = 6 15 Β· 6 2 = 6 15 Β· 6 2 = 6 17
    • ΠŸΡ€Π΅Π΄ΡΡ‚Π°Π²ΠΈΡ‚ΡŒ Π² Π²ΠΈΠ΄Π΅ стСпСни.
      (0,8) 3 Β· (0,8) 12 = (0,8) 3 + 12 = (0,8) 15

    Π’Π°ΠΆΠ½ΠΎ!

    ΠžΠ±Ρ€Π°Ρ‚ΠΈΡ‚Π΅ Π²Π½ΠΈΠΌΠ°Π½ΠΈΠ΅, Ρ‡Ρ‚ΠΎ Π² ΡƒΠΊΠ°Π·Π°Π½Π½ΠΎΠΌ свойствС Ρ€Π΅Ρ‡ΡŒ шла Ρ‚ΠΎΠ»ΡŒΠΊΠΎ ΠΎΠ± ΡƒΠΌΠ½ΠΎΠΆΠ΅Π½ΠΈΠΈ стСпСнСй с ΠΎΠ΄ΠΈΠ½Π°ΠΊΠΎΠ²Ρ‹ΠΌΠΈ основаниями . Оно Π½Π΅ относится ΠΊ ΠΈΡ… слоТСнию.

    НСльзя Π·Π°ΠΌΠ΅Π½ΡΡ‚ΡŒ сумму (3 3 + 3 2) Π½Π° 3 5 . Π­Ρ‚ΠΎ понятно, Ссли
    ΠΏΠΎΡΡ‡ΠΈΡ‚Π°Ρ‚ΡŒ (3 3 + 3 2) = (27 + 9) = 36 , Π° 3 5 = 243

    Бвойство β„– 2


    ЧастноС стСпСнСй

    Π—Π°ΠΏΠΎΠΌΠ½ΠΈΡ‚Π΅!

    ΠŸΡ€ΠΈ Π΄Π΅Π»Π΅Π½ΠΈΠΈ стСпСнСй с ΠΎΠ΄ΠΈΠ½Π°ΠΊΠΎΠ²Ρ‹ΠΌΠΈ основаниями основаниС остаётся Π±Π΅Π· ΠΈΠ·ΠΌΠ΅Π½Π΅Π½ΠΈΠΉ, Π° ΠΈΠ· показатСля стСпСни Π΄Π΅Π»ΠΈΠΌΠΎΠ³ΠΎ Π²Ρ‹Ρ‡ΠΈΡ‚Π°ΡŽΡ‚ ΠΏΠΎΠΊΠ°Π·Π°Ρ‚Π΅Π»ΡŒ стСпСни дСлитСля.

    = 11 3 βˆ’ 2 Β· 4 2 βˆ’ 1 = 11 Β· 4 = 44

  • ΠŸΡ€ΠΈΠΌΠ΅Ρ€. Π Π΅ΡˆΠΈΡ‚ΡŒ ΡƒΡ€Π°Π²Π½Π΅Π½ΠΈΠ΅. Π˜ΡΠΏΠΎΠ»ΡŒΠ·ΡƒΠ΅ΠΌ свойство частного стСпСнСй.
    3 8: t = 3 4

    T = 3 8 βˆ’ 4

    ΠžΡ‚Π²Π΅Ρ‚: t = 3 4 = 81
  • ΠŸΠΎΠ»ΡŒΠ·ΡƒΡΡΡŒ свойствами β„– 1 ΠΈ β„– 2, ΠΌΠΎΠΆΠ½ΠΎ Π»Π΅Π³ΠΊΠΎ ΡƒΠΏΡ€ΠΎΡ‰Π°Ρ‚ΡŒ выраТСния ΠΈ ΠΏΡ€ΠΎΠΈΠ·Π²ΠΎΠ΄ΠΈΡ‚ΡŒ вычислСния.

    • ΠŸΡ€ΠΈΠΌΠ΅Ρ€. Π£ΠΏΡ€ΠΎΡΡ‚ΠΈΡ‚ΡŒ Π²Ρ‹Ρ€Π°ΠΆΠ΅Π½ΠΈΠ΅.
      4 5m + 6 Β· 4 m + 2: 4 4m + 3 = 4 5m + 6 + m + 2: 4 4m + 3 = 4 6m + 8 βˆ’ 4m βˆ’ 3 = 4 2m + 5
    • ΠŸΡ€ΠΈΠΌΠ΅Ρ€. Найти Π·Π½Π°Ρ‡Π΅Π½ΠΈΠ΅ выраТСния, ΠΈΡΠΏΠΎΠ»ΡŒΠ·ΡƒΡ свойства стСпСни.
      = = =2 9 + 2
      2 5
      =2 11
      2 5
      = 2 11 βˆ’ 5 = 2 6 = 64

      Π’Π°ΠΆΠ½ΠΎ!

      ΠžΠ±Ρ€Π°Ρ‚ΠΈΡ‚Π΅ Π²Π½ΠΈΠΌΠ°Π½ΠΈΠ΅, Ρ‡Ρ‚ΠΎ Π² свойствС 2 Ρ€Π΅Ρ‡ΡŒ шла Ρ‚ΠΎΠ»ΡŒΠΊΠΎ ΠΎ Π΄Π΅Π»Π΅Π½ΠΈΠΈ стСпСнСй с ΠΎΠ΄ΠΈΠ½Π°ΠΊΠΎΠ²Ρ‹ΠΌΠΈ основаниями.

      НСльзя Π·Π°ΠΌΠ΅Π½ΡΡ‚ΡŒ Ρ€Π°Π·Π½ΠΎΡΡ‚ΡŒ (4 3 βˆ’4 2) Π½Π° 4 1 . Π­Ρ‚ΠΎ понятно, Ссли ΠΏΠΎΡΡ‡ΠΈΡ‚Π°Ρ‚ΡŒ (4 3 βˆ’4 2) = (64 βˆ’ 16) = 48 , Π° 4 1 = 4

      Π‘ΡƒΠ΄ΡŒΡ‚Π΅ Π²Π½ΠΈΠΌΠ°Ρ‚Π΅Π»ΡŒΠ½Ρ‹!

      Бвойство β„– 3


      Π’ΠΎΠ·Π²Π΅Π΄Π΅Π½ΠΈΠ΅ стСпСни Π² ΡΡ‚Π΅ΠΏΠ΅Π½ΡŒ

      Π—Π°ΠΏΠΎΠΌΠ½ΠΈΡ‚Π΅!

      ΠŸΡ€ΠΈ Π²ΠΎΠ·Π²Π΅Π΄Π΅Π½ΠΈΠΈ стСпСни Π² ΡΡ‚Π΅ΠΏΠ΅Π½ΡŒ основаниС стСпСни остаётся Π±Π΅Π· измСнСния, Π° ΠΏΠΎΠΊΠ°Π·Π°Ρ‚Π΅Π»ΠΈ стСпСнСй ΠΏΠ΅Ρ€Π΅ΠΌΠ½ΠΎΠΆΠ°ΡŽΡ‚ΡΡ.

      (a n) m = a n Β· m , Π³Π΄Π΅ Β«a Β» β€” любоС число, Π° Β«m Β», Β«n Β» β€” Π»ΡŽΠ±Ρ‹Π΅ Π½Π°Ρ‚ΡƒΡ€Π°Π»ΡŒΠ½Ρ‹Π΅ числа.


      Бвойства 4


      Π‘Ρ‚Π΅ΠΏΠ΅Π½ΡŒ произвСдСния

      Π—Π°ΠΏΠΎΠΌΠ½ΠΈΡ‚Π΅!

      ΠŸΡ€ΠΈ Π²ΠΎΠ·Π²Π΅Π΄Π΅Π½ΠΈΠΈ Π² ΡΡ‚Π΅ΠΏΠ΅Π½ΡŒ произвСдСния ΠΊΠ°ΠΆΠ΄Ρ‹ΠΉ ΠΈΠ· ΠΌΠ½ΠΎΠΆΠΈΡ‚Π΅Π»Π΅ΠΉ возводится Π² ΡΡ‚Π΅ΠΏΠ΅Π½ΡŒ. Π—Π°Ρ‚Π΅ΠΌ ΠΏΠΎΠ»ΡƒΡ‡Π΅Π½Π½Ρ‹Π΅ Ρ€Π΅Π·ΡƒΠ»ΡŒΡ‚Π°Ρ‚Ρ‹ ΠΏΠ΅Ρ€Π΅ΠΌΠ½ΠΎΠΆΠ°ΡŽΡ‚ΡΡ.

      (a Β· b) n = a n Β· b n , Π³Π΄Π΅ Β«a Β», Β«b Β» β€” Π»ΡŽΠ±Ρ‹Π΅ Ρ€Π°Ρ†ΠΈΠΎΠ½Π°Π»ΡŒΠ½Ρ‹Π΅ числа; Β«n Β» β€” любоС Π½Π°Ρ‚ΡƒΡ€Π°Π»ΡŒΠ½ΠΎΠ΅ число.

      • ΠŸΡ€ΠΈΠΌΠ΅Ρ€ 1.
        (6 · a 2 · b 3 · c) 2 = 6 2 · a 2 · 2 · b 3 · 2 · с 1 · 2 = 36 a 4 · b 6 · с 2
      • ΠŸΡ€ΠΈΠΌΠ΅Ρ€ 2.
        (βˆ’x 2 Β· y) 6 = ((βˆ’1) 6 Β· x 2 Β· 6 Β· y 1 Β· 6) = x 12 Β· y 6

      Π’Π°ΠΆΠ½ΠΎ!

      ΠžΠ±Ρ€Π°Ρ‚ΠΈΡ‚Π΅ Π²Π½ΠΈΠΌΠ°Π½ΠΈΠ΅, Ρ‡Ρ‚ΠΎ свойство β„– 4, ΠΊΠ°ΠΊ ΠΈ Π΄Ρ€ΡƒΠ³ΠΈΠ΅ свойства стСпСнСй, ΠΏΡ€ΠΈΠΌΠ΅Π½ΡΡŽΡ‚ ΠΈ Π² ΠΎΠ±Ρ€Π°Ρ‚Π½ΠΎΠΌ порядкС.

      (a n Β· b n)= (a Β· b) n

      Π’ΠΎ Π΅ΡΡ‚ΡŒ, Ρ‡Ρ‚ΠΎΠ±Ρ‹ ΠΏΠ΅Ρ€Π΅ΠΌΠ½ΠΎΠΆΠΈΡ‚ΡŒ стСпСни с ΠΎΠ΄ΠΈΠ½Π°ΠΊΠΎΠ²Ρ‹ΠΌΠΈ показатСлями ΠΌΠΎΠΆΠ½ΠΎ ΠΏΠ΅Ρ€Π΅ΠΌΠ½ΠΎΠΆΠΈΡ‚ΡŒ основания, Π° ΠΏΠΎΠΊΠ°Π·Π°Ρ‚Π΅Π»ΡŒ стСпСни ΠΎΡΡ‚Π°Π²ΠΈΡ‚ΡŒ Π½Π΅ΠΈΠ·ΠΌΠ΅Π½Π½Ρ‹ΠΌ.

      • ΠŸΡ€ΠΈΠΌΠ΅Ρ€. Π’Ρ‹Ρ‡ΠΈΡΠ»ΠΈΡ‚ΡŒ.
        2 4 Β· 5 4 = (2 Β· 5) 4 = 10 4 = 10 000
      • ΠŸΡ€ΠΈΠΌΠ΅Ρ€. Π’Ρ‹Ρ‡ΠΈΡΠ»ΠΈΡ‚ΡŒ.
        0,5 16 Β· 2 16 = (0,5 Β· 2) 16 = 1

      Π’ Π±ΠΎΠ»Π΅Π΅ слоТных ΠΏΡ€ΠΈΠΌΠ΅Ρ€Π°Ρ… ΠΌΠΎΠ³ΡƒΡ‚ Π²ΡΡ‚Ρ€Π΅Ρ‚ΠΈΡ‚ΡŒΡΡ случаи, ΠΊΠΎΠ³Π΄Π° ΡƒΠΌΠ½ΠΎΠΆΠ΅Π½ΠΈΠ΅ ΠΈ Π΄Π΅Π»Π΅Π½ΠΈΠ΅ Π½Π°Π΄ΠΎ Π²Ρ‹ΠΏΠΎΠ»Π½ΠΈΡ‚ΡŒ Π½Π°Π΄ стСпСнями с Ρ€Π°Π·Π½Ρ‹ΠΌΠΈ основаниями ΠΈ Ρ€Π°Π·Π½Ρ‹ΠΌΠΈ показатСлями. Π’ этом случаС совСтуСм ΠΏΠΎΡΡ‚ΡƒΠΏΠ°Ρ‚ΡŒ ΡΠ»Π΅Π΄ΡƒΡŽΡ‰ΠΈΠΌ ΠΎΠ±Ρ€Π°Π·ΠΎΠΌ.

      НапримСр, 4 5 Β· 3 2 = 4 3 Β· 4 2 Β· 3 2 = 4 3 Β· (4 Β· 3) 2 = 64 Β· 12 2 = 64 Β· 144 = 9216

      ΠŸΡ€ΠΈΠΌΠ΅Ρ€ возвСдСния Π² ΡΡ‚Π΅ΠΏΠ΅Π½ΡŒ дСсятичной Π΄Ρ€ΠΎΠ±ΠΈ.

      4 21 Β· (βˆ’0,25) 20 = 4 Β· 4 20 Β· (βˆ’0,25) 20 = 4 Β· (4 Β· (βˆ’0,25)) 20 = 4 Β· (βˆ’1) 20 = 4 Β· 1 = 4

      Бвойства 5


      Π‘Ρ‚Π΅ΠΏΠ΅Π½ΡŒ частного (Π΄Ρ€ΠΎΠ±ΠΈ)

      Π—Π°ΠΏΠΎΠΌΠ½ΠΈΡ‚Π΅!

      Π§Ρ‚ΠΎΠ±Ρ‹ возвСсти Π² ΡΡ‚Π΅ΠΏΠ΅Π½ΡŒ частноС, ΠΌΠΎΠΆΠ½ΠΎ возвСсти Π² эту ΡΡ‚Π΅ΠΏΠ΅Π½ΡŒ ΠΎΡ‚Π΄Π΅Π»ΡŒΠ½ΠΎ Π΄Π΅Π»ΠΈΠΌΠΎΠ΅ ΠΈ Π΄Π΅Π»ΠΈΡ‚Π΅Π»ΡŒ, ΠΈ ΠΏΠ΅Ρ€Π²Ρ‹ΠΉ Ρ€Π΅Π·ΡƒΠ»ΡŒΡ‚Π°Ρ‚ Ρ€Π°Π·Π΄Π΅Π»ΠΈΡ‚ΡŒ Π½Π° Π²Ρ‚ΠΎΡ€ΠΎΠΉ.

      (a: b) n = a n: b n , Π³Π΄Π΅ Β«a Β», Β«b Β» β€” Π»ΡŽΠ±Ρ‹Π΅ Ρ€Π°Ρ†ΠΈΠΎΠ½Π°Π»ΡŒΠ½Ρ‹Π΅ числа, b β‰  0, n β€” любоС Π½Π°Ρ‚ΡƒΡ€Π°Π»ΡŒΠ½ΠΎΠ΅ число.

      • ΠŸΡ€ΠΈΠΌΠ΅Ρ€. ΠŸΡ€Π΅Π΄ΡΡ‚Π°Π²ΠΈΡ‚ΡŒ Π²Ρ‹Ρ€Π°ΠΆΠ΅Π½ΠΈΠ΅ Π² Π²ΠΈΠ΄Π΅ частного стСпСнСй.
        (5: 3) 12 = 5 12: 3 12

      НапоминаСм, Ρ‡Ρ‚ΠΎ частноС ΠΌΠΎΠΆΠ½ΠΎ ΠΏΡ€Π΅Π΄ΡΡ‚Π°Π²ΠΈΡ‚ΡŒ Π² Π²ΠΈΠ΄Π΅ Π΄Ρ€ΠΎΠ±ΠΈ. ΠŸΠΎΡΡ‚ΠΎΠΌΡƒ Π½Π° Ρ‚Π΅ΠΌΠ΅ Π²ΠΎΠ·Π²Π΅Π΄Π΅Π½ΠΈΠ΅ Π΄Ρ€ΠΎΠ±ΠΈ Π² ΡΡ‚Π΅ΠΏΠ΅Π½ΡŒ ΠΌΡ‹ остановимся Π±ΠΎΠ»Π΅Π΅ ΠΏΠΎΠ΄Ρ€ΠΎΠ±Π½ΠΎ Π½Π° ΡΠ»Π΅Π΄ΡƒΡŽΡ‰Π΅ΠΉ страницС.

    Π’ ΠΏΡ€Π΅Π΄Ρ‹Π΄ΡƒΡ‰Π΅ΠΉ ΡΡ‚Π°Ρ‚ΡŒΠ΅ ΠΌΡ‹ рассказали, Ρ‡Ρ‚ΠΎ ΠΈΠ· сСбя ΠΏΡ€Π΅Π΄ΡΡ‚Π°Π²Π»ΡΡŽΡ‚ ΠΎΠ΄Π½ΠΎΡ‡Π»Π΅Π½Ρ‹. Π’ этом ΠΌΠ°Ρ‚Π΅Ρ€ΠΈΠ°Π»Π΅ Ρ€Π°Π·Π±Π΅Ρ€Π΅ΠΌ, ΠΊΠ°ΠΊ Ρ€Π΅ΡˆΠ°Ρ‚ΡŒ ΠΏΡ€ΠΈΠΌΠ΅Ρ€Ρ‹ ΠΈ Π·Π°Π΄Π°Ρ‡ΠΈ, Π² ΠΊΠΎΡ‚ΠΎΡ€Ρ‹Ρ… ΠΎΠ½ΠΈ ΠΏΡ€ΠΈΠΌΠ΅Π½ΡΡŽΡ‚ΡΡ. Π—Π΄Π΅ΡΡŒ Π±ΡƒΠ΄ΡƒΡ‚ рассмотрСны Ρ‚Π°ΠΊΠΈΠ΅ дСйствия, ΠΊΠ°ΠΊ Π²Ρ‹Ρ‡ΠΈΡ‚Π°Π½ΠΈΠ΅, слоТСниС, ΡƒΠΌΠ½ΠΎΠΆΠ΅Π½ΠΈΠ΅, Π΄Π΅Π»Π΅Π½ΠΈΠ΅ ΠΎΠ΄Π½ΠΎΡ‡Π»Π΅Π½ΠΎΠ² ΠΈ Π²ΠΎΠ·Π²Π΅Π΄Π΅Π½ΠΈΠ΅ ΠΈΡ… Π² ΡΡ‚Π΅ΠΏΠ΅Π½ΡŒ с Π½Π°Ρ‚ΡƒΡ€Π°Π»ΡŒΠ½Ρ‹ΠΌ ΠΏΠΎΠΊΠ°Π·Π°Ρ‚Π΅Π»Π΅ΠΌ. ΠœΡ‹ ΠΏΠΎΠΊΠ°ΠΆΠ΅ΠΌ, ΠΊΠ°ΠΊ ΠΎΠΏΡ€Π΅Π΄Π΅Π»ΡΡŽΡ‚ΡΡ Ρ‚Π°ΠΊΠΈΠ΅ ΠΎΠΏΠ΅Ρ€Π°Ρ†ΠΈΠΈ, ΠΎΠ±ΠΎΠ·Π½Π°Ρ‡ΠΈΠΌ основныС ΠΏΡ€Π°Π²ΠΈΠ»Π° ΠΈΡ… выполнСния ΠΈ Ρ‚ΠΎ, Ρ‡Ρ‚ΠΎ Π΄ΠΎΠ»ΠΆΠ½ΠΎ получится Π² Ρ€Π΅Π·ΡƒΠ»ΡŒΡ‚Π°Ρ‚Π΅. ВсС тСорСтичСскиС полоТСния, ΠΊΠ°ΠΊ ΠΎΠ±Ρ‹Ρ‡Π½ΠΎ, Π±ΡƒΠ΄ΡƒΡ‚ ΠΏΡ€ΠΎΠΈΠ»Π»ΡŽΡΡ‚Ρ€ΠΈΡ€ΠΎΠ²Π°Π½Ρ‹ ΠΏΡ€ΠΈΠΌΠ΅Ρ€Π°ΠΌΠΈ Π·Π°Π΄Π°Ρ‡ с описаниями Ρ€Π΅ΡˆΠ΅Π½ΠΈΠΉ.

    Π£Π΄ΠΎΠ±Π½Π΅Π΅ всСго Ρ€Π°Π±ΠΎΡ‚Π°Ρ‚ΡŒ со стандартной записью ΠΎΠ΄Π½ΠΎΡ‡Π»Π΅Π½ΠΎΠ², поэтому всС выраТСния, ΠΊΠΎΡ‚ΠΎΡ€Ρ‹Π΅ Π±ΡƒΠ΄ΡƒΡ‚ ΠΈΡΠΏΠΎΠ»ΡŒΠ·ΠΎΠ²Π°Π½Ρ‹ Π² ΡΡ‚Π°Ρ‚ΡŒΠ΅, ΠΌΡ‹ ΠΏΡ€ΠΈΠ²ΠΎΠ΄ΠΈΠΌ Π² стандартном Π²ΠΈΠ΄Π΅. Если ΠΈΠ·Π½Π°Ρ‡Π°Π»ΡŒΠ½ΠΎ ΠΎΠ½ΠΈ Π·Π°Π΄Π°Π½Ρ‹ ΠΈΠ½Π°Ρ‡Π΅, рСкомСндуСтся сначала привСсти ΠΈΡ… ΠΊ общСпринятой Ρ„ΠΎΡ€ΠΌΠ΅.

    ΠŸΡ€Π°Π²ΠΈΠ»Π° слоТСния ΠΈ вычитания ΠΎΠ΄Π½ΠΎΡ‡Π»Π΅Π½ΠΎΠ²

    НаиболСС простыС дСйствия, ΠΊΠΎΡ‚ΠΎΡ€Ρ‹Π΅ ΠΌΠΎΠΆΠ½ΠΎ ΠΏΡ€ΠΎΠ²ΠΎΠ΄ΠΈΡ‚ΡŒ с ΠΎΠ΄Π½ΠΎΡ‡Π»Π΅Π½Π°ΠΌΠΈ – это Π²Ρ‹Ρ‡ΠΈΡ‚Π°Π½ΠΈΠ΅ ΠΈ слоТСниС. Π’ ΠΎΠ±Ρ‰Π΅ΠΌ случаС Ρ€Π΅Π·ΡƒΠ»ΡŒΡ‚Π°Ρ‚ΠΎΠΌ этих дСйствий Π±ΡƒΠ΄Π΅Ρ‚ ΡΠ²Π»ΡΡ‚ΡŒΡΡ ΠΌΠ½ΠΎΠ³ΠΎΡ‡Π»Π΅Π½ (ΠΎΠ΄Π½ΠΎΡ‡Π»Π΅Π½ Π²ΠΎΠ·ΠΌΠΎΠΆΠ΅Π½ Π² Π½Π΅ΠΊΠΎΡ‚ΠΎΡ€Ρ‹Ρ… частных случаях).

    Когда ΠΌΡ‹ складываСм ΠΈΠ»ΠΈ Π²Ρ‹Ρ‡ΠΈΡ‚Π°Π΅ΠΌ ΠΎΠ΄Π½ΠΎΡ‡Π»Π΅Π½Ρ‹, сначала записываСм Π² общСпринятой Ρ„ΠΎΡ€ΠΌΠ΅ ΡΠΎΠΎΡ‚Π²Π΅Ρ‚ΡΡ‚Π²ΡƒΡŽΡ‰ΡƒΡŽ сумму ΠΈ Ρ€Π°Π·Π½ΠΎΡΡ‚ΡŒ, послС Ρ‡Π΅Π³ΠΎ ΡƒΠΏΡ€ΠΎΡ‰Π°Π΅ΠΌ ΠΏΠΎΠ»ΡƒΡ‡ΠΈΠ²ΡˆΠ΅Π΅ΡΡ Π²Ρ‹Ρ€Π°ΠΆΠ΅Π½ΠΈΠ΅. Если Π΅ΡΡ‚ΡŒ ΠΏΠΎΠ΄ΠΎΠ±Π½Ρ‹Π΅ слагаСмыС, ΠΈΡ… Π½ΡƒΠΆΠ½ΠΎ привСсти, скобки – Ρ€Π°ΡΠΊΡ€Ρ‹Ρ‚ΡŒ. Поясним Π½Π° ΠΏΡ€ΠΈΠΌΠ΅Ρ€Π΅.

    ΠŸΡ€ΠΈΠΌΠ΅Ρ€ 1

    УсловиС: Π²Ρ‹ΠΏΠΎΠ»Π½ΠΈΡ‚Π΅ слоТСниС ΠΎΠ΄Π½ΠΎΡ‡Π»Π΅Π½ΠΎΠ² βˆ’ 3 Β· x ΠΈ 2 , 72 Β· x 3 Β· y 5 Β· z .

    РСшСниС

    Π—Π°ΠΏΠΈΡˆΠ΅ΠΌ сумму исходных Π²Ρ‹Ρ€Π°ΠΆΠ΅Π½ΠΈΠΉ. Π”ΠΎΠ±Π°Π²ΠΈΠΌ скобки ΠΈ поставим ΠΌΠ΅ΠΆΠ΄Ρƒ Π½ΠΈΠΌΠΈ плюс. Π£ нас получится ΡΠ»Π΅Π΄ΡƒΡŽΡ‰Π΅Π΅:

    (βˆ’ 3 Β· x) + (2 , 72 Β· x 3 Β· y 5 Β· z)

    Когда ΠΌΡ‹ Π²Ρ‹ΠΏΠΎΠ»Π½ΠΈΠΌ раскрытиС скобок, получится — 3 Β· x + 2 , 72 Β· x 3 Β· y 5 Β· z . Π­Ρ‚ΠΎ ΠΌΠ½ΠΎΠ³ΠΎΡ‡Π»Π΅Π½, записанный Π² стандартной Ρ„ΠΎΡ€ΠΌΠ΅, ΠΊΠΎΡ‚ΠΎΡ€Ρ‹ΠΉ ΠΈ Π±ΡƒΠ΄Π΅Ρ‚ Ρ€Π΅Π·ΡƒΠ»ΡŒΡ‚Π°Ρ‚ΠΎΠΌ слоТСния Π΄Π°Π½Π½Ρ‹Ρ… ΠΎΠ΄Π½ΠΎΡ‡Π»Π΅Π½ΠΎΠ².

    ΠžΡ‚Π²Π΅Ρ‚: (βˆ’ 3 Β· x) + (2 , 72 Β· x 3 Β· y 5 Β· z) = βˆ’ 3 Β· x + 2 , 72 Β· x 3 Β· y 5 Β· z .

    Если Ρƒ нас Π·Π°Π΄Π°Π½ΠΎ Ρ‚Ρ€ΠΈ, Ρ‡Π΅Ρ‚Ρ‹Ρ€Π΅ ΠΈ большС слагаСмых, ΠΌΡ‹ осущСствляСм это дСйствиС Ρ‚ΠΎΡ‡Π½ΠΎ Ρ‚Π°ΠΊ ΠΆΠ΅.

    ΠŸΡ€ΠΈΠΌΠ΅Ρ€ 2

    УсловиС: ΠΏΡ€ΠΎΠ²Π΅Π΄ΠΈΡ‚Π΅ Π² ΠΏΡ€Π°Π²ΠΈΠ»ΡŒΠ½ΠΎΠΌ порядкС ΡƒΠΊΠ°Π·Π°Π½Π½Ρ‹Π΅ дСйствия с ΠΌΠ½ΠΎΠ³ΠΎΡ‡Π»Π΅Π½Π°ΠΌΠΈ

    3 Β· a 2 — (- 4 Β· a Β· c) + a 2 — 7 Β· a 2 + 4 9 — 2 2 3 Β· a Β· c

    РСшСниС

    НачнСм с раскрытия скобок.

    3 Β· a 2 + 4 Β· a Β· c + a 2 — 7 Β· a 2 + 4 9 — 2 2 3 Β· a Β· c

    ΠœΡ‹ Π²ΠΈΠ΄ΠΈΠΌ, Ρ‡Ρ‚ΠΎ ΠΏΠΎΠ»ΡƒΡ‡Π΅Π½Π½ΠΎΠ΅ Π²Ρ‹Ρ€Π°ΠΆΠ΅Π½ΠΈΠ΅ ΠΌΠΎΠΆΠ½ΠΎ ΡƒΠΏΡ€ΠΎΡΡ‚ΠΈΡ‚ΡŒ ΠΏΡƒΡ‚Π΅ΠΌ привСдСния ΠΏΠΎΠ΄ΠΎΠ±Π½Ρ‹Ρ… слагаСмых:

    3 Β· a 2 + 4 Β· a Β· c + a 2 — 7 Β· a 2 + 4 9 — 2 2 3 Β· a Β· c = = (3 Β· a 2 + a 2 — 7 Β· a 2) + 4 Β· a Β· c — 2 2 3 Β· a Β· c + 4 9 = = — 3 Β· a 2 + 1 1 3 Β· a Β· c + 4 9

    Π£ нас получился ΠΌΠ½ΠΎΠ³ΠΎΡ‡Π»Π΅Π½, ΠΊΠΎΡ‚ΠΎΡ€Ρ‹ΠΉ ΠΈ Π±ΡƒΠ΄Π΅Ρ‚ Ρ€Π΅Π·ΡƒΠ»ΡŒΡ‚Π°Ρ‚ΠΎΠΌ Π΄Π°Π½Π½ΠΎΠ³ΠΎ дСйствия.

    ΠžΡ‚Π²Π΅Ρ‚: 3 Β· a 2 — (- 4 Β· a Β· c) + a 2 — 7 Β· a 2 + 4 9 — 2 2 3 Β· a Β· c = — 3 Β· a 2 + 1 1 3 Β· a Β· c + 4 9

    Π’ ΠΏΡ€ΠΈΠ½Ρ†ΠΈΠΏΠ΅, ΠΌΡ‹ ΠΌΠΎΠΆΠ΅ΠΌ Π²Ρ‹ΠΏΠΎΠ»Π½ΠΈΡ‚ΡŒ слоТСниС ΠΈ Π²Ρ‹Ρ‡ΠΈΡ‚Π°Π½ΠΈΠ΅ Π΄Π²ΡƒΡ… ΠΎΠ΄Π½ΠΎΡ‡Π»Π΅Π½ΠΎΠ² с Π½Π΅ΠΊΠΎΡ‚ΠΎΡ€Ρ‹ΠΌΠΈ ограничСниями Ρ‚Π°ΠΊ, Ρ‡Ρ‚ΠΎΠ±Ρ‹ ΠΏΠΎΠ»ΡƒΡ‡ΠΈΡ‚ΡŒ Π² ΠΈΡ‚ΠΎΠ³Π΅ ΠΎΠ΄Π½ΠΎΡ‡Π»Π΅Π½. Для этого Π½ΡƒΠΆΠ½ΠΎ ΡΠΎΠ±Π»ΡŽΡΡ‚ΠΈ Π½Π΅ΠΊΠΎΡ‚ΠΎΡ€Ρ‹Π΅ условия, ΠΊΠ°ΡΠ°ΡŽΡ‰ΠΈΠ΅ΡΡ слагаСмых ΠΈ Π²Ρ‹Ρ‡ΠΈΡ‚Π°Π΅ΠΌΡ‹Ρ… ΠΎΠ΄Π½ΠΎΡ‡Π»Π΅Π½ΠΎΠ². О Ρ‚ΠΎΠΌ, ΠΊΠ°ΠΊ это дСлаСтся, ΠΌΡ‹ расскаТСм Π² ΠΎΡ‚Π΄Π΅Π»ΡŒΠ½ΠΎΠΉ ΡΡ‚Π°Ρ‚ΡŒΠ΅.

    ΠŸΡ€Π°Π²ΠΈΠ»Π° умноТСния ΠΎΠ΄Π½ΠΎΡ‡Π»Π΅Π½ΠΎΠ²

    ДСйствиС умноТСния Π½Π΅ Π½Π°Π»Π°Π³Π°Π΅Ρ‚ Π½ΠΈΠΊΠ°ΠΊΠΈΡ… ΠΎΠ³Ρ€Π°Π½ΠΈΡ‡Π΅Π½ΠΈΠΉ Π½Π° ΠΌΠ½ΠΎΠΆΠΈΡ‚Π΅Π»ΠΈ. Π£ΠΌΠ½ΠΎΠΆΠ°Π΅ΠΌΡ‹Π΅ ΠΎΠ΄Π½ΠΎΡ‡Π»Π΅Π½Ρ‹ Π½Π΅ Π΄ΠΎΠ»ΠΆΠ½Ρ‹ ΡΠΎΠΎΡ‚Π²Π΅Ρ‚ΡΡ‚Π²ΠΎΠ²Π°Ρ‚ΡŒ Π½ΠΈΠΊΠ°ΠΊΠΈΠΌ Π΄ΠΎΠΏΠΎΠ»Π½ΠΈΡ‚Π΅Π»ΡŒΠ½Ρ‹ΠΌ условиям, Ρ‡Ρ‚ΠΎΠ±Ρ‹ Π² Ρ€Π΅Π·ΡƒΠ»ΡŒΡ‚Π°Ρ‚Π΅ получится ΠΎΠ΄Π½ΠΎΡ‡Π»Π΅Π½.

    Π§Ρ‚ΠΎΠ±Ρ‹ Π²Ρ‹ΠΏΠΎΠ»Π½ΠΈΡ‚ΡŒ ΡƒΠΌΠ½ΠΎΠΆΠ΅Π½ΠΈΠ΅ ΠΎΠ΄Π½ΠΎΡ‡Π»Π΅Π½ΠΎΠ², Π½ΡƒΠΆΠ½ΠΎ Π²Ρ‹ΠΏΠΎΠ»Π½ΠΈΡ‚ΡŒ ΡΠ»Π΅Π΄ΡƒΡŽΡ‰ΠΈΠ΅ шаги:

    1. ΠŸΡ€Π°Π²ΠΈΠ»ΡŒΠ½ΠΎ Π·Π°ΠΏΠΈΡΠ°Ρ‚ΡŒ ΠΏΡ€ΠΎΠΈΠ·Π²Π΅Π΄Π΅Π½ΠΈΠ΅.
    2. Π Π°ΡΠΊΡ€Ρ‹Ρ‚ΡŒ скобки Π² ΠΏΠΎΠ»ΡƒΡ‡Π΅Π½Π½ΠΎΠΌ Π²Ρ‹Ρ€Π°ΠΆΠ΅Π½ΠΈΠΈ.
    3. Π‘Π³Ρ€ΡƒΠΏΠΏΠΈΡ€ΠΎΠ²Π°Ρ‚ΡŒ ΠΏΠΎ возмоТности ΠΌΠ½ΠΎΠΆΠΈΡ‚Π΅Π»ΠΈ с ΠΎΠ΄ΠΈΠ½Π°ΠΊΠΎΠ²Ρ‹ΠΌΠΈ ΠΏΠ΅Ρ€Π΅ΠΌΠ΅Π½Π½Ρ‹ΠΌΠΈ ΠΈ числовыС ΠΌΠ½ΠΎΠΆΠΈΡ‚Π΅Π»ΠΈ ΠΎΡ‚Π΄Π΅Π»ΡŒΠ½ΠΎ.
    4. Π’Ρ‹ΠΏΠΎΠ»Π½ΠΈΡ‚ΡŒ Π½Π΅ΠΎΠ±Ρ…ΠΎΠ΄ΠΈΠΌΡ‹Π΅ дСйствия с числами ΠΈ ΠΏΡ€ΠΈΠΌΠ΅Π½ΠΈΡ‚ΡŒ ΠΊ ΠΎΡΡ‚Π°Π²ΡˆΠΈΠΌΡΡ мноТитСлям свойство умноТСния стСпСнСй с ΠΎΠ΄ΠΈΠ½Π°ΠΊΠΎΠ²Ρ‹ΠΌΠΈ основаниями.

    ΠŸΠΎΡΠΌΠΎΡ‚Ρ€ΠΈΠΌ, ΠΊΠ°ΠΊ это дСлаСтся Π½Π° ΠΏΡ€Π°ΠΊΡ‚ΠΈΠΊΠ΅.

    ΠŸΡ€ΠΈΠΌΠ΅Ρ€ 3

    УсловиС: Π²Ρ‹ΠΏΠΎΠ»Π½ΠΈΡ‚Π΅ ΡƒΠΌΠ½ΠΎΠΆΠ΅Π½ΠΈΠ΅ ΠΎΠ΄Π½ΠΎΡ‡Π»Π΅Π½ΠΎΠ² 2 Β· x 4 Β· y Β· z ΠΈ — 7 16 Β· t 2 Β· x 2 Β· z 11 .

    РСшСниС

    НачнСм с составлСния произвСдСния.

    РаскрываСм Π² Π½Π΅ΠΌ скобки ΠΈ ΠΏΠΎΠ»ΡƒΡ‡Π°Π΅ΠΌ ΡΠ»Π΅Π΄ΡƒΡŽΡ‰Π΅Π΅:

    2 Β· x 4 Β· y Β· z Β· — 7 16 Β· t 2 Β· x 2 Β· z 11

    2 Β· — 7 16 Β· t 2 Β· x 4 Β· x 2 Β· y Β· z 3 Β· z 11

    ВсС, Ρ‡Ρ‚ΠΎ Π½Π°ΠΌ ΠΎΡΡ‚Π°Π»ΠΎΡΡŒ ΡΠ΄Π΅Π»Π°Ρ‚ΡŒ – это ΡƒΠΌΠ½ΠΎΠΆΠΈΡ‚ΡŒ числа Π² ΠΏΠ΅Ρ€Π²Ρ‹Ρ… скобках ΠΈ ΠΏΡ€ΠΈΠΌΠ΅Π½ΠΈΡ‚ΡŒ свойство стСпСнСй для Π²Ρ‚ΠΎΡ€Ρ‹Ρ…. Π’ ΠΈΡ‚ΠΎΠ³Π΅ ΠΏΠΎΠ»ΡƒΡ‡ΠΈΠΌ ΡΠ»Π΅Π΄ΡƒΡŽΡ‰Π΅Π΅:

    2 Β· — 7 16 Β· t 2 Β· x 4 Β· x 2 Β· y Β· z 3 Β· z 11 = — 7 8 Β· t 2 Β· x 4 + 2 Β· y Β· z 3 + 11 = = — 7 8 Β· t 2 Β· x 6 Β· y Β· z 14

    ΠžΡ‚Π²Π΅Ρ‚: 2 Β· x 4 Β· y Β· z Β· — 7 16 Β· t 2 Β· x 2 Β· z 11 = — 7 8 Β· t 2 Β· x 6 Β· y Β· z 14 .

    Если Ρƒ нас Π² условии стоят Ρ‚Ρ€ΠΈ ΠΌΠ½ΠΎΠ³ΠΎΡ‡Π»Π΅Π½Π° ΠΈ большС, ΠΌΡ‹ ΡƒΠΌΠ½ΠΎΠΆΠ°Π΅ΠΌ ΠΈΡ… ΠΏΠΎ Ρ‚ΠΎΡ‡Π½ΠΎ Ρ‚Π°ΠΊΠΎΠΌΡƒ ΠΆΠ΅ Π°Π»Π³ΠΎΡ€ΠΈΡ‚ΠΌΡƒ. Π‘ΠΎΠ»Π΅Π΅ ΠΏΠΎΠ΄Ρ€ΠΎΠ±Π½ΠΎ вопрос умноТСния ΠΎΠ΄Π½ΠΎΡ‡Π»Π΅Π½ΠΎΠ² ΠΌΡ‹ рассмотрим Π² Ρ€Π°ΠΌΠΊΠ°Ρ… ΠΎΡ‚Π΄Π΅Π»ΡŒΠ½ΠΎΠ³ΠΎ ΠΌΠ°Ρ‚Π΅Ρ€ΠΈΠ°Π»Π°.

    ΠŸΡ€Π°Π²ΠΈΠ»Π° возвСдСния ΠΎΠ΄Π½ΠΎΡ‡Π»Π΅Π½Π° Π² ΡΡ‚Π΅ΠΏΠ΅Π½ΡŒ

    ΠœΡ‹ Π·Π½Π°Π΅ΠΌ, Ρ‡Ρ‚ΠΎ ΡΡ‚Π΅ΠΏΠ΅Π½ΡŒΡŽ с Π½Π°Ρ‚ΡƒΡ€Π°Π»ΡŒΠ½Ρ‹ΠΌ ΠΏΠΎΠΊΠ°Π·Π°Ρ‚Π΅Π»Π΅ΠΌ Π½Π°Π·Ρ‹Π²Π°ΡŽΡ‚ ΠΏΡ€ΠΎΠΈΠ·Π²Π΅Π΄Π΅Π½ΠΈΠ΅ Π½Π΅ΠΊΠΎΡ‚ΠΎΡ€ΠΎΠ³ΠΎ числа ΠΎΠ΄ΠΈΠ½Π°ΠΊΠΎΠ²Ρ‹Ρ… ΠΌΠ½ΠΎΠΆΠΈΡ‚Π΅Π»Π΅ΠΉ. На ΠΈΡ… количСство ΡƒΠΊΠ°Π·Ρ‹Π²Π°Π΅Ρ‚ число Π² ΠΏΠΎΠΊΠ°Π·Π°Ρ‚Π΅Π»Π΅. Богласно этому ΠΎΠΏΡ€Π΅Π΄Π΅Π»Π΅Π½ΠΈΡŽ, Π²ΠΎΠ·Π²Π΅Π΄Π΅Π½ΠΈΠ΅ ΠΎΠ΄Π½ΠΎΡ‡Π»Π΅Π½Π° Π² ΡΡ‚Π΅ΠΏΠ΅Π½ΡŒ Ρ€Π°Π²Π½ΠΎΠ·Π½Π°Ρ‡Π½ΠΎ ΡƒΠΌΠ½ΠΎΠΆΠ΅Π½ΠΈΡŽ ΡƒΠΊΠ°Π·Π°Π½Π½ΠΎΠ³ΠΎ числа ΠΎΠ΄ΠΈΠ½Π°ΠΊΠΎΠ²Ρ‹Ρ… ΠΎΠ΄Π½ΠΎΡ‡Π»Π΅Π½ΠΎΠ². ΠŸΠΎΡΠΌΠΎΡ‚Ρ€ΠΈΠΌ, ΠΊΠ°ΠΊ это дСлаСтся.

    ΠŸΡ€ΠΈΠΌΠ΅Ρ€ 4

    УсловиС: Π²Ρ‹ΠΏΠΎΠ»Π½ΠΈΡ‚Π΅ Π²ΠΎΠ·Π²Π΅Π΄Π΅Π½ΠΈΠ΅ ΠΎΠ΄Π½ΠΎΡ‡Π»Π΅Π½Π° βˆ’ 2 Β· a Β· b 4 Π² ΡΡ‚Π΅ΠΏΠ΅Π½ΡŒ 3 .

    РСшСниС

    ΠœΡ‹ ΠΌΠΎΠΆΠ΅ΠΌ Π·Π°ΠΌΠ΅Π½ΠΈΡ‚ΡŒ Π²ΠΎΠ·Π²Π΅Π΄Π΅Π½ΠΈΠ΅ Π² ΡΡ‚Π΅ΠΏΠ΅Π½ΡŒ Π½Π° ΡƒΠΌΠ½ΠΎΠΆΠ΅Π½ΠΈΠ΅ 3 -Ρ… ΠΎΠ΄Π½ΠΎΡ‡Π»Π΅Π½ΠΎΠ² βˆ’ 2 Β· a Β· b 4 . Π—Π°ΠΏΠΈΡˆΠ΅ΠΌ ΠΈ ΠΏΠΎΠ»ΡƒΡ‡ΠΈΠΌ Π½ΡƒΠΆΠ½Ρ‹ΠΉ ΠΎΡ‚Π²Π΅Ρ‚:

    (βˆ’ 2 Β· a Β· b 4) 3 = (βˆ’ 2 Β· a Β· b 4) Β· (βˆ’ 2 Β· a Β· b 4) Β· (βˆ’ 2 Β· a Β· b 4) = = ((βˆ’ 2) Β· (βˆ’ 2) Β· (βˆ’ 2)) Β· (a Β· a Β· a) Β· (b 4 Β· b 4 Β· b 4) = βˆ’ 8 Β· a 3 Β· b 12

    ΠžΡ‚Π²Π΅Ρ‚: (βˆ’ 2 Β· a Β· b 4) 3 = βˆ’ 8 Β· a 3 Β· b 12 .

    А ΠΊΠ°ΠΊ Π±Ρ‹Ρ‚ΡŒ Π² Ρ‚ΠΎΠΌ случаС, ΠΊΠΎΠ³Π΄Π° ΡΡ‚Π΅ΠΏΠ΅Π½ΡŒ ΠΈΠΌΠ΅Π΅Ρ‚ большой ΠΏΠΎΠΊΠ°Π·Π°Ρ‚Π΅Π»ΡŒ? Π—Π°ΠΏΠΈΡΡ‹Π²Π°Ρ‚ΡŒ большоС количСство ΠΌΠ½ΠΎΠΆΠΈΡ‚Π΅Π»Π΅ΠΉ Π½Π΅ΡƒΠ΄ΠΎΠ±Π½ΠΎ. Π’ΠΎΠ³Π΄Π° для Ρ€Π΅ΡˆΠ΅Π½ΠΈΡ Ρ‚Π°ΠΊΠΎΠΉ Π·Π°Π΄Π°Ρ‡ΠΈ Π½Π°ΠΌ Π½Π°Π΄ΠΎ ΠΏΡ€ΠΈΠΌΠ΅Π½ΠΈΡ‚ΡŒ свойства стСпСни, Π° ΠΈΠΌΠ΅Π½Π½ΠΎ свойство стСпСни произвСдСния ΠΈ свойство стСпСни Π² стСпСни.

    РСшим Π·Π°Π΄Π°Ρ‡Ρƒ, ΠΊΠΎΡ‚ΠΎΡ€ΡƒΡŽ ΠΌΡ‹ ΠΏΡ€ΠΈΠ²Π΅Π»ΠΈ Π²Ρ‹ΡˆΠ΅, ΡƒΠΊΠ°Π·Π°Π½Π½Ρ‹ΠΌ способом.

    ΠŸΡ€ΠΈΠΌΠ΅Ρ€ 5

    УсловиС: Π²Ρ‹ΠΏΠΎΠ»Π½ΠΈΡ‚Π΅ Π²ΠΎΠ·Π²Π΅Π΄Π΅Π½ΠΈΠ΅ βˆ’ 2 Β· a Β· b 4 Π² Ρ‚Ρ€Π΅Ρ‚ΡŒΡŽ ΡΡ‚Π΅ΠΏΠ΅Π½ΡŒ.

    РСшСниС

    Зная свойство стСпСни Π² стСпСни, ΠΌΡ‹ ΠΌΠΎΠΆΠ΅ΠΌ ΠΏΠ΅Ρ€Π΅ΠΉΡ‚ΠΈ ΠΊ Π²Ρ‹Ρ€Π°ΠΆΠ΅Π½ΠΈΡŽ ΡΠ»Π΅Π΄ΡƒΡŽΡ‰Π΅Π³ΠΎ Π²ΠΈΠ΄Π°:

    (βˆ’ 2 Β· a Β· b 4) 3 = (βˆ’ 2) 3 Β· a 3 Β· (b 4) 3 .

    ПослС этого ΠΌΡ‹ Π²ΠΎΠ·Π²ΠΎΠ΄ΠΈΠΌ Π² ΡΡ‚Π΅ΠΏΠ΅Π½ΡŒ — 2 ΠΈ примСняСм свойство стСпСни Π² стСпСни:

    (βˆ’ 2) 3 Β· (a) 3 Β· (b 4) 3 = βˆ’ 8 Β· a 3 Β· b 4 Β· 3 = βˆ’ 8 Β· a 3 Β· b 12 .

    ΠžΡ‚Π²Π΅Ρ‚: βˆ’ 2 Β· a Β· b 4 = βˆ’ 8 Β· a 3 Β· b 12 .

    Π’ΠΎΠ·Π²Π΅Π΄Π΅Π½ΠΈΡŽ ΠΎΠ΄Π½ΠΎΡ‡Π»Π΅Π½Π° Π² ΡΡ‚Π΅ΠΏΠ΅Π½ΡŒ ΠΌΡ‹ Ρ‚Π°ΠΊΠΆΠ΅ посвятили ΠΎΡ‚Π΄Π΅Π»ΡŒΠ½ΡƒΡŽ ΡΡ‚Π°Ρ‚ΡŒΡŽ.

    ΠŸΡ€Π°Π²ΠΈΠ»Π° дСлСния ΠΎΠ΄Π½ΠΎΡ‡Π»Π΅Π½ΠΎΠ²

    ПослСднСС дСйствиС с ΠΎΠ΄Π½ΠΎΡ‡Π»Π΅Π½Π°ΠΌΠΈ, ΠΊΠΎΡ‚ΠΎΡ€ΠΎΠ΅ ΠΌΡ‹ Ρ€Π°Π·Π±Π΅Ρ€Π΅ΠΌ Π² Π΄Π°Π½Π½ΠΎΠΌ ΠΌΠ°Ρ‚Π΅Ρ€ΠΈΠ°Π»Π΅, – Π΄Π΅Π»Π΅Π½ΠΈΠ΅ ΠΎΠ΄Π½ΠΎΡ‡Π»Π΅Π½Π° Π½Π° ΠΎΠ΄Π½ΠΎΡ‡Π»Π΅Π½. Π’ Ρ€Π΅Π·ΡƒΠ»ΡŒΡ‚Π°Ρ‚Π΅ ΠΌΡ‹ Π΄ΠΎΠ»ΠΆΠ½Ρ‹ ΠΏΠΎΠ»ΡƒΡ‡ΠΈΡ‚ΡŒ Ρ€Π°Ρ†ΠΈΠΎΠ½Π°Π»ΡŒΠ½ΡƒΡŽ (Π°Π»Π³Π΅Π±Ρ€Π°ΠΈΡ‡Π΅ΡΠΊΡƒΡŽ) Π΄Ρ€ΠΎΠ±ΡŒ (Π² Π½Π΅ΠΊΠΎΡ‚ΠΎΡ€Ρ‹Ρ… случаях Π²ΠΎΠ·ΠΌΠΎΠΆΠ½ΠΎ ΠΏΠΎΠ»ΡƒΡ‡Π΅Π½ΠΈΠ΅ ΠΎΠ΄Π½ΠΎΡ‡Π»Π΅Π½Π°). Π‘Ρ€Π°Π·Ρƒ ΡƒΡ‚ΠΎΡ‡Π½ΠΈΠΌ, Ρ‡Ρ‚ΠΎ Π΄Π΅Π»Π΅Π½ΠΈΠ΅ Π½Π° Π½ΡƒΠ»Π΅Π²ΠΎΠΉ ΠΎΠ΄Π½ΠΎΡ‡Π»Π΅Π½ Π½Π΅ опрСдСляСтся, ΠΏΠΎΡΠΊΠΎΠ»ΡŒΠΊΡƒ Π½Π΅ опрСдСляСтся Π΄Π΅Π»Π΅Π½ΠΈΠ΅ Π½Π° 0.

    Для выполнСния дСлСния Π½Π°ΠΌ Π½ΡƒΠΆΠ½ΠΎ Π·Π°ΠΏΠΈΡΠ°Ρ‚ΡŒ ΡƒΠΊΠ°Π·Π°Π½Π½Ρ‹Π΅ ΠΎΠ΄Π½ΠΎΡ‡Π»Π΅Π½Ρ‹ Π² Ρ„ΠΎΡ€ΠΌΠ΅ Π΄Ρ€ΠΎΠ±ΠΈ ΠΈ ΡΠΎΠΊΡ€Π°Ρ‚ΠΈΡ‚ΡŒ Π΅Π΅, Ссли Π΅ΡΡ‚ΡŒ такая Π²ΠΎΠ·ΠΌΠΎΠΆΠ½ΠΎΡΡ‚ΡŒ.

    ΠŸΡ€ΠΈΠΌΠ΅Ρ€ 6

    УсловиС: Π²Ρ‹ΠΏΠΎΠ»Π½ΠΈΡ‚Π΅ Π΄Π΅Π»Π΅Π½ΠΈΠ΅ ΠΎΠ΄Π½ΠΎΡ‡Π»Π΅Π½Π° βˆ’ 9 Β· x 4 Β· y 3 Β· z 7 Π½Π° βˆ’ 6 Β· p 3 Β· t 5 Β· x 2 Β· y 2 .

    РСшСниС

    НачнСм с записи ΠΎΠ΄Π½ΠΎΡ‡Π»Π΅Π½ΠΎΠ² Π² Ρ„ΠΎΡ€ΠΌΠ΅ Π΄Ρ€ΠΎΠ±ΠΈ.

    9 Β· x 4 Β· y 3 Β· z 7 — 6 Β· p 3 Β· t 5 Β· x 2 Β· y 2

    Π­Ρ‚Ρƒ Π΄Ρ€ΠΎΠ±ΡŒ ΠΌΠΎΠΆΠ½ΠΎ ΡΠΎΠΊΡ€Π°Ρ‚ΠΈΡ‚ΡŒ. ПослС выполнСния этого дСйствия ΠΏΠΎΠ»ΡƒΡ‡ΠΈΠΌ:

    3 Β· x 2 Β· y Β· z 7 2 Β· p 3 Β· t 5

    ΠžΡ‚Π²Π΅Ρ‚: — 9 Β· x 4 Β· y 3 Β· z 7 — 6 Β· p 3 Β· t 5 Β· x 2 Β· y 2 = 3 Β· x 2 Β· y Β· z 7 2 Β· p 3 Β· t 5 .

    Условия, ΠΏΡ€ΠΈ ΠΊΠΎΡ‚ΠΎΡ€Ρ‹Ρ… Π² Ρ€Π΅Π·ΡƒΠ»ΡŒΡ‚Π°Ρ‚Π΅ дСлСния ΠΎΠ΄Π½ΠΎΡ‡Π»Π΅Π½ΠΎΠ² ΠΌΡ‹ ΠΏΠΎΠ»ΡƒΡ‡ΠΈΠΌ ΠΎΠ΄Π½ΠΎΡ‡Π»Π΅Π½, приводятся Π² ΠΎΡ‚Π΄Π΅Π»ΡŒΠ½ΠΎΠΉ ΡΡ‚Π°Ρ‚ΡŒΠ΅.

    Если Π²Ρ‹ Π·Π°ΠΌΠ΅Ρ‚ΠΈΠ»ΠΈ ΠΎΡˆΠΈΠ±ΠΊΡƒ Π² тСкстС, поТалуйста, Π²Ρ‹Π΄Π΅Π»ΠΈΡ‚Π΅ Π΅Ρ‘ ΠΈ Π½Π°ΠΆΠΌΠΈΡ‚Π΅ Ctrl+Enter

    Как Ρ€Π΅ΡˆΠ°Ρ‚ΡŒ выраТСния со стСпСнями ΠΈ дробями. Π‘Π»ΠΎΠΆΠ΅Π½ΠΈΠ΅, Π²Ρ‹Ρ‡ΠΈΡ‚Π°Π½ΠΈΠ΅, ΡƒΠΌΠ½ΠΎΠΆΠ΅Π½ΠΈΠ΅, ΠΈ Π΄Π΅Π»Π΅Π½ΠΈΠ΅ стСпСнСй

    Π Π°Π·Π΄Π΅Π»Ρ‹: ΠœΠ°Ρ‚Π΅ΠΌΠ°Ρ‚ΠΈΠΊΠ°

    Π’ΠΈΠΏ ΡƒΡ€ΠΎΠΊΠ°: ΡƒΡ€ΠΎΠΊ обобщСния ΠΈ систСматизации Π·Π½Π°Π½ΠΈΠΉ

    Π¦Π΅Π»ΠΈ:

  • ΠΎΠ±ΡƒΡ‡Π°ΡŽΡ‰ΠΈΠ΅ – ΠΏΠΎΠ²Ρ‚ΠΎΡ€ΠΈΡ‚ΡŒ ΠΎΠΏΡ€Π΅Π΄Π΅Π»Π΅Π½ΠΈΠ΅ стСпСни, ΠΏΡ€Π°Π²ΠΈΠ»Π° умноТСния ΠΈ дСлСния стСпСнСй, возвСдСния стСпСни Π² ΡΡ‚Π΅ΠΏΠ΅Π½ΡŒ, Π·Π°ΠΊΡ€Π΅ΠΏΠΈΡ‚ΡŒ умСния Ρ€Π΅ΡˆΠ΅Π½ΠΈΡ ΠΏΡ€ΠΈΠΌΠ΅Ρ€ΠΎΠ², содСрТащих стСпСни,
  • Ρ€Π°Π·Π²ΠΈΠ²Π°ΡŽΡ‰ΠΈΠ΅ – Ρ€Π°Π·Π²ΠΈΡ‚ΠΈΠ΅ логичСского ΠΌΡ‹ΡˆΠ»Π΅Π½ΠΈΡ учащихся, интСрСса ΠΊ ΠΈΠ·ΡƒΡ‡Π°Π΅ΠΌΠΎΠΌΡƒ ΠΌΠ°Ρ‚Π΅Ρ€ΠΈΠ°Π»Ρƒ,
  • Π²ΠΎΡΠΏΠΈΡ‚Ρ‹Π²Π°ΡŽΡ‰ΠΈΠ΅ – воспитаниС отвСтствСнного ΠΎΡ‚Π½ΠΎΡˆΠ΅Π½ΠΈΡ ΠΊ ΡƒΡ‡Π΅Π±Π΅, ΠΊΡƒΠ»ΡŒΡ‚ΡƒΡ€Ρ‹ общСния, чувства ΠΊΠΎΠ»Π»Π΅ΠΊΡ‚ΠΈΠ²ΠΈΠ·ΠΌΠ°.
  • ΠžΠ±ΠΎΡ€ΡƒΠ΄ΠΎΠ²Π°Π½ΠΈΠ΅: ΠΊΠΎΠΌΠΏΡŒΡŽΡ‚Π΅Ρ€, ΠΌΡƒΠ»ΡŒΡ‚ΠΈΠΌΠ΅Π΄ΠΈΠΉΠ½Ρ‹ΠΉ ΠΏΡ€ΠΎΠ΅ΠΊΡ‚ΠΎΡ€, интСрактивная доска, прСзСнтация β€œΠ‘Ρ‚Π΅ΠΏΠ΅Π½ΠΈβ€ для устного счСта, ΠΊΠ°Ρ€Ρ‚ΠΎΡ‡ΠΊΠΈ с заданиями, Ρ€Π°Π·Π΄Π°Ρ‚ΠΎΡ‡Π½Ρ‹ΠΉ ΠΌΠ°Ρ‚Π΅Ρ€ΠΈΠ°Π».

    План ΡƒΡ€ΠΎΠΊΠ°:

  • ΠžΡ€Π³Π°Π½ΠΈΠ·Π°Ρ†ΠΈΠΎΠ½Π½Ρ‹ΠΉ ΠΌΠΎΠΌΠ΅Π½Ρ‚.
  • ΠŸΠΎΠ²Ρ‚ΠΎΡ€Π΅Π½ΠΈΠ΅ ΠΏΡ€Π°Π²ΠΈΠ»
  • Устный счСт.
  • Π˜ΡΡ‚ΠΎΡ€ΠΈΡ‡Π΅ΡΠΊΠ°Ρ справка.
  • Π Π°Π±ΠΎΡ‚Π° Ρƒ доски.
  • Π€ΠΈΠ·ΠΊΡƒΠ»ΡŒΡ‚ΠΌΠΈΠ½ΡƒΡ‚ΠΊΠ°.
  • Π Π°Π±ΠΎΡ‚Π° Π½Π° ΠΈΠ½Ρ‚Π΅Ρ€Π°ΠΊΡ‚ΠΈΠ²Π½ΠΎΠΉ доскС.
  • Π‘Π°ΠΌΠΎΡΡ‚ΠΎΡΡ‚Π΅Π»ΡŒΠ½Π°Ρ Ρ€Π°Π±ΠΎΡ‚Π°.
  • Π”ΠΎΠΌΠ°ΡˆΠ½Π΅Π΅ Π·Π°Π΄Π°Π½ΠΈΠ΅.
  • ПодвСдСниС ΠΈΡ‚ΠΎΠ³ΠΎΠ² ΡƒΡ€ΠΎΠΊΠ°.
  • Π₯ΠΎΠ΄ ΡƒΡ€ΠΎΠΊΠ°

    I. ΠžΡ€Π³Π°Π½ΠΈΠ·Π°Ρ†ΠΈΠΎΠ½Π½Ρ‹ΠΉ ΠΌΠΎΠΌΠ΅Π½Ρ‚

    Π‘ΠΎΠΎΠ±Ρ‰Π΅Π½ΠΈΠ΅ Ρ‚Π΅ΠΌΡ‹ ΠΈ Ρ†Π΅Π»Π΅ΠΉ ΡƒΡ€ΠΎΠΊΠ°.

    На ΠΏΡ€Π΅Π΄Ρ‹Π΄ΡƒΡ‰ΠΈΡ… ΡƒΡ€ΠΎΠΊΠ°Ρ… Π²Ρ‹ ΠΎΡ‚ΠΊΡ€Ρ‹Π»ΠΈ для сСбя ΡƒΠ΄ΠΈΠ²ΠΈΡ‚Π΅Π»ΡŒΠ½Ρ‹ΠΉ ΠΌΠΈΡ€ стСпСнСй, Π½Π°ΡƒΡ‡ΠΈΠ»ΠΈΡΡŒ ΡƒΠΌΠ½ΠΎΠΆΠ°Ρ‚ΡŒ ΠΈ Π΄Π΅Π»ΠΈΡ‚ΡŒ стСпСни, Π²ΠΎΠ·Π²ΠΎΠ΄ΠΈΡ‚ΡŒ ΠΈΡ… Π² ΡΡ‚Π΅ΠΏΠ΅Π½ΡŒ. БСгодня ΠΌΡ‹ Π΄ΠΎΠ»ΠΆΠ½Ρ‹ Π·Π°ΠΊΡ€Π΅ΠΏΠΈΡ‚ΡŒ ΠΏΠΎΠ»ΡƒΡ‡Π΅Π½Π½Ρ‹Π΅ знания ΠΏΡ€ΠΈ Ρ€Π΅ΡˆΠ΅Π½ΠΈΠΈ ΠΏΡ€ΠΈΠΌΠ΅Ρ€ΠΎΠ².

    II. ΠŸΠΎΠ²Ρ‚ΠΎΡ€Π΅Π½ΠΈΠ΅ ΠΏΡ€Π°Π²ΠΈΠ» (устно)

    1. Π”Π°ΠΉΡ‚Π΅ ΠΎΠΏΡ€Π΅Π΄Π΅Π»Π΅Π½ΠΈΠ΅ стСпСни с Π½Π°Ρ‚ΡƒΡ€Π°Π»ΡŒΠ½Ρ‹ΠΌ ΠΏΠΎΠΊΠ°Π·Π°Ρ‚Π΅Π»Π΅ΠΌ? (Π‘Ρ‚Π΅ΠΏΠ΅Π½ΡŒΡŽ числа Π° с Π½Π°Ρ‚ΡƒΡ€Π°Π»ΡŒΠ½Ρ‹ΠΌ ΠΏΠΎΠΊΠ°Π·Π°Ρ‚Π΅Π»Π΅ΠΌ, большим 1, называСтся ΠΏΡ€ΠΎΠΈΠ·Π²Π΅Π΄Π΅Π½ΠΈΠ΅ n ΠΌΠ½ΠΎΠΆΠΈΡ‚Π΅Π»Π΅ΠΉ, ΠΊΠ°ΠΆΠ΄Ρ‹ΠΉ ΠΈΠ· ΠΊΠΎΡ‚ΠΎΡ€Ρ‹Ρ… Ρ€Π°Π²Π΅Π½ Π° .)
    2. Как ΡƒΠΌΠ½ΠΎΠΆΠΈΡ‚ΡŒ Π΄Π²Π΅ стСпСни? (Π§Ρ‚ΠΎΠ±Ρ‹ ΡƒΠΌΠ½ΠΎΠΆΠΈΡ‚ΡŒ стСпСни с ΠΎΠ΄ΠΈΠ½Π°ΠΊΠΎΠ²Ρ‹ΠΌΠΈ основаниями, Π½Π°Π΄ΠΎ основаниС ΠΎΡΡ‚Π°Π²ΠΈΡ‚ΡŒ Ρ‚Π΅ΠΌ ΠΆΠ΅, Π° ΠΏΠΎΠΊΠ°Π·Π°Ρ‚Π΅Π»ΠΈ ΡΠ»ΠΎΠΆΠΈΡ‚ΡŒ.)
    3. Как Ρ€Π°Π·Π΄Π΅Π»ΠΈΡ‚ΡŒ ΡΡ‚Π΅ΠΏΠ΅Π½ΡŒ Π½Π° ΡΡ‚Π΅ΠΏΠ΅Π½ΡŒ? (Π§Ρ‚ΠΎΠ±Ρ‹ Ρ€Π°Π·Π΄Π΅Π»ΠΈΡ‚ΡŒ стСпСни с ΠΎΠ΄ΠΈΠ½Π°ΠΊΠΎΠ²Ρ‹ΠΌΠΈ основаниями, Π½Π°Π΄ΠΎ основаниС ΠΎΡΡ‚Π°Π²ΠΈΡ‚ΡŒ Ρ‚Π΅ΠΌ ΠΆΠ΅, Π° ΠΏΠΎΠΊΠ°Π·Π°Ρ‚Π΅Π»ΠΈ Π²Ρ‹Ρ‡Π΅ΡΡ‚ΡŒ. )
    4. Как возвСсти ΠΏΡ€ΠΎΠΈΠ·Π²Π΅Π΄Π΅Π½ΠΈΠ΅ Π² ΡΡ‚Π΅ΠΏΠ΅Π½ΡŒ? (Π§Ρ‚ΠΎΠ±Ρ‹ возвСсти ΠΏΡ€ΠΎΠΈΠ·Π²Π΅Π΄Π΅Π½ΠΈΠ΅ Π² ΡΡ‚Π΅ΠΏΠ΅Π½ΡŒ, Π½Π°Π΄ΠΎ ΠΊΠ°ΠΆΠ΄Ρ‹ΠΉ ΠΌΠ½ΠΎΠΆΠΈΡ‚Π΅Π»ΡŒ возвСсти Π² эту ΡΡ‚Π΅ΠΏΠ΅Π½ΡŒ)
    5. Как возвСсти ΡΡ‚Π΅ΠΏΠ΅Π½ΡŒ Π² ΡΡ‚Π΅ΠΏΠ΅Π½ΡŒ? (Π§Ρ‚ΠΎΠ±Ρ‹ возвСсти ΡΡ‚Π΅ΠΏΠ΅Π½ΡŒ Π² ΡΡ‚Π΅ΠΏΠ΅Π½ΡŒ, Π½Π°Π΄ΠΎ основаниС ΠΎΡΡ‚Π°Π²ΠΈΡ‚ΡŒ Ρ‚Π΅ΠΌ ΠΆΠ΅, Π° ΠΏΠΎΠΊΠ°Π·Π°Ρ‚Π΅Π»ΠΈ ΠΏΠ΅Ρ€Π΅ΠΌΠ½ΠΎΠΆΠΈΡ‚ΡŒ)
    6. III. Устный счСт (ΠΏΠΎ ΠΌΡƒΠ»ΡŒΡ‚ΠΈΠΌΠ΅Π΄ΠΈΠ°)

      IV. Π˜ΡΡ‚ΠΎΡ€ΠΈΡ‡Π΅ΡΠΊΠ°Ρ справка

      ВсС Π·Π°Π΄Π°Ρ‡ΠΈ ΠΈΠ· папируса АхмСса, ΠΊΠΎΡ‚ΠΎΡ€Ρ‹ΠΉ записан ΠΎΠΊΠΎΠ»ΠΎ 1650 Π³ΠΎΠ΄Π° Π΄ΠΎ Π½. э. связаны с ΠΏΡ€Π°ΠΊΡ‚ΠΈΠΊΠΎΠΉ ΡΡ‚Ρ€ΠΎΠΈΡ‚Π΅Π»ΡŒΡΡ‚Π²Π°, Ρ€Π°Π·ΠΌΠ΅ΠΆΠ΅Π²Π°Π½ΠΈΠ΅ΠΌ Π·Π΅ΠΌΠ΅Π»ΡŒΠ½Ρ‹Ρ… Π½Π°Π΄Π΅Π»ΠΎΠ² ΠΈ Ρ‚. ΠΏ. Π—Π°Π΄Π°Ρ‡ΠΈ сгруппированы ΠΏΠΎ Ρ‚Π΅ΠΌΠ°Ρ‚ΠΈΠΊΠ΅. По прСимущСству это Π·Π°Π΄Π°Ρ‡ΠΈ Π½Π° Π½Π°Ρ…ΠΎΠΆΠ΄Π΅Π½ΠΈΠ΅ ΠΏΠ»ΠΎΡ‰Π°Π΄Π΅ΠΉ Ρ‚Ρ€Π΅ΡƒΠ³ΠΎΠ»ΡŒΠ½ΠΈΠΊΠ°, Ρ‡Π΅Ρ‚Ρ‹Ρ€Ρ‘Ρ…ΡƒΠ³ΠΎΠ»ΡŒΠ½ΠΈΠΊΠΎΠ² ΠΈ ΠΊΡ€ΡƒΠ³Π°, Ρ€Π°Π·Π½ΠΎΠΎΠ±Ρ€Π°Π·Π½Ρ‹Π΅ дСйствия с Ρ†Π΅Π»Ρ‹ΠΌΠΈ числами ΠΈ дробями, ΠΏΡ€ΠΎΠΏΠΎΡ€Ρ†ΠΈΠΎΠ½Π°Π»ΡŒΠ½ΠΎΠ΅ Π΄Π΅Π»Π΅Π½ΠΈΠ΅, Π½Π°Ρ…ΠΎΠΆΠ΄Π΅Π½ΠΈΠ΅ ΠΎΡ‚Π½ΠΎΡˆΠ΅Π½ΠΈΠΉ, здСсь присутствуСт ΠΈ Π²ΠΎΠ·Π²Π΅Π΄Π΅Π½ΠΈΠ΅ Π² Ρ€Π°Π·Π½Ρ‹Π΅ стСпСни, Ρ€Π΅ΡˆΠ΅Π½ΠΈΠ΅ ΡƒΡ€Π°Π²Π½Π΅Π½ΠΈΠΉ ΠΏΠ΅Ρ€Π²ΠΎΠΉ ΠΈ Π²Ρ‚ΠΎΡ€ΠΎΠΉ стСпСни с ΠΎΠ΄Π½ΠΈΠΌ нСизвСстным.

      ΠŸΠΎΠ»Π½ΠΎΡΡ‚ΡŒΡŽ ΠΎΡ‚ΡΡƒΡ‚ΡΡ‚Π²ΡƒΡŽΡ‚ ΠΊΠ°ΠΊΠΈΠ΅ Π±Ρ‹ Ρ‚ΠΎ Π½ΠΈ Π±Ρ‹Π»ΠΎ объяснСния ΠΈΠ»ΠΈ Π΄ΠΎΠΊΠ°Π·Π°Ρ‚Π΅Π»ΡŒΡΡ‚Π²Π°. Π˜ΡΠΊΠΎΠΌΡ‹ΠΉ Ρ€Π΅Π·ΡƒΠ»ΡŒΡ‚Π°Ρ‚ Π»ΠΈΠ±ΠΎ даётся прямо, Π»ΠΈΠ±ΠΎ приводится ΠΊΡ€Π°Ρ‚ΠΊΠΈΠΉ Π°Π»Π³ΠΎΡ€ΠΈΡ‚ΠΌ Π΅Π³ΠΎ вычислСния. Π’Π°ΠΊΠΎΠΉ способ излоТСния, Ρ‚ΠΈΠΏΠΈΡ‡Π½Ρ‹ΠΉ для Π½Π°ΡƒΠΊΠΈ стран Π΄Ρ€Π΅Π²Π½Π΅Π³ΠΎ Востока, Π½Π°Π²ΠΎΠ΄ΠΈΡ‚ Π½Π° ΠΌΡ‹ΡΠ»ΡŒ ΠΎ Ρ‚ΠΎΠΌ, Ρ‡Ρ‚ΠΎ ΠΌΠ°Ρ‚Π΅ΠΌΠ°Ρ‚ΠΈΠΊΠ° Ρ‚Π°ΠΌ Ρ€Π°Π·Π²ΠΈΠ²Π°Π»Π°ΡΡŒ ΠΏΡƒΡ‚Ρ‘ΠΌ ΠΎΠ±ΠΎΠ±Ρ‰Π΅Π½ΠΈΠΉ ΠΈ Π΄ΠΎΠ³Π°Π΄ΠΎΠΊ, Π½Π΅ ΠΎΠ±Ρ€Π°Π·ΡƒΡŽΡ‰ΠΈΡ… Π½ΠΈΠΊΠ°ΠΊΠΎΠΉ ΠΎΠ±Ρ‰Π΅ΠΉ Ρ‚Π΅ΠΎΡ€ΠΈΠΈ. Π’Π΅ΠΌ Π½Π΅ ΠΌΠ΅Π½Π΅Π΅, Π² папирусС Π΅ΡΡ‚ΡŒ Ρ†Π΅Π»Ρ‹ΠΉ ряд ΡΠ²ΠΈΠ΄Π΅Ρ‚Π΅Π»ΡŒΡΡ‚Π² Ρ‚ΠΎΠ³ΠΎ, Ρ‡Ρ‚ΠΎ СгипСтскиС ΠΌΠ°Ρ‚Π΅ΠΌΠ°Ρ‚ΠΈΠΊΠΈ ΡƒΠΌΠ΅Π»ΠΈ ΠΈΠ·Π²Π»Π΅ΠΊΠ°Ρ‚ΡŒ ΠΊΠΎΡ€Π½ΠΈ ΠΈ Π²ΠΎΠ·Π²ΠΎΠ΄ΠΈΡ‚ΡŒ Π² ΡΡ‚Π΅ΠΏΠ΅Π½ΡŒ, Ρ€Π΅ΡˆΠ°Ρ‚ΡŒ уравнСния, ΠΈ Π΄Π°ΠΆΠ΅ Π²Π»Π°Π΄Π΅Π»ΠΈ Π·Π°Ρ‡Π°Ρ‚ΠΊΠ°ΠΌΠΈ Π°Π»Π³Π΅Π±Ρ€Ρ‹.

      V. Π Π°Π±ΠΎΡ‚Π° Ρƒ доски

      НайдитС Π·Π½Π°Ρ‡Π΅Π½ΠΈΠ΅ выраТСния Ρ€Π°Ρ†ΠΈΠΎΠ½Π°Π»ΡŒΠ½Ρ‹ΠΌ способом:

      ВычислитС Π·Π½Π°Ρ‡Π΅Π½ΠΈΠ΅ выраТСния:

      VI. Π€ΠΈΠ·ΠΊΡƒΠ»ΡŒΡ‚ΠΌΠΈΠ½ΡƒΡ‚ΠΊΠ°

    7. для глаз
    8. для шСи
    9. для Ρ€ΡƒΠΊ
    10. для Ρ‚ΡƒΠ»ΠΎΠ²ΠΈΡ‰Π°
    11. для ног
    12. VII. РСшСниС Π·Π°Π΄Π°Ρ‡ (с ΠΏΠΎΠΊΠ°Π·ΠΎΠΌ Π½Π° ΠΈΠ½Ρ‚Π΅Ρ€Π°ΠΊΡ‚ΠΈΠ²Π½ΠΎΠΉ доскС)

      ЯвляСтся Π»ΠΈ ΠΊΠΎΡ€Π΅Π½ΡŒ уравнСния ΠΏΠΎΠ»ΠΎΠΆΠΈΡ‚Π΅Π»ΡŒΠ½Ρ‹ΠΌ числом?

      xn--i1abbnckbmcl9fb.xn--p1ai

      Π€ΠΎΡ€ΠΌΡƒΠ»Ρ‹ стСпСнСй ΠΈ ΠΊΠΎΡ€Π½Π΅ΠΉ.

      Π€ΠΎΡ€ΠΌΡƒΠ»Ρ‹ стСпСнСй ΠΈΡΠΏΠΎΠ»ΡŒΠ·ΡƒΡŽΡ‚ Π² процСссС сокращСния ΠΈ упрощСния слоТных Π²Ρ‹Ρ€Π°ΠΆΠ΅Π½ΠΈΠΉ, Π² Ρ€Π΅ΡˆΠ΅Π½ΠΈΠΈ ΡƒΡ€Π°Π²Π½Π΅Π½ΠΈΠΉ ΠΈ нСравСнств.

      Число c являСтся n -Π½ΠΎΠΉ ΡΡ‚Π΅ΠΏΠ΅Π½ΡŒΡŽ числа a ΠΊΠΎΠ³Π΄Π°:

      ΠžΠΏΠ΅Ρ€Π°Ρ†ΠΈΠΈ со стСпСнями.

      1. УмноТая стСпСни с ΠΎΠ΄ΠΈΠ½Π°ΠΊΠΎΠ²Ρ‹ΠΌ основаниСм ΠΈΡ… ΠΏΠΎΠΊΠ°Π·Π°Ρ‚Π΅Π»ΠΈ ΡΠΊΠ»Π°Π΄Ρ‹Π²Π°ΡŽΡ‚ΡΡ:

      2. Π’ Π΄Π΅Π»Π΅Π½ΠΈΠΈ стСпСнСй с ΠΎΠ΄ΠΈΠ½Π°ΠΊΠΎΠ²Ρ‹ΠΌ основаниСм ΠΈΡ… ΠΏΠΎΠΊΠ°Π·Π°Ρ‚Π΅Π»ΠΈ Π²Ρ‹Ρ‡ΠΈΡ‚Π°ΡŽΡ‚ΡΡ:

      3. Π‘Ρ‚Π΅ΠΏΠ΅Π½ΡŒ произвСдСния 2-Ρ… Π»ΠΈΠ±ΠΎ большСго числа ΠΌΠ½ΠΎΠΆΠΈΡ‚Π΅Π»Π΅ΠΉ равняСтся ΠΏΡ€ΠΎΠΈΠ·Π²Π΅Π΄Π΅Π½ΠΈΡŽ стСпСнСй этих сомноТитСлСй:

      (abc…) n = a n Β· b n Β· c n …

      4. Π‘Ρ‚Π΅ΠΏΠ΅Π½ΡŒ Π΄Ρ€ΠΎΠ±ΠΈ равняСтся ΠΎΡ‚Π½ΠΎΡˆΠ΅Π½ΠΈΡŽ стСпСнСй Π΄Π΅Π»ΠΈΠΌΠΎΠ³ΠΎ ΠΈ дСлитСля:

      5. Возводя ΡΡ‚Π΅ΠΏΠ΅Π½ΡŒ Π² ΡΡ‚Π΅ΠΏΠ΅Π½ΡŒ, ΠΏΠΎΠΊΠ°Π·Π°Ρ‚Π΅Π»ΠΈ стСпСнСй ΠΏΠ΅Ρ€Π΅ΠΌΠ½ΠΎΠΆΠ°ΡŽΡ‚:

      КаТдая Π²Ρ‹ΡˆΠ΅ΠΏΡ€ΠΈΠ²Π΅Π΄Π΅Π½Π½Π°Ρ Ρ„ΠΎΡ€ΠΌΡƒΠ»Π° Π²Π΅Ρ€Π½Π° Π² направлСниях слСва Π½Π°ΠΏΡ€Π°Π²ΠΎ ΠΈ Π½Π°ΠΎΠ±ΠΎΡ€ΠΎΡ‚.

      ΠžΠΏΠ΅Ρ€Π°Ρ†ΠΈΠΈ с корнями.

      1. ΠšΠΎΡ€Π΅Π½ΡŒ ΠΈΠ· произвСдСния Π½Π΅ΡΠΊΠΎΠ»ΡŒΠΊΠΈΡ… сомноТитСлСй равняСтся ΠΏΡ€ΠΎΠΈΠ·Π²Π΅Π΄Π΅Π½ΠΈΡŽ ΠΊΠΎΡ€Π½Π΅ΠΉ ΠΈΠ· этих сомноТитСлСй:

      2. ΠšΠΎΡ€Π΅Π½ΡŒ ΠΈΠ· ΠΎΡ‚Π½ΠΎΡˆΠ΅Π½ΠΈΡ Ρ€Π°Π²Π΅Π½ ΠΎΡ‚Π½ΠΎΡˆΠ΅Π½ΠΈΡŽ Π΄Π΅Π»ΠΈΠΌΠΎΠ³ΠΎ ΠΈ дСлитСля ΠΊΠΎΡ€Π½Π΅ΠΉ:

      3. ΠŸΡ€ΠΈ Π²ΠΎΠ·Π²Π΅Π΄Π΅Π½ΠΈΠΈ корня Π² ΡΡ‚Π΅ΠΏΠ΅Π½ΡŒ довольно возвСсти Π² эту ΡΡ‚Π΅ΠΏΠ΅Π½ΡŒ ΠΏΠΎΠ΄ΠΊΠΎΡ€Π΅Π½Π½ΠΎΠ΅ число:

      4. Если ΡƒΠ²Π΅Π»ΠΈΡ‡ΠΈΡ‚ΡŒ ΡΡ‚Π΅ΠΏΠ΅Π½ΡŒ корня Π² n Ρ€Π°Π· ΠΈ Π² Ρ‚ΠΎΠΆΠ΅ врСмя возвСсти Π² n -ΡƒΡŽ ΡΡ‚Π΅ΠΏΠ΅Π½ΡŒ ΠΏΠΎΠ΄ΠΊΠΎΡ€Π΅Π½Π½ΠΎΠ΅ число, Ρ‚ΠΎ Π·Π½Π°Ρ‡Π΅Π½ΠΈΠ΅ корня Π½Π΅ помСняСтся:

      5. Если ΡƒΠΌΠ΅Π½ΡŒΡˆΠΈΡ‚ΡŒ ΡΡ‚Π΅ΠΏΠ΅Π½ΡŒ корня Π² n Ρ€Π°Π· ΠΈ Π² Ρ‚ΠΎΠΆΠ΅ врСмя ΠΈΠ·Π²Π»Π΅Ρ‡ΡŒ ΠΊΠΎΡ€Π΅Π½ΡŒ n -ΠΎΠΉ стСпСни ΠΈΠ· ΠΏΠΎΠ΄ΠΊΠΎΡ€Π΅Π½Π½ΠΎΠ³ΠΎ числа, Ρ‚ΠΎ Π·Π½Π°Ρ‡Π΅Π½ΠΈΠ΅ корня Π½Π΅ помСняСтся:

      Π‘Ρ‚Π΅ΠΏΠ΅Π½ΡŒ Π½Π΅ΠΊΠΎΡ‚ΠΎΡ€ΠΎΠ³ΠΎ числа с Π½Π΅ΠΏΠΎΠ»ΠΎΠΆΠΈΡ‚Π΅Π»ΡŒΠ½Ρ‹ΠΌ (Ρ†Π΅Π»Ρ‹ΠΌ) ΠΏΠΎΠΊΠ°Π·Π°Ρ‚Π΅Π»Π΅ΠΌ ΠΎΠΏΡ€Π΅Π΄Π΅Π»ΡΡŽΡ‚ ΠΊΠ°ΠΊ Π΅Π΄ΠΈΠ½ΠΈΡ†Ρƒ, Π΄Π΅Π»Π΅Π½Π½ΡƒΡŽ Π½Π° ΡΡ‚Π΅ΠΏΠ΅Π½ΡŒ Ρ‚ΠΎΠ³ΠΎ ΠΆΠ΅ числа с ΠΏΠΎΠΊΠ°Π·Π°Ρ‚Π΅Π»Π΅ΠΌ, Ρ€Π°Π²Π½Ρ‹ΠΌ Π°Π±ΡΠΎΠ»ΡŽΡ‚Π½ΠΎΠΉ Π²Π΅Π»ΠΈΡ‡ΠΈΠ½Π΅ Π½Π΅ΠΏΠΎΠ»ΠΎΠΆΠΈΡ‚Π΅Π»ΡŒΠ½ΠΎΠ³ΠΎ показатСля:

      Π€ΠΎΡ€ΠΌΡƒΠ»Ρƒ a m :a n =a m β€” n ΠΌΠΎΠΆΠ½ΠΎ ΠΈΡΠΏΠΎΠ»ΡŒΠ·ΠΎΠ²Π°Ρ‚ΡŒ Π½Π΅ Ρ‚ΠΎΠ»ΡŒΠΊΠΎ ΠΏΡ€ΠΈ m > n , Π½ΠΎ ΠΈ ΠΏΡ€ΠΈ m 4:a 7 = a 4 β€” 7 = a -3 .

      Π§Ρ‚ΠΎΠ±Ρ‹ Ρ„ΠΎΡ€ΠΌΡƒΠ»Π° a m :a n =a m β€” n стала справСдливой ΠΏΡ€ΠΈ m=n , Π½ΡƒΠΆΠ½ΠΎ присутствиС Π½ΡƒΠ»Π΅Π²ΠΎΠΉ стСпСни.

      Π‘Ρ‚Π΅ΠΏΠ΅Π½ΡŒ всякого числа, Π½Π΅ Ρ€Π°Π²Π½ΠΎΠ³ΠΎ Π½ΡƒΠ»ΡŽ, с Π½ΡƒΠ»Π΅Π²Ρ‹ΠΌ ΠΏΠΎΠΊΠ°Π·Π°Ρ‚Π΅Π»Π΅ΠΌ равняСтся Π΅Π΄ΠΈΠ½ΠΈΡ†Π΅.

      Π§Ρ‚ΠΎΠ±Ρ‹ возвСсти Π΄Π΅ΠΉΡΡ‚Π²ΠΈΡ‚Π΅Π»ΡŒΠ½ΠΎΠ΅ число Π° Π² ΡΡ‚Π΅ΠΏΠ΅Π½ΡŒ m/n , Π½Π΅ΠΎΠ±Ρ…ΠΎΠ΄ΠΈΠΌΠΎ ΠΈΠ·Π²Π»Π΅Ρ‡ΡŒ ΠΊΠΎΡ€Π΅Π½ΡŒ n –ой стСпСни ΠΈΠ· m -ΠΎΠΉ стСпСни этого числа Π° :

      Π€ΠΎΡ€ΠΌΡƒΠ»Ρ‹ стСпСнСй.

      6. a β€” n = β€” Π΄Π΅Π»Π΅Π½ΠΈΠ΅ стСпСнСй;

      7. β€” Π΄Π΅Π»Π΅Π½ΠΈΠ΅ стСпСнСй;

      8. a 1/n = ;

      Π‘Ρ‚Π΅ΠΏΠ΅Π½ΠΈ ΠΏΡ€Π°Π²ΠΈΠ»Π° дСйствия со стСпСнями

      1. Π‘Ρ‚Π΅ΠΏΠ΅Π½ΡŒ произвСдСния Π΄Π²ΡƒΡ… ΠΈΠ»ΠΈ Π½Π΅ΡΠΊΠΎΠ»ΡŒΠΊΠΈΡ… сомноТитСлСй Ρ€Π°Π²Π½Π° ΠΏΡ€ΠΎΠΈΠ·Π²Π΅Π΄Π΅Π½ΠΈΡŽ стСпСнСй этих сомноТитСлСй (с Ρ‚Π΅ΠΌ ΠΆΠ΅ ΠΏΠΎΠΊΠ°Π·Π°Ρ‚Π΅Π»Π΅ΠΌ):

      (abc…) n = a n b n c n …

      ΠŸΡ€ΠΈΠΌΠ΅Ρ€ 1. (7 2 10) 2 = 7 2 2 2 10 2 = 49 4 100 = 19600. ΠŸΡ€ΠΈΠΌΠ΅Ρ€ 2. (x 2 –a 2) 3 = [(x +a)(x β€” a)] 3 =(x +a) 3 (x β€” a) 3

      ΠŸΡ€Π°ΠΊΡ‚ΠΈΡ‡Π΅ΡΠΊΠΈ Π±ΠΎΠ»Π΅Π΅ Π²Π°ΠΆΠ½ΠΎ ΠΎΠ±Ρ€Π°Ρ‚Π½ΠΎΠ΅ ΠΏΡ€Π΅ΠΎΠ±Ρ€Π°Π·ΠΎΠ²Π°Π½ΠΈΠ΅:

      a n b n c n … = (abc…) n

      Ρ‚.Π΅. ΠΏΡ€ΠΎΠΈΠ·Π²Π΅Π΄Π΅Π½ΠΈΠ΅ ΠΎΠ΄ΠΈΠ½Π°ΠΊΠΎΠ²Ρ‹Ρ… стСпСнСй Π½Π΅ΡΠΊΠΎΠ»ΡŒΠΊΠΈΡ… Π²Π΅Π»ΠΈΡ‡ΠΈΠ½ Ρ€Π°Π²Π½ΠΎ Ρ‚ΠΎΠΉ ΠΆΠ΅ стСпСни произвСдСния этих Π²Π΅Π»ΠΈΡ‡ΠΈΠ½.

      ΠŸΡ€ΠΈΠΌΠ΅Ρ€ 3. ΠŸΡ€ΠΈΠΌΠ΅Ρ€ 4. (a +b) 2 (a 2 – ab +b 2) 2 =[(a +b)(a 2 – ab +b 2)] 2 =(a 3 +b 3) 2

      2. Π‘Ρ‚Π΅ΠΏΠ΅Π½ΡŒ частного (Π΄Ρ€ΠΎΠ±ΠΈ) Ρ€Π°Π²Π½Π° частному ΠΎΡ‚ дСлСния Ρ‚ΠΎΠΉ ΠΆΠ΅ стСпСни Π΄Π΅Π»ΠΈΠΌΠΎΠ³ΠΎ Π½Π° Ρ‚Ρƒ ΠΆΠ΅ ΡΡ‚Π΅ΠΏΠ΅Π½ΡŒ дСлитСля:

      ΠŸΡ€ΠΈΠΌΠ΅Ρ€ 5. ΠŸΡ€ΠΈΠΌΠ΅Ρ€ 6.

      ΠžΠ±Ρ€Π°Ρ‚Π½ΠΎΠ΅ ΠΏΡ€Π΅ΠΎΠ±Ρ€Π°Π·ΠΎΠ²Π°Π½ΠΈΠ΅:. ΠŸΡ€ΠΈΠΌΠ΅Ρ€ 7.. ΠŸΡ€ΠΈΠΌΠ΅Ρ€ 8..

      3. ΠŸΡ€ΠΈ ΡƒΠΌΠ½ΠΎΠΆΠ΅Π½ΠΈΠΈ стСпСнСй с ΠΎΠ΄ΠΈΠ½Π°ΠΊΠΎΠ²Ρ‹ΠΌΠΈ основаниями ΠΏΠΎΠΊΠ°Π·Π°Ρ‚Π΅Π»ΠΈ стСпСнСй ΡΠΊΠ»Π°Π΄Ρ‹Π²Π°ΡŽΡ‚ΡΡ:

      ΠŸΡ€ΠΈΠΌΠ΅Ρ€ 9. 2 2 2 5 =2 2+5 =2 7 =128. ΠŸΡ€ΠΈΠΌΠ΅Ρ€ 10. (a – 4c +x) 2 (a – 4c +x) 3 =(a – 4c + x) 5 .

      4. ΠŸΡ€ΠΈ Π΄Π΅Π»Π΅Π½ΠΈΠΈ стСпСнСй с ΠΎΠ΄ΠΈΠ½Π°ΠΊΠΎΠ²Ρ‹ΠΌΠΈ основаниями ΠΏΠΎΠΊΠ°Π·Π°Ρ‚Π΅Π»ΡŒ стСпСни дСлитСля вычитаСтся ΠΈΠ· показатСля стСпСни Π΄Π΅Π»ΠΈΠΌΠΎΠ³ΠΎ

      ΠŸΡ€ΠΈΠΌΠ΅Ρ€ 11. 12 5:12 3 =12 5-3 =12 2 =144. ΠŸΡ€ΠΈΠΌΠ΅Ρ€ 12. (x-y) 3:(x-y) 2 =x-y.

      5. ΠŸΡ€ΠΈ Π²ΠΎΠ·Π²Π΅Π΄Π΅Π½ΠΈΠΈ стСпСни Π² ΡΡ‚Π΅ΠΏΠ΅Π½ΡŒ ΠΏΠΎΠΊΠ°Π·Π°Ρ‚Π΅Π»ΠΈ стСпСнСй ΠΏΠ΅Ρ€Π΅ΠΌΠ½ΠΎΠΆΠ°ΡŽΡ‚ΡΡ:

      ΠŸΡ€ΠΈΠΌΠ΅Ρ€ 13. (2 3) 2 =2 6 =64. ΠŸΡ€ΠΈΠΌΠ΅Ρ€ 14.

      www.maths.yfa1.ru

      Π‘Ρ‚Π΅ΠΏΠ΅Π½ΠΈ ΠΈ ΠΊΠΎΡ€Π½ΠΈ

      ΠžΠΏΠ΅Ρ€Π°Ρ†ΠΈΠΈ со стСпСнями ΠΈ корнями. Π‘Ρ‚Π΅ΠΏΠ΅Π½ΡŒ с ΠΎΡ‚Ρ€ΠΈΡ†Π°Ρ‚Π΅Π»ΡŒΠ½Ρ‹ΠΌ ,

      Π½ΡƒΠ»Π΅Π²Ρ‹ΠΌ ΠΈ Π΄Ρ€ΠΎΠ±Π½Ρ‹ΠΌ ΠΏΠΎΠΊΠ°Π·Π°Ρ‚Π΅Π»Π΅ΠΌ. О выраТСниях, Π½Π΅ ΠΈΠΌΠ΅ΡŽΡ‰ΠΈΡ… смысла.

      ΠžΠΏΠ΅Ρ€Π°Ρ†ΠΈΠΈ со стСпСнями.

      1. ΠŸΡ€ΠΈ ΡƒΠΌΠ½ΠΎΠΆΠ΅Π½ΠΈΠΈ стСпСнСй с ΠΎΠ΄ΠΈΠ½Π°ΠΊΠΎΠ²Ρ‹ΠΌ основаниСм ΠΈΡ… ΠΏΠΎΠΊΠ°Π·Π°Ρ‚Π΅Π»ΠΈ ΡΠΊΠ»Π°Π΄Ρ‹Π²Π°ΡŽΡ‚ΡΡ:

      a m Β· a n = a m + n .

      2. ΠŸΡ€ΠΈ Π΄Π΅Π»Π΅Π½ΠΈΠΈ стСпСнСй с ΠΎΠ΄ΠΈΠ½Π°ΠΊΠΎΠ²Ρ‹ΠΌ основаниСм ΠΈΡ… ΠΏΠΎΠΊΠ°Π·Π°Ρ‚Π΅Π»ΠΈ Π²Ρ‹Ρ‡ΠΈΡ‚Π°ΡŽΡ‚ΡΡ .

      3. Π‘Ρ‚Π΅ΠΏΠ΅Π½ΡŒ произвСдСния Π΄Π²ΡƒΡ… ΠΈΠ»ΠΈ Π½Π΅ΡΠΊΠΎΠ»ΡŒΠΊΠΈΡ… сомноТитСлСй Ρ€Π°Π²Π½Π° ΠΏΡ€ΠΎΠΈΠ·Π²Π΅Π΄Π΅Π½ΠΈΡŽ стСпСнСй этих сомноТитСлСй.

      4. Π‘Ρ‚Π΅ΠΏΠ΅Π½ΡŒ ΠΎΡ‚Π½ΠΎΡˆΠ΅Π½ΠΈΡ (Π΄Ρ€ΠΎΠ±ΠΈ) Ρ€Π°Π²Π½Π° ΠΎΡ‚Π½ΠΎΡˆΠ΅Π½ΠΈΡŽ стСпСнСй Π΄Π΅Π»ΠΈΠΌΠΎΠ³ΠΎ (числитСля) ΠΈ дСлитСля (знамСнатСля):

      (a / b ) n = a n / b n .

      5. ΠŸΡ€ΠΈ Π²ΠΎΠ·Π²Π΅Π΄Π΅Π½ΠΈΠΈ стСпСни Π² ΡΡ‚Π΅ΠΏΠ΅Π½ΡŒ ΠΈΡ… ΠΏΠΎΠΊΠ°Π·Π°Ρ‚Π΅Π»ΠΈ ΠΏΠ΅Ρ€Π΅ΠΌΠ½ΠΎΠΆΠ°ΡŽΡ‚ΡΡ:

      ВсС Π²Ρ‹ΡˆΠ΅ΠΏΡ€ΠΈΠ²Π΅Π΄Π΅Π½Π½Ρ‹Π΅ Ρ„ΠΎΡ€ΠΌΡƒΠ»Ρ‹ Ρ‡ΠΈΡ‚Π°ΡŽΡ‚ΡΡ ΠΈ Π²Ρ‹ΠΏΠΎΠ»Π½ΡΡŽΡ‚ΡΡ Π² ΠΎΠ±ΠΎΠΈΡ… направлСниях слСва Π½Π°ΠΏΡ€Π°Π²ΠΎ ΠΈ Π½Π°ΠΎΠ±ΠΎΡ€ΠΎΡ‚.

      П Ρ€ ΠΈ ΠΌ Π΅ Ρ€. (2 Β· 3 Β· 5 / 15) Β² = 2 Β² Β· 3 Β² Β· 5 Β² / 15 Β² = 900 / 225 = 4 .

      ΠžΠΏΠ΅Ρ€Π°Ρ†ΠΈΠΈ с корнями. Π’ΠΎ всСх Π½ΠΈΠΆΠ΅ΠΏΡ€ΠΈΠ²Π΅Π΄Π΅Π½Π½Ρ‹Ρ… Ρ„ΠΎΡ€ΠΌΡƒΠ»Π°Ρ… символ ΠΎΠ·Π½Π°Ρ‡Π°Π΅Ρ‚ арифмСтичСский ΠΊΠΎΡ€Π΅Π½ΡŒ (ΠΏΠΎΠ΄ΠΊΠΎΡ€Π΅Π½Π½ΠΎΠ΅ Π²Ρ‹Ρ€Π°ΠΆΠ΅Π½ΠΈΠ΅ ΠΏΠΎΠ»ΠΎΠΆΠΈΡ‚Π΅Π»ΡŒΠ½ΠΎ).

      1. ΠšΠΎΡ€Π΅Π½ΡŒ ΠΈΠ· произвСдСния Π½Π΅ΡΠΊΠΎΠ»ΡŒΠΊΠΈΡ… сомноТитСлСй Ρ€Π°Π²Π΅Π½ ΠΏΡ€ΠΎΠΈΠ·Π²Π΅Π΄Π΅Π½ΠΈΡŽ ΠΊΠΎΡ€Π½Π΅ΠΉ ΠΈΠ· этих сомноТитСлСй:

      2. ΠšΠΎΡ€Π΅Π½ΡŒ ΠΈΠ· ΠΎΡ‚Π½ΠΎΡˆΠ΅Π½ΠΈΡ Ρ€Π°Π²Π΅Π½ ΠΎΡ‚Π½ΠΎΡˆΠ΅Π½ΠΈΡŽ ΠΊΠΎΡ€Π½Π΅ΠΉ Π΄Π΅Π»ΠΈΠΌΠΎΠ³ΠΎ ΠΈ дСлитСля:

      3. ΠŸΡ€ΠΈ Π²ΠΎΠ·Π²Π΅Π΄Π΅Π½ΠΈΠΈ корня Π² ΡΡ‚Π΅ΠΏΠ΅Π½ΡŒ достаточно возвСсти Π² эту ΡΡ‚Π΅ΠΏΠ΅Π½ΡŒ ΠΏΠΎΠ΄ΠΊΠΎΡ€Π΅Π½Π½ΠΎΠ΅ число:

      4. Если ΡƒΠ²Π΅Π»ΠΈΡ‡ΠΈΡ‚ΡŒ ΡΡ‚Π΅ΠΏΠ΅Π½ΡŒ корня Π² m Ρ€Π°Π· ΠΈ ΠΎΠ΄Π½ΠΎΠ²Ρ€Π΅ΠΌΠ΅Π½Π½ΠΎ возвСсти Π² m -ΡƒΡŽ ΡΡ‚Π΅ΠΏΠ΅Π½ΡŒ ΠΏΠΎΠ΄ΠΊΠΎΡ€Π΅Π½Π½ΠΎΠ΅ число, Ρ‚ΠΎ Π·Π½Π°Ρ‡Π΅Π½ΠΈΠ΅ корня Π½Π΅ измСнится:

      5. Если ΡƒΠΌΠ΅Π½ΡŒΡˆΠΈΡ‚ΡŒ ΡΡ‚Π΅ΠΏΠ΅Π½ΡŒ корня Π² m Ρ€Π°Π· ΠΈ ΠΎΠ΄Π½ΠΎΠ²Ρ€Π΅ΠΌΠ΅Π½Π½ΠΎ ΠΈΠ·Π²Π»Π΅Ρ‡ΡŒ ΠΊΠΎΡ€Π΅Π½ΡŒ m -ΠΎΠΉ стСпСни ΠΈΠ· ΠΏΠΎΠ΄ΠΊΠΎΡ€Π΅Π½Π½ΠΎΠ³ΠΎ числа, Ρ‚ΠΎ Π·Π½Π°Ρ‡Π΅Π½ΠΈΠ΅ корня Π½Π΅ измСнится:


      Π Π°ΡΡˆΠΈΡ€Π΅Π½ΠΈΠ΅ понятия стСпСни. Π”ΠΎ сих ΠΏΠΎΡ€ ΠΌΡ‹ рассматривали стСпСни Ρ‚ΠΎΠ»ΡŒΠΊΠΎ с Π½Π°Ρ‚ΡƒΡ€Π°Π»ΡŒΠ½Ρ‹ΠΌ ΠΏΠΎΠΊΠ°Π·Π°Ρ‚Π΅Π»Π΅ΠΌ; Π½ΠΎ дСйствия со стСпСнями ΠΈ корнями ΠΌΠΎΠ³ΡƒΡ‚ ΠΏΡ€ΠΈΠ²ΠΎΠ΄ΠΈΡ‚ΡŒ Ρ‚Π°ΠΊΠΆΠ΅ ΠΊ ΠΎΡ‚Ρ€ΠΈΡ†Π°Ρ‚Π΅Π»ΡŒΠ½Ρ‹ΠΌ , Π½ΡƒΠ»Π΅Π²Ρ‹ΠΌ ΠΈ Π΄Ρ€ΠΎΠ±Π½Ρ‹ΠΌ показатСлям. ВсС эти ΠΏΠΎΠΊΠ°Π·Π°Ρ‚Π΅Π»ΠΈ стСпСнСй Ρ‚Ρ€Π΅Π±ΡƒΡŽΡ‚ Π΄ΠΎΠΏΠΎΠ»Π½ΠΈΡ‚Π΅Π»ΡŒΠ½ΠΎΠ³ΠΎ опрСдСлСния.

      Π‘Ρ‚Π΅ΠΏΠ΅Π½ΡŒ с ΠΎΡ‚Ρ€ΠΈΡ†Π°Ρ‚Π΅Π»ΡŒΠ½Ρ‹ΠΌ ΠΏΠΎΠΊΠ°Π·Π°Ρ‚Π΅Π»Π΅ΠΌ. Π‘Ρ‚Π΅ΠΏΠ΅Π½ΡŒ Π½Π΅ΠΊΠΎΡ‚ΠΎΡ€ΠΎΠ³ΠΎ числа с ΠΎΡ‚Ρ€ΠΈΡ†Π°Ρ‚Π΅Π»ΡŒΠ½Ρ‹ΠΌ (Ρ†Π΅Π»Ρ‹ΠΌ) ΠΏΠΎΠΊΠ°Π·Π°Ρ‚Π΅Π»Π΅ΠΌ опрСдСляСтся ΠΊΠ°ΠΊ Π΅Π΄ΠΈΠ½ΠΈΡ†Π°, дСлённая Π½Π° ΡΡ‚Π΅ΠΏΠ΅Π½ΡŒ Ρ‚ΠΎΠ³ΠΎ ΠΆΠ΅ числа с ΠΏΠΎΠΊΠ°Π·Π°Ρ‚Π΅Π»Π΅ΠΌ, Ρ€Π°Π²Π½Ρ‹ΠΌ Π°Π±ΡΠΎΠ»ΡŽΡ‚Π½ΠΎΠΉ Π²Π΅Π»Π΅Ρ‡ΠΈΠ½Π΅ ΠΎΡ‚Ρ€ΠΈΡ†Π°Ρ‚Π΅Π»ΡŒΠ½ΠΎΠ³ΠΎ показатСля:

      Π’ Π΅ΠΏΠ΅Ρ€ΡŒ Ρ„ΠΎΡ€ΠΌΡƒΠ»Π° a m : a n = a m β€” n ΠΌΠΎΠΆΠ΅Ρ‚ Π±Ρ‹Ρ‚ΡŒ использована Π½Π΅ Ρ‚ΠΎΠ»ΡŒΠΊΠΎ ΠΏΡ€ΠΈ m , большСм, Ρ‡Π΅ΠΌ n , Π½ΠΎ ΠΈ ΠΏΡ€ΠΈ m , мСньшСм, Ρ‡Π΅ΠΌ n .

      П Ρ€ ΠΈ ΠΌ Π΅ Ρ€. a 4: a 7 = a 4 β€” 7 = a β€” 3 .

      Если ΠΌΡ‹ Ρ…ΠΎΡ‚ΠΈΠΌ, Ρ‡Ρ‚ΠΎΠ±Ρ‹ Ρ„ΠΎΡ€ΠΌΡƒΠ»Π° a m : a n = a m β€” n Π±Ρ‹Π»Π° справСдлива ΠΏΡ€ΠΈ m = n , Π½Π°ΠΌ Π½Π΅ΠΎΠ±Ρ…ΠΎΠ΄ΠΈΠΌΠΎ ΠΎΠΏΡ€Π΅Π΄Π΅Π»Π΅Π½ΠΈΠ΅ Π½ΡƒΠ»Π΅Π²ΠΎΠΉ стСпСни.

      Π‘Ρ‚Π΅ΠΏΠ΅Π½ΡŒ с Π½ΡƒΠ»Π΅Π²Ρ‹ΠΌ ΠΏΠΎΠΊΠ°Π·Π°Ρ‚Π΅Π»Π΅ΠΌ. Π‘Ρ‚Π΅ΠΏΠ΅Π½ΡŒ любого Π½Π΅Π½ΡƒΠ»Π΅Π²ΠΎΠ³ΠΎ числа с Π½ΡƒΠ»Π΅Π²Ρ‹ΠΌ ΠΏΠΎΠΊΠ°Π·Π°Ρ‚Π΅Π»Π΅ΠΌ Ρ€Π°Π²Π½Π° 1.

      П Ρ€ ΠΈ ΠΌ Π΅ Ρ€ Ρ‹. 2 0 = 1, (– 5) 0 = 1, (– 3 / 5) 0 = 1.

      Π‘Ρ‚Π΅ΠΏΠ΅Π½ΡŒ с Π΄Ρ€ΠΎΠ±Π½Ρ‹ΠΌ ΠΏΠΎΠΊΠ°Π·Π°Ρ‚Π΅Π»Π΅ΠΌ. Для Ρ‚ΠΎΠ³ΠΎ, Ρ‡Ρ‚ΠΎΠ±Ρ‹ возвСсти Π΄Π΅ΠΉΡΡ‚Π²ΠΈΡ‚Π΅Π»ΡŒΠ½ΠΎΠ΅ число Π° Π² ΡΡ‚Π΅ΠΏΠ΅Π½ΡŒ m / n , Π½ΡƒΠΆΠ½ΠΎ ΠΈΠ·Π²Π»Π΅Ρ‡ΡŒ ΠΊΠΎΡ€Π΅Π½ΡŒ n –ой стСпСни ΠΈΠ· m -ΠΎΠΉ стСпСни этого числа Π°:

      О выраТСниях, Π½Π΅ ΠΈΠΌΠ΅ΡŽΡ‰ΠΈΡ… смысла. Π•ΡΡ‚ΡŒ нСсколько Ρ‚Π°ΠΊΠΈΡ… Π²Ρ‹Ρ€Π°ΠΆΠ΅Π½ΠΈΠΉ.

      Π³Π΄Π΅ a β‰  0 , Π½Π΅ сущСствуСт.

      Π’ самом Π΄Π΅Π»Π΅, Ссли ΠΏΡ€Π΅Π΄ΠΏΠΎΠ»ΠΎΠΆΠΈΡ‚ΡŒ, Ρ‡Ρ‚ΠΎ x – Π½Π΅ΠΊΠΎΡ‚ΠΎΡ€ΠΎΠ΅ число, Ρ‚ΠΎ Π² соотвСтствии с ΠΎΠΏΡ€Π΅Π΄Π΅Π»Π΅Π½ΠΈΠ΅ΠΌ ΠΎΠΏΠ΅Ρ€Π°Ρ†ΠΈΠΈ дСлСния ΠΈΠΌΠ΅Π΅ΠΌ: a = 0Β· x , Ρ‚.e. a = 0, Ρ‡Ρ‚ΠΎ ΠΏΡ€ΠΎΡ‚ΠΈΠ²ΠΎΡ€Π΅Ρ‡ΠΈΡ‚ ΡƒΡΠ»ΠΎΠ²ΠΈΡŽ: a β‰  0

      β€” любоС число.

      Π’ самом Π΄Π΅Π»Π΅, Ссли ΠΏΡ€Π΅Π΄ΠΏΠΎΠ»ΠΎΠΆΠΈΡ‚ΡŒ, Ρ‡Ρ‚ΠΎ это Π²Ρ‹Ρ€Π°ΠΆΠ΅Π½ΠΈΠ΅ Ρ€Π°Π²Π½ΠΎ Π½Π΅ΠΊΠΎΡ‚ΠΎΡ€ΠΎΠΌΡƒ числу x , Ρ‚ΠΎ согласно ΠΎΠΏΡ€Π΅Π΄Π΅Π»Π΅Π½ΠΈΡŽ ΠΎΠΏΠ΅Ρ€Π°Ρ†ΠΈΠΈ дСлСния ΠΈΠΌΠ΅Π΅ΠΌ: 0 = 0 Β· x . Но это равСнство ΠΈΠΌΠ΅Π΅Ρ‚ мСсто ΠΏΡ€ΠΈ любом числС x , Ρ‡Ρ‚ΠΎ ΠΈ Ρ‚Ρ€Π΅Π±ΠΎΠ²Π°Π»ΠΎΡΡŒ Π΄ΠΎΠΊΠ°Π·Π°Ρ‚ΡŒ.

      0 0 β€” любоС число.

      Π  Π΅ ш Π΅ Π½ ΠΈ Π΅. Рассмотрим Ρ‚Ρ€ΠΈ основных случая:

      1) x = 0 – это Π·Π½Π°Ρ‡Π΅Π½ΠΈΠ΅ Π½Π΅ удовлСтворяСт Π΄Π°Π½Π½ΠΎΠΌΡƒ ΡƒΡ€Π°Π²Π½Π΅Π½ΠΈΡŽ

      2) ΠΏΡ€ΠΈ x > 0 ΠΏΠΎΠ»ΡƒΡ‡Π°Π΅ΠΌ: x / x = 1, Ρ‚.e. 1 = 1, ΠΎΡ‚ΠΊΡƒΠ΄Π° слСдуСт,

      Ρ‡Ρ‚ΠΎ x – любоС число; Π½ΠΎ принимая Π²ΠΎ Π²Π½ΠΈΠΌΠ°Π½ΠΈΠ΅, Ρ‡Ρ‚ΠΎ Π²

      нашСм случаС x > 0 , ΠΎΡ‚Π²Π΅Ρ‚ΠΎΠΌ являСтся x > 0 ;

      Бвойства стСпСни

      НапоминаСм, Ρ‡Ρ‚ΠΎ Π² Π΄Π°Π½Π½ΠΎΠΌ ΡƒΡ€ΠΎΠΊΠ΅ Ρ€Π°Π·Π±ΠΈΡ€Π°ΡŽΡ‚ΡΡ свойства стСпСнСй с Π½Π°Ρ‚ΡƒΡ€Π°Π»ΡŒΠ½Ρ‹ΠΌΠΈ показатСлями ΠΈ Π½ΡƒΠ»Ρ‘ΠΌ. Π‘Ρ‚Π΅ΠΏΠ΅Π½ΠΈ с Ρ€Π°Ρ†ΠΈΠΎΠ½Π°Π»ΡŒΠ½Ρ‹ΠΌΠΈ показатСлями ΠΈ ΠΈΡ… свойства Π±ΡƒΠ΄ΡƒΡ‚ рассмотрСны Π² ΡƒΡ€ΠΎΠΊΠ°Ρ… для 8 классов.

      Π‘Ρ‚Π΅ΠΏΠ΅Π½ΡŒ с Π½Π°Ρ‚ΡƒΡ€Π°Π»ΡŒΠ½Ρ‹ΠΌ ΠΏΠΎΠΊΠ°Π·Π°Ρ‚Π΅Π»Π΅ΠΌ ΠΎΠ±Π»Π°Π΄Π°Π΅Ρ‚ нСсколькими Π²Π°ΠΆΠ½Ρ‹ΠΌΠΈ свойствами, ΠΊΠΎΡ‚ΠΎΡ€Ρ‹Π΅ ΠΏΠΎΠ·Π²ΠΎΠ»ΡΡŽΡ‚ ΡƒΠΏΡ€ΠΎΡ‰Π°Ρ‚ΡŒ вычислСния Π² ΠΏΡ€ΠΈΠΌΠ΅Ρ€Π°Ρ… со стСпСнями.

      Бвойство β„– 1


      ΠŸΡ€ΠΎΠΈΠ·Π²Π΅Π΄Π΅Π½ΠΈΠ΅ стСпСнСй

      ΠŸΡ€ΠΈ ΡƒΠΌΠ½ΠΎΠΆΠ΅Π½ΠΈΠΈ стСпСнСй с ΠΎΠ΄ΠΈΠ½Π°ΠΊΠΎΠ²Ρ‹ΠΌΠΈ основаниями основаниС остаётся Π±Π΅Π· ΠΈΠ·ΠΌΠ΅Π½Π΅Π½ΠΈΠΉ, Π° ΠΏΠΎΠΊΠ°Π·Π°Ρ‚Π΅Π»ΠΈ стСпСнСй ΡΠΊΠ»Π°Π΄Ρ‹Π²Π°ΡŽΡ‚ΡΡ.

      a m Β· a n = a m + n , Π³Π΄Π΅ Β« a Β» — любоС число, Π° Β« m Β», Β« n Β» — Π»ΡŽΠ±Ρ‹Π΅ Π½Π°Ρ‚ΡƒΡ€Π°Π»ΡŒΠ½Ρ‹Π΅ числа.

      Π”Π°Π½Π½ΠΎΠ΅ свойство стСпСнСй Ρ‚Π°ΠΊΠΆΠ΅ дСйствуСт Π½Π° ΠΏΡ€ΠΎΠΈΠ·Π²Π΅Π΄Π΅Π½ΠΈΠ΅ Ρ‚Ρ€Ρ‘Ρ… ΠΈ Π±ΠΎΠ»Π΅Π΅ стСпСнСй.

    • Π£ΠΏΡ€ΠΎΡΡ‚ΠΈΡ‚ΡŒ Π²Ρ‹Ρ€Π°ΠΆΠ΅Π½ΠΈΠ΅.
      b Β· b 2 Β· b 3 Β· b 4 Β· b 5 = b 1 + 2 + 3 + 4 + 5 = b 15
    • ΠŸΡ€Π΅Π΄ΡΡ‚Π°Π²ΠΈΡ‚ΡŒ Π² Π²ΠΈΠ΄Π΅ стСпСни.
      6 15 Β· 36 = 6 15 Β· 6 2 = 6 15 Β· 6 2 = 6 17
    • ΠŸΡ€Π΅Π΄ΡΡ‚Π°Π²ΠΈΡ‚ΡŒ Π² Π²ΠΈΠ΄Π΅ стСпСни.
      (0,8) 3 Β· (0,8) 12 = (0,8) 3 + 12 = (0,8) 15
    • ΠžΠ±Ρ€Π°Ρ‚ΠΈΡ‚Π΅ Π²Π½ΠΈΠΌΠ°Π½ΠΈΠ΅, Ρ‡Ρ‚ΠΎ Π² ΡƒΠΊΠ°Π·Π°Π½Π½ΠΎΠΌ свойствС Ρ€Π΅Ρ‡ΡŒ шла Ρ‚ΠΎΠ»ΡŒΠΊΠΎ ΠΎΠ± ΡƒΠΌΠ½ΠΎΠΆΠ΅Π½ΠΈΠΈ стСпСнСй с ΠΎΠ΄ΠΈΠ½Π°ΠΊΠΎΠ²Ρ‹ΠΌΠΈ основаниями . Оно Π½Π΅ относится ΠΊ ΠΈΡ… слоТСнию.

      НСльзя Π·Π°ΠΌΠ΅Π½ΡΡ‚ΡŒ сумму (3 3 + 3 2) Π½Π° 3 5 . Π­Ρ‚ΠΎ понятно, Ссли
      ΠΏΠΎΡΡ‡ΠΈΡ‚Π°Ρ‚ΡŒ (3 3 + 3 2) = (27 + 9) = 36 , Π° 3 5 = 243

      Бвойство β„– 2


      ЧастноС стСпСнСй

      ΠŸΡ€ΠΈ Π΄Π΅Π»Π΅Π½ΠΈΠΈ стСпСнСй с ΠΎΠ΄ΠΈΠ½Π°ΠΊΠΎΠ²Ρ‹ΠΌΠΈ основаниями основаниС остаётся Π±Π΅Π· ΠΈΠ·ΠΌΠ΅Π½Π΅Π½ΠΈΠΉ, Π° ΠΈΠ· показатСля стСпСни Π΄Π΅Π»ΠΈΠΌΠΎΠ³ΠΎ Π²Ρ‹Ρ‡ΠΈΡ‚Π°ΡŽΡ‚ ΠΏΠΎΠΊΠ°Π·Π°Ρ‚Π΅Π»ΡŒ стСпСни дСлитСля.

    • Π—Π°ΠΏΠΈΡΠ°Ρ‚ΡŒ частноС Π² Π²ΠΈΠ΄Π΅ стСпСни
      (2b) 5: (2b) 3 = (2b) 5 βˆ’ 3 = (2b) 2
    • Π’Ρ‹Ρ‡ΠΈΡΠ»ΠΈΡ‚ΡŒ.

    11 3 βˆ’ 2 Β· 4 2 βˆ’ 1 = 11 Β· 4 = 44
    ΠŸΡ€ΠΈΠΌΠ΅Ρ€. Π Π΅ΡˆΠΈΡ‚ΡŒ ΡƒΡ€Π°Π²Π½Π΅Π½ΠΈΠ΅. Π˜ΡΠΏΠΎΠ»ΡŒΠ·ΡƒΠ΅ΠΌ свойство частного стСпСнСй.
    3 8: t = 3 4

    ΠžΡ‚Π²Π΅Ρ‚: t = 3 4 = 81

    ΠŸΠΎΠ»ΡŒΠ·ΡƒΡΡΡŒ свойствами β„– 1 ΠΈ β„– 2, ΠΌΠΎΠΆΠ½ΠΎ Π»Π΅Π³ΠΊΠΎ ΡƒΠΏΡ€ΠΎΡ‰Π°Ρ‚ΡŒ выраТСния ΠΈ ΠΏΡ€ΠΎΠΈΠ·Π²ΠΎΠ΄ΠΈΡ‚ΡŒ вычислСния.

    ΠŸΡ€ΠΈΠΌΠ΅Ρ€. Π£ΠΏΡ€ΠΎΡΡ‚ΠΈΡ‚ΡŒ Π²Ρ‹Ρ€Π°ΠΆΠ΅Π½ΠΈΠ΅.
    4 5m + 6 Β· 4 m + 2: 4 4m + 3 = 4 5m + 6 + m + 2: 4 4m + 3 = 4 6m + 8 βˆ’ 4m βˆ’ 3 = 4 2m + 5

    ΠŸΡ€ΠΈΠΌΠ΅Ρ€. Найти Π·Π½Π°Ρ‡Π΅Π½ΠΈΠ΅ выраТСния, ΠΈΡΠΏΠΎΠ»ΡŒΠ·ΡƒΡ свойства стСпСни.

    2 11 βˆ’ 5 = 2 6 = 64

    ΠžΠ±Ρ€Π°Ρ‚ΠΈΡ‚Π΅ Π²Π½ΠΈΠΌΠ°Π½ΠΈΠ΅, Ρ‡Ρ‚ΠΎ Π² свойствС 2 Ρ€Π΅Ρ‡ΡŒ шла Ρ‚ΠΎΠ»ΡŒΠΊΠΎ ΠΎ Π΄Π΅Π»Π΅Π½ΠΈΠΈ стСпСнСй с ΠΎΠ΄ΠΈΠ½Π°ΠΊΠΎΠ²Ρ‹ΠΌΠΈ основаниями.

    НСльзя Π·Π°ΠΌΠ΅Π½ΡΡ‚ΡŒ Ρ€Π°Π·Π½ΠΎΡΡ‚ΡŒ (4 3 βˆ’4 2) Π½Π° 4 1 . Π­Ρ‚ΠΎ понятно, Ссли ΠΏΠΎΡΡ‡ΠΈΡ‚Π°Ρ‚ΡŒ (4 3 βˆ’4 2) = (64 βˆ’ 16) = 48 , Π° 4 1 = 4

    Бвойство β„– 3


    Π’ΠΎΠ·Π²Π΅Π΄Π΅Π½ΠΈΠ΅ стСпСни Π² ΡΡ‚Π΅ΠΏΠ΅Π½ΡŒ

    ΠŸΡ€ΠΈ Π²ΠΎΠ·Π²Π΅Π΄Π΅Π½ΠΈΠΈ стСпСни Π² ΡΡ‚Π΅ΠΏΠ΅Π½ΡŒ основаниС стСпСни остаётся Π±Π΅Π· измСнСния, Π° ΠΏΠΎΠΊΠ°Π·Π°Ρ‚Π΅Π»ΠΈ стСпСнСй ΠΏΠ΅Ρ€Π΅ΠΌΠ½ΠΎΠΆΠ°ΡŽΡ‚ΡΡ.

    (a n) m = a n Β· m , Π³Π΄Π΅ Β« a Β» — любоС число, Π° Β« m Β», Β« n Β» — Π»ΡŽΠ±Ρ‹Π΅ Π½Π°Ρ‚ΡƒΡ€Π°Π»ΡŒΠ½Ρ‹Π΅ числа.

  • ΠŸΡ€ΠΈΠΌΠ΅Ρ€.
    (a 4) 6 = a 4 Β· 6 = a 24
  • ΠŸΡ€ΠΈΠΌΠ΅Ρ€. ΠŸΡ€Π΅Π΄ΡΡ‚Π°Π²ΠΈΡ‚ΡŒ 3 20 Π² Π²ΠΈΠ΄Π΅ стСпСни с основаниСм 3 2 .
  • По свойству возвСдСния стСпСни Π² ΡΡ‚Π΅ΠΏΠ΅Π½ΡŒ извСстно, Ρ‡Ρ‚ΠΎ ΠΏΡ€ΠΈ Π²ΠΎΠ·Π²Π΅Π΄Π΅Π½ΠΈΠΈ Π² ΡΡ‚Π΅ΠΏΠ΅Π½ΡŒ ΠΏΠΎΠΊΠ°Π·Π°Ρ‚Π΅Π»ΠΈ ΠΏΠ΅Ρ€Π΅ΠΌΠ½ΠΎΠΆΠ°ΡŽΡ‚ΡΡ, Π·Π½Π°Ρ‡ΠΈΡ‚:

    Бвойства 4


    Π‘Ρ‚Π΅ΠΏΠ΅Π½ΡŒ произвСдСния

    ΠŸΡ€ΠΈ Π²ΠΎΠ·Π²Π΅Π΄Π΅Π½ΠΈΠΈ стСпСни Π² ΡΡ‚Π΅ΠΏΠ΅Π½ΡŒ произвСдСния Π² эту ΡΡ‚Π΅ΠΏΠ΅Π½ΡŒ возводится ΠΊΠ°ΠΆΠ΄Ρ‹ΠΉ ΠΌΠ½ΠΎΠΆΠΈΡ‚Π΅Π»ΡŒ ΠΈ Ρ€Π΅Π·ΡƒΠ»ΡŒΡ‚Π°Ρ‚Ρ‹ ΠΏΠ΅Ρ€Π΅ΠΌΠ½ΠΎΠΆΠ°ΡŽΡ‚ΡΡ.

    (a Β· b) n = a n Β· b n , Π³Π΄Π΅ Β« a Β», Β« b Β» — Π»ΡŽΠ±Ρ‹Π΅ Ρ€Π°Ρ†ΠΈΠΎΠ½Π°Π»ΡŒΠ½Ρ‹Π΅ числа; Β« n Β» — любоС Π½Π°Ρ‚ΡƒΡ€Π°Π»ΡŒΠ½ΠΎΠ΅ число.

    • ΠŸΡ€ΠΈΠΌΠ΅Ρ€ 1.
      (6 · a 2 · b 3 · c) 2 = 6 2 · a 2 · 2 · b 3 · 2 · с 1 · 2 = 36 a 4 · b 6 · с 2
    • ΠŸΡ€ΠΈΠΌΠ΅Ρ€ 2.
      (βˆ’x 2 Β· y) 6 = ((βˆ’1) 6 Β· x 2 Β· 6 Β· y 1 Β· 6) = x 12 Β· y 6
    • ΠžΠ±Ρ€Π°Ρ‚ΠΈΡ‚Π΅ Π²Π½ΠΈΠΌΠ°Π½ΠΈΠ΅, Ρ‡Ρ‚ΠΎ свойство β„– 4, ΠΊΠ°ΠΊ ΠΈ Π΄Ρ€ΡƒΠ³ΠΈΠ΅ свойства стСпСнСй, ΠΏΡ€ΠΈΠΌΠ΅Π½ΡΡŽΡ‚ ΠΈ Π² ΠΎΠ±Ρ€Π°Ρ‚Π½ΠΎΠΌ порядкС.

      (a n Β· b n)= (a Β· b) n

      Π’ΠΎ Π΅ΡΡ‚ΡŒ, Ρ‡Ρ‚ΠΎΠ±Ρ‹ ΠΏΠ΅Ρ€Π΅ΠΌΠ½ΠΎΠΆΠΈΡ‚ΡŒ стСпСни с ΠΎΠ΄ΠΈΠ½Π°ΠΊΠΎΠ²Ρ‹ΠΌΠΈ показатСлями ΠΌΠΎΠΆΠ½ΠΎ ΠΏΠ΅Ρ€Π΅ΠΌΠ½ΠΎΠΆΠΈΡ‚ΡŒ основания, Π° ΠΏΠΎΠΊΠ°Π·Π°Ρ‚Π΅Π»ΡŒ стСпСни ΠΎΡΡ‚Π°Π²ΠΈΡ‚ΡŒ Π½Π΅ΠΈΠ·ΠΌΠ΅Π½Π½Ρ‹ΠΌ.

    • ΠŸΡ€ΠΈΠΌΠ΅Ρ€. Π’Ρ‹Ρ‡ΠΈΡΠ»ΠΈΡ‚ΡŒ.
      2 4 Β· 5 4 = (2 Β· 5) 4 = 10 4 = 10 000
    • ΠŸΡ€ΠΈΠΌΠ΅Ρ€. Π’Ρ‹Ρ‡ΠΈΡΠ»ΠΈΡ‚ΡŒ.
      0,5 16 Β· 2 16 = (0,5 Β· 2) 16 = 1
    • Π’ Π±ΠΎΠ»Π΅Π΅ слоТных ΠΏΡ€ΠΈΠΌΠ΅Ρ€Π°Ρ… ΠΌΠΎΠ³ΡƒΡ‚ Π²ΡΡ‚Ρ€Π΅Ρ‚ΠΈΡ‚ΡŒΡΡ случаи, ΠΊΠΎΠ³Π΄Π° ΡƒΠΌΠ½ΠΎΠΆΠ΅Π½ΠΈΠ΅ ΠΈ Π΄Π΅Π»Π΅Π½ΠΈΠ΅ Π½Π°Π΄ΠΎ Π²Ρ‹ΠΏΠΎΠ»Π½ΠΈΡ‚ΡŒ Π½Π°Π΄ стСпСнями с Ρ€Π°Π·Π½Ρ‹ΠΌΠΈ основаниями ΠΈ Ρ€Π°Π·Π½Ρ‹ΠΌΠΈ показатСлями. Π’ этом случаС совСтуСм ΠΏΠΎΡΡ‚ΡƒΠΏΠ°Ρ‚ΡŒ ΡΠ»Π΅Π΄ΡƒΡŽΡ‰ΠΈΠΌ ΠΎΠ±Ρ€Π°Π·ΠΎΠΌ.

      НапримСр, 4 5 Β· 3 2 = 4 3 Β· 4 2 Β· 3 2 = 4 3 Β· (4 Β· 3) 2 = 64 Β· 12 2 = 64 Β· 144 = 9216

      ΠŸΡ€ΠΈΠΌΠ΅Ρ€ возвСдСния Π² ΡΡ‚Π΅ΠΏΠ΅Π½ΡŒ дСсятичной Π΄Ρ€ΠΎΠ±ΠΈ.

      4 21 Β· (βˆ’0,25) 20 = 4 Β· 4 20 Β· (βˆ’0,25) 20 = 4 Β· (4 Β· (βˆ’0,25)) 20 = 4 Β· (βˆ’1) 20 = 4 Β· 1 = 4

      Бвойства 5


      Π‘Ρ‚Π΅ΠΏΠ΅Π½ΡŒ частного (Π΄Ρ€ΠΎΠ±ΠΈ)

      Π§Ρ‚ΠΎΠ±Ρ‹ возвСсти Π² ΡΡ‚Π΅ΠΏΠ΅Π½ΡŒ частноС, ΠΌΠΎΠΆΠ½ΠΎ возвСсти Π² эту ΡΡ‚Π΅ΠΏΠ΅Π½ΡŒ ΠΎΡ‚Π΄Π΅Π»ΡŒΠ½ΠΎ Π΄Π΅Π»ΠΈΠΌΠΎΠ΅ ΠΈ Π΄Π΅Π»ΠΈΡ‚Π΅Π»ΡŒ, ΠΈ ΠΏΠ΅Ρ€Π²Ρ‹ΠΉ Ρ€Π΅Π·ΡƒΠ»ΡŒΡ‚Π°Ρ‚ Ρ€Π°Π·Π΄Π΅Π»ΠΈΡ‚ΡŒ Π½Π° Π²Ρ‚ΠΎΡ€ΠΎΠΉ.

      (a: b) n = a n: b n , Π³Π΄Π΅ Β« a Β», Β« b Β» — Π»ΡŽΠ±Ρ‹Π΅ Ρ€Π°Ρ†ΠΈΠΎΠ½Π°Π»ΡŒΠ½Ρ‹Π΅ числа, b β‰  0, n — любоС Π½Π°Ρ‚ΡƒΡ€Π°Π»ΡŒΠ½ΠΎΠ΅ число.

    • ΠŸΡ€ΠΈΠΌΠ΅Ρ€. ΠŸΡ€Π΅Π΄ΡΡ‚Π°Π²ΠΈΡ‚ΡŒ Π²Ρ‹Ρ€Π°ΠΆΠ΅Π½ΠΈΠ΅ Π² Π²ΠΈΠ΄Π΅ частного стСпСнСй.
      (5: 3) 12 = 5 12: 3 12
    • НапоминаСм, Ρ‡Ρ‚ΠΎ частноС ΠΌΠΎΠΆΠ½ΠΎ ΠΏΡ€Π΅Π΄ΡΡ‚Π°Π²ΠΈΡ‚ΡŒ Π² Π²ΠΈΠ΄Π΅ Π΄Ρ€ΠΎΠ±ΠΈ. ΠŸΠΎΡΡ‚ΠΎΠΌΡƒ Π½Π° Ρ‚Π΅ΠΌΠ΅ Π²ΠΎΠ·Π²Π΅Π΄Π΅Π½ΠΈΠ΅ Π΄Ρ€ΠΎΠ±ΠΈ Π² ΡΡ‚Π΅ΠΏΠ΅Π½ΡŒ ΠΌΡ‹ остановимся Π±ΠΎΠ»Π΅Π΅ ΠΏΠΎΠ΄Ρ€ΠΎΠ±Π½ΠΎ Π½Π° ΡΠ»Π΅Π΄ΡƒΡŽΡ‰Π΅ΠΉ страницС.

    ΠžΡ‡Π΅Π²ΠΈΠ΄Π½ΠΎ, Ρ‡Ρ‚ΠΎ числа со стСпСнями ΠΌΠΎΠ³ΡƒΡ‚ ΡΠ»Π°Π³Π°Ρ‚ΡŒΡΡ, ΠΊΠ°ΠΊ Π΄Ρ€ΡƒΠ³ΠΈΠ΅ Π²Π΅Π»ΠΈΡ‡ΠΈΠ½Ρ‹ , ΠΏΡƒΡ‚Π΅ΠΌ ΠΈΡ… слоТСния ΠΎΠ΄Π½ΠΎ Π·Π° Π΄Ρ€ΡƒΠ³ΠΈΠΌ со своими Π·Π½Π°ΠΊΠ°ΠΌΠΈ .

    Π’Π°ΠΊ, сумма a 3 ΠΈ b 2 Π΅ΡΡ‚ΡŒ a 3 + b 2 .
    Π‘ΡƒΠΌΠΌΠ° a 3 — b n ΠΈ h 5 -d 4 Π΅ΡΡ‚ΡŒ a 3 — b n + h 5 — d 4 .

    ΠšΠΎΡΡ„Ρ„ΠΈΡ†ΠΈΠ΅Π½Ρ‚Ρ‹ ΠΎΠ΄ΠΈΠ½Π°ΠΊΠΎΠ²Ρ‹Ρ… стСпСнСй ΠΎΠ΄ΠΈΠ½Π°ΠΊΠΎΠ²Ρ‹Ρ… ΠΏΠ΅Ρ€Π΅ΠΌΠ΅Π½Π½Ρ‹Ρ… ΠΌΠΎΠ³ΡƒΡ‚ ΡΠ»Π°Π³Π°Ρ‚ΡŒΡΡ ΠΈΠ»ΠΈ Π²Ρ‹Ρ‡ΠΈΡ‚Π°Ρ‚ΡŒΡΡ.

    Π’Π°ΠΊ, сумма 2a 2 ΠΈ 3a 2 Ρ€Π°Π²Π½Π° 5a 2 .

    Π­Ρ‚ΠΎ Ρ‚Π°ΠΊ ΠΆΠ΅ ΠΎΡ‡Π΅Π²ΠΈΠ΄Π½ΠΎ, Ρ‡Ρ‚ΠΎ Ссли Π²Π·ΡΡ‚ΡŒ Π΄Π²Π° ΠΊΠ²Π°Π΄Ρ€Π°Ρ‚Π° Π°, ΠΈΠ»ΠΈ Ρ‚Ρ€ΠΈ ΠΊΠ²Π°Π΄Ρ€Π°Ρ‚Π° Π°, ΠΈΠ»ΠΈ ΠΏΡΡ‚ΡŒ ΠΊΠ²Π°Π΄Ρ€Π°Ρ‚ΠΎΠ² Π°.

    Но стСпСни Ρ€Π°Π·Π»ΠΈΡ‡Π½Ρ‹Ρ… ΠΏΠ΅Ρ€Π΅ΠΌΠ΅Π½Π½Ρ‹Ρ… ΠΈ Ρ€Π°Π·Π»ΠΈΡ‡Π½Ρ‹Π΅ стСпСни ΠΎΠ΄ΠΈΠ½Π°ΠΊΠΎΠ²Ρ‹Ρ… ΠΏΠ΅Ρ€Π΅ΠΌΠ΅Π½Π½Ρ‹Ρ… , Π΄ΠΎΠ»ΠΆΠ½Ρ‹ ΡΠ»Π°Π³Π°Ρ‚ΡŒΡΡ ΠΈΡ… слоТСниСм с ΠΈΡ… Π·Π½Π°ΠΊΠ°ΠΌΠΈ.

    Π’Π°ΠΊ, сумма a 2 ΠΈ a 3 Π΅ΡΡ‚ΡŒ сумма a 2 + a 3 .

    Π­Ρ‚ΠΎ ΠΎΡ‡Π΅Π²ΠΈΠ΄Π½ΠΎ, Ρ‡Ρ‚ΠΎ ΠΊΠ²Π°Π΄Ρ€Π°Ρ‚ числа a, ΠΈ ΠΊΡƒΠ± числа a, Π½Π΅ Ρ€Π°Π²Π½ΠΎ Π½ΠΈ ΡƒΠ΄Π²ΠΎΠ΅Π½Π½ΠΎΠΌΡƒ ΠΊΠ²Π°Π΄Ρ€Π°Ρ‚Ρƒ a, Π½ΠΎ ΡƒΠ΄Π²ΠΎΠ΅Π½Π½ΠΎΠΌΡƒ ΠΊΡƒΠ±Ρƒ a.

    Π‘ΡƒΠΌΠΌΠ° a 3 b n ΠΈ 3a 5 b 6 Π΅ΡΡ‚ΡŒ a 3 b n + 3a 5 b 6 .

    Π’Ρ‹Ρ‡ΠΈΡ‚Π°Π½ΠΈΠ΅ стСпСнСй проводится Ρ‚Π°ΠΊΠΈΠΌ ΠΆΠ΅ ΠΎΠ±Ρ€Π°Π·ΠΎΠΌ, Ρ‡Ρ‚ΠΎ ΠΈ слоТСниС, Π·Π° ΠΈΡΠΊΠ»ΡŽΡ‡Π΅Π½ΠΈΠ΅ΠΌ Ρ‚ΠΎΠ³ΠΎ, Ρ‡Ρ‚ΠΎ Π·Π½Π°ΠΊΠΈ Π²Ρ‹Ρ‡ΠΈΡ‚Π°Π΅ΠΌΡ‹Ρ… Π΄ΠΎΠ»ΠΆΠ½Ρ‹ соотвСтствСнно Π±Ρ‹Ρ‚ΡŒ ΠΈΠ·ΠΌΠ΅Π½Π΅Π½Ρ‹.

    Или:
    2a 4 — (-6a 4) = 8a 4
    3h 2 b 6 — 4h 2 b 6 = -h 2 b 6
    5(a — h) 6 — 2(a — h) 6 = 3(a — h) 6

    Π£ΠΌΠ½ΠΎΠΆΠ΅Π½ΠΈΠ΅ стСпСнСй

    Числа со стСпСнями ΠΌΠΎΠ³ΡƒΡ‚ Π±Ρ‹Ρ‚ΡŒ ΡƒΠΌΠ½ΠΎΠΆΠ΅Π½Ρ‹, ΠΊΠ°ΠΊ ΠΈ Π΄Ρ€ΡƒΠ³ΠΈΠ΅ Π²Π΅Π»ΠΈΡ‡ΠΈΠ½Ρ‹, ΠΏΡƒΡ‚Π΅ΠΌ написания ΠΈΡ… ΠΎΠ΄Π½ΠΎ Π·Π° Π΄Ρ€ΡƒΠ³ΠΈΠΌ, со Π·Π½Π°ΠΊΠΎΠΌ умноТСния ΠΈΠ»ΠΈ Π±Π΅Π· Π½Π΅Π³ΠΎ ΠΌΠ΅ΠΆΠ΄Ρƒ Π½ΠΈΠΌΠΈ.

    Π’Π°ΠΊ, Ρ€Π΅Π·ΡƒΠ»ΡŒΡ‚Π°Ρ‚ умноТСния a 3 Π½Π° b 2 Ρ€Π°Π²Π΅Π½ a 3 b 2 ΠΈΠ»ΠΈ aaabb.

    Или:
    x -3 β‹… a m = a m x -3
    3a 6 y 2 β‹… (-2x) = -6a 6 xy 2
    a 2 b 3 y 2 β‹… a 3 b 2 y = a 2 b 3 y 2 a 3 b 2 y

    Π Π΅Π·ΡƒΠ»ΡŒΡ‚Π°Ρ‚ Π² послСднСм ΠΏΡ€ΠΈΠΌΠ΅Ρ€Π΅ ΠΌΠΎΠΆΠ΅Ρ‚ Π±Ρ‹Ρ‚ΡŒ упорядочСн ΠΏΡƒΡ‚Ρ‘ΠΌ слоТСния ΠΎΠ΄ΠΈΠ½Π°ΠΊΠΎΠ²Ρ‹Ρ… ΠΏΠ΅Ρ€Π΅ΠΌΠ΅Π½Π½Ρ‹Ρ….
    Π’Ρ‹Ρ€Π°ΠΆΠ΅Π½ΠΈΠ΅ ΠΏΡ€ΠΈΠΌΠ΅Ρ‚ Π²ΠΈΠ΄: a 5 b 5 y 3 .

    Бравнивая нСсколько чисСл(ΠΏΠ΅Ρ€Π΅ΠΌΠ΅Π½Π½Ρ‹Ρ…) со стСпСнями, ΠΌΡ‹ ΠΌΠΎΠΆΠ΅ΠΌ ΡƒΠ²ΠΈΠ΄Π΅Ρ‚ΡŒ, Ρ‡Ρ‚ΠΎ Ссли Π»ΡŽΠ±Ρ‹Π΅ Π΄Π²Π° ΠΈΠ· Π½ΠΈΡ… ΡƒΠΌΠ½ΠΎΠΆΠ°ΡŽΡ‚ΡΡ, Ρ‚ΠΎ Ρ€Π΅Π·ΡƒΠ»ΡŒΡ‚Π°Ρ‚ — это число (пСрСмСнная) со ΡΡ‚Π΅ΠΏΠ΅Π½ΡŒΡŽ, Ρ€Π°Π²Π½ΠΎΠΉ суммС стСпСнСй слагаСмых.

    Π’Π°ΠΊ, a 2 .a 3 = aa.aaa = aaaaa = a 5 .

    Π—Π΄Π΅ΡΡŒ 5 — это ΡΡ‚Π΅ΠΏΠ΅Π½ΡŒ Ρ€Π΅Π·ΡƒΠ»ΡŒΡ‚Π°Ρ‚Π° умноТСния, равная 2 + 3, суммС стСпСнСй слагаСмых.

    Π’Π°ΠΊ, a n .a m = a m+n .

    Для a n , a бСрётся ΠΊΠ°ΠΊ ΠΌΠ½ΠΎΠΆΠΈΡ‚Π΅Π»ΡŒ ΡΡ‚ΠΎΠ»ΡŒΠΊΠΎ Ρ€Π°Π·, сколько Ρ€Π°Π²Π½Π° ΡΡ‚Π΅ΠΏΠ΅Π½ΡŒ n;

    И a m , бСрётся ΠΊΠ°ΠΊ ΠΌΠ½ΠΎΠΆΠΈΡ‚Π΅Π»ΡŒ ΡΡ‚ΠΎΠ»ΡŒΠΊΠΎ Ρ€Π°Π·, сколько Ρ€Π°Π²Π½Π° ΡΡ‚Π΅ΠΏΠ΅Π½ΡŒ m;

    ΠŸΠΎΡΡ‚ΠΎΠΌΡƒ, стСпСни с ΠΎΠ΄ΠΈΠ½Π°ΠΊΠΎΠ²Ρ‹ΠΌΠΈ основами ΠΌΠΎΠ³ΡƒΡ‚ Π±Ρ‹Ρ‚ΡŒ ΡƒΠΌΠ½ΠΎΠΆΠ΅Π½Ρ‹ ΠΏΡƒΡ‚Ρ‘ΠΌ слоТСния ΠΏΠΎΠΊΠ°Π·Π°Ρ‚Π΅Π»Π΅ΠΉ стСпСнСй.

    Вак, a 2 .a 6 = a 2+6 = a 8 . И x 3 .x 2 .x = x 3+2+1 = x 6 .

    Или:
    4a n β‹… 2a n = 8a 2n
    b 2 y 3 β‹… b 4 y = b 6 y 4
    (b + h — y) n β‹… (b + h — y) = (b + h — y) n+1

    Π£ΠΌΠ½ΠΎΠΆΡŒΡ‚Π΅ (x 3 + x 2 y + xy 2 + y 3) β‹… (x — y).
    ΠžΡ‚Π²Π΅Ρ‚: x 4 — y 4 .
    Π£ΠΌΠ½ΠΎΠΆΡŒΡ‚Π΅ (x 3 + x — 5) β‹… (2x 3 + x + 1).

    Π­Ρ‚ΠΎ ΠΏΡ€Π°Π²ΠΈΠ»ΠΎ справСдливо ΠΈ для чисСл, ΠΏΠΎΠΊΠ°Π·Π°Ρ‚Π΅Π»ΠΈ стСпСни ΠΊΠΎΡ‚ΠΎΡ€Ρ‹Ρ… — ΠΎΡ‚Ρ€ΠΈΡ†Π°Ρ‚Π΅Π»ΡŒΠ½Ρ‹Π΅ .

    1. Π’Π°ΠΊ, a -2 .a -3 = a -5 . Π­Ρ‚ΠΎ ΠΌΠΎΠΆΠ½ΠΎ Π·Π°ΠΏΠΈΡΠ°Ρ‚ΡŒ Π² Π²ΠΈΠ΄Π΅ (1/aa).(1/aaa) = 1/aaaaa.

    2. y -n .y -m = y -n-m .

    3. a -n .a m = a m-n .

    Если a + b ΡƒΠΌΠ½ΠΎΠΆΠ°ΡŽΡ‚ΡΡ Π½Π° a — b, Ρ€Π΅Π·ΡƒΠ»ΡŒΡ‚Π°Ρ‚ Π±ΡƒΠ΄Π΅Ρ‚ Ρ€Π°Π²Π΅Π½ a 2 — b 2: Ρ‚ΠΎ Π΅ΡΡ‚ΡŒ

    Π Π΅Π·ΡƒΠ»ΡŒΡ‚Π°Ρ‚ умноТСния суммы ΠΈΠ»ΠΈ Ρ€Π°Π·Π½ΠΈΡ†Ρ‹ Π΄Π²ΡƒΡ… чисСл Ρ€Π°Π²Π΅Π½ суммС ΠΈΠ»ΠΈ Ρ€Π°Π·Π½ΠΈΡ†Π΅ ΠΈΡ… ΠΊΠ²Π°Π΄Ρ€Π°Ρ‚ΠΎΠ².

    Если умноТаСтся сумма ΠΈ Ρ€Π°Π·Π½ΠΈΡ†Π° Π΄Π²ΡƒΡ… чисСл, Π²ΠΎΠ·Π²Π΅Π΄Ρ‘Π½Π½Ρ‹Ρ… Π² ΠΊΠ²Π°Π΄Ρ€Π°Ρ‚ , Ρ€Π΅Π·ΡƒΠ»ΡŒΡ‚Π°Ρ‚ Π±ΡƒΠ΄Π΅Ρ‚ Ρ€Π°Π²Π΅Π½ суммС ΠΈΠ»ΠΈ Ρ€Π°Π·Π½ΠΈΡ†Π΅ этих чисСл Π² Ρ‡Π΅Ρ‚Π²Ρ‘Ρ€Ρ‚ΠΎΠΉ стСпСни. 5}$. ΠžΡ‚Π²Π΅Ρ‚: $\frac{2x}{1}$ ΠΈΠ»ΠΈ 2x.

    3. Π£ΠΌΠ΅Π½ΡŒΡˆΠΈΡ‚Π΅ ΠΏΠΎΠΊΠ°Π·Π°Ρ‚Π΅Π»ΠΈ стСпСнСй a 2 /a 3 ΠΈ a -3 /a -4 ΠΈ ΠΏΡ€ΠΈΠ²Π΅Π΄ΠΈΡ‚Π΅ ΠΊ ΠΎΠ±Ρ‰Π΅ΠΌΡƒ Π·Π½Π°ΠΌΠ΅Π½Π°Ρ‚Π΅Π»ΡŽ.
    a 2 .a -4 Π΅ΡΡ‚ΡŒ a -2 ΠΏΠ΅Ρ€Π²Ρ‹ΠΉ Ρ‡ΠΈΡΠ»ΠΈΡ‚Π΅Π»ΡŒ.
    a 3 .a -3 Π΅ΡΡ‚ΡŒ a 0 = 1, Π²Ρ‚ΠΎΡ€ΠΎΠΉ Ρ‡ΠΈΡΠ»ΠΈΡ‚Π΅Π»ΡŒ.
    a 3 .a -4 Π΅ΡΡ‚ΡŒ a -1 , ΠΎΠ±Ρ‰ΠΈΠΉ Ρ‡ΠΈΡΠ»ΠΈΡ‚Π΅Π»ΡŒ.
    ПослС упрощСния: a -2 /a -1 ΠΈ 1/a -1 .

    4. Π£ΠΌΠ΅Π½ΡŒΡˆΠΈΡ‚Π΅ ΠΏΠΎΠΊΠ°Π·Π°Ρ‚Π΅Π»ΠΈ стСпСнСй 2a 4 /5a 3 ΠΈ 2 /a 4 ΠΈ ΠΏΡ€ΠΈΠ²Π΅Π΄ΠΈΡ‚Π΅ ΠΊ ΠΎΠ±Ρ‰Π΅ΠΌΡƒ Π·Π½Π°ΠΌΠ΅Π½Π°Ρ‚Π΅Π»ΡŽ.
    ΠžΡ‚Π²Π΅Ρ‚: 2a 3 /5a 7 ΠΈ 5a 5 /5a 7 ΠΈΠ»ΠΈ 2a 3 /5a 2 ΠΈ 5/5a 2 .

    5. Π£ΠΌΠ½ΠΎΠΆΡŒΡ‚Π΅ (a 3 + b)/b 4 Π½Π° (a — b)/3.

    6. Π£ΠΌΠ½ΠΎΠΆΡŒΡ‚Π΅ (a 5 + 1)/x 2 Π½Π° (b 2 — 1)/(x + a).

    7. Π£ΠΌΠ½ΠΎΠΆΡŒΡ‚Π΅ b 4 /a -2 Π½Π° h -3 /x ΠΈ a n /y -3 .

    8. Π Π°Π·Π΄Π΅Π»ΠΈΡ‚Π΅ a 4 /y 3 Π½Π° a 3 /y 2 . ΠžΡ‚Π²Π΅Ρ‚: a/y.

    9. Π Π°Π·Π΄Π΅Π»ΠΈΡ‚Π΅ (h 3 — 1)/d 4 Π½Π° (d n + 1)/h.

    Рассмотрим Ρ‚Π΅ΠΌΡƒ прСобразования Π²Ρ‹Ρ€Π°ΠΆΠ΅Π½ΠΈΠΉ со стСпСнями, Π½ΠΎ ΠΏΡ€Π΅ΠΆΠ΄Π΅ остановимся Π½Π° рядС ΠΏΡ€Π΅ΠΎΠ±Ρ€Π°Π·ΠΎΠ²Π°Π½ΠΈΠΉ, ΠΊΠΎΡ‚ΠΎΡ€Ρ‹Π΅ ΠΌΠΎΠΆΠ½ΠΎ ΠΏΡ€ΠΎΠ²ΠΎΠ΄ΠΈΡ‚ΡŒ с Π»ΡŽΠ±Ρ‹ΠΌΠΈ выраТСниями, Π² Ρ‚ΠΎΠΌ числС со стСпСнными. ΠœΡ‹ научимся Ρ€Π°ΡΠΊΡ€Ρ‹Π²Π°Ρ‚ΡŒ скобки, ΠΏΡ€ΠΈΠ²ΠΎΠ΄ΠΈΡ‚ΡŒ ΠΏΠΎΠ΄ΠΎΠ±Π½Ρ‹Π΅ слагаСмыС, Ρ€Π°Π±ΠΎΡ‚Π°Ρ‚ΡŒ с основаниСм ΠΈ ΠΏΠΎΠΊΠ°Π·Π°Ρ‚Π΅Π»Π΅ΠΌ стСпСни, ΠΈΡΠΏΠΎΠ»ΡŒΠ·ΠΎΠ²Π°Ρ‚ΡŒ свойства стСпСнСй.

    Yandex.RTB R-A-339285-1

    Π§Ρ‚ΠΎ ΠΏΡ€Π΅Π΄ΡΡ‚Π°Π²Π»ΡΡŽΡ‚ собой стСпСнныС выраТСния?

    Π’ школьном курсС ΠΌΠ°Π»ΠΎ ΠΊΡ‚ΠΎ ΠΈΡΠΏΠΎΠ»ΡŒΠ·ΡƒΠ΅Ρ‚ словосочСтаниС «стСпСнныС выраТСния», Π·Π°Ρ‚ΠΎ этот Ρ‚Π΅Ρ€ΠΌΠΈΠ½ постоянно встрСчаСтся Π² сборниках для ΠΏΠΎΠ΄Π³ΠΎΡ‚ΠΎΠ²ΠΊΠΈ ΠΊ Π•Π“Π­. Π’ Π±ΠΎΠ»ΡŒΡˆΠΈΠ½ΡΡ‚Π²Π° случаСв словосочСтаниСм ΠΎΠ±ΠΎΠ·Π½Π°Ρ‡Π°ΡŽΡ‚ΡΡ выраТСния, ΠΊΠΎΡ‚ΠΎΡ€Ρ‹Π΅ содСрТат Π² своих записях стСпСни. Π­Ρ‚ΠΎ ΠΌΡ‹ ΠΈ ΠΎΡ‚Ρ€Π°Π·ΠΈΠΌ Π² нашСм ΠΎΠΏΡ€Π΅Π΄Π΅Π»Π΅Π½ΠΈΠΈ.

    ΠžΠΏΡ€Π΅Π΄Π΅Π»Π΅Π½ΠΈΠ΅ 1

    Π‘Ρ‚Π΅ΠΏΠ΅Π½Π½ΠΎΠ΅ Π²Ρ‹Ρ€Π°ΠΆΠ΅Π½ΠΈΠ΅ – это Π²Ρ‹Ρ€Π°ΠΆΠ΅Π½ΠΈΠ΅, ΠΊΠΎΡ‚ΠΎΡ€ΠΎΠ΅ содСрТит стСпСни.

    ΠŸΡ€ΠΈΠ²Π΅Π΄Π΅ΠΌ нСсколько ΠΏΡ€ΠΈΠΌΠ΅Ρ€ΠΎΠ² стСпСнных Π²Ρ‹Ρ€Π°ΠΆΠ΅Π½ΠΈΠΉ, начиная со стСпСни с Π½Π°Ρ‚ΡƒΡ€Π°Π»ΡŒΠ½Ρ‹ΠΌ ΠΏΠΎΠΊΠ°Π·Π°Ρ‚Π΅Π»Π΅ΠΌ ΠΈ заканчивая ΡΡ‚Π΅ΠΏΠ΅Π½ΡŒΡŽ с Π΄Π΅ΠΉΡΡ‚Π²ΠΈΡ‚Π΅Π»ΡŒΠ½Ρ‹ΠΌ ΠΏΠΎΠΊΠ°Π·Π°Ρ‚Π΅Π»Π΅ΠΌ.

    Π‘Π°ΠΌΡ‹ΠΌΠΈ простыми стСпСнными выраТСниями ΠΌΠΎΠΆΠ½ΠΎ ΡΡ‡ΠΈΡ‚Π°Ρ‚ΡŒ стСпСни числа с Π½Π°Ρ‚ΡƒΡ€Π°Π»ΡŒΠ½Ρ‹ΠΌ ΠΏΠΎΠΊΠ°Π·Π°Ρ‚Π΅Π»Π΅ΠΌ: 3 2 , 7 5 + 1 , (2 + 1) 5 , (βˆ’ 0 , 1) 4 , 2 2 3 3 , 3 Β· a 2 βˆ’ a + a 2 , x 3 βˆ’ 1 , (a 2) 3 . А Ρ‚Π°ΠΊΠΆΠ΅ стСпСни с Π½ΡƒΠ»Π΅Π²Ρ‹ΠΌ ΠΏΠΎΠΊΠ°Π·Π°Ρ‚Π΅Π»Π΅ΠΌ: 5 0 , (a + 1) 0 , 3 + 5 2 βˆ’ 3 , 2 0 . И стСпСни с Ρ†Π΅Π»Ρ‹ΠΌΠΈ ΠΎΡ‚Ρ€ΠΈΡ†Π°Ρ‚Π΅Π»ΡŒΠ½Ρ‹ΠΌΠΈ стСпСнями: (0 , 5) 2 + (0 , 5) — 2 2 .

    Π§ΡƒΡ‚ΡŒ слоТнСС Ρ€Π°Π±ΠΎΡ‚Π°Ρ‚ΡŒ со ΡΡ‚Π΅ΠΏΠ΅Π½ΡŒΡŽ, ΠΈΠΌΠ΅ΡŽΡ‰Π΅ΠΉ Ρ€Π°Ρ†ΠΈΠΎΠ½Π°Π»ΡŒΠ½Ρ‹ΠΉ ΠΈ ΠΈΡ€Ρ€Π°Ρ†ΠΈΠΎΠ½Π°Π»ΡŒΠ½Ρ‹ΠΉ ΠΏΠΎΠΊΠ°Π·Π°Ρ‚Π΅Π»ΠΈ: 264 1 4 — 3 Β· 3 Β· 3 1 2 , 2 3 , 5 Β· 2 — 2 2 — 1 , 5 , 1 a 1 4 Β· a 1 2 — 2 Β· a — 1 6 Β· b 1 2 , x Ο€ Β· x 1 — Ο€ , 2 3 3 + 5 .

    Π’ качСствС показатСля ΠΌΠΎΠΆΠ΅Ρ‚ Π²Ρ‹ΡΡ‚ΡƒΠΏΠ°Ρ‚ΡŒ пСрСмСнная 3 x — 54 — 7 Β· 3 x — 58 ΠΈΠ»ΠΈ Π»ΠΎΠ³Π°Ρ€ΠΈΡ„ΠΌ x 2 Β· l g x βˆ’ 5 Β· x l g x .

    Π‘ вопросом ΠΎ Ρ‚ΠΎΠΌ, Ρ‡Ρ‚ΠΎ Ρ‚Π°ΠΊΠΎΠ΅ стСпСнныС выраТСния, ΠΌΡ‹ Ρ€Π°Π·ΠΎΠ±Ρ€Π°Π»ΠΈΡΡŒ. Π’Π΅ΠΏΠ΅Ρ€ΡŒ займСмся ΠΈΡ… ΠΏΡ€Π΅ΠΎΠ±Ρ€Π°Π·ΠΎΠ²Π°Π½ΠΈΠ΅ΠΌ.

    ΠžΡΠ½ΠΎΠ²Π½Ρ‹Π΅ Π²ΠΈΠ΄Ρ‹ ΠΏΡ€Π΅ΠΎΠ±Ρ€Π°Π·ΠΎΠ²Π°Π½ΠΈΠΉ стСпСнных Π²Ρ‹Ρ€Π°ΠΆΠ΅Π½ΠΈΠΉ

    Π’ ΠΏΠ΅Ρ€Π²ΡƒΡŽ ΠΎΡ‡Π΅Ρ€Π΅Π΄ΡŒ ΠΌΡ‹ рассмотрим основныС тоТдСствСнныС прСобразования Π²Ρ‹Ρ€Π°ΠΆΠ΅Π½ΠΈΠΉ, ΠΊΠΎΡ‚ΠΎΡ€Ρ‹Π΅ ΠΌΠΎΠΆΠ½ΠΎ Π²Ρ‹ΠΏΠΎΠ»Π½ΡΡ‚ΡŒ со стСпСнными выраТСниями.

    ΠŸΡ€ΠΈΠΌΠ΅Ρ€ 1

    ВычислитС Π·Π½Π°Ρ‡Π΅Π½ΠΈΠ΅ стСпСнного выраТСния 2 3 Β· (4 2 βˆ’ 12) .

    РСшСниС

    ВсС прСобразования ΠΌΡ‹ Π±ΡƒΠ΄Π΅ΠΌ ΠΏΡ€ΠΎΠ²ΠΎΠ΄ΠΈΡ‚ΡŒ с соблюдСниСм порядка выполнСния дСйствий. Π’ Π΄Π°Π½Π½ΠΎΠΌ случаС Π½Π°Ρ‡Π½Π΅ΠΌ ΠΌΡ‹ с выполнСния дСйствий Π² скобках: Π·Π°ΠΌΠ΅Π½ΠΈΠΌ ΡΡ‚Π΅ΠΏΠ΅Π½ΡŒ Π½Π° Ρ†ΠΈΡ„Ρ€ΠΎΠ²ΠΎΠ΅ Π·Π½Π°Ρ‡Π΅Π½ΠΈΠ΅ ΠΈ вычислим Ρ€Π°Π·Π½ΠΎΡΡ‚ΡŒ Π΄Π²ΡƒΡ… чисСл. ИмССм 2 3 Β· (4 2 βˆ’ 12) = 2 3 Β· (16 βˆ’ 12) = 2 3 Β· 4 .

    Нам остаСтся Π·Π°ΠΌΠ΅Π½ΠΈΡ‚ΡŒ ΡΡ‚Π΅ΠΏΠ΅Π½ΡŒ 2 3 Π΅Π΅ Π·Π½Π°Ρ‡Π΅Π½ΠΈΠ΅ΠΌ 8 ΠΈ Π²Ρ‹Ρ‡ΠΈΡΠ»ΠΈΡ‚ΡŒ ΠΏΡ€ΠΎΠΈΠ·Π²Π΅Π΄Π΅Π½ΠΈΠ΅ 8 Β· 4 = 32 . Π’ΠΎΡ‚ наш ΠΎΡ‚Π²Π΅Ρ‚.

    ΠžΡ‚Π²Π΅Ρ‚: 2 3 Β· (4 2 βˆ’ 12) = 32 .

    ΠŸΡ€ΠΈΠΌΠ΅Ρ€ 2

    УпроститС Π²Ρ‹Ρ€Π°ΠΆΠ΅Π½ΠΈΠ΅ со стСпСнями 3 Β· a 4 Β· b βˆ’ 7 βˆ’ 1 + 2 Β· a 4 Β· b βˆ’ 7 .

    РСшСниС

    Π”Π°Π½Π½ΠΎΠ΅ Π½Π°ΠΌ Π² условии Π·Π°Π΄Π°Ρ‡ΠΈ Π²Ρ‹Ρ€Π°ΠΆΠ΅Π½ΠΈΠ΅ содСрТит ΠΏΠΎΠ΄ΠΎΠ±Π½Ρ‹Π΅ слагаСмыС, ΠΊΠΎΡ‚ΠΎΡ€Ρ‹Π΅ ΠΌΡ‹ ΠΌΠΎΠΆΠ΅ΠΌ привСсти: 3 Β· a 4 Β· b βˆ’ 7 βˆ’ 1 + 2 Β· a 4 Β· b βˆ’ 7 = 5 Β· a 4 Β· b βˆ’ 7 βˆ’ 1 .

    ΠžΡ‚Π²Π΅Ρ‚: 3 Β· a 4 Β· b βˆ’ 7 βˆ’ 1 + 2 Β· a 4 Β· b βˆ’ 7 = 5 Β· a 4 Β· b βˆ’ 7 βˆ’ 1 .

    ΠŸΡ€ΠΈΠΌΠ΅Ρ€ 3

    ΠŸΡ€Π΅Π΄ΡΡ‚Π°Π²ΡŒΡ‚Π΅ Π²Ρ‹Ρ€Π°ΠΆΠ΅Π½ΠΈΠ΅ со стСпСнями 9 — b 3 Β· Ο€ — 1 2 Π² Π²ΠΈΠ΄Π΅ произвСдСния.

    РСшСниС

    ΠŸΡ€Π΅Π΄ΡΡ‚Π°Π²ΠΈΠΌ число 9 ΠΊΠ°ΠΊ ΡΡ‚Π΅ΠΏΠ΅Π½ΡŒ 3 2 ΠΈ ΠΏΡ€ΠΈΠΌΠ΅Π½ΠΈΠΌ Ρ„ΠΎΡ€ΠΌΡƒΠ»Ρƒ сокращСнного умноТСния:

    9 — b 3 Β· Ο€ — 1 2 = 3 2 — b 3 Β· Ο€ — 1 2 = = 3 — b 3 Β· Ο€ — 1 3 + b 3 Β· Ο€ — 1

    ΠžΡ‚Π²Π΅Ρ‚: 9 — b 3 Β· Ο€ — 1 2 = 3 — b 3 Β· Ο€ — 1 3 + b 3 Β· Ο€ — 1 .

    А Ρ‚Π΅ΠΏΠ΅Ρ€ΡŒ ΠΏΠ΅Ρ€Π΅ΠΉΠ΄Π΅ΠΌ ΠΊ Ρ€Π°Π·Π±ΠΎΡ€Ρƒ тоТдСствСнных ΠΏΡ€Π΅ΠΎΠ±Ρ€Π°Π·ΠΎΠ²Π°Π½ΠΈΠΉ, ΠΊΠΎΡ‚ΠΎΡ€Ρ‹Π΅ ΠΌΠΎΠ³ΡƒΡ‚ ΠΏΡ€ΠΈΠΌΠ΅Π½ΡΡ‚ΡŒΡΡ ΠΈΠΌΠ΅Π½Π½ΠΎ Π² ΠΎΡ‚Π½ΠΎΡˆΠ΅Π½ΠΈΠΈ стСпСнных Π²Ρ‹Ρ€Π°ΠΆΠ΅Π½ΠΈΠΉ.

    Π Π°Π±ΠΎΡ‚Π° с основаниСм ΠΈ ΠΏΠΎΠΊΠ°Π·Π°Ρ‚Π΅Π»Π΅ΠΌ стСпСни

    Π‘Ρ‚Π΅ΠΏΠ΅Π½ΡŒ Π² основании ΠΈΠ»ΠΈ ΠΏΠΎΠΊΠ°Π·Π°Ρ‚Π΅Π»Π΅ ΠΌΠΎΠΆΠ΅Ρ‚ ΠΈΠΌΠ΅Ρ‚ΡŒ ΠΈ числа, ΠΈ ΠΏΠ΅Ρ€Π΅ΠΌΠ΅Π½Π½Ρ‹Π΅, ΠΈ Π½Π΅ΠΊΠΎΡ‚ΠΎΡ€Ρ‹Π΅ выраТСния. НапримСр, (2 + 0 , 3 Β· 7) 5 βˆ’ 3 , 7 ΠΈ . Π Π°Π±ΠΎΡ‚Π°Ρ‚ΡŒ с Ρ‚Π°ΠΊΠΈΠΌΠΈ записями слоТно. Намного ΠΏΡ€ΠΎΡ‰Π΅ Π·Π°ΠΌΠ΅Π½ΠΈΡ‚ΡŒ Π²Ρ‹Ρ€Π°ΠΆΠ΅Π½ΠΈΠ΅ Π² основании стСпСни ΠΈΠ»ΠΈ Π²Ρ‹Ρ€Π°ΠΆΠ΅Π½ΠΈΠ΅ Π² ΠΏΠΎΠΊΠ°Π·Π°Ρ‚Π΅Π»Π΅ тоТдСствСнно Ρ€Π°Π²Π½Ρ‹ΠΌ Π²Ρ‹Ρ€Π°ΠΆΠ΅Π½ΠΈΠ΅ΠΌ.

    ΠŸΡ€ΠΎΠ²ΠΎΠ΄ΡΡ‚ΡΡ прСобразования стСпСни ΠΈ показатСля ΠΏΠΎ извСстным Π½Π°ΠΌ ΠΏΡ€Π°Π²ΠΈΠ»Π°ΠΌ ΠΎΡ‚Π΄Π΅Π»ΡŒΠ½ΠΎ Π΄Ρ€ΡƒΠ³ ΠΎΡ‚ Π΄Ρ€ΡƒΠ³Π°. Π‘Π°ΠΌΠΎΠ΅ Π³Π»Π°Π²Π½ΠΎΠ΅, Ρ‡Ρ‚ΠΎΠ±Ρ‹ Π² Ρ€Π΅Π·ΡƒΠ»ΡŒΡ‚Π°Ρ‚Π΅ ΠΏΡ€Π΅ΠΎΠ±Ρ€Π°Π·ΠΎΠ²Π°Π½ΠΈΠΉ ΠΏΠΎΠ»ΡƒΡ‡ΠΈΠ»ΠΎΡΡŒ Π²Ρ‹Ρ€Π°ΠΆΠ΅Π½ΠΈΠ΅, тоТдСствСнноС исходному.

    ЦСль ΠΏΡ€Π΅ΠΎΠ±Ρ€Π°Π·ΠΎΠ²Π°Π½ΠΈΠΉ – ΡƒΠΏΡ€ΠΎΡΡ‚ΠΈΡ‚ΡŒ исходноС Π²Ρ‹Ρ€Π°ΠΆΠ΅Π½ΠΈΠ΅ ΠΈΠ»ΠΈ ΠΏΠΎΠ»ΡƒΡ‡ΠΈΡ‚ΡŒ Ρ€Π΅ΡˆΠ΅Π½ΠΈΠ΅ Π·Π°Π΄Π°Ρ‡ΠΈ. НапримСр, Π² ΠΏΡ€ΠΈΠΌΠ΅Ρ€Π΅, ΠΊΠΎΡ‚ΠΎΡ€Ρ‹ΠΉ ΠΌΡ‹ ΠΏΡ€ΠΈΠ²Π΅Π»ΠΈ Π²Ρ‹ΡˆΠ΅, (2 + 0 , 3 Β· 7) 5 βˆ’ 3 , 7 ΠΌΠΎΠΆΠ½ΠΎ Π²Ρ‹ΠΏΠΎΠ»Π½ΠΈΡ‚ΡŒ дСйствия для ΠΏΠ΅Ρ€Π΅Ρ…ΠΎΠ΄Π° ΠΊ стСпСни 4 , 1 1 , 3 . Раскрыв скобки, ΠΌΡ‹ ΠΌΠΎΠΆΠ΅ΠΌ привСсти ΠΏΠΎΠ΄ΠΎΠ±Π½Ρ‹Π΅ слагаСмыС Π² основании стСпСни (a Β· (a + 1) βˆ’ a 2) 2 Β· (x + 1) ΠΈ ΠΏΠΎΠ»ΡƒΡ‡ΠΈΡ‚ΡŒ стСпСнноС Π²Ρ‹Ρ€Π°ΠΆΠ΅Π½ΠΈΠ΅ Π±ΠΎΠ»Π΅Π΅ простого Π²ΠΈΠ΄Π° a 2 Β· (x + 1) .

    ИспользованиС свойств стСпСнСй

    Бвойства стСпСнСй, записанныС Π² Π²ΠΈΠ΄Π΅ равСнств, ΡΠ²Π»ΡΡŽΡ‚ΡΡ ΠΎΠ΄Π½ΠΈΠΌ ΠΈΠ· Π³Π»Π°Π²Π½Ρ‹Ρ… инструмСнтов прСобразования Π²Ρ‹Ρ€Π°ΠΆΠ΅Π½ΠΈΠΉ со стСпСнями. ΠŸΡ€ΠΈΠ²Π΅Π΄Π΅ΠΌ здСсь основныС ΠΈΠ· Π½ΠΈΡ…, учитывая, Ρ‡Ρ‚ΠΎ a ΠΈ b – это Π»ΡŽΠ±Ρ‹Π΅ ΠΏΠΎΠ»ΠΎΠΆΠΈΡ‚Π΅Π»ΡŒΠ½Ρ‹Π΅ числа, Π° r ΠΈ s — ΠΏΡ€ΠΎΠΈΠ·Π²ΠΎΠ»ΡŒΠ½Ρ‹Π΅ Π΄Π΅ΠΉΡΡ‚Π²ΠΈΡ‚Π΅Π»ΡŒΠ½Ρ‹Π΅ числа:

    ΠžΠΏΡ€Π΅Π΄Π΅Π»Π΅Π½ΠΈΠ΅ 2

    • a r Β· a s = a r + s ;
    • a r: a s = a r βˆ’ s ;
    • (a Β· b) r = a r Β· b r ;
    • (a: b) r = a r: b r ;
    • (a r) s = a r Β· s .

    Π’ Ρ‚Π΅Ρ… случаях, ΠΊΠΎΠ³Π΄Π° ΠΌΡ‹ ΠΈΠΌΠ΅Π΅ΠΌ Π΄Π΅Π»ΠΎ с Π½Π°Ρ‚ΡƒΡ€Π°Π»ΡŒΠ½Ρ‹ΠΌΠΈ, Ρ†Π΅Π»Ρ‹ΠΌΠΈ, ΠΏΠΎΠ»ΠΎΠΆΠΈΡ‚Π΅Π»ΡŒΠ½Ρ‹ΠΌΠΈ показатСлями стСпСни, ограничСния Π½Π° числа a ΠΈ b ΠΌΠΎΠ³ΡƒΡ‚ Π±Ρ‹Ρ‚ΡŒ Π³ΠΎΡ€Π°Π·Π΄ΠΎ ΠΌΠ΅Π½Π΅Π΅ строгими. Π’Π°ΠΊ, Π½Π°ΠΏΡ€ΠΈΠΌΠ΅Ρ€, Ссли Ρ€Π°ΡΡΠΌΠΎΡ‚Ρ€Π΅Ρ‚ΡŒ равСнство a m Β· a n = a m + n , Π³Π΄Π΅ m ΠΈ n – Π½Π°Ρ‚ΡƒΡ€Π°Π»ΡŒΠ½Ρ‹Π΅ числа, Ρ‚ΠΎ ΠΎΠ½ΠΎ Π±ΡƒΠ΄Π΅Ρ‚ Π²Π΅Ρ€Π½ΠΎ для Π»ΡŽΠ±Ρ‹Ρ… Π·Π½Π°Ρ‡Π΅Π½ΠΈΠΉ a , ΠΊΠ°ΠΊ ΠΏΠΎΠ»ΠΎΠΆΠΈΡ‚Π΅Π»ΡŒΠ½Ρ‹Ρ…, Ρ‚Π°ΠΊ ΠΈ ΠΎΡ‚Ρ€ΠΈΡ†Π°Ρ‚Π΅Π»ΡŒΠ½Ρ‹Ρ…, Π° Ρ‚Π°ΠΊΠΆΠ΅ для a = 0 .

    ΠŸΡ€ΠΈΠΌΠ΅Π½ΡΡ‚ΡŒ свойства стСпСнСй Π±Π΅Π· ΠΎΠ³Ρ€Π°Π½ΠΈΡ‡Π΅Π½ΠΈΠΉ ΠΌΠΎΠΆΠ½ΠΎ Π² Ρ‚Π΅Ρ… случаях, ΠΊΠΎΠ³Π΄Π° основания стСпСнСй ΠΏΠΎΠ»ΠΎΠΆΠΈΡ‚Π΅Π»ΡŒΠ½Ρ‹Π΅ ΠΈΠ»ΠΈ содСрТат ΠΏΠ΅Ρ€Π΅ΠΌΠ΅Π½Π½Ρ‹Π΅, ΠΎΠ±Π»Π°ΡΡ‚ΡŒ допустимых Π·Π½Π°Ρ‡Π΅Π½ΠΈΠΉ ΠΊΠΎΡ‚ΠΎΡ€Ρ‹Ρ… Ρ‚Π°ΠΊΠΎΠ²Π°, Ρ‡Ρ‚ΠΎ Π½Π° Π½Π΅ΠΉ основания ΠΏΡ€ΠΈΠ½ΠΈΠΌΠ°ΡŽΡ‚ лишь ΠΏΠΎΠ»ΠΎΠΆΠΈΡ‚Π΅Π»ΡŒΠ½Ρ‹Π΅ значСния. ЀактичСски, Π² Ρ€Π°ΠΌΠΊΠ°Ρ… школьной ΠΏΡ€ΠΎΠ³Ρ€Π°ΠΌΠΌΡ‹ ΠΏΠΎ ΠΌΠ°Ρ‚Π΅ΠΌΠ°Ρ‚ΠΈΠΊΠ΅ Π·Π°Π΄Π°Ρ‡Π΅ΠΉ учащСгося являСтся Π²Ρ‹Π±ΠΎΡ€ подходящСго свойства ΠΈ ΠΏΡ€Π°Π²ΠΈΠ»ΡŒΠ½ΠΎΠ΅ Π΅Π³ΠΎ ΠΏΡ€ΠΈΠΌΠ΅Π½Π΅Π½ΠΈΠ΅.

    ΠŸΡ€ΠΈ ΠΏΠΎΠ΄Π³ΠΎΡ‚ΠΎΠ²ΠΊΠ΅ ΠΊ ΠΏΠΎΡΡ‚ΡƒΠΏΠ»Π΅Π½ΠΈΡŽ Π² Π’ΡƒΠ·Ρ‹ ΠΌΠΎΠ³ΡƒΡ‚ Π²ΡΡ‚Ρ€Π΅Ρ‡Π°Ρ‚ΡŒΡΡ Π·Π°Π΄Π°Ρ‡ΠΈ, Π² ΠΊΠΎΡ‚ΠΎΡ€Ρ‹Ρ… Π½Π΅Π°ΠΊΠΊΡƒΡ€Π°Ρ‚Π½ΠΎΠ΅ ΠΏΡ€ΠΈΠΌΠ΅Π½Π΅Π½ΠΈΠ΅ свойств Π±ΡƒΠ΄Π΅Ρ‚ ΠΏΡ€ΠΈΠ²ΠΎΠ΄ΠΈΡ‚ΡŒ ΠΊ ΡΡƒΠΆΠ΅Π½ΠΈΡŽ ΠžΠ”Π— ΠΈ Π΄Ρ€ΡƒΠ³ΠΈΠΌ слоТностям с Ρ€Π΅ΡˆΠ΅Π½ΠΈΠ΅ΠΌ. Π’ Π΄Π°Π½Π½ΠΎΠΌ Ρ€Π°Π·Π΄Π΅Π»Π΅ ΠΌΡ‹ Ρ€Π°Π·Π±Π΅Ρ€Π΅ΠΌ всСго Π΄Π²Π° Ρ‚Π°ΠΊΠΈΡ… случая. Π‘ΠΎΠ»ΡŒΡˆΠ΅ ΠΈΠ½Ρ„ΠΎΡ€ΠΌΠ°Ρ†ΠΈΠΈ ΠΏΠΎ вопросу ΠΌΠΎΠΆΠ½ΠΎ Π½Π°ΠΉΡ‚ΠΈ Π² Ρ‚Π΅ΠΌΠ΅ Β«ΠŸΡ€Π΅ΠΎΠ±Ρ€Π°Π·ΠΎΠ²Π°Π½ΠΈΠ΅ Π²Ρ‹Ρ€Π°ΠΆΠ΅Π½ΠΈΠΉ с использованиСм свойств стСпСнСй».

    ΠŸΡ€ΠΈΠΌΠ΅Ρ€ 4

    ΠŸΡ€Π΅Π΄ΡΡ‚Π°Π²ΡŒΡ‚Π΅ Π²Ρ‹Ρ€Π°ΠΆΠ΅Π½ΠΈΠ΅ a 2 , 5 Β· (a 2) βˆ’ 3: a βˆ’ 5 , 5 Π² Π²ΠΈΠ΄Π΅ стСпСни с основаниСм a .

    РСшСниС

    Для Π½Π°Ρ‡Π°Π»Π° ΠΈΡΠΏΠΎΠ»ΡŒΠ·ΡƒΠ΅ΠΌ свойство возвСдСния Π² ΡΡ‚Π΅ΠΏΠ΅Π½ΡŒ ΠΈ ΠΏΡ€Π΅ΠΎΠ±Ρ€Π°Π·ΡƒΠ΅ΠΌ ΠΏΠΎ Π½Π΅ΠΌΡƒ Π²Ρ‚ΠΎΡ€ΠΎΠΉ ΠΌΠ½ΠΎΠΆΠΈΡ‚Π΅Π»ΡŒ (a 2) βˆ’ 3 . Π—Π°Ρ‚Π΅ΠΌ ΠΈΡΠΏΠΎΠ»ΡŒΠ·ΡƒΠ΅ΠΌ свойства умноТСния ΠΈ дСлСния стСпСнСй с ΠΎΠ΄ΠΈΠ½Π°ΠΊΠΎΠ²Ρ‹ΠΌ основаниСм:

    a 2 , 5 Β· a βˆ’ 6: a βˆ’ 5 , 5 = a 2 , 5 βˆ’ 6: a βˆ’ 5 , 5 = a βˆ’ 3 , 5: a βˆ’ 5 , 5 = a βˆ’ 3 , 5 βˆ’ (βˆ’ 5 , 5) = a 2 .

    ΠžΡ‚Π²Π΅Ρ‚: a 2 , 5 Β· (a 2) βˆ’ 3: a βˆ’ 5 , 5 = a 2 .

    ΠŸΡ€Π΅ΠΎΠ±Ρ€Π°Π·ΠΎΠ²Π°Π½ΠΈΠ΅ стСпСнных Π²Ρ‹Ρ€Π°ΠΆΠ΅Π½ΠΈΠΉ согласно свойству стСпСнСй ΠΌΠΎΠΆΠ΅Ρ‚ ΠΏΡ€ΠΎΠΈΠ·Π²ΠΎΠ΄ΠΈΡ‚ΡŒΡΡ ΠΊΠ°ΠΊ слСва Π½Π°ΠΏΡ€Π°Π²ΠΎ, Ρ‚Π°ΠΊ ΠΈ Π² ΠΎΠ±Ρ€Π°Ρ‚Π½ΠΎΠΌ Π½Π°ΠΏΡ€Π°Π²Π»Π΅Π½ΠΈΠΈ.

    ΠŸΡ€ΠΈΠΌΠ΅Ρ€ 5

    Найти Π·Π½Π°Ρ‡Π΅Π½ΠΈΠ΅ стСпСнного выраТСния 3 1 3 Β· 7 1 3 Β· 21 2 3 .

    РСшСниС

    Если ΠΌΡ‹ ΠΏΡ€ΠΈΠΌΠ΅Π½ΠΈΠΌ равСнство (a Β· b) r = a r Β· b r , справа Π½Π°Π»Π΅Π²ΠΎ, Ρ‚ΠΎ ΠΏΠΎΠ»ΡƒΡ‡ΠΈΠΌ ΠΏΡ€ΠΎΠΈΠ·Π²Π΅Π΄Π΅Π½ΠΈΠ΅ Π²ΠΈΠ΄Π° 3 Β· 7 1 3 Β· 21 2 3 ΠΈ дальшС 21 1 3 Β· 21 2 3 . Π‘Π»ΠΎΠΆΠΈΠΌ ΠΏΠΎΠΊΠ°Π·Π°Ρ‚Π΅Π»ΠΈ ΠΏΡ€ΠΈ ΡƒΠΌΠ½ΠΎΠΆΠ΅Π½ΠΈΠΈ стСпСнСй с ΠΎΠ΄ΠΈΠ½Π°ΠΊΠΎΠ²Ρ‹ΠΌΠΈ основаниями: 21 1 3 Β· 21 2 3 = 21 1 3 + 2 3 = 21 1 = 21 .

    Π•ΡΡ‚ΡŒ Π΅Ρ‰Π΅ ΠΎΠ΄ΠΈΠ½ способ провСсти прСобразования:

    3 1 3 Β· 7 1 3 Β· 21 2 3 = 3 1 3 Β· 7 1 3 Β· (3 Β· 7) 2 3 = 3 1 3 Β· 7 1 3 Β· 3 2 3 Β· 7 2 3 = = 3 1 3 Β· 3 2 3 Β· 7 1 3 Β· 7 2 3 = 3 1 3 + 2 3 Β· 7 1 3 + 2 3 = 3 1 Β· 7 1 = 21

    ΠžΡ‚Π²Π΅Ρ‚: 3 1 3 Β· 7 1 3 Β· 21 2 3 = 3 1 Β· 7 1 = 21

    ΠŸΡ€ΠΈΠΌΠ΅Ρ€ 6

    Π”Π°Π½ΠΎ стСпСнноС Π²Ρ‹Ρ€Π°ΠΆΠ΅Π½ΠΈΠ΅ a 1 , 5 βˆ’ a 0 , 5 βˆ’ 6 , Π²Π²Π΅Π΄ΠΈΡ‚Π΅ Π½ΠΎΠ²ΡƒΡŽ ΠΏΠ΅Ρ€Π΅ΠΌΠ΅Π½Π½ΡƒΡŽ t = a 0 , 5 .

    РСшСниС

    ΠŸΡ€Π΅Π΄ΡΡ‚Π°Π²ΠΈΠΌ ΡΡ‚Π΅ΠΏΠ΅Π½ΡŒ a 1 , 5 ΠΊΠ°ΠΊ a 0 , 5 Β· 3 . Π˜ΡΠΏΠΎΠ»ΡŒΠ·ΡƒΠ΅ΠΌ свойство стСпСни Π² стСпСни (a r) s = a r Β· s справа Π½Π°Π»Π΅Π²ΠΎ ΠΈ ΠΏΠΎΠ»ΡƒΡ‡ΠΈΠΌ (a 0 , 5) 3: a 1 , 5 βˆ’ a 0 , 5 βˆ’ 6 = (a 0 , 5) 3 βˆ’ a 0 , 5 βˆ’ 6 . Π’ ΠΏΠΎΠ»ΡƒΡ‡Π΅Π½Π½ΠΎΠ΅ Π²Ρ‹Ρ€Π°ΠΆΠ΅Π½ΠΈΠ΅ ΠΌΠΎΠΆΠ½ΠΎ Π±Π΅Π· ΠΏΡ€ΠΎΠ±Π»Π΅ΠΌ Π²Π²ΠΎΠ΄ΠΈΡ‚ΡŒ Π½ΠΎΠ²ΡƒΡŽ ΠΏΠ΅Ρ€Π΅ΠΌΠ΅Π½Π½ΡƒΡŽ t = a 0 , 5 : ΠΏΠΎΠ»ΡƒΡ‡Π°Π΅ΠΌ t 3 βˆ’ t βˆ’ 6 .

    ΠžΡ‚Π²Π΅Ρ‚: t 3 βˆ’ t βˆ’ 6 .

    ΠŸΡ€Π΅ΠΎΠ±Ρ€Π°Π·ΠΎΠ²Π°Π½ΠΈΠ΅ Π΄Ρ€ΠΎΠ±Π΅ΠΉ, содСрТащих стСпСни

    ΠžΠ±Ρ‹Ρ‡Π½ΠΎ ΠΌΡ‹ ΠΈΠΌΠ΅Π΅ΠΌ Π΄Π΅Π»ΠΎ с двумя Π²Π°Ρ€ΠΈΠ°Π½Ρ‚Π°ΠΌΠΈ стСпСнных Π²Ρ‹Ρ€Π°ΠΆΠ΅Π½ΠΈΠΉ с дробями: Π²Ρ‹Ρ€Π°ΠΆΠ΅Π½ΠΈΠ΅ прСдставляСт собой Π΄Ρ€ΠΎΠ±ΡŒ со ΡΡ‚Π΅ΠΏΠ΅Π½ΡŒΡŽ ΠΈΠ»ΠΈ содСрТит Ρ‚Π°ΠΊΡƒΡŽ Π΄Ρ€ΠΎΠ±ΡŒ. К Ρ‚Π°ΠΊΠΈΠΌ выраТСниям ΠΏΡ€ΠΈΠΌΠ΅Π½ΠΈΠΌΡ‹ всС основныС прСобразования Π΄Ρ€ΠΎΠ±Π΅ΠΉ Π±Π΅Π· ΠΎΠ³Ρ€Π°Π½ΠΈΡ‡Π΅Π½ΠΈΠΉ. Π˜Ρ… ΠΌΠΎΠΆΠ½ΠΎ ΡΠΎΠΊΡ€Π°Ρ‰Π°Ρ‚ΡŒ, ΠΏΡ€ΠΈΠ²ΠΎΠ΄ΠΈΡ‚ΡŒ ΠΊ Π½ΠΎΠ²ΠΎΠΌΡƒ Π·Π½Π°ΠΌΠ΅Π½Π°Ρ‚Π΅Π»ΡŽ, Ρ€Π°Π±ΠΎΡ‚Π°Ρ‚ΡŒ ΠΎΡ‚Π΄Π΅Π»ΡŒΠ½ΠΎ с числитСлСм ΠΈ Π·Π½Π°ΠΌΠ΅Π½Π°Ρ‚Π΅Π»Π΅ΠΌ. ΠŸΡ€ΠΎΠΈΠ»Π»ΡŽΡΡ‚Ρ€ΠΈΡ€ΡƒΠ΅ΠΌ это ΠΏΡ€ΠΈΠΌΠ΅Ρ€Π°ΠΌΠΈ.

    ΠŸΡ€ΠΈΠΌΠ΅Ρ€ 7

    Π£ΠΏΡ€ΠΎΡΡ‚ΠΈΡ‚ΡŒ стСпСнноС Π²Ρ‹Ρ€Π°ΠΆΠ΅Π½ΠΈΠ΅ 3 Β· 5 2 3 Β· 5 1 3 — 5 — 2 3 1 + 2 Β· x 2 — 3 — 3 Β· x 2 .

    РСшСниС

    ΠœΡ‹ ΠΈΠΌΠ΅Π΅ΠΌ Π΄Π΅Π»ΠΎ с Π΄Ρ€ΠΎΠ±ΡŒΡŽ, поэтому ΠΏΡ€ΠΎΠ²Π΅Π΄Π΅ΠΌ прСобразования ΠΈ Π² числитСлС, ΠΈ Π² Π·Π½Π°ΠΌΠ΅Π½Π°Ρ‚Π΅Π»Π΅:

    3 Β· 5 2 3 Β· 5 1 3 — 5 — 2 3 1 + 2 Β· x 2 — 3 — 3 Β· x 2 = 3 Β· 5 2 3 Β· 5 1 3 — 3 Β· 5 2 3 Β· 5 — 2 3 — 2 — x 2 = = 3 Β· 5 2 3 + 1 3 — 3 Β· 5 2 3 + — 2 3 — 2 — x 2 = 3 Β· 5 1 — 3 Β· 5 0 — 2 — x 2

    ΠŸΠΎΠΌΠ΅ΡΡ‚ΠΈΠΌ минус ΠΏΠ΅Ρ€Π΅Π΄ Π΄Ρ€ΠΎΠ±ΡŒΡŽ для Ρ‚ΠΎΠ³ΠΎ, Ρ‡Ρ‚ΠΎΠ±Ρ‹ ΠΈΠ·ΠΌΠ΅Π½ΠΈΡ‚ΡŒ Π·Π½Π°ΠΊ знамСнатСля: 12 — 2 — x 2 = — 12 2 + x 2

    ΠžΡ‚Π²Π΅Ρ‚: 3 Β· 5 2 3 Β· 5 1 3 — 5 — 2 3 1 + 2 Β· x 2 — 3 — 3 Β· x 2 = — 12 2 + x 2

    Π”Ρ€ΠΎΠ±ΠΈ, содСрТащиС стСпСни, приводятся ΠΊ Π½ΠΎΠ²ΠΎΠΌΡƒ Π·Π½Π°ΠΌΠ΅Π½Π°Ρ‚Π΅Π»ΡŽ Ρ‚ΠΎΡ‡Π½ΠΎ Ρ‚Π°ΠΊΠΆΠ΅, ΠΊΠ°ΠΊ ΠΈ Ρ€Π°Ρ†ΠΈΠΎΠ½Π°Π»ΡŒΠ½Ρ‹Π΅ Π΄Ρ€ΠΎΠ±ΠΈ. Для этого Π½Π΅ΠΎΠ±Ρ…ΠΎΠ΄ΠΈΠΌΠΎ Π½Π°ΠΉΡ‚ΠΈ Π΄ΠΎΠΏΠΎΠ»Π½ΠΈΡ‚Π΅Π»ΡŒΠ½Ρ‹ΠΉ ΠΌΠ½ΠΎΠΆΠΈΡ‚Π΅Π»ΡŒ ΠΈ ΡƒΠΌΠ½ΠΎΠΆΠΈΡ‚ΡŒ Π½Π° Π½Π΅Π³ΠΎ Ρ‡ΠΈΡΠ»ΠΈΡ‚Π΅Π»ΡŒ ΠΈ Π·Π½Π°ΠΌΠ΅Π½Π°Ρ‚Π΅Π»ΡŒ Π΄Ρ€ΠΎΠ±ΠΈ. ΠŸΠΎΠ΄Π±ΠΈΡ€Π°Ρ‚ΡŒ Π΄ΠΎΠΏΠΎΠ»Π½ΠΈΡ‚Π΅Π»ΡŒΠ½Ρ‹ΠΉ ΠΌΠ½ΠΎΠΆΠΈΡ‚Π΅Π»ΡŒ Π½Π΅ΠΎΠ±Ρ…ΠΎΠ΄ΠΈΠΌΠΎ Ρ‚Π°ΠΊΠΈΠΌ ΠΎΠ±Ρ€Π°Π·ΠΎΠΌ, Ρ‡Ρ‚ΠΎΠ±Ρ‹ ΠΎΠ½ Π½Π΅ обращался Π² Π½ΡƒΠ»ΡŒ Π½ΠΈ ΠΏΡ€ΠΈ ΠΊΠ°ΠΊΠΈΡ… значСниях ΠΏΠ΅Ρ€Π΅ΠΌΠ΅Π½Π½Ρ‹Ρ… ΠΈΠ· ΠžΠ”Π— ΠΏΠ΅Ρ€Π΅ΠΌΠ΅Π½Π½Ρ‹Ρ… для исходного выраТСния.

    ΠŸΡ€ΠΈΠΌΠ΅Ρ€ 8

    ΠŸΡ€ΠΈΠ²Π΅Π΄ΠΈΡ‚Π΅ Π΄Ρ€ΠΎΠ±ΠΈ ΠΊ Π½ΠΎΠ²ΠΎΠΌΡƒ Π·Π½Π°ΠΌΠ΅Π½Π°Ρ‚Π΅Π»ΡŽ: Π°) a + 1 a 0 , 7 ΠΊ Π·Π½Π°ΠΌΠ΅Π½Π°Ρ‚Π΅Π»ΡŽ a , Π±) 1 x 2 3 — 2 Β· x 1 3 Β· y 1 6 + 4 Β· y 1 3 ΠΊ Π·Π½Π°ΠΌΠ΅Π½Π°Ρ‚Π΅Π»ΡŽ x + 8 Β· y 1 2 .

    РСшСниС

    Π°) ΠŸΠΎΠ΄Π±Π΅Ρ€Π΅ΠΌ ΠΌΠ½ΠΎΠΆΠΈΡ‚Π΅Π»ΡŒ, ΠΊΠΎΡ‚ΠΎΡ€Ρ‹ΠΉ ΠΏΠΎΠ·Π²ΠΎΠ»ΠΈΡ‚ Π½Π°ΠΌ произвСсти ΠΏΡ€ΠΈΠ²Π΅Π΄Π΅Π½ΠΈΠ΅ ΠΊ Π½ΠΎΠ²ΠΎΠΌΡƒ Π·Π½Π°ΠΌΠ΅Π½Π°Ρ‚Π΅Π»ΡŽ. a 0 , 7 Β· a 0 , 3 = a 0 , 7 + 0 , 3 = a , ΡΠ»Π΅Π΄ΠΎΠ²Π°Ρ‚Π΅Π»ΡŒΠ½ΠΎ, Π² качСствС Π΄ΠΎΠΏΠΎΠ»Π½ΠΈΡ‚Π΅Π»ΡŒΠ½ΠΎΠ³ΠΎ мноТитСля ΠΌΡ‹ возьмСм a 0 , 3 . ΠžΠ±Π»Π°ΡΡ‚ΡŒ допустимых Π·Π½Π°Ρ‡Π΅Π½ΠΈΠΉ ΠΏΠ΅Ρ€Π΅ΠΌΠ΅Π½Π½ΠΎΠΉ Π° Π²ΠΊΠ»ΡŽΡ‡Π°Π΅Ρ‚ мноТСство всСх ΠΏΠΎΠ»ΠΎΠΆΠΈΡ‚Π΅Π»ΡŒΠ½Ρ‹Ρ… Π΄Π΅ΠΉΡΡ‚Π²ΠΈΡ‚Π΅Π»ΡŒΠ½Ρ‹Ρ… чисСл. Π’ этой области ΡΡ‚Π΅ΠΏΠ΅Π½ΡŒ a 0 , 3 Π½Π΅ обращаСтся Π² Π½ΡƒΠ»ΡŒ.

    Π’Ρ‹ΠΏΠΎΠ»Π½ΠΈΠΌ ΡƒΠΌΠ½ΠΎΠΆΠ΅Π½ΠΈΠ΅ числитСля ΠΈ знамСнатСля Π΄Ρ€ΠΎΠ±ΠΈ Π½Π° a 0 , 3 :

    a + 1 a 0 , 7 = a + 1 Β· a 0 , 3 a 0 , 7 Β· a 0 , 3 = a + 1 Β· a 0 , 3 a

    Π±) ΠžΠ±Ρ€Π°Ρ‚ΠΈΠΌ Π²Π½ΠΈΠΌΠ°Π½ΠΈΠ΅ Π½Π° Π·Π½Π°ΠΌΠ΅Π½Π°Ρ‚Π΅Π»ΡŒ:

    x 2 3 — 2 Β· x 1 3 Β· y 1 6 + 4 Β· y 1 3 = = x 1 3 2 — x 1 3 Β· 2 Β· y 1 6 + 2 Β· y 1 6 2

    Π£ΠΌΠ½ΠΎΠΆΠΈΠΌ это Π²Ρ‹Ρ€Π°ΠΆΠ΅Π½ΠΈΠ΅ Π½Π° x 1 3 + 2 Β· y 1 6 , ΠΏΠΎΠ»ΡƒΡ‡ΠΈΠΌ сумму ΠΊΡƒΠ±ΠΎΠ² x 1 3 ΠΈ 2 Β· y 1 6 , Ρ‚. Π΅. x + 8 Β· y 1 2 . Π­Ρ‚ΠΎ наш Π½ΠΎΠ²Ρ‹ΠΉ Π·Π½Π°ΠΌΠ΅Π½Π°Ρ‚Π΅Π»ΡŒ, ΠΊ ΠΊΠΎΡ‚ΠΎΡ€ΠΎΠΌΡƒ Π½Π°ΠΌ Π½Π°Π΄ΠΎ привСсти ΠΈΡΡ…ΠΎΠ΄Π½ΡƒΡŽ Π΄Ρ€ΠΎΠ±ΡŒ.

    Π’Π°ΠΊ ΠΌΡ‹ нашли Π΄ΠΎΠΏΠΎΠ»Π½ΠΈΡ‚Π΅Π»ΡŒΠ½Ρ‹ΠΉ ΠΌΠ½ΠΎΠΆΠΈΡ‚Π΅Π»ΡŒ x 1 3 + 2 Β· y 1 6 . На области допустимых Π·Π½Π°Ρ‡Π΅Π½ΠΈΠΉ ΠΏΠ΅Ρ€Π΅ΠΌΠ΅Π½Π½Ρ‹Ρ… x ΠΈ y Π²Ρ‹Ρ€Π°ΠΆΠ΅Π½ΠΈΠ΅ x 1 3 + 2 Β· y 1 6 Π½Π΅ обращаСтся Π² Π½ΡƒΠ»ΡŒ, поэтому, ΠΌΡ‹ ΠΌΠΎΠΆΠ΅ΠΌ ΡƒΠΌΠ½ΠΎΠΆΠΈΡ‚ΡŒ Π½Π° Π½Π΅Π³ΠΎ Ρ‡ΠΈΡΠ»ΠΈΡ‚Π΅Π»ΡŒ ΠΈ Π·Π½Π°ΠΌΠ΅Π½Π°Ρ‚Π΅Π»ΡŒ Π΄Ρ€ΠΎΠ±ΠΈ:
    1 x 2 3 — 2 Β· x 1 3 Β· y 1 6 + 4 Β· y 1 3 = = x 1 3 + 2 Β· y 1 6 x 1 3 + 2 Β· y 1 6 x 2 3 — 2 Β· x 1 3 Β· y 1 6 + 4 Β· y 1 3 = = x 1 3 + 2 Β· y 1 6 x 1 3 3 + 2 Β· y 1 6 3 = x 1 3 + 2 Β· y 1 6 x + 8 Β· y 1 2

    ΠžΡ‚Π²Π΅Ρ‚: Π°) a + 1 a 0 , 7 = a + 1 Β· a 0 , 3 a , Π±) 1 x 2 3 — 2 Β· x 1 3 Β· y 1 6 + 4 Β· y 1 3 = x 1 3 + 2 Β· y 1 6 x + 8 Β· y 1 2 .

    ΠŸΡ€ΠΈΠΌΠ΅Ρ€ 9

    Π‘ΠΎΠΊΡ€Π°Ρ‚ΠΈΡ‚Π΅ Π΄Ρ€ΠΎΠ±ΡŒ: Π°) 30 Β· x 3 Β· (x 0 , 5 + 1) Β· x + 2 Β· x 1 1 3 — 5 3 45 Β· x 0 , 5 + 1 2 Β· x + 2 Β· x 1 1 3 — 5 3 , Π±) a 1 4 — b 1 4 a 1 2 — b 1 2 .

    РСшСниС

    Π°) Π˜ΡΠΏΠΎΠ»ΡŒΠ·ΡƒΠ΅ΠΌ наибольший ΠΎΠ±Ρ‰ΠΈΠΉ Π·Π½Π°ΠΌΠ΅Π½Π°Ρ‚Π΅Π»ΡŒ (ΠΠžΠ”), Π½Π° ΠΊΠΎΡ‚ΠΎΡ€Ρ‹ΠΉ ΠΌΠΎΠΆΠ½ΠΎ ΡΠΎΠΊΡ€Π°Ρ‚ΠΈΡ‚ΡŒ Ρ‡ΠΈΡΠ»ΠΈΡ‚Π΅Π»ΡŒ ΠΈ Π·Π½Π°ΠΌΠ΅Π½Π°Ρ‚Π΅Π»ΡŒ. Для чисСл 30 ΠΈ 45 это 15 . Π’Π°ΠΊΠΆΠ΅ ΠΌΡ‹ ΠΌΠΎΠΆΠ΅ΠΌ произвСсти сокращСниС Π½Π° x 0 , 5 + 1 ΠΈ Π½Π° x + 2 Β· x 1 1 3 — 5 3 .

    ΠŸΠΎΠ»ΡƒΡ‡Π°Π΅ΠΌ:

    30 Β· x 3 Β· (x 0 , 5 + 1) Β· x + 2 Β· x 1 1 3 — 5 3 45 Β· x 0 , 5 + 1 2 Β· x + 2 Β· x 1 1 3 — 5 3 = 2 Β· x 3 3 Β· (x 0 , 5 + 1)

    Π±) Π—Π΄Π΅ΡΡŒ Π½Π°Π»ΠΈΡ‡ΠΈΠ΅ ΠΎΠ΄ΠΈΠ½Π°ΠΊΠΎΠ²Ρ‹Ρ… ΠΌΠ½ΠΎΠΆΠΈΡ‚Π΅Π»Π΅ΠΉ Π½Π΅ΠΎΡ‡Π΅Π²ΠΈΠ΄Π½ΠΎ. ΠŸΡ€ΠΈΠ΄Π΅Ρ‚ΡΡ Π²Ρ‹ΠΏΠΎΠ»Π½ΠΈΡ‚ΡŒ Π½Π΅ΠΊΠΎΡ‚ΠΎΡ€Ρ‹Π΅ прСобразования для Ρ‚ΠΎΠ³ΠΎ, Ρ‡Ρ‚ΠΎΠ±Ρ‹ ΠΏΠΎΠ»ΡƒΡ‡ΠΈΡ‚ΡŒ ΠΎΠ΄ΠΈΠ½Π°ΠΊΠΎΠ²Ρ‹Π΅ ΠΌΠ½ΠΎΠΆΠΈΡ‚Π΅Π»ΠΈ Π² числитСлС ΠΈ Π·Π½Π°ΠΌΠ΅Π½Π°Ρ‚Π΅Π»Π΅. Для этого Ρ€Π°Π·Π»ΠΎΠΆΠΈΠΌ Π·Π½Π°ΠΌΠ΅Π½Π°Ρ‚Π΅Π»ΡŒ, ΠΈΡΠΏΠΎΠ»ΡŒΠ·ΡƒΡ Ρ„ΠΎΡ€ΠΌΡƒΠ»Ρƒ разности ΠΊΠ²Π°Π΄Ρ€Π°Ρ‚ΠΎΠ²:

    a 1 4 — b 1 4 a 1 2 — b 1 2 = a 1 4 — b 1 4 a 1 4 2 — b 1 2 2 = = a 1 4 — b 1 4 a 1 4 + b 1 4 Β· a 1 4 — b 1 4 = 1 a 1 4 + b 1 4

    ΠžΡ‚Π²Π΅Ρ‚: Π°) 30 Β· x 3 Β· (x 0 , 5 + 1) Β· x + 2 Β· x 1 1 3 — 5 3 45 Β· x 0 , 5 + 1 2 Β· x + 2 Β· x 1 1 3 — 5 3 = 2 Β· x 3 3 Β· (x 0 , 5 + 1) , Π±) a 1 4 — b 1 4 a 1 2 — b 1 2 = 1 a 1 4 + b 1 4 .

    К числу основных дСйствий с дробями относится ΠΏΡ€ΠΈΠ²Π΅Π΄Π΅Π½ΠΈΠ΅ ΠΊ Π½ΠΎΠ²ΠΎΠΌΡƒ Π·Π½Π°ΠΌΠ΅Π½Π°Ρ‚Π΅Π»ΡŽ ΠΈ сокращСниС Π΄Ρ€ΠΎΠ±Π΅ΠΉ. Оба дСйствия Π²Ρ‹ΠΏΠΎΠ»Π½ΡΡŽΡ‚ с соблюдСниСм ряда ΠΏΡ€Π°Π²ΠΈΠ». ΠŸΡ€ΠΈ слоТСнии ΠΈ Π²Ρ‹Ρ‡ΠΈΡ‚Π°Π½ΠΈΠΈ Π΄Ρ€ΠΎΠ±Π΅ΠΉ сначала Π΄Ρ€ΠΎΠ±ΠΈ приводятся ΠΊ ΠΎΠ±Ρ‰Π΅ΠΌΡƒ Π·Π½Π°ΠΌΠ΅Π½Π°Ρ‚Π΅Π»ΡŽ, послС Ρ‡Π΅Π³ΠΎ проводятся дСйствия (слоТСниС ΠΈΠ»ΠΈ Π²Ρ‹Ρ‡ΠΈΡ‚Π°Π½ΠΈΠ΅) с числитСлями. Π—Π½Π°ΠΌΠ΅Π½Π°Ρ‚Π΅Π»ΡŒ остаСтся ΠΏΡ€Π΅ΠΆΠ½ΠΈΠΌ. Π Π΅Π·ΡƒΠ»ΡŒΡ‚Π°Ρ‚ΠΎΠΌ Π½Π°ΡˆΠΈΡ… дСйствий являСтся новая Π΄Ρ€ΠΎΠ±ΡŒ, Ρ‡ΠΈΡΠ»ΠΈΡ‚Π΅Π»ΡŒ ΠΊΠΎΡ‚ΠΎΡ€ΠΎΠΉ являСтся ΠΏΡ€ΠΎΠΈΠ·Π²Π΅Π΄Π΅Π½ΠΈΠ΅ΠΌ числитСлСй, Π° Π·Π½Π°ΠΌΠ΅Π½Π°Ρ‚Π΅Π»ΡŒ Π΅ΡΡ‚ΡŒ ΠΏΡ€ΠΎΠΈΠ·Π²Π΅Π΄Π΅Π½ΠΈΠ΅ Π·Π½Π°ΠΌΠ΅Π½Π°Ρ‚Π΅Π»Π΅ΠΉ.

    ΠŸΡ€ΠΈΠΌΠ΅Ρ€ 10

    Π’Ρ‹ΠΏΠΎΠ»Π½ΠΈΡ‚Π΅ дСйствия x 1 2 + 1 x 1 2 — 1 — x 1 2 — 1 x 1 2 + 1 Β· 1 x 1 2 .

    РСшСниС

    НачнСм с вычитания Π΄Ρ€ΠΎΠ±Π΅ΠΉ, ΠΊΠΎΡ‚ΠΎΡ€Ρ‹Π΅ Ρ€Π°ΡΠΏΠΎΠ»Π°Π³Π°ΡŽΡ‚ΡΡ Π² скобках. ΠŸΡ€ΠΈΠ²Π΅Π΄Π΅ΠΌ ΠΈΡ… ΠΊ ΠΎΠ±Ρ‰Π΅ΠΌΡƒ Π·Π½Π°ΠΌΠ΅Π½Π°Ρ‚Π΅Π»ΡŽ:

    x 1 2 — 1 Β· x 1 2 + 1

    Π’Ρ‹Ρ‡Ρ‚Π΅ΠΌ числитСли:

    x 1 2 + 1 x 1 2 — 1 — x 1 2 — 1 x 1 2 + 1 Β· 1 x 1 2 = = x 1 2 + 1 Β· x 1 2 + 1 x 1 2 — 1 Β· x 1 2 + 1 — x 1 2 — 1 Β· x 1 2 — 1 x 1 2 + 1 Β· x 1 2 — 1 Β· 1 x 1 2 = = x 1 2 + 1 2 — x 1 2 — 1 2 x 1 2 — 1 Β· x 1 2 + 1 Β· 1 x 1 2 = = x 1 2 2 + 2 Β· x 1 2 + 1 — x 1 2 2 — 2 Β· x 1 2 + 1 x 1 2 — 1 Β· x 1 2 + 1 Β· 1 x 1 2 = = 4 Β· x 1 2 x 1 2 — 1 Β· x 1 2 + 1 Β· 1 x 1 2

    Π’Π΅ΠΏΠ΅Ρ€ΡŒ ΡƒΠΌΠ½ΠΎΠΆΠ°Π΅ΠΌ Π΄Ρ€ΠΎΠ±ΠΈ:

    4 Β· x 1 2 x 1 2 — 1 Β· x 1 2 + 1 Β· 1 x 1 2 = = 4 Β· x 1 2 x 1 2 — 1 Β· x 1 2 + 1 Β· x 1 2

    ΠŸΡ€ΠΎΠΈΠ·Π²Π΅Π΄Π΅ΠΌ сокращСниС Π½Π° ΡΡ‚Π΅ΠΏΠ΅Π½ΡŒ x 1 2 , ΠΏΠΎΠ»ΡƒΡ‡ΠΈΠΌ 4 x 1 2 — 1 Β· x 1 2 + 1 .

    Π”ΠΎΠΏΠΎΠ»Π½ΠΈΡ‚Π΅Π»ΡŒΠ½ΠΎ ΠΌΠΎΠΆΠ½ΠΎ ΡƒΠΏΡ€ΠΎΡΡ‚ΠΈΡ‚ΡŒ стСпСнноС Π²Ρ‹Ρ€Π°ΠΆΠ΅Π½ΠΈΠ΅ Π² Π·Π½Π°ΠΌΠ΅Π½Π°Ρ‚Π΅Π»Π΅, ΠΈΡΠΏΠΎΠ»ΡŒΠ·ΡƒΡ Ρ„ΠΎΡ€ΠΌΡƒΠ»Ρƒ разности ΠΊΠ²Π°Π΄Ρ€Π°Ρ‚ΠΎΠ²: ΠΊΠ²Π°Π΄Ρ€Π°Ρ‚ΠΎΠ²: 4 x 1 2 — 1 Β· x 1 2 + 1 = 4 x 1 2 2 — 1 2 = 4 x — 1 .

    ΠžΡ‚Π²Π΅Ρ‚: x 1 2 + 1 x 1 2 — 1 — x 1 2 — 1 x 1 2 + 1 Β· 1 x 1 2 = 4 x — 1

    ΠŸΡ€ΠΈΠΌΠ΅Ρ€ 11

    УпроститС стСпСнноС Π²Ρ‹Ρ€Π°ΠΆΠ΅Π½ΠΈΠ΅ x 3 4 Β· x 2 , 7 + 1 2 x — 5 8 Β· x 2 , 7 + 1 3 .
    РСшСниС

    ΠœΡ‹ ΠΌΠΎΠΆΠ΅ΠΌ произвСсти сокращСниС Π΄Ρ€ΠΎΠ±ΠΈ Π½Π° (x 2 , 7 + 1) 2 . ΠŸΠΎΠ»ΡƒΡ‡Π°Π΅ΠΌ Π΄Ρ€ΠΎΠ±ΡŒ x 3 4 x — 5 8 Β· x 2 , 7 + 1 .

    ΠŸΡ€ΠΎΠ΄ΠΎΠ»ΠΆΠΈΠΌ прСобразования стСпСнСй икса x 3 4 x — 5 8 Β· 1 x 2 , 7 + 1 . Π’Π΅ΠΏΠ΅Ρ€ΡŒ ΠΌΠΎΠΆΠ½ΠΎ ΠΈΡΠΏΠΎΠ»ΡŒΠ·ΠΎΠ²Π°Ρ‚ΡŒ свойство дСлСния стСпСнСй с ΠΎΠ΄ΠΈΠ½Π°ΠΊΠΎΠ²Ρ‹ΠΌΠΈ основаниями: x 3 4 x — 5 8 Β· 1 x 2 , 7 + 1 = x 3 4 — — 5 8 Β· 1 x 2 , 7 + 1 = x 1 1 8 Β· 1 x 2 , 7 + 1 .

    ΠŸΠ΅Ρ€Π΅Ρ…ΠΎΠ΄ΠΈΠΌ ΠΎΡ‚ послСднСго произвСдСния ΠΊ Π΄Ρ€ΠΎΠ±ΠΈ x 1 3 8 x 2 , 7 + 1 .

    ΠžΡ‚Π²Π΅Ρ‚: x 3 4 Β· x 2 , 7 + 1 2 x — 5 8 Β· x 2 , 7 + 1 3 = x 1 3 8 x 2 , 7 + 1 .

    ΠœΠ½ΠΎΠΆΠΈΡ‚Π΅Π»ΠΈ с ΠΎΡ‚Ρ€ΠΈΡ†Π°Ρ‚Π΅Π»ΡŒΠ½Ρ‹ΠΌΠΈ показатСлями стСпСни Π² Π±ΠΎΠ»ΡŒΡˆΠΈΠ½ΡΡ‚Π²Π΅ случаСв ΡƒΠ΄ΠΎΠ±Π½Π΅Π΅ ΠΏΠ΅Ρ€Π΅Π½ΠΎΡΠΈΡ‚ΡŒ ΠΈΠ· числитСля Π² Π·Π½Π°ΠΌΠ΅Π½Π°Ρ‚Π΅Π»ΡŒ ΠΈ ΠΎΠ±Ρ€Π°Ρ‚Π½ΠΎ, измСняя Π·Π½Π°ΠΊ показатСля. Π­Ρ‚ΠΎ дСйствиС позволяСт ΡƒΠΏΡ€ΠΎΡΡ‚ΠΈΡ‚ΡŒ дальнСйшСС Ρ€Π΅ΡˆΠ΅Π½ΠΈΠ΅. ΠŸΡ€ΠΈΠ²Π΅Π΄Π΅ΠΌ ΠΏΡ€ΠΈΠΌΠ΅Ρ€: стСпСнноС Π²Ρ‹Ρ€Π°ΠΆΠ΅Π½ΠΈΠ΅ (x + 1) — 0 , 2 3 Β· x — 1 ΠΌΠΎΠΆΠ½ΠΎ Π·Π°ΠΌΠ΅Π½ΠΈΡ‚ΡŒ Π½Π° x 3 Β· (x + 1) 0 , 2 .

    ΠŸΡ€Π΅ΠΎΠ±Ρ€Π°Π·ΠΎΠ²Π°Π½ΠΈΠ΅ Π²Ρ‹Ρ€Π°ΠΆΠ΅Π½ΠΈΠΉ с корнями ΠΈ стСпСнями

    Π’ Π·Π°Π΄Π°Ρ‡Π°Ρ… Π²ΡΡ‚Ρ€Π΅Ρ‡Π°ΡŽΡ‚ΡΡ стСпСнныС выраТСния, ΠΊΠΎΡ‚ΠΎΡ€Ρ‹Π΅ содСрТат Π½Π΅ Ρ‚ΠΎΠ»ΡŒΠΊΠΎ стСпСни с Π΄Ρ€ΠΎΠ±Π½Ρ‹ΠΌΠΈ показатСлями, Π½ΠΎ ΠΈ ΠΊΠΎΡ€Π½ΠΈ. Π’Π°ΠΊΠΈΠ΅ выраТСния ΠΆΠ΅Π»Π°Ρ‚Π΅Π»ΡŒΠ½ΠΎ привСсти Ρ‚ΠΎΠ»ΡŒΠΊΠΎ ΠΊ корням ΠΈΠ»ΠΈ Ρ‚ΠΎΠ»ΡŒΠΊΠΎ ΠΊ стСпСням. ΠŸΠ΅Ρ€Π΅Ρ…ΠΎΠ΄ ΠΊ стСпСням ΠΏΡ€Π΅Π΄ΠΏΠΎΡ‡Ρ‚ΠΈΡ‚Π΅Π»ΡŒΠ½Π΅Π΅, Ρ‚Π°ΠΊ ΠΊΠ°ΠΊ с Π½ΠΈΠΌΠΈ ΠΏΡ€ΠΎΡ‰Π΅ Ρ€Π°Π±ΠΎΡ‚Π°Ρ‚ΡŒ. Π’Π°ΠΊΠΎΠΉ ΠΏΠ΅Ρ€Π΅Ρ…ΠΎΠ΄ являСтся особСнно ΠΏΡ€Π΅Π΄ΠΏΠΎΡ‡Ρ‚ΠΈΡ‚Π΅Π»ΡŒΠ½Ρ‹ΠΌ, ΠΊΠΎΠ³Π΄Π° ΠžΠ”Π— ΠΏΠ΅Ρ€Π΅ΠΌΠ΅Π½Π½Ρ‹Ρ… для исходного выраТСния позволяСт Π·Π°ΠΌΠ΅Π½ΠΈΡ‚ΡŒ ΠΊΠΎΡ€Π½ΠΈ стСпСнями Π±Π΅Π· нСобходимости ΠΎΠ±Ρ€Π°Ρ‰Π°Ρ‚ΡŒΡΡ ΠΊ ΠΌΠΎΠ΄ΡƒΠ»ΡŽ ΠΈΠ»ΠΈ Ρ€Π°Π·Π±ΠΈΠ²Π°Ρ‚ΡŒ ΠžΠ”Π— Π½Π° нСсколько ΠΏΡ€ΠΎΠΌΠ΅ΠΆΡƒΡ‚ΠΊΠΎΠ².

    ΠŸΡ€ΠΈΠΌΠ΅Ρ€ 12

    ΠŸΡ€Π΅Π΄ΡΡ‚Π°Π²ΡŒΡ‚Π΅ Π²Ρ‹Ρ€Π°ΠΆΠ΅Π½ΠΈΠ΅ x 1 9 Β· x Β· x 3 6 Π² Π²ΠΈΠ΄Π΅ стСпСни.

    РСшСниС

    ΠžΠ±Π»Π°ΡΡ‚ΡŒ допустимых Π·Π½Π°Ρ‡Π΅Π½ΠΈΠΉ ΠΏΠ΅Ρ€Π΅ΠΌΠ΅Π½Π½ΠΎΠΉ x опрСдСляСтся двумя нСравСнствами x β‰₯ 0 ΠΈ x Β· x 3 β‰₯ 0 , ΠΊΠΎΡ‚ΠΎΡ€Ρ‹Π΅ Π·Π°Π΄Π°ΡŽΡ‚ мноТСство [ 0 , + ∞) .

    На этом мноТСствС ΠΌΡ‹ ΠΈΠΌΠ΅Π΅ΠΌ ΠΏΡ€Π°Π²ΠΎ ΠΏΠ΅Ρ€Π΅ΠΉΡ‚ΠΈ ΠΎΡ‚ ΠΊΠΎΡ€Π½Π΅ΠΉ ΠΊ стСпСням:

    x 1 9 Β· x Β· x 3 6 = x 1 9 Β· x Β· x 1 3 1 6

    Π˜ΡΠΏΠΎΠ»ΡŒΠ·ΡƒΡ свойства стСпСнСй, упростим ΠΏΠΎΠ»ΡƒΡ‡Π΅Π½Π½ΠΎΠ΅ стСпСнноС Π²Ρ‹Ρ€Π°ΠΆΠ΅Π½ΠΈΠ΅.

    x 1 9 Β· x Β· x 1 3 1 6 = x 1 9 Β· x 1 6 Β· x 1 3 1 6 = x 1 9 Β· x 1 6 Β· x 1 Β· 1 3 Β· 6 = = x 1 9 Β· x 1 6 Β· x 1 18 = x 1 9 + 1 6 + 1 18 = x 1 3

    ΠžΡ‚Π²Π΅Ρ‚: x 1 9 Β· x Β· x 3 6 = x 1 3 .

    ΠŸΡ€Π΅ΠΎΠ±Ρ€Π°Π·ΠΎΠ²Π°Π½ΠΈΠ΅ стСпСнСй с ΠΏΠ΅Ρ€Π΅ΠΌΠ΅Π½Π½Ρ‹ΠΌΠΈ Π² ΠΏΠΎΠΊΠ°Π·Π°Ρ‚Π΅Π»Π΅

    Π”Π°Π½Π½Ρ‹Π΅ прСобразования достаточно просто произвСсти, Ссли Π³Ρ€Π°ΠΌΠΎΡ‚Π½ΠΎ ΠΈΡΠΏΠΎΠ»ΡŒΠ·ΠΎΠ²Π°Ρ‚ΡŒ свойства стСпСни. НапримСр, 5 2 Β· x + 1 βˆ’ 3 Β· 5 x Β· 7 x βˆ’ 14 Β· 7 2 Β· x βˆ’ 1 = 0 .

    ΠœΡ‹ ΠΌΠΎΠΆΠ΅ΠΌ Π·Π°ΠΌΠ΅Π½ΠΈΡ‚ΡŒ ΠΏΡ€ΠΎΠΈΠ·Π²Π΅Π΄Π΅Π½ΠΈΠ΅ΠΌ стСпСни, Π² показатСлях ΠΊΠΎΡ‚ΠΎΡ€Ρ‹Ρ… находится сумма Π½Π΅ΠΊΠΎΡ‚ΠΎΡ€ΠΎΠΉ ΠΏΠ΅Ρ€Π΅ΠΌΠ΅Π½Π½ΠΎΠΉ ΠΈ числа. Π’ Π»Π΅Π²ΠΎΠΉ части это ΠΌΠΎΠΆΠ½ΠΎ ΠΏΡ€ΠΎΠ΄Π΅Π»Π°Ρ‚ΡŒ с ΠΏΠ΅Ρ€Π²Ρ‹ΠΌ ΠΈ послСдним слагаСмыми Π»Π΅Π²ΠΎΠΉ части выраТСния:

    5 2 Β· x Β· 5 1 βˆ’ 3 Β· 5 x Β· 7 x βˆ’ 14 Β· 7 2 Β· x Β· 7 βˆ’ 1 = 0 , 5 Β· 5 2 Β· x βˆ’ 3 Β· 5 x Β· 7 x βˆ’ 2 Β· 7 2 Β· x = 0 .

    Π’Π΅ΠΏΠ΅Ρ€ΡŒ ΠΏΠΎΠ΄Π΅Π»ΠΈΠΌ ΠΎΠ±Π΅ части равСнства Π½Π° 7 2 Β· x . Π­Ρ‚ΠΎ Π²Ρ‹Ρ€Π°ΠΆΠ΅Π½ΠΈΠ΅ Π½Π° ΠžΠ”Π— ΠΏΠ΅Ρ€Π΅ΠΌΠ΅Π½Π½ΠΎΠΉ x ΠΏΡ€ΠΈΠ½ΠΈΠΌΠ°Π΅Ρ‚ Ρ‚ΠΎΠ»ΡŒΠΊΠΎ ΠΏΠΎΠ»ΠΎΠΆΠΈΡ‚Π΅Π»ΡŒΠ½Ρ‹Π΅ значСния:

    5 Β· 5 — 3 Β· 5 x Β· 7 x — 2 Β· 7 2 Β· x 7 2 Β· x = 0 7 2 Β· x , 5 Β· 5 2 Β· x 7 2 Β· x — 3 Β· 5 x Β· 7 x 7 2 Β· x — 2 Β· 7 2 Β· x 7 2 Β· x = 0 , 5 Β· 5 2 Β· x 7 2 Β· x — 3 Β· 5 x Β· 7 x 7 x Β· 7 x — 2 Β· 7 2 Β· x 7 2 Β· x = 0

    Π‘ΠΎΠΊΡ€Π°Ρ‚ΠΈΠΌ Π΄Ρ€ΠΎΠ±ΠΈ со стСпСнями, ΠΏΠΎΠ»ΡƒΡ‡ΠΈΠΌ: 5 Β· 5 2 Β· x 7 2 Β· x — 3 Β· 5 x 7 x — 2 = 0 .

    НаконСц, ΠΎΡ‚Π½ΠΎΡˆΠ΅Π½ΠΈΠ΅ стСпСнСй с ΠΎΠ΄ΠΈΠ½Π°ΠΊΠΎΠ²Ρ‹ΠΌΠΈ показатСлями замСняСтся стСпСнями ΠΎΡ‚Π½ΠΎΡˆΠ΅Π½ΠΈΠΉ, Ρ‡Ρ‚ΠΎ ΠΏΡ€ΠΈΠ²ΠΎΠ΄ΠΈΡ‚ ΠΊ ΡƒΡ€Π°Π²Π½Π΅Π½ΠΈΡŽ 5 Β· 5 7 2 Β· x — 3 Β· 5 7 x — 2 = 0 , ΠΊΠΎΡ‚ΠΎΡ€ΠΎΠ΅ Ρ€Π°Π²Π½ΠΎΡΠΈΠ»ΡŒΠ½ΠΎ 5 Β· 5 7 x 2 — 3 Β· 5 7 x — 2 = 0 .

    Π’Π²Π΅Π΄Π΅ΠΌ Π½ΠΎΠ²ΡƒΡŽ ΠΏΠ΅Ρ€Π΅ΠΌΠ΅Π½Π½ΡƒΡŽ t = 5 7 x , Ρ‡Ρ‚ΠΎ сводит Ρ€Π΅ΡˆΠ΅Π½ΠΈΠ΅ исходного ΠΏΠΎΠΊΠ°Π·Π°Ρ‚Π΅Π»ΡŒΠ½ΠΎΠ³ΠΎ уравнСния ΠΊ Ρ€Π΅ΡˆΠ΅Π½ΠΈΡŽ ΠΊΠ²Π°Π΄Ρ€Π°Ρ‚Π½ΠΎΠ³ΠΎ уравнСния 5 Β· t 2 βˆ’ 3 Β· t βˆ’ 2 = 0 .

    ΠŸΡ€Π΅ΠΎΠ±Ρ€Π°Π·ΠΎΠ²Π°Π½ΠΈΠ΅ Π²Ρ‹Ρ€Π°ΠΆΠ΅Π½ΠΈΠΉ со стСпСнями ΠΈ Π»ΠΎΠ³Π°Ρ€ΠΈΡ„ΠΌΠ°ΠΌΠΈ

    ВыраТСния, содСрТащиС с записи стСпСни ΠΈ Π»ΠΎΠ³Π°Ρ€ΠΈΡ„ΠΌΡ‹, Ρ‚Π°ΠΊΠΆΠ΅ Π²ΡΡ‚Ρ€Π΅Ρ‡Π°ΡŽΡ‚ΡΡ Π² Π·Π°Π΄Π°Ρ‡Π°Ρ…. ΠŸΡ€ΠΈΠΌΠ΅Ρ€ΠΎΠΌ Ρ‚Π°ΠΊΠΈΡ… Π²Ρ‹Ρ€Π°ΠΆΠ΅Π½ΠΈΠΉ ΠΌΠΎΠ³ΡƒΡ‚ ΡΠ»ΡƒΠΆΠΈΡ‚ΡŒ: 1 4 1 — 5 Β· log 2 3 ΠΈΠ»ΠΈ log 3 27 9 + 5 (1 — log 3 5) Β· log 5 3 . ΠŸΡ€Π΅ΠΎΠ±Ρ€Π°Π·ΠΎΠ²Π°Π½ΠΈΠ΅ ΠΏΠΎΠ΄ΠΎΠ±Π½Ρ‹Ρ… Π²Ρ‹Ρ€Π°ΠΆΠ΅Π½ΠΈΠΉ проводится с использованиСм Ρ€Π°Π·ΠΎΠ±Ρ€Π°Π½Π½Ρ‹Ρ… Π²Ρ‹ΡˆΠ΅ ΠΏΠΎΠ΄Ρ…ΠΎΠ΄ΠΎΠ² ΠΈ свойств Π»ΠΎΠ³Π°Ρ€ΠΈΡ„ΠΌΠΎΠ², ΠΊΠΎΡ‚ΠΎΡ€Ρ‹Π΅ ΠΌΡ‹ ΠΏΠΎΠ΄Ρ€ΠΎΠ±Π½ΠΎ Ρ€Π°Π·ΠΎΠ±Ρ€Π°Π»ΠΈ Π² Ρ‚Π΅ΠΌΠ΅ Β«ΠŸΡ€Π΅ΠΎΠ±Ρ€Π°Π·ΠΎΠ²Π°Π½ΠΈΠ΅ логарифмичСских Π²Ρ‹Ρ€Π°ΠΆΠ΅Π½ΠΈΠΉΒ».

    Если Π²Ρ‹ Π·Π°ΠΌΠ΅Ρ‚ΠΈΠ»ΠΈ ΠΎΡˆΠΈΠ±ΠΊΡƒ Π² тСкстС, поТалуйста, Π²Ρ‹Π΄Π΅Π»ΠΈΡ‚Π΅ Π΅Ρ‘ ΠΈ Π½Π°ΠΆΠΌΠΈΡ‚Π΅ Ctrl+Enter

    Π’ΠΈΠΏ ΡƒΡ€ΠΎΠΊΠ°: ΡƒΡ€ΠΎΠΊ обобщСния ΠΈ систСматизации Π·Π½Π°Π½ΠΈΠΉ

    Π¦Π΅Π»ΠΈ:

    • ΠΎΠ±ΡƒΡ‡Π°ΡŽΡ‰ΠΈΠ΅ – ΠΏΠΎΠ²Ρ‚ΠΎΡ€ΠΈΡ‚ΡŒ ΠΎΠΏΡ€Π΅Π΄Π΅Π»Π΅Π½ΠΈΠ΅ стСпСни, ΠΏΡ€Π°Π²ΠΈΠ»Π° умноТСния ΠΈ дСлСния стСпСнСй, возвСдСния стСпСни Π² ΡΡ‚Π΅ΠΏΠ΅Π½ΡŒ, Π·Π°ΠΊΡ€Π΅ΠΏΠΈΡ‚ΡŒ умСния Ρ€Π΅ΡˆΠ΅Π½ΠΈΡ ΠΏΡ€ΠΈΠΌΠ΅Ρ€ΠΎΠ², содСрТащих стСпСни,
    • Ρ€Π°Π·Π²ΠΈΠ²Π°ΡŽΡ‰ΠΈΠ΅ – Ρ€Π°Π·Π²ΠΈΡ‚ΠΈΠ΅ логичСского ΠΌΡ‹ΡˆΠ»Π΅Π½ΠΈΡ учащихся, интСрСса ΠΊ ΠΈΠ·ΡƒΡ‡Π°Π΅ΠΌΠΎΠΌΡƒ ΠΌΠ°Ρ‚Π΅Ρ€ΠΈΠ°Π»Ρƒ,
    • Π²ΠΎΡΠΏΠΈΡ‚Ρ‹Π²Π°ΡŽΡ‰ΠΈΠ΅ – воспитаниС отвСтствСнного ΠΎΡ‚Π½ΠΎΡˆΠ΅Π½ΠΈΡ ΠΊ ΡƒΡ‡Π΅Π±Π΅, ΠΊΡƒΠ»ΡŒΡ‚ΡƒΡ€Ρ‹ общСния, чувства ΠΊΠΎΠ»Π»Π΅ΠΊΡ‚ΠΈΠ²ΠΈΠ·ΠΌΠ°.

    ΠžΠ±ΠΎΡ€ΡƒΠ΄ΠΎΠ²Π°Π½ΠΈΠ΅: ΠΊΠΎΠΌΠΏΡŒΡŽΡ‚Π΅Ρ€, ΠΌΡƒΠ»ΡŒΡ‚ΠΈΠΌΠ΅Π΄ΠΈΠΉΠ½Ρ‹ΠΉ ΠΏΡ€ΠΎΠ΅ΠΊΡ‚ΠΎΡ€, интСрактивная доска, прСзСнтация β€œΠ‘Ρ‚Π΅ΠΏΠ΅Π½ΠΈβ€ для устного счСта, ΠΊΠ°Ρ€Ρ‚ΠΎΡ‡ΠΊΠΈ с заданиями, Ρ€Π°Π·Π΄Π°Ρ‚ΠΎΡ‡Π½Ρ‹ΠΉ ΠΌΠ°Ρ‚Π΅Ρ€ΠΈΠ°Π».

    План ΡƒΡ€ΠΎΠΊΠ°:

    1. ΠžΡ€Π³Π°Π½ΠΈΠ·Π°Ρ†ΠΈΠΎΠ½Π½Ρ‹ΠΉ ΠΌΠΎΠΌΠ΅Π½Ρ‚.
    2. ΠŸΠΎΠ²Ρ‚ΠΎΡ€Π΅Π½ΠΈΠ΅ ΠΏΡ€Π°Π²ΠΈΠ»
    3. Устный счСт.
    4. Π˜ΡΡ‚ΠΎΡ€ΠΈΡ‡Π΅ΡΠΊΠ°Ρ справка.
    5. Π Π°Π±ΠΎΡ‚Π° Ρƒ доски.
    6. Π€ΠΈΠ·ΠΊΡƒΠ»ΡŒΡ‚ΠΌΠΈΠ½ΡƒΡ‚ΠΊΠ°.
    7. Π Π°Π±ΠΎΡ‚Π° Π½Π° ΠΈΠ½Ρ‚Π΅Ρ€Π°ΠΊΡ‚ΠΈΠ²Π½ΠΎΠΉ доскС.
    8. Π‘Π°ΠΌΠΎΡΡ‚ΠΎΡΡ‚Π΅Π»ΡŒΠ½Π°Ρ Ρ€Π°Π±ΠΎΡ‚Π°.
    9. Π”ΠΎΠΌΠ°ΡˆΠ½Π΅Π΅ Π·Π°Π΄Π°Π½ΠΈΠ΅.
    10. ПодвСдСниС ΠΈΡ‚ΠΎΠ³ΠΎΠ² ΡƒΡ€ΠΎΠΊΠ°.

    Π₯ΠΎΠ΄ ΡƒΡ€ΠΎΠΊΠ°

    I. ΠžΡ€Π³Π°Π½ΠΈΠ·Π°Ρ†ΠΈΠΎΠ½Π½Ρ‹ΠΉ ΠΌΠΎΠΌΠ΅Π½Ρ‚

    Π‘ΠΎΠΎΠ±Ρ‰Π΅Π½ΠΈΠ΅ Ρ‚Π΅ΠΌΡ‹ ΠΈ Ρ†Π΅Π»Π΅ΠΉ ΡƒΡ€ΠΎΠΊΠ°.

    На ΠΏΡ€Π΅Π΄Ρ‹Π΄ΡƒΡ‰ΠΈΡ… ΡƒΡ€ΠΎΠΊΠ°Ρ… Π²Ρ‹ ΠΎΡ‚ΠΊΡ€Ρ‹Π»ΠΈ для сСбя ΡƒΠ΄ΠΈΠ²ΠΈΡ‚Π΅Π»ΡŒΠ½Ρ‹ΠΉ ΠΌΠΈΡ€ стСпСнСй, Π½Π°ΡƒΡ‡ΠΈΠ»ΠΈΡΡŒ ΡƒΠΌΠ½ΠΎΠΆΠ°Ρ‚ΡŒ ΠΈ Π΄Π΅Π»ΠΈΡ‚ΡŒ стСпСни, Π²ΠΎΠ·Π²ΠΎΠ΄ΠΈΡ‚ΡŒ ΠΈΡ… Π² ΡΡ‚Π΅ΠΏΠ΅Π½ΡŒ. БСгодня ΠΌΡ‹ Π΄ΠΎΠ»ΠΆΠ½Ρ‹ Π·Π°ΠΊΡ€Π΅ΠΏΠΈΡ‚ΡŒ ΠΏΠΎΠ»ΡƒΡ‡Π΅Π½Π½Ρ‹Π΅ знания ΠΏΡ€ΠΈ Ρ€Π΅ΡˆΠ΅Π½ΠΈΠΈ ΠΏΡ€ΠΈΠΌΠ΅Ρ€ΠΎΠ².

    II. ΠŸΠΎΠ²Ρ‚ΠΎΡ€Π΅Π½ΠΈΠ΅ ΠΏΡ€Π°Π²ΠΈΠ» (устно)

    1. Π”Π°ΠΉΡ‚Π΅ ΠΎΠΏΡ€Π΅Π΄Π΅Π»Π΅Π½ΠΈΠ΅ стСпСни с Π½Π°Ρ‚ΡƒΡ€Π°Π»ΡŒΠ½Ρ‹ΠΌ ΠΏΠΎΠΊΠ°Π·Π°Ρ‚Π΅Π»Π΅ΠΌ? (Π‘Ρ‚Π΅ΠΏΠ΅Π½ΡŒΡŽ числа Π° с Π½Π°Ρ‚ΡƒΡ€Π°Π»ΡŒΠ½Ρ‹ΠΌ ΠΏΠΎΠΊΠ°Π·Π°Ρ‚Π΅Π»Π΅ΠΌ, большим 1, называСтся ΠΏΡ€ΠΎΠΈΠ·Π²Π΅Π΄Π΅Π½ΠΈΠ΅n ΠΌΠ½ΠΎΠΆΠΈΡ‚Π΅Π»Π΅ΠΉ, ΠΊΠ°ΠΆΠ΄Ρ‹ΠΉ ΠΈΠ· ΠΊΠΎΡ‚ΠΎΡ€Ρ‹Ρ… Ρ€Π°Π²Π΅Π½ Π° . )
    2. Как ΡƒΠΌΠ½ΠΎΠΆΠΈΡ‚ΡŒ Π΄Π²Π΅ стСпСни? (Π§Ρ‚ΠΎΠ±Ρ‹ ΡƒΠΌΠ½ΠΎΠΆΠΈΡ‚ΡŒ стСпСни с ΠΎΠ΄ΠΈΠ½Π°ΠΊΠΎΠ²Ρ‹ΠΌΠΈ основаниями, Π½Π°Π΄ΠΎ основаниС ΠΎΡΡ‚Π°Π²ΠΈΡ‚ΡŒ Ρ‚Π΅ΠΌ ΠΆΠ΅, Π° ΠΏΠΎΠΊΠ°Π·Π°Ρ‚Π΅Π»ΠΈ ΡΠ»ΠΎΠΆΠΈΡ‚ΡŒ.)
    3. Как Ρ€Π°Π·Π΄Π΅Π»ΠΈΡ‚ΡŒ ΡΡ‚Π΅ΠΏΠ΅Π½ΡŒ Π½Π° ΡΡ‚Π΅ΠΏΠ΅Π½ΡŒ? (Π§Ρ‚ΠΎΠ±Ρ‹ Ρ€Π°Π·Π΄Π΅Π»ΠΈΡ‚ΡŒ стСпСни с ΠΎΠ΄ΠΈΠ½Π°ΠΊΠΎΠ²Ρ‹ΠΌΠΈ основаниями, Π½Π°Π΄ΠΎ основаниС ΠΎΡΡ‚Π°Π²ΠΈΡ‚ΡŒ Ρ‚Π΅ΠΌ ΠΆΠ΅, Π° ΠΏΠΎΠΊΠ°Π·Π°Ρ‚Π΅Π»ΠΈ Π²Ρ‹Ρ‡Π΅ΡΡ‚ΡŒ.)
    4. Как возвСсти ΠΏΡ€ΠΎΠΈΠ·Π²Π΅Π΄Π΅Π½ΠΈΠ΅ Π² ΡΡ‚Π΅ΠΏΠ΅Π½ΡŒ? (Π§Ρ‚ΠΎΠ±Ρ‹ возвСсти ΠΏΡ€ΠΎΠΈΠ·Π²Π΅Π΄Π΅Π½ΠΈΠ΅ Π² ΡΡ‚Π΅ΠΏΠ΅Π½ΡŒ, Π½Π°Π΄ΠΎ ΠΊΠ°ΠΆΠ΄Ρ‹ΠΉ ΠΌΠ½ΠΎΠΆΠΈΡ‚Π΅Π»ΡŒ возвСсти Π² эту ΡΡ‚Π΅ΠΏΠ΅Π½ΡŒ)
    5. Как возвСсти ΡΡ‚Π΅ΠΏΠ΅Π½ΡŒ Π² ΡΡ‚Π΅ΠΏΠ΅Π½ΡŒ? (Π§Ρ‚ΠΎΠ±Ρ‹ возвСсти ΡΡ‚Π΅ΠΏΠ΅Π½ΡŒ Π² ΡΡ‚Π΅ΠΏΠ΅Π½ΡŒ, Π½Π°Π΄ΠΎ основаниС ΠΎΡΡ‚Π°Π²ΠΈΡ‚ΡŒ Ρ‚Π΅ΠΌ ΠΆΠ΅, Π° ΠΏΠΎΠΊΠ°Π·Π°Ρ‚Π΅Π»ΠΈ ΠΏΠ΅Ρ€Π΅ΠΌΠ½ΠΎΠΆΠΈΡ‚ΡŒ)

    III. Устный счСт (ΠΏΠΎ ΠΌΡƒΠ»ΡŒΡ‚ΠΈΠΌΠ΅Π΄ΠΈΠ°)

    IV. Π˜ΡΡ‚ΠΎΡ€ΠΈΡ‡Π΅ΡΠΊΠ°Ρ справка

    ВсС Π·Π°Π΄Π°Ρ‡ΠΈ ΠΈΠ· папируса АхмСса, ΠΊΠΎΡ‚ΠΎΡ€Ρ‹ΠΉ записан ΠΎΠΊΠΎΠ»ΠΎ 1650 Π³ΠΎΠ΄Π° Π΄ΠΎ Π½. э. связаны с ΠΏΡ€Π°ΠΊΡ‚ΠΈΠΊΠΎΠΉ ΡΡ‚Ρ€ΠΎΠΈΡ‚Π΅Π»ΡŒΡΡ‚Π²Π°, Ρ€Π°Π·ΠΌΠ΅ΠΆΠ΅Π²Π°Π½ΠΈΠ΅ΠΌ Π·Π΅ΠΌΠ΅Π»ΡŒΠ½Ρ‹Ρ… Π½Π°Π΄Π΅Π»ΠΎΠ² ΠΈ Ρ‚. ΠΏ. Π—Π°Π΄Π°Ρ‡ΠΈ сгруппированы ΠΏΠΎ Ρ‚Π΅ΠΌΠ°Ρ‚ΠΈΠΊΠ΅. По прСимущСству это Π·Π°Π΄Π°Ρ‡ΠΈ Π½Π° Π½Π°Ρ…ΠΎΠΆΠ΄Π΅Π½ΠΈΠ΅ ΠΏΠ»ΠΎΡ‰Π°Π΄Π΅ΠΉ Ρ‚Ρ€Π΅ΡƒΠ³ΠΎΠ»ΡŒΠ½ΠΈΠΊΠ°, Ρ‡Π΅Ρ‚Ρ‹Ρ€Ρ‘Ρ…ΡƒΠ³ΠΎΠ»ΡŒΠ½ΠΈΠΊΠΎΠ² ΠΈ ΠΊΡ€ΡƒΠ³Π°, Ρ€Π°Π·Π½ΠΎΠΎΠ±Ρ€Π°Π·Π½Ρ‹Π΅ дСйствия с Ρ†Π΅Π»Ρ‹ΠΌΠΈ числами ΠΈ дробями, ΠΏΡ€ΠΎΠΏΠΎΡ€Ρ†ΠΈΠΎΠ½Π°Π»ΡŒΠ½ΠΎΠ΅ Π΄Π΅Π»Π΅Π½ΠΈΠ΅, Π½Π°Ρ…ΠΎΠΆΠ΄Π΅Π½ΠΈΠ΅ ΠΎΡ‚Π½ΠΎΡˆΠ΅Π½ΠΈΠΉ, здСсь присутствуСт ΠΈ Π²ΠΎΠ·Π²Π΅Π΄Π΅Π½ΠΈΠ΅ Π² Ρ€Π°Π·Π½Ρ‹Π΅ стСпСни, Ρ€Π΅ΡˆΠ΅Π½ΠΈΠ΅ ΡƒΡ€Π°Π²Π½Π΅Π½ΠΈΠΉ ΠΏΠ΅Ρ€Π²ΠΎΠΉ ΠΈ Π²Ρ‚ΠΎΡ€ΠΎΠΉ стСпСни с ΠΎΠ΄Π½ΠΈΠΌ нСизвСстным.

    ΠŸΠΎΠ»Π½ΠΎΡΡ‚ΡŒΡŽ ΠΎΡ‚ΡΡƒΡ‚ΡΡ‚Π²ΡƒΡŽΡ‚ ΠΊΠ°ΠΊΠΈΠ΅ Π±Ρ‹ Ρ‚ΠΎ Π½ΠΈ Π±Ρ‹Π»ΠΎ объяснСния ΠΈΠ»ΠΈ Π΄ΠΎΠΊΠ°Π·Π°Ρ‚Π΅Π»ΡŒΡΡ‚Π²Π°. Π˜ΡΠΊΠΎΠΌΡ‹ΠΉ Ρ€Π΅Π·ΡƒΠ»ΡŒΡ‚Π°Ρ‚ Π»ΠΈΠ±ΠΎ даётся прямо, Π»ΠΈΠ±ΠΎ приводится ΠΊΡ€Π°Ρ‚ΠΊΠΈΠΉ Π°Π»Π³ΠΎΡ€ΠΈΡ‚ΠΌ Π΅Π³ΠΎ вычислСния. Π’Π°ΠΊΠΎΠΉ способ излоТСния, Ρ‚ΠΈΠΏΠΈΡ‡Π½Ρ‹ΠΉ для Π½Π°ΡƒΠΊΠΈ стран Π΄Ρ€Π΅Π²Π½Π΅Π³ΠΎ Востока, Π½Π°Π²ΠΎΠ΄ΠΈΡ‚ Π½Π° ΠΌΡ‹ΡΠ»ΡŒ ΠΎ Ρ‚ΠΎΠΌ, Ρ‡Ρ‚ΠΎ ΠΌΠ°Ρ‚Π΅ΠΌΠ°Ρ‚ΠΈΠΊΠ° Ρ‚Π°ΠΌ Ρ€Π°Π·Π²ΠΈΠ²Π°Π»Π°ΡΡŒ ΠΏΡƒΡ‚Ρ‘ΠΌ ΠΎΠ±ΠΎΠ±Ρ‰Π΅Π½ΠΈΠΉ ΠΈ Π΄ΠΎΠ³Π°Π΄ΠΎΠΊ, Π½Π΅ ΠΎΠ±Ρ€Π°Π·ΡƒΡŽΡ‰ΠΈΡ… Π½ΠΈΠΊΠ°ΠΊΠΎΠΉ ΠΎΠ±Ρ‰Π΅ΠΉ Ρ‚Π΅ΠΎΡ€ΠΈΠΈ. Π’Π΅ΠΌ Π½Π΅ ΠΌΠ΅Π½Π΅Π΅, Π² папирусС Π΅ΡΡ‚ΡŒ Ρ†Π΅Π»Ρ‹ΠΉ ряд ΡΠ²ΠΈΠ΄Π΅Ρ‚Π΅Π»ΡŒΡΡ‚Π² Ρ‚ΠΎΠ³ΠΎ, Ρ‡Ρ‚ΠΎ СгипСтскиС ΠΌΠ°Ρ‚Π΅ΠΌΠ°Ρ‚ΠΈΠΊΠΈ ΡƒΠΌΠ΅Π»ΠΈ ΠΈΠ·Π²Π»Π΅ΠΊΠ°Ρ‚ΡŒ ΠΊΠΎΡ€Π½ΠΈ ΠΈ Π²ΠΎΠ·Π²ΠΎΠ΄ΠΈΡ‚ΡŒ Π² ΡΡ‚Π΅ΠΏΠ΅Π½ΡŒ, Ρ€Π΅ΡˆΠ°Ρ‚ΡŒ уравнСния, ΠΈ Π΄Π°ΠΆΠ΅ Π²Π»Π°Π΄Π΅Π»ΠΈ Π·Π°Ρ‡Π°Ρ‚ΠΊΠ°ΠΌΠΈ Π°Π»Π³Π΅Π±Ρ€Ρ‹.

    V. Π Π°Π±ΠΎΡ‚Π° Ρƒ доски

    НайдитС Π·Π½Π°Ρ‡Π΅Π½ΠΈΠ΅ выраТСния Ρ€Π°Ρ†ΠΈΠΎΠ½Π°Π»ΡŒΠ½Ρ‹ΠΌ способом:

    ВычислитС Π·Π½Π°Ρ‡Π΅Π½ΠΈΠ΅ выраТСния:

    VI. Π€ΠΈΠ·ΠΊΡƒΠ»ΡŒΡ‚ΠΌΠΈΠ½ΡƒΡ‚ΠΊΠ°

    1. для глаз
    2. для шСи
    3. для Ρ€ΡƒΠΊ
    4. для Ρ‚ΡƒΠ»ΠΎΠ²ΠΈΡ‰Π°
    5. для ног

    VII. РСшСниС Π·Π°Π΄Π°Ρ‡ (с ΠΏΠΎΠΊΠ°Π·ΠΎΠΌ Π½Π° ΠΈΠ½Ρ‚Π΅Ρ€Π°ΠΊΡ‚ΠΈΠ²Π½ΠΎΠΉ доскС)

    ЯвляСтся Π»ΠΈ ΠΊΠΎΡ€Π΅Π½ΡŒ уравнСния ΠΏΠΎΠ»ΠΎΠΆΠΈΡ‚Π΅Π»ΡŒΠ½Ρ‹ΠΌ числом?

    Π°) 3x + (-0,1) 7 = (-0,496) 4 (x > 0)

    Π±) (10,381) 5 = (-0,012) 3 — 2x (x

    VIII. Π‘Π°ΠΌΠΎΡΡ‚ΠΎΡΡ‚Π΅Π»ΡŒΠ½Π°Ρ Ρ€Π°Π±ΠΎΡ‚Π°

    IX. Π”ΠΎΠΌΠ°ΡˆΠ½Π΅Π΅ Π·Π°Π΄Π°Π½ΠΈΠ΅

    Π₯. ПодвСдСниС ΠΈΡ‚ΠΎΠ³ΠΎΠ² ΡƒΡ€ΠΎΠΊΠ°

    Анализ Ρ€Π΅Π·ΡƒΠ»ΡŒΡ‚Π°Ρ‚ΠΎΠ², объявлСниС ΠΎΡ†Π΅Π½ΠΎΠΊ.

    ΠŸΠΎΠ»ΡƒΡ‡Π΅Π½Π½Ρ‹Π΅ знания ΠΎ стСпСнях ΠΌΡ‹ Π±ΡƒΠ΄Π΅ΠΌ ΠΏΡ€ΠΈΠΌΠ΅Π½ΡΡ‚ΡŒ ΠΏΡ€ΠΈ Ρ€Π΅ΡˆΠ΅Π½ΠΈΠΈ ΡƒΡ€Π°Π²Π½Π΅Π½ΠΈΠΉ, Π·Π°Π΄Π°Ρ‡ Π² ΡΡ‚Π°Ρ€ΡˆΠΈΡ… классах, Ρ‚Π°ΠΊΠΆΠ΅ ΠΎΠ½ΠΈ часто Π²ΡΡ‚Ρ€Π΅Ρ‡Π°ΡŽΡ‚ΡΡ Π² Π•Π“Π­.

    Π€ΠΎΡ€ΠΌΡƒΠ»Ρ‹ стСпСнСй ΠΈΡΠΏΠΎΠ»ΡŒΠ·ΡƒΡŽΡ‚ Π² процСссС сокращСния ΠΈ упрощСния слоТных Π²Ρ‹Ρ€Π°ΠΆΠ΅Π½ΠΈΠΉ, Π² Ρ€Π΅ΡˆΠ΅Π½ΠΈΠΈ ΡƒΡ€Π°Π²Π½Π΅Π½ΠΈΠΉ ΠΈ нСравСнств.

    Число c являСтся n -Π½ΠΎΠΉ ΡΡ‚Π΅ΠΏΠ΅Π½ΡŒΡŽ числа a ΠΊΠΎΠ³Π΄Π°:

    ΠžΠΏΠ΅Ρ€Π°Ρ†ΠΈΠΈ со стСпСнями.

    1. УмноТая стСпСни с ΠΎΠ΄ΠΈΠ½Π°ΠΊΠΎΠ²Ρ‹ΠΌ основаниСм ΠΈΡ… ΠΏΠΎΠΊΠ°Π·Π°Ρ‚Π΅Π»ΠΈ ΡΠΊΠ»Π°Π΄Ρ‹Π²Π°ΡŽΡ‚ΡΡ:

    a m Β·a n = a m + n .

    2. Π’ Π΄Π΅Π»Π΅Π½ΠΈΠΈ стСпСнСй с ΠΎΠ΄ΠΈΠ½Π°ΠΊΠΎΠ²Ρ‹ΠΌ основаниСм ΠΈΡ… ΠΏΠΎΠΊΠ°Π·Π°Ρ‚Π΅Π»ΠΈ Π²Ρ‹Ρ‡ΠΈΡ‚Π°ΡŽΡ‚ΡΡ:

    3. Π‘Ρ‚Π΅ΠΏΠ΅Π½ΡŒ произвСдСния 2-Ρ… Π»ΠΈΠ±ΠΎ большСго числа ΠΌΠ½ΠΎΠΆΠΈΡ‚Π΅Π»Π΅ΠΉ равняСтся ΠΏΡ€ΠΎΠΈΠ·Π²Π΅Π΄Π΅Π½ΠΈΡŽ стСпСнСй этих сомноТитСлСй:

    (abc…) n = a n Β· b n Β· c n …

    4. Π‘Ρ‚Π΅ΠΏΠ΅Π½ΡŒ Π΄Ρ€ΠΎΠ±ΠΈ равняСтся ΠΎΡ‚Π½ΠΎΡˆΠ΅Π½ΠΈΡŽ стСпСнСй Π΄Π΅Π»ΠΈΠΌΠΎΠ³ΠΎ ΠΈ дСлитСля:

    (a/b) n = a n /b n .

    5. Возводя ΡΡ‚Π΅ΠΏΠ΅Π½ΡŒ Π² ΡΡ‚Π΅ΠΏΠ΅Π½ΡŒ, ΠΏΠΎΠΊΠ°Π·Π°Ρ‚Π΅Π»ΠΈ стСпСнСй ΠΏΠ΅Ρ€Π΅ΠΌΠ½ΠΎΠΆΠ°ΡŽΡ‚:

    (a m) n = a m n .

    КаТдая Π²Ρ‹ΡˆΠ΅ΠΏΡ€ΠΈΠ²Π΅Π΄Π΅Π½Π½Π°Ρ Ρ„ΠΎΡ€ΠΌΡƒΠ»Π° Π²Π΅Ρ€Π½Π° Π² направлСниях слСва Π½Π°ΠΏΡ€Π°Π²ΠΎ ΠΈ Π½Π°ΠΎΠ±ΠΎΡ€ΠΎΡ‚.

    НапримСр . (2Β·3Β·5/15)Β² = 2Β²Β·3Β²Β·5Β²/15Β² = 900/225 = 4 .

    ΠžΠΏΠ΅Ρ€Π°Ρ†ΠΈΠΈ с корнями.

    1. ΠšΠΎΡ€Π΅Π½ΡŒ ΠΈΠ· произвСдСния Π½Π΅ΡΠΊΠΎΠ»ΡŒΠΊΠΈΡ… сомноТитСлСй равняСтся ΠΏΡ€ΠΎΠΈΠ·Π²Π΅Π΄Π΅Π½ΠΈΡŽ ΠΊΠΎΡ€Π½Π΅ΠΉ ΠΈΠ· этих сомноТитСлСй:

    2. ΠšΠΎΡ€Π΅Π½ΡŒ ΠΈΠ· ΠΎΡ‚Π½ΠΎΡˆΠ΅Π½ΠΈΡ Ρ€Π°Π²Π΅Π½ ΠΎΡ‚Π½ΠΎΡˆΠ΅Π½ΠΈΡŽ Π΄Π΅Π»ΠΈΠΌΠΎΠ³ΠΎ ΠΈ дСлитСля ΠΊΠΎΡ€Π½Π΅ΠΉ:

    3. ΠŸΡ€ΠΈ Π²ΠΎΠ·Π²Π΅Π΄Π΅Π½ΠΈΠΈ корня Π² ΡΡ‚Π΅ΠΏΠ΅Π½ΡŒ довольно возвСсти Π² эту ΡΡ‚Π΅ΠΏΠ΅Π½ΡŒ ΠΏΠΎΠ΄ΠΊΠΎΡ€Π΅Π½Π½ΠΎΠ΅ число:

    4. Если ΡƒΠ²Π΅Π»ΠΈΡ‡ΠΈΡ‚ΡŒ ΡΡ‚Π΅ΠΏΠ΅Π½ΡŒ корня Π² n Ρ€Π°Π· ΠΈ Π² Ρ‚ΠΎΠΆΠ΅ врСмя возвСсти Π² n -ΡƒΡŽ ΡΡ‚Π΅ΠΏΠ΅Π½ΡŒ ΠΏΠΎΠ΄ΠΊΠΎΡ€Π΅Π½Π½ΠΎΠ΅ число, Ρ‚ΠΎ Π·Π½Π°Ρ‡Π΅Π½ΠΈΠ΅ корня Π½Π΅ помСняСтся:

    5. Если ΡƒΠΌΠ΅Π½ΡŒΡˆΠΈΡ‚ΡŒ ΡΡ‚Π΅ΠΏΠ΅Π½ΡŒ корня Π² n Ρ€Π°Π· ΠΈ Π² Ρ‚ΠΎΠΆΠ΅ врСмя ΠΈΠ·Π²Π»Π΅Ρ‡ΡŒ ΠΊΠΎΡ€Π΅Π½ΡŒ n -ΠΎΠΉ стСпСни ΠΈΠ· ΠΏΠΎΠ΄ΠΊΠΎΡ€Π΅Π½Π½ΠΎΠ³ΠΎ числа, Ρ‚ΠΎ Π·Π½Π°Ρ‡Π΅Π½ΠΈΠ΅ корня Π½Π΅ помСняСтся:

    Π‘Ρ‚Π΅ΠΏΠ΅Π½ΡŒ с ΠΎΡ‚Ρ€ΠΈΡ†Π°Ρ‚Π΅Π»ΡŒΠ½Ρ‹ΠΌ ΠΏΠΎΠΊΠ°Π·Π°Ρ‚Π΅Π»Π΅ΠΌ. Π‘Ρ‚Π΅ΠΏΠ΅Π½ΡŒ Π½Π΅ΠΊΠΎΡ‚ΠΎΡ€ΠΎΠ³ΠΎ числа с Π½Π΅ΠΏΠΎΠ»ΠΎΠΆΠΈΡ‚Π΅Π»ΡŒΠ½Ρ‹ΠΌ (Ρ†Π΅Π»Ρ‹ΠΌ) ΠΏΠΎΠΊΠ°Π·Π°Ρ‚Π΅Π»Π΅ΠΌ ΠΎΠΏΡ€Π΅Π΄Π΅Π»ΡΡŽΡ‚ ΠΊΠ°ΠΊ Π΅Π΄ΠΈΠ½ΠΈΡ†Ρƒ, Π΄Π΅Π»Π΅Π½Π½ΡƒΡŽ Π½Π° ΡΡ‚Π΅ΠΏΠ΅Π½ΡŒ Ρ‚ΠΎΠ³ΠΎ ΠΆΠ΅ числа с ΠΏΠΎΠΊΠ°Π·Π°Ρ‚Π΅Π»Π΅ΠΌ, Ρ€Π°Π²Π½Ρ‹ΠΌ Π°Π±ΡΠΎΠ»ΡŽΡ‚Π½ΠΎΠΉ Π²Π΅Π»ΠΈΡ‡ΠΈΠ½Π΅ Π½Π΅ΠΏΠΎΠ»ΠΎΠΆΠΈΡ‚Π΅Π»ΡŒΠ½ΠΎΠ³ΠΎ показатСля:

    Π€ΠΎΡ€ΠΌΡƒΠ»Ρƒ a m :a n =a m — n ΠΌΠΎΠΆΠ½ΠΎ ΠΈΡΠΏΠΎΠ»ΡŒΠ·ΠΎΠ²Π°Ρ‚ΡŒ Π½Π΅ Ρ‚ΠΎΠ»ΡŒΠΊΠΎ ΠΏΡ€ΠΈ m > n , Π½ΠΎ ΠΈ ΠΏΡ€ΠΈ m n .

    НапримСр . a 4:a 7 = a 4 — 7 = a -3 .

    Π§Ρ‚ΠΎΠ±Ρ‹ Ρ„ΠΎΡ€ΠΌΡƒΠ»Π° a m :a n =a m — n стала справСдливой ΠΏΡ€ΠΈ m=n , Π½ΡƒΠΆΠ½ΠΎ присутствиС Π½ΡƒΠ»Π΅Π²ΠΎΠΉ стСпСни.

    Π‘Ρ‚Π΅ΠΏΠ΅Π½ΡŒ с Π½ΡƒΠ»Π΅Π²Ρ‹ΠΌ ΠΏΠΎΠΊΠ°Π·Π°Ρ‚Π΅Π»Π΅ΠΌ. Π‘Ρ‚Π΅ΠΏΠ΅Π½ΡŒ всякого числа, Π½Π΅ Ρ€Π°Π²Π½ΠΎΠ³ΠΎ Π½ΡƒΠ»ΡŽ, с Π½ΡƒΠ»Π΅Π²Ρ‹ΠΌ ΠΏΠΎΠΊΠ°Π·Π°Ρ‚Π΅Π»Π΅ΠΌ равняСтся Π΅Π΄ΠΈΠ½ΠΈΡ†Π΅.

    НапримСр . 2 0 = 1,(-5) 0 = 1,(-3/5) 0 = 1.

    Π‘Ρ‚Π΅ΠΏΠ΅Π½ΡŒ с Π΄Ρ€ΠΎΠ±Π½Ρ‹ΠΌ ΠΏΠΎΠΊΠ°Π·Π°Ρ‚Π΅Π»Π΅ΠΌ. Π§Ρ‚ΠΎΠ±Ρ‹ возвСсти Π΄Π΅ΠΉΡΡ‚Π²ΠΈΡ‚Π΅Π»ΡŒΠ½ΠΎΠ΅ число Π° Π² ΡΡ‚Π΅ΠΏΠ΅Π½ΡŒ m/n , Π½Π΅ΠΎΠ±Ρ…ΠΎΠ΄ΠΈΠΌΠΎ ΠΈΠ·Π²Π»Π΅Ρ‡ΡŒ ΠΊΠΎΡ€Π΅Π½ΡŒ n -ΠΎΠΉ стСпСни ΠΈΠ· m -ΠΎΠΉ стСпСни этого числа Π° .

    Π§Ρ‚ΠΎ Π΄Π΅Π»Π°Ρ‚ΡŒ со стСпСнями ΠΏΡ€ΠΈ слоТСнии чисСл?


    Π§Ρ‚ΠΎ Π΄Π΅Π»Π°Ρ‚ΡŒ со стСпСнями ΠΏΡ€ΠΈ слоТСнии чисСл?

    Как ΡΠΊΠ»Π°Π΄Ρ‹Π²Π°Ρ‚ΡŒ числа с ΠΎΠ΄ΠΈΠ½Π°ΠΊΠΎΠ²Ρ‹ΠΌΠΈ стСпСнями Если стСпСни ΠΎΠ΄ΠΈΠ½Π°ΠΊΠΎΠ²Ρ‹Π΅, Π° основания Ρ€Π°Π·Π½Ρ‹Π΅, Ρ‚ΠΎ нСльзя ΡΠ»ΠΎΠΆΠΈΡ‚ΡŒ основания ΠΈ Π·Π°Ρ‚Π΅ΠΌ эту сумму Π²ΠΎΠ·Π²ΠΎΠ΄ΠΈΡ‚ΡŒ Π² ΡΡ‚Π΅ΠΏΠ΅Π½ΡŒ. Π‘Π½Π°Ρ‡Π°Π»Π° Π²ΠΎΠ·Π²ΠΎΠ΄ΠΈΠΌ ΠΊΠ°ΠΆΠ΄ΠΎΠ΅ число Π² ΡΡ‚Π΅ΠΏΠ΅Π½ΡŒ ΠΈ Π·Π°Ρ‚Π΅ΠΌ выполняСм слоТСниС. Π’ уравнСниях это Π±ΡƒΠ΄Π΅Ρ‚ ΠΏΡ€ΠΎΠΈΡΡ…ΠΎΠ΄ΠΈΡ‚ΡŒ Π½Π΅ΠΌΠ½ΠΎΠ³ΠΎ ΠΈΠ½Π°Ρ‡Π΅.

    Π§Ρ‚ΠΎ происходит с показатСлями стСпСни ΠΏΡ€ΠΈ слоТСнии?

    Бвойство β„– 1 ΠŸΡ€ΠΈ ΡƒΠΌΠ½ΠΎΠΆΠ΅Π½ΠΈΠΈ стСпСнСй с ΠΎΠ΄ΠΈΠ½Π°ΠΊΠΎΠ²Ρ‹ΠΌΠΈ основаниями основаниС остаётся Π±Π΅Π· ΠΈΠ·ΠΌΠ΅Π½Π΅Π½ΠΈΠΉ, Π° ΠΏΠΎΠΊΠ°Π·Π°Ρ‚Π΅Π»ΠΈ стСпСнСй ΡΠΊΠ»Π°Π΄Ρ‹Π²Π°ΡŽΡ‚ΡΡ. am Β· an = am + n, Π³Π΄Π΅ Β«aΒ» β€” любоС число, Π° Β«mΒ», Β«nΒ» β€” Π»ΡŽΠ±Ρ‹Π΅ Π½Π°Ρ‚ΡƒΡ€Π°Π»ΡŒΠ½Ρ‹Π΅ числа. Π”Π°Π½Π½ΠΎΠ΅ свойство стСпСнСй Ρ‚Π°ΠΊΠΆΠ΅ дСйствуСт Π½Π° ΠΏΡ€ΠΎΠΈΠ·Π²Π΅Π΄Π΅Π½ΠΈΠ΅ Ρ‚Ρ€Ρ‘Ρ… ΠΈ Π±ΠΎΠ»Π΅Π΅ стСпСнСй. ΠŸΡ€ΠΈΠΌΠ΅Ρ€Ρ‹.

    Как Ρ€Π°Π·Π΄Π΅Π»ΠΈΡ‚ΡŒ стСпСни с ΠΎΠ΄ΠΈΠ½Π°ΠΊΠΎΠ²Ρ‹ΠΌΠΈ основаниями?

    Π§Ρ‚ΠΎΠ±Ρ‹ Ρ€Π°Π·Π΄Π΅Π»ΠΈΡ‚ΡŒ Π΄Ρ€ΡƒΠ³ Π½Π° Π΄Ρ€ΡƒΠ³Π° стСпСни с ΠΎΠ΄ΠΈΠ½Π°ΠΊΠΎΠ²Ρ‹ΠΌΠΈ показатСлями, достаточно Ρ€Π°Π·Π΄Π΅Π»ΠΈΡ‚ΡŒ ΠΎΠ΄Π½ΠΎ основаниС Π½Π° Π΄Ρ€ΡƒΠ³ΠΎΠ΅, Π° ΠΏΠΎΠΊΠ°Π·Π°Ρ‚Π΅Π»ΡŒ стСпСни ΠΎΡΡ‚Π°Π²ΠΈΡ‚ΡŒ Π½Π΅ΠΈΠ·ΠΌΠ΅Π½Π½Ρ‹ΠΌ.

    Π§Ρ‚ΠΎ Π½ΡƒΠΆΠ½ΠΎ Π΄Π΅Π»Π°Ρ‚ΡŒ со стСпСнями ΠΏΡ€ΠΈ Π΄Π΅Π»Π΅Π½ΠΈΠΈ?

    ΠŸΡ€ΠΈ Π΄Π΅Π»Π΅Π½ΠΈΠΈ стСпСнСй с ΠΎΠ΄ΠΈΠ½Π°ΠΊΠΎΠ²Ρ‹ΠΌ основаниСм ΠΈΡ… ΠΏΠΎΠΊΠ°Π·Π°Ρ‚Π΅Π»ΠΈ Π²Ρ‹Ρ‡ΠΈΡ‚Π°ΡŽΡ‚ΡΡ. … Π‘Ρ‚Π΅ΠΏΠ΅Π½ΡŒ произвСдСния Π΄Π²ΡƒΡ… ΠΈΠ»ΠΈ Π½Π΅ΡΠΊΠΎΠ»ΡŒΠΊΠΈΡ… сомноТитСлСй Ρ€Π°Π²Π½Π° ΠΏΡ€ΠΎΠΈΠ·Π²Π΅Π΄Π΅Π½ΠΈΡŽ стСпСнСй этих сомноТитСлСй.

    Как ΡƒΠΌΠ½ΠΎΠΆΠ°Ρ‚ΡŒ ΠΈ Π΄Π΅Π»ΠΈΡ‚ΡŒ числа с Ρ€Π°Π·Π½Ρ‹ΠΌΠΈ основаниями ΠΈ стСпСнями?

    Как ΠΏΠ΅Ρ€Π΅ΠΌΠ½ΠΎΠΆΠΈΡ‚ΡŒ стСпСни с Ρ€Π°Π·Π½Ρ‹ΠΌΠΈ основаниями Π² Π²ΠΈΠ΄Π΅ чисСл?

    1. Если Π½Π°Π΄ΠΎ ΡƒΠΌΠ½ΠΎΠΆΠΈΡ‚ΡŒ Π΄Π²Π° числа с ΠΎΠ΄ΠΈΠ½Π°ΠΊΠΎΠ²Ρ‹ΠΌΠΈ основаниями, Π½ΠΎ Ρ€Π°Π·Π½Ρ‹ΠΌΠΈ показатСлями стСпСнСй, Ρ‚ΠΎ ΠΎΠ±Ρ‰Π΅Π΅ основаниС возводится Π² сумму стСпСнСй.
    2. Если основания Ρ€Π°Π·Π½Ρ‹Π΅, Π° ΠΏΠΎΠΊΠ°Π·Π°Ρ‚Π΅Π»ΠΈ ΠΎΠ΄ΠΈΠ½Π°ΠΊΠΎΠ²Ρ‹Π΅, Ρ‚ΠΎ Π½ΡƒΠΆΠ½ΠΎ Π²ΠΎΠ·Π²ΠΎΠ΄ΠΈΡ‚ΡŒ Π² ΡΡ‚Π΅ΠΏΠ΅Π½ΡŒ ΠΏΡ€ΠΎΠΈΠ·Π²Π΅Π΄Π΅Π½ΠΈΠ΅ оснований.

    Как Ρ€Π΅ΡˆΠΈΡ‚ΡŒ число Π² стСпСни?

    Π§Ρ‚ΠΎΠ±Ρ‹ Π²Ρ‹Ρ‡ΠΈΡΠ»ΠΈΡ‚ΡŒ Π·Π½Π°Ρ‡Π΅Π½ΠΈΠ΅ стСпСни, Π½ΡƒΠΆΠ½ΠΎ Π²Ρ‹ΠΏΠΎΠ»Π½ΠΈΡ‚ΡŒ дСйствиС умноТСния, Ρ‚ΠΎ Π΅ΡΡ‚ΡŒ ΠΏΠ΅Ρ€Π΅ΠΌΠ½ΠΎΠΆΠΈΡ‚ΡŒ основания стСпСни ΡƒΠΊΠ°Π·Π°Π½Π½ΠΎΠ΅ число Ρ€Π°Π·. На ΡƒΠΌΠ΅Π½ΠΈΠΈ быстро ΡƒΠΌΠ½ΠΎΠΆΠ°Ρ‚ΡŒ ΠΈ основано само понятиС стСпСни с Π½Π°Ρ‚ΡƒΡ€Π°Π»ΡŒΠ½Ρ‹ΠΌ ΠΏΠΎΠΊΠ°Π·Π°Ρ‚Π΅Π»Π΅ΠΌ.

    Как считаСтся ΡΡ‚Π΅ΠΏΠ΅Π½ΡŒ числа?

    Π‘Ρ‚Π΅ΠΏΠ΅Π½ΡŒΡŽ числа Β«aΒ» с Π½Π°Ρ‚ΡƒΡ€Π°Π»ΡŒΠ½Ρ‹ΠΌ ΠΏΠΎΠΊΠ°Π·Π°Ρ‚Π΅Π»Π΅ΠΌ Β«nΒ», бóльшим 1, называСтся ΠΏΡ€ΠΎΠΈΠ·Π²Π΅Π΄Π΅Π½ΠΈΠ΅ Β«nΒ» ΠΎΠ΄ΠΈΠ½Π°ΠΊΠΎΠ²Ρ‹Ρ… ΠΌΠ½ΠΎΠΆΠΈΡ‚Π΅Π»Π΅ΠΉ, ΠΊΠ°ΠΆΠ΄Ρ‹ΠΉ ΠΈΠ· ΠΊΠΎΡ‚ΠΎΡ€Ρ‹Ρ… Ρ€Π°Π²Π΅Π½ числу Β«aΒ». Π—Π°ΠΏΠΈΡΡŒ Β«anΒ» читаСтся Ρ‚Π°ΠΊ: Β«Π° Π² стСпСни nΒ» ΠΈΠ»ΠΈ Β«n-ая ΡΡ‚Π΅ΠΏΠ΅Π½ΡŒ числа aΒ».

    Когда ΡƒΠΌΠ½ΠΎΠΆΠ°ΡŽΡ‚ΡΡ стСпСни?

    Когда Π²ΠΎΠ·Π²ΠΎΠ΄ΠΈΠΌ ΡΡ‚Π΅ΠΏΠ΅Π½ΡŒ Π² ΡΡ‚Π΅ΠΏΠ΅Π½ΡŒ, Ρ‚ΠΎ основаниС стСпСни остаСтся Π½Π΅ΠΈΠ·ΠΌΠ΅Π½Π΅Π½Π½Ρ‹ΠΌ, Π° ΠΏΠΎΠΊΠ°Π·Π°Ρ‚Π΅Π»ΠΈ стСпСнСй ΡƒΠΌΠ½ΠΎΠΆΠ°ΡŽΡ‚ΡΡ Π΄Ρ€ΡƒΠ³ Π½Π° Π΄Ρ€ΡƒΠ³Π°.

    Как Ρ€Π°Π·Π΄Π΅Π»ΠΈΡ‚ΡŒ число Π² ΠΎΡ‚Ρ€ΠΈΡ†Π°Ρ‚Π΅Π»ΡŒΠ½ΠΎΠΉ стСпСни?

    ΠŸΡ€ΠΈ Π΄Π΅Π»Π΅Π½ΠΈΠΈ стСпСнСй с ΠΎΠ΄ΠΈΠ½Π°ΠΊΠΎΠ²Ρ‹ΠΌΠΈ основаниями ΠΈΠ· показатСля стСпСни Π΄Π΅Π»ΠΈΠΌΠΎΠ³ΠΎ Π²Ρ‹Ρ‡ΠΈΡ‚Π°ΡŽΡ‚ ΠΏΠΎΠΊΠ°Π·Π°Ρ‚Π΅Π»ΡŒ стСпСни дСлитСля. Π‘Π»Π΅Π΄ΠΎΠ²Π°Ρ‚Π΅Π»ΡŒΠ½ΠΎ, Ссли ΡΡ‚Π΅ΠΏΠ΅Π½ΡŒ Π΄Π΅Π»ΠΈΠΌΠΎΠ³ΠΎ Π±ΡƒΠ΄Π΅Ρ‚ мСньшС стСпСни дСлитСля, Ρ‚ΠΎ Π² Ρ€Π΅Π·ΡƒΠ»ΡŒΡ‚Π°Ρ‚Π΅ получится число с ΠΎΡ‚Ρ€ΠΈΡ†Π°Ρ‚Π΅Π»ΡŒΠ½ΠΎΠΉ ΡΡ‚Π΅ΠΏΠ΅Π½ΡŒΡŽ: a 5 : a 8 = a5 — 8 = a -3. = x -2.

    Как ΡƒΠΌΠ½ΠΎΠΆΠΈΡ‚ΡŒ число Π½Π° ΡΡ‚Π΅ΠΏΠ΅Π½ΡŒ?

    Если Π³ΠΎΠ²ΠΎΡ€ΠΈΡ‚ΡŒ ΠΏΡ€ΠΎΡ‰Π΅ Ρ‚ΠΎ, ΡΡ‚Π΅ΠΏΠ΅Π½ΡŒ, Π° Ρ‚ΠΎΡ‡Π½Π΅Π΅ ΠΏΠΎΠΊΠ°Π·Π°Ρ‚Π΅Π»ΡŒ стСпСни (n), Π³ΠΎΠ²ΠΎΡ€ΠΈΡ‚ Π½Π°ΠΌ ΠΎ Ρ‚ΠΎΠΌ, сколько Ρ€Π°Π· слСдуСт ΡƒΠΌΠ½ΠΎΠΆΠΈΡ‚ΡŒ это число (основаниС стСпСни) Π½Π° само сСбя. А Π·Π½Π°Ρ‡ΠΈΡ‚, Ссли Ρƒ нас Π΅ΡΡ‚ΡŒ Π·Π°Π΄Π°Ρ‡ΠΊΠ°, Π³Π΄Π΅ ΡΠΏΡ€Π°ΡˆΠΈΠ²Π°ΡŽΡ‚, ΠΊΠ°ΠΊ возвСсти число Π² ΡΡ‚Π΅ΠΏΠ΅Π½ΡŒ, Π½Π°ΠΏΡ€ΠΈΠΌΠ΅Ρ€, число β€” ΠΎΠ½Π° Ρ€Π΅ΡˆΠ°Π΅Ρ‚ΡΡ довольно просто: 23 = 2Β·2Β·2, Π³Π΄Π΅

    ΠŸΠΎΡ‡Π΅ΠΌΡƒ любоС число Π² Π½ΡƒΠ»Π΅Π²ΠΎΠΉ стСпСни Ρ€Π°Π²Π½ΠΎ Π΅Π΄ΠΈΠ½ΠΈΡ†Π΅?

    Π’Π°ΠΊΠΈΠΌ ΠΎΠ±Ρ€Π°Π·ΠΎΠΌ Π½ΡƒΠ»Π΅Π²ΠΎΠΉ ΠΏΠΎΠΊΠ°Π·Π°Ρ‚Π΅Π»ΡŒ стСпСни ΠΊΠ°ΠΊ Π±Ρ‹ Π³ΠΎΠ²ΠΎΡ€ΠΈΡ‚ ΠΎ Ρ‚ΠΎΠΌ, Ρ‡Ρ‚ΠΎ число Π½Π΅ умноТаСтся само Π½Π° сСбя, Π° дСлится само Π½Π° сСбя. …

    ΠŸΠΎΡ‡Π΅ΠΌΡƒ число Π² Π½ΡƒΠ»Π΅Π²ΠΎΠΉ стСпСни Ρ€Π°Π²Π½ΠΎ Π΅Π΄ΠΈΠ½ΠΈΡ†Π΅?

    Π˜Π·Π²Π΅ΡΡ‚Π½ΠΎ, Ρ‡Ρ‚ΠΎ Π°Π±ΡΠΎΠ»ΡŽΡ‚Π½ΠΎ любоС число Π² Π½ΡƒΠ»Π΅Π²ΠΎΠΉ стСпСни Ρ€Π°Π²Π½ΠΎ Π΅Π΄ΠΈΠ½ΠΈΡ†Π΅. ΠŸΠΎΡ‡Π΅ΠΌΡƒ 0 Π² стСпСни 0 Ρ€Π°Π²Π½ΠΎ 1? Π”ΠΎΠΊΠ°Π·Π°Ρ‚ΡŒ это просто, ΠΏΡ€ΠΈ ΡƒΠΌΠ½ΠΎΠΆΠ΅Π½ΠΈΠΈ стСпСнСй с ΠΎΠ΄ΠΈΠ½Π°ΠΊΠΎΠ²Ρ‹ΠΌ основаниСм ΠΈΡ… ΠΏΠΎΠΊΠ°Π·Π°Ρ‚Π΅Π»ΠΈ ΡΠΊΠ»Π°Π΄Ρ‹Π²Π°ΡŽΡ‚ΡΡ, Π° ΠΏΡ€ΠΈ Π΄Π΅Π»Π΅Π½ΠΈΠΈ – Π²Ρ‹Ρ‡ΠΈΡ‚Π°ΡŽΡ‚ΡΡ. … Но вСдь ΠΌΡ‹ Π΄Π΅Π»ΠΈΠΌ число Π½Π° сСбя, это всСгда Π±ΡƒΠ΄Π΅Ρ‚ Ρ€Π°Π²Π½ΠΎ Π΅Π΄ΠΈΠ½ΠΈΡ†Π΅!

    Π§Π΅ΠΌΡƒ Ρ€Π°Π²Π½ΠΎ число Π΅ Π² ΠΏΠ΅Ρ€Π²ΠΎΠΉ стСпСни?

    Π’ΠΎΠ·Π²Π΅Π΄Π΅Π½ΠΈΠ΅ числа Π΅ Π² ΡΡ‚Π΅ΠΏΠ΅Π½ΡŒ ΠΎΠ·Π½Π°Ρ‡Π°Π΅Ρ‚ Π²ΠΎΠ·Π²Π΅Π΄Π΅Π½ΠΈΠ΅ Π² ΡΡ‚Π΅ΠΏΠ΅Π½ΡŒ числа Π­ΠΉΠ»Π΅Ρ€Π° e x = exp (x). Число Π΅ Π² 1-ΠΉ стСпСни, ΠΊΠ°ΠΊ ΠΈ любоС число Π² этой стСпСни, Π±ΡƒΠ΄Π΅Ρ‚ Ρ€Π°Π²Π½ΠΎ самому сСбС, Ρ‚. Π΅. 2.

    Как ΠΏΠΎΡΡ‡ΠΈΡ‚Π°Ρ‚ΡŒ Π΅ Π² минусовой стСпСни?

    ΠŸΡ€ΠΈ Π²ΠΎΠ·Π²Π΅Π΄Π΅Π½ΠΈΠΈ экспонСнты Π² ΠΎΡ‚Ρ€ΠΈΡ†Π°Ρ‚Π΅Π»ΡŒΠ½ΡƒΡŽ ΡΡ‚Π΅ΠΏΠ΅Π½ΡŒ Π½ΡƒΠΆΠ½ΠΎ 1 Π΄Π΅Π»ΠΈΡ‚ΡŒ Π½Π° число Π΅ Π² Π·Π°Π΄Π°Π½Π½ΠΎΠΉ стСпСни, Π½ΠΎ со Π·Π½Π°ΠΊΠΎΠΌ плюс.

    Π§Π΅ΠΌΡƒ Π±ΡƒΠ΄Π΅Ρ‚ Ρ€Π°Π²Π½ΠΎ число Π² минус ΠΏΠ΅Ρ€Π²ΠΎΠΉ стСпСни?

    Π‘Ρ‚Π΅ΠΏΠ΅Π½ΡŒ с ΠΎΡ‚Ρ€ΠΈΡ†Π°Ρ‚Π΅Π»ΡŒΠ½Ρ‹ΠΌ ΠΏΠΎΠΊΠ°Π·Π°Ρ‚Π΅Π»Π΅ΠΌ К ΠΏΡ€ΠΈΠΌΠ΅Ρ€Ρƒ, 4 Π² минус 2 стСпСни β€” это 1/42, 2 Π² минус 3 стСпСни β€” это 1/23, 3 Π² минус 1 стСпСни β€” это 1/3, 10 Π² минус ΠΏΠ΅Ρ€Π²ΠΎΠΉ стСпСни β€” это 1/10 (0,1). Π‘Ρ‚Π΅ΠΏΠ΅Π½ΠΈ с ΠΎΡ‚Ρ€ΠΈΡ†Π°Ρ‚Π΅Π»ΡŒΠ½Ρ‹ΠΌ показатСлям ΠΏΠΎΠΌΠΎΠ³Π°ΡŽΡ‚ ΠΊΠΎΠΌΠΏΠ°ΠΊΡ‚Π½ΠΎ Π·Π°ΠΏΠΈΡΡ‹Π²Π°Ρ‚ΡŒ ΠΊΡ€Π°ΠΉΠ½Π΅ ΠΌΠ°Π»Ρ‹Π΅ ΠΈΠ»ΠΈ постоянно ΡƒΠΌΠ΅Π½ΡŒΡˆΠ°ΡŽΡ‰ΠΈΠ΅ΡΡ Π²Π΅Π»ΠΈΡ‡ΠΈΠ½Ρ‹.

    Π§Ρ‚ΠΎ Ρ‚Π°ΠΊΠΎΠ΅ Π΅ Π² ΠΌΠ°Ρ‚Π΅ΠΌΠ°Ρ‚ΠΈΠΊΠ΅?

    Π‘Π°ΠΌΠΎ число e β€” ΠΈΡ€Ρ€Π°Ρ†ΠΈΠΎΠ½Π°Π»ΡŒΠ½ΠΎΠ΅, Ρ‚ΠΎ Π΅ΡΡ‚ΡŒ выраТаСтся бСсконСчной нСпСриодичСской дСсятичной Π΄Ρ€ΠΎΠ±ΡŒΡŽ. ΠŸΡ€ΠΈΠ±Π»ΠΈΠ·ΠΈΡ‚Π΅Π»ΡŒΠ½ΠΎ ΠΎΠ½ΠΎ Ρ€Π°Π²Π½ΠΎ 2,718. … Π’Π°ΠΊ Π½Π°Π·Ρ‹Π²Π°ΡŽΡ‚ числа, ΠΊΠΎΡ‚ΠΎΡ€Ρ‹Π΅ ΠΌΠΎΠΆΠ½ΠΎ Π²ΡΡ‚Ρ€Π΅Ρ‚ΠΈΡ‚ΡŒ Π² матСматичСских Ρ„ΠΎΡ€ΠΌΡƒΠ»Π°Ρ…, Π²Ρ‹Ρ€Π°ΠΆΠ°ΡŽΡ‰ΠΈΡ… Ρ„ΡƒΠ½Π΄Π°ΠΌΠ΅Π½Ρ‚Π°Π»ΡŒΠ½Ρ‹Π΅ Π·Π°ΠΊΠΎΠ½Ρ‹ ΠΏΡ€ΠΈΡ€ΠΎΠ΄Ρ‹, β€” Π² Ρ„ΠΈΠ·ΠΈΠΊΠ΅, статистикС, Π±ΠΈΠΎΠ»ΠΎΠ³ΠΈΠΈ ΠΈΠ»ΠΈ экономикС.

    Π§Ρ‚ΠΎ Π·Π½Π°Ρ‡ΠΈΡ‚ 1Π΅ 5?

    1e5 — это 100000. 5 ΠΎΠ±ΠΎΠ·Π½Π°Ρ‡Π°Π΅Ρ‚ количСство Π½ΡƒΠ»Π΅ΠΉ, ΠΊΠΎΡ‚ΠΎΡ€Ρ‹Π΅ Π²Ρ‹ добавляСтС Π·Π° этим числом. НапримСр, ΠΏΡ€Π΅Π΄ΠΏΠΎΠ»ΠΎΠΆΠΈΠΌ, Ρ‡Ρ‚ΠΎ Ρƒ мСня Π΅ΡΡ‚ΡŒ 1e7. Π― Π±Ρ‹ поставил 7 Π½ΡƒΠ»Π΅ΠΉ Π·Π° 1, Ρ‚Π°ΠΊ Ρ‡Ρ‚ΠΎ ΠΎΠ½ станСт

    Π§Ρ‚ΠΎ Π·Π½Π°Ρ‡ΠΈΡ‚ 1Π΅ 9?

    e (ΠΈΠ»ΠΈ e ) ΠΎΠ·Π½Π°Ρ‡Π°Π΅Ρ‚ «times 10-to-the», поэтому 1e9 «ΠΎΠ΄ΠΈΠ½ Ρ€Π°Π· Π΄Π΅ΡΡΡ‚ΡŒ Π΄ΠΎ дСвятой мощности», Π° 1e-9 ΠΎΠ·Π½Π°Ρ‡Π°Π΅Ρ‚ «ΠΎΠ΄ΠΈΠ½ Ρ€Π°Π· Π΄Π΅ΡΡΡ‚ΡŒ Π΄ΠΎ ΠΎΡ‚Ρ€ΠΈΡ†Π°Ρ‚Π΅Π»ΡŒΠ½ΠΎΠ³ΠΎ дСвятой власти». Π’ матСматичСских Π½Π°ΡƒΡ‡Π½Ρ‹Ρ… обозначСниях это ΠΎΠ±Ρ‹Ρ‡Π½ΠΎ обозначаСтся надстрочным индСксом: 1 Γ— 10 —9 ΠΈΠ»ΠΈ —1 Γ— 10 9 .

    Π§Ρ‚ΠΎ Π·Π½Π°Ρ‡ΠΈΡ‚ 1 Π΅7?

    Π’ Π±ΠΎΠ»ΡŒΡˆΠΈΠ½ΡΡ‚Π²Π΅ случаСв ΠΊΠΎΠ΄ ошибки 1Π• / 1C / E7 прямо ΡƒΠΊΠ°Π·Ρ‹Π²Π°Π΅Ρ‚ Π½Π° ΠΏΠΎΠ»ΠΎΠΌΠΊΡƒ прСссостата, ΠΎΠ΄Π½Π°ΠΊΠΎ, Π² Π½Π΅ΠΊΠΎΡ‚ΠΎΡ€Ρ‹Ρ… случаях, ΠΈΡΠΏΡ€Π°Π²ΠΈΡ‚ΡŒ Π²ΠΎΠ·Π½ΠΈΠΊΡˆΡƒΡŽ Π½Π΅ΠΏΠΎΠ»Π°Π΄ΠΊΡƒ ΠΌΠΎΠΆΠ½ΠΎ своими силами.

    Как Π½Π° ΠΊΠ°Π»ΡŒΠΊΡƒΠ»ΡΡ‚ΠΎΡ€Π΅ ΠΏΠΎΡΡ‡ΠΈΡ‚Π°Ρ‚ΡŒ экспонСнту?

    Π’ Π²ΠΈΠ½Π΄ΠΎΠ²ΠΎΠΌ ΠΊΠ°Π»ΡŒΠΊΡƒΠ»ΡΡ‚ΠΎΡ€Π΅ Π΅ΡΡ‚ΡŒ нСпримСтная Π³Π°Π»ΠΎΡ‡ΠΊΠ° Inv, выставив ΠΊΠΎΡ‚ΠΎΡ€ΡƒΡŽ, ΠΏΠΎΠ»ΡŒΠ·ΠΎΠ²Π°Ρ‚Π΅Π»ΡŒ ΠΈΠ½Ρ„ΠΎΡ€ΠΌΠΈΡ€ΡƒΠ΅Ρ‚ ΠΊΠ°Π»ΡŒΠΊΡƒΠ»ΡΡ‚ΠΎΡ€, Ρ‡Ρ‚ΠΎ Ρ…ΠΎΡ‡Π΅Ρ‚ ΠΏΡ€ΠΈΠΌΠ΅Π½ΠΈΡ‚ΡŒ Ρ„ΡƒΠ½ΠΊΡ†ΠΈΡŽ, ΠΎΠ±Ρ€Π°Ρ‚Π½ΡƒΡŽ Π²Ρ‹Π·Ρ‹Π²Π°Π΅ΠΌΠΎΠΉ. Π€ΡƒΠ½ΠΊΡ†ΠΈΠ΅ΠΉ, ΠΎΠ±Ρ€Π°Ρ‚Π½ΠΎΠΉ экспонСнтС, являСтся Π½Π°Ρ‚ΡƒΡ€Π°Π»ΡŒΠ½Ρ‹ΠΉ Π»ΠΎΠ³Π°Ρ€ΠΈΡ„ΠΌ — ΡΠ»Π΅Π΄ΠΎΠ²Π°Ρ‚Π΅Π»ΡŒΠ½ΠΎ, Ρ‡Ρ‚ΠΎΠ±Ρ‹ Π²Ρ‹Ρ‡ΠΈΡΠ»ΠΈΡ‚ΡŒ экспонСнту числа, Π½ΡƒΠΆΠ½ΠΎ Π½Π°ΠΆΠ°Ρ‚ΡŒ сначала Inv, ΠΏΠΎΡ‚ΠΎΠΌ ln.

    Бвойства стСпСнСй, ΠΊΠΎΡ‚ΠΎΡ€Ρ‹Π΅ пригодятся Π² ΠΆΠΈΠ·Π½ΠΈ ΠΈ школьникам, ΠΈ ΠΈΡ… родитСлям / Π‘Π΅Ρ€ΠΈ ΠΈ Π΄Π΅Π»Π°ΠΉ

    Π’Β ΠΆΠΈΠ·Π½ΠΈ Π½Π΅Ρ€Π΅Π΄ΠΊΠΎ Π²ΠΎΠ·Π½ΠΈΠΊΠ°ΡŽΡ‚ ситуации, ΠΊΠΎΠ³Π΄Π° ΠΏΠΎΠ»Π΅Π·Π½ΠΎ ΡƒΠΌΠ΅Ρ‚ΡŒ ΠΏΡ€Π°Π²ΠΈΠ»ΡŒΠ½ΠΎ Π²ΠΎΠ·Π²ΠΎΠ΄ΠΈΡ‚ΡŒ число Π²Β ΡΡ‚Π΅ΠΏΠ΅Π½ΡŒ, Π°Β Ρ‚Π°ΠΊΠΆΠ΅ Ρ€Π΅ΡˆΠ°Ρ‚ΡŒ Π·Π°Π΄Π°Ρ‡ΠΈ, ΠΎΠΏΠΈΡ€Π°ΡΡΡŒ на особСнности и свойства стСпСнСй. ΠšΒ ΠΏΡ€ΠΈΠΌΠ΅Ρ€Ρƒ, Π±Π΅Π· матСматичСских ΠΎΠΏΠ΅Ρ€Π°Ρ†ΠΈΠΉ Π½Π°Π΄ числами в стСпСни вы нС смоТСтС Ρ€Π°ΡΡΡ‡ΠΈΡ‚Π°Ρ‚ΡŒ, сколько ΠΏΠ»ΠΈΡ‚ΠΊΠΈ Π²Π°ΠΌ Π½ΡƒΠΆΠ½ΠΎ приобрСсти, Ρ‡Ρ‚ΠΎΠ±Ρ‹ Π²Ρ‹Π»ΠΎΠΆΠΈΡ‚ΡŒ ΠΏΠΎΠ» и стСны Π²Β Π²Π°Π½Π½ΠΎΠΉ ΠΊΠΎΠΌΠ½Π°Ρ‚Π΅.

    Β«Π‘Π΅Ρ€ΠΈ ΠΈΒ Π”Π΅Π»Π°ΠΉΒ» ΠΊΡ€Π°Ρ‚ΠΊΠΎ ΠΎΠ±ΡŠΡΡΠ½ΡΠ΅Ρ‚, ΠΊΠ°ΠΊΠΈΠΌΠΈ свойствами ΠΎΠ±Π»Π°Π΄Π°ΡŽΡ‚ стСпСни ΠΈΒ ΠΊΠ°ΠΊΠΈΠ΅ ошибки ΠΌΡ‹Β Ρ‡Π°Ρ‰Π΅ всСго допускаСм, Ρ€Π΅ΡˆΠ°Ρ Π΄Π°ΠΆΠ΅ простыС выраТСния с подобными числами. Π‘ΠΎΡ…Ρ€Π°Π½ΠΈΡ‚Π΅ эту ΡΡ‚Π°Ρ‚ΡŒΡŽ, Ρ‡Ρ‚ΠΎΠ±Ρ‹ всСгда ΠΈΠΌΠ΅Ρ‚ΡŒ ΠΏΠΎΠ΄ Ρ€ΡƒΠΊΠΎΠΉ Π²Β Π½ΡƒΠΆΠ½Ρ‹ΠΉ ΠΌΠΎΠΌΠ΅Π½Ρ‚.

    Π§Ρ‚ΠΎ Ρ‚Π°ΠΊΠΎΠ΅ ΡΡ‚Π΅ΠΏΠ΅Π½ΡŒ числа

    Когда ΠΌΡ‹Β ΡƒΠΌΠ½ΠΎΠΆΠ°Π΅ΠΌ число само на сСбя, ΠΌΡ‹Β Π½Π°Π·Ρ‹Π²Π°Π΅ΠΌ этот процСсс Π²ΠΎΠ·Π²Π΅Π΄Π΅Π½ΠΈΠ΅ΠΌ числа Π²Β ΡΡ‚Π΅ΠΏΠ΅Π½ΡŒ. Π’Π°ΠΊ, Ссли ΡƒΠΌΠ½ΠΎΠΆΠΈΡ‚ΡŒ число 3 само на сСбя, Ρ‚ΠΎΒ ΠΌΡ‹Β Π·Π°ΠΏΠΈΡˆΠ΅ΠΌ это ΠΊΠ°ΠΊ 32,ΠΈΠ»ΠΈ Β«Ρ‚Ρ€ΠΈ Π²ΠΎΒ Π²Ρ‚ΠΎΡ€ΠΎΠΉ стСпСни».

    Для 2-ΠΉ ΠΈΒ 3-ΠΉ стСпСнСй часто ΠΈΡΠΏΠΎΠ»ΡŒΠ·ΡƒΡŽΡ‚ ΡΠΏΠ΅Ρ†ΠΈΠ°Π»ΡŒΠ½Ρ‹Π΅ названия:

    • Π²ΠΎΠ·Π²Π΅Π΄Π΅Π½ΠΈΠ΅ Π²Β ΠΊΠ²Π°Π΄Ρ€Π°Ρ‚ (ΠΈΠ»ΠΈ Π²ΠΎΠ·Π²Π΅Π΄Π΅Π½ΠΈΠ΅ Π²ΠΎΒ 2-ю ΡΡ‚Π΅ΠΏΠ΅Π½ΡŒ), Π½Π°ΠΏΡ€ΠΈΠΌΠ΅Ρ€ a2
    • Π²ΠΎΠ·Π²Π΅Π΄Π΅Π½ΠΈΠ΅ Π²Β ΠΊΡƒΠ± (ΠΈΠ»ΠΈ Π²ΠΎΠ·Π²Π΅Π΄Π΅Π½ΠΈΠ΅ Π²Β 3-ю ΡΡ‚Π΅ΠΏΠ΅Π½ΡŒ), Π½Π°ΠΏΡ€ΠΈΠΌΠ΅Ρ€ a3

    Если по извСстным значСниям стСпСни и показатСля ΠΎΠΏΡ€Π΅Π΄Π΅Π»ΡΡŽΡ‚ нСизвСстноС основаниС, это называСтся ΠΈΠ·Π²Π»Π΅Ρ‡Π΅Π½ΠΈΠ΅ΠΌ корня.

    Бвойства стСпСнСй

    У стСпСни Π΅ΡΡ‚ΡŒ основаниС ΠΈΒ ΠΏΠΎΠΊΠ°Π·Π°Ρ‚Π΅Π»ΡŒ. ΠŸΠΎΡΠΌΠΎΡ‚Ρ€ΠΈΡ‚Π΅ Π½Π°Β ΠΊΠ°Ρ€Ρ‚ΠΈΠ½ΠΊΡƒ Π²Ρ‹ΡˆΠ΅. ОснованиС стСпСни — это a, Ρ‚ΠΎΒ Π΅ΡΡ‚ΡŒ ΠΏΠΎΠ²Ρ‚ΠΎΡ€ΡΡŽΡ‰ΠΈΠΉΡΡ ΠΌΠ½ΠΎΠΆΠΈΡ‚Π΅Π»ΡŒ, Π°Β ΠΏΠΎΠΊΠ°Π·Π°Ρ‚Π΅Π»ΡŒ стСпСни — это b, Ρ‚ΠΎΒ Π΅ΡΡ‚ΡŒ число, ΡƒΠΊΠ°Π·Ρ‹Π²Π°ΡŽΡ‰Π΅Π΅ число ΠΏΠΎΠ²Ρ‚ΠΎΡ€Π΅Π½ΠΈΠΉ мноТитСля. ЧитаСтся это ΠΊΠ°ΠΊ Β«a в стСпСни bΒ».

    Если ΠΏΠΎΠΊΠ°Π·Π°Ρ‚Π΅Π»ΡŒ стСпСни являСтся Π½Π°Ρ‚ΡƒΡ€Π°Π»ΡŒΠ½Ρ‹ΠΌ числом, Ρ‚ΠΎΒ Π΅Π³ΠΎ Π½Π°Π·Ρ‹Π²Π°ΡŽΡ‚ Π½Π°Ρ‚ΡƒΡ€Π°Π»ΡŒΠ½Ρ‹ΠΌ ΠΏΠΎΠΊΠ°Π·Π°Ρ‚Π΅Π»Π΅ΠΌ. Π˜Π·Β ΠΎΠΏΡ€Π΅Π΄Π΅Π»Π΅Π½ΠΈΡ Π½Π°Ρ‚ΡƒΡ€Π°Π»ΡŒΠ½Ρ‹Ρ… чисСл слСдуСт, Ρ‡Ρ‚ΠΎ Π½Π°Ρ‚ΡƒΡ€Π°Π»ΡŒΠ½Ρ‹ΠΉ ΠΏΠΎΠΊΠ°Π·Π°Ρ‚Π΅Π»ΡŒ стСпСни Π½Π΅Β ΠΌΠΎΠΆΠ΅Ρ‚ ΡΠ²Π»ΡΡ‚ΡŒΡΡ Π½ΡƒΠ»Π΅ΠΌ, ΠΎΡ‚Ρ€ΠΈΡ†Π°Ρ‚Π΅Π»ΡŒΠ½Ρ‹ΠΌ ΠΈΠ»ΠΈ Π½Π΅Ρ†Π΅Π»Ρ‹ΠΌ числом.

    У стСпСни ΡΒ Π½Π°Ρ‚ΡƒΡ€Π°Π»ΡŒΠ½Ρ‹ΠΌ ΠΏΠΎΠΊΠ°Π·Π°Ρ‚Π΅Π»Π΅ΠΌ Π΅ΡΡ‚ΡŒ нСсколько свойств, благодаря ΠΊΠΎΡ‚ΠΎΡ€Ρ‹ΠΌ процСсс вычислСний ΠΌΠΎΠΆΠ½ΠΎ ΡƒΠΏΡ€ΠΎΡΡ‚ΠΈΡ‚ΡŒ. Они пСрСчислСны Π²Ρ‹ΡˆΠ΅. Π”Π°Π²Π°ΠΉΡ‚Π΅ разбСрСмся с каТдым ΠΈΠ·Β Π½ΠΈΡ… ΠΎΡ‚Π΄Π΅Π»ΡŒΠ½ΠΎ.

    Бвойство β„–Β 1: Π’Β 1-ю ΡΡ‚Π΅ΠΏΠ΅Π½ΡŒ ΠΌΠΎΠΆΠ½ΠΎ возвСсти любоС число. Π˜Β Ρ‚ΠΎΠ³Π΄Π° число Π²Β 1-ΠΉ стСпСни Π±ΡƒΠ΄Π΅Ρ‚ Ρ€Π°Π²Π½ΡΡ‚ΡŒΡΡ самому числу.

    НапримСр, 11 = 1, Π°Β 31 = 3. Π’Β Π°Π»Π³Π΅Π±Ρ€Π΅ 1-я ΡΡ‚Π΅ΠΏΠ΅Π½ΡŒ ΠΎΠ±Ρ‹Ρ‡Π½ΠΎ нС записываСтся, Π½ΠΎΒ ΠΏΡ€ΠΈ дСйствиях со стСпСнями учитываСтся. НапримСр, a4 Γ— a = a4 Γ— a1 = a4 + 1 = a5.

    Бвойство β„–Β 2: Π‘Ρ‚Π΅ΠΏΠ΅Π½ΡŒ произвСдСния. Если Π½ΡƒΠΆΠ½ΠΎ возвСсти Π²Β ΡΡ‚Π΅ΠΏΠ΅Π½ΡŒ ΠΏΡ€ΠΎΠΈΠ·Π²Π΅Π΄Π΅Π½ΠΈΠ΅ ΠΌΠ½ΠΎΠΆΠΈΡ‚Π΅Π»Π΅ΠΉ, Ρ‚ΠΎΒ ΠΊΠ°ΠΆΠ΄Ρ‹ΠΉ ΠΈΠ·Β ΠΌΠ½ΠΎΠΆΠΈΡ‚Π΅Π»Π΅ΠΉ возводят Π²Β ΡΡ‚Π΅ΠΏΠ΅Π½ΡŒ, Π°Β Π·Π°Ρ‚Π΅ΠΌ Ρ€Π΅Π·ΡƒΠ»ΡŒΡ‚Π°Ρ‚Ρ‹ ΠΏΠ΅Ρ€Π΅ΠΌΠ½ΠΎΠΆΠ°ΡŽΡ‚. ΠŸΡ€ΠΈ этом для Π²Ρ‹ΡˆΠ΅ΡƒΠΊΠ°Π·Π°Π½Π½ΠΎΠΉ Ρ„ΠΎΡ€ΠΌΡƒΠ»Ρ‹ Π²Π°ΠΆΠ½ΠΎ, Ρ‡Ρ‚ΠΎ aΒ ΠΈΒ bΒ β€” это основания стСпСнСй (Π½Π΅Β Ρ€Π°Π²Π½Ρ‹Π΅ Π½ΡƒΠ»ΡŽ), Π°Β nΒ β€” это ΠΏΠΎΠΊΠ°Π·Π°Ρ‚Π΅Π»ΡŒ стСпСни, Π½Π°Ρ‚ΡƒΡ€Π°Π»ΡŒΠ½ΠΎΠ΅ число.

    ΠŸΡ€ΠΈΠΌΠ΅Ρ€ использования: (3a)2 = (3 Γ— a)2 = 32 Γ— a2 = 32a2.

    Бвойство β„–Β 3: Π‘Ρ‚Π΅ΠΏΠ΅Π½ΡŒ частного. Если Π½ΡƒΠΆΠ½ΠΎ возвСсти Π²Β ΡΡ‚Π΅ΠΏΠ΅Π½ΡŒ Ρ€Π΅Π·ΡƒΠ»ΡŒΡ‚Π°Ρ‚ дСлСния Π΄Π΅Π»ΠΈΠΌΠΎΠ³ΠΎ Π½Π°Β Π΄Π΅Π»ΠΈΡ‚Π΅Π»ΡŒ, Ρ‚ΠΎΒ ΠΌΠΎΠΆΠ½ΠΎ возвСсти в эту ΡΡ‚Π΅ΠΏΠ΅Π½ΡŒ ΠΎΡ‚Π΄Π΅Π»ΡŒΠ½ΠΎ Π΄Π΅Π»ΠΈΠΌΠΎΠ΅ ΠΈΒ Π΄Π΅Π»ΠΈΡ‚Π΅Π»ΡŒ, Π°Β Π·Π°Ρ‚Π΅ΠΌ ΠΏΠ΅Ρ€Π²Ρ‹ΠΉ Ρ€Π΅Π·ΡƒΠ»ΡŒΡ‚Π°Ρ‚ Ρ€Π°Π·Π΄Π΅Π»ΠΈΡ‚ΡŒ Π½Π°Β Π²Ρ‚ΠΎΡ€ΠΎΠΉ. ΠŸΡ€ΠΈ этом для Π²Ρ‹ΡˆΠ΅ΡƒΠΊΠ°Π·Π°Π½Π½ΠΎΠΉ Ρ„ΠΎΡ€ΠΌΡƒΠ»Ρ‹ Π²Π°ΠΆΠ½ΠΎ, Ρ‡Ρ‚ΠΎ aΒ ΠΈΒ bΒ β€” это основания стСпСнСй (Π½Π΅Β Ρ€Π°Π²Π½Ρ‹Π΅ Π½ΡƒΠ»ΡŽ) ΠΈΒ Π»ΡŽΠ±Ρ‹Π΅ Ρ€Π°Ρ†ΠΈΠΎΠ½Π°Π»ΡŒΠ½Ρ‹Π΅ числа, Π½ΠΎΒ b β‰  0. А nΒ β€” это Π½Π°Ρ‚ΡƒΡ€Π°Π»ΡŒΠ½Ρ‹ΠΉ ΠΏΠΎΠΊΠ°Π·Π°Ρ‚Π΅Π»ΡŒ стСпСни.

    ΠŸΡ€ΠΈΠΌΠ΅Ρ€ использования: (5/8)2 = 52 Γ· 82.

    Бвойство β„–Β 4: ΠŸΡ€ΠΎΠΈΠ·Π²Π΅Π΄Π΅Π½ΠΈΠ΅ стСпСнСй. Если Π½ΡƒΠΆΠ½ΠΎ ΡƒΠΌΠ½ΠΎΠΆΠΈΡ‚ΡŒ стСпСни с одинаковыми основаниями, то основаниС ΠΎΡΡ‚Π°Π²Π»ΡΡŽΡ‚ Π±Π΅Π· ΠΈΠ·ΠΌΠ΅Π½Π΅Π½ΠΈΠΉ, Π°Β ΠΏΠΎΠΊΠ°Π·Π°Ρ‚Π΅Π»ΠΈ стСпСнСй ΡΠΊΠ»Π°Π΄Ρ‹Π²Π°ΡŽΡ‚. ΠŸΡ€ΠΈ этом для Π²Ρ‹ΡˆΠ΅ΡƒΠΊΠ°Π·Π°Π½Π½ΠΎΠΉ Ρ„ΠΎΡ€ΠΌΡƒΠ»Ρ‹ Π²Π°ΠΆΠ½ΠΎ, Ρ‡Ρ‚ΠΎΒ n, mΒ β€” это Π½Π°Ρ‚ΡƒΡ€Π°Π»ΡŒΠ½Ρ‹Π΅ ΠΏΠΎΠΊΠ°Π·Π°Ρ‚Π΅Π»ΠΈ стСпСни.

    ΠŸΡ€ΠΈΠΌΠ΅Ρ€ использования: 3435 = 34 Γ— 35 = 34 + 5 = 39.

    Бвойство β„–Β 5: ЧастноС стСпСнСй. Если Π½ΡƒΠΆΠ½ΠΎ Ρ€Π°Π·Π΄Π΅Π»ΠΈΡ‚ΡŒ стСпСни с одинаковыми основаниями, то основаниС ΠΎΡΡ‚Π°Π²Π»ΡΡŽΡ‚ Π±Π΅Π· ΠΈΠ·ΠΌΠ΅Π½Π΅Π½ΠΈΠΉ, а из показатСля стСпСни Π΄Π΅Π»ΠΈΠΌΠΎΠ³ΠΎ Π²Ρ‹Ρ‡ΠΈΡ‚Π°ΡŽΡ‚ ΠΏΠΎΠΊΠ°Π·Π°Ρ‚Π΅Π»ΡŒ стСпСни дСлитСля. ΠŸΡ€ΠΈ этом для Π²Ρ‹ΡˆΠ΅ΡƒΠΊΠ°Π·Π°Π½Π½ΠΎΠΉ Ρ„ΠΎΡ€ΠΌΡƒΠ»Ρ‹ Π²Π°ΠΆΠ½ΠΎ, Ρ‡Ρ‚ΠΎ aΒ β€” любоС число, Π½Π΅Β Ρ€Π°Π²Π½ΠΎΠ΅ Π½ΡƒΠ»ΡŽ, aΒ n, mΒ β€” это Π»ΡŽΠ±Ρ‹Π΅ Π½Π°Ρ‚ΡƒΡ€Π°Π»ΡŒΠ½Ρ‹Π΅ числа, Π½ΠΎΒ Ρ‚Π°ΠΊΠΈΠ΅, Ρ‡Ρ‚ΠΎ nΒ > m.

    ΠŸΡ€ΠΈΠΌΠ΅Ρ€ использования: 48 Γ· 45 = 48 βˆ’ 5 = 43.

    Бвойство β„–Β 6: Π’ΠΎΠ·Π²Π΅Π΄Π΅Π½ΠΈΠ΅ стСпСни Π²Β ΠΊΠ²Π°Π΄Ρ€Π°Ρ‚. Если Π½ΡƒΠΆΠ½ΠΎ возвСсти ΡΡ‚Π΅ΠΏΠ΅Π½ΡŒ Π²Β ΡΡ‚Π΅ΠΏΠ΅Π½ΡŒ, то основаниС стСпСни ΠΎΡΡ‚Π°Π²Π»ΡΡŽΡ‚ Π±Π΅Π· ΠΈΠ·ΠΌΠ΅Π½Π΅Π½ΠΈΠΉ, Π°Β ΠΏΠΎΠΊΠ°Π·Π°Ρ‚Π΅Π»ΠΈ стСпСнСй ΡƒΠΌΠ½ΠΎΠΆΠ°ΡŽΡ‚ Π΄Ρ€ΡƒΠ³ Π½Π°Β Π΄Ρ€ΡƒΠ³Π°. ΠŸΡ€ΠΈ этом для Π²Ρ‹ΡˆΠ΅ΡƒΠΊΠ°Π·Π°Π½Π½ΠΎΠΉ Ρ„ΠΎΡ€ΠΌΡƒΠ»Ρ‹ Π²Π°ΠΆΠ½ΠΎ, Ρ‡Ρ‚ΠΎ aΒ β€” основаниС стСпСни (Π½Π΅Β Ρ€Π°Π²Π½ΠΎΠ΅ Π½ΡƒΠ»ΡŽ), aΒ n, mΒ β€” Π½Π°Ρ‚ΡƒΡ€Π°Π»ΡŒΠ½Ρ‹Π΅ ΠΏΠΎΠΊΠ°Π·Π°Ρ‚Π΅Π»ΠΈ стСпСни.

    ΠŸΡ€ΠΈΠΌΠ΅Ρ€ использования: (42)3 = 42 Γ— 3 = 46.

    Π”Ρ€ΡƒΠ³ΠΈΠ΅ свойства стСпСнСй

    Число с дробным ΠΏΠΎΠΊΠ°Π·Π°Ρ‚Π΅Π»Π΅ΠΌ стСпСни равняСтся ΠΊΠΎΡ€Π½ΡŽ с показатСлСм, Ρ€Π°Π²Π½Ρ‹ΠΌ Π·Π½Π°ΠΌΠ΅Π½Π°Ρ‚Π΅Π»ΡŽ, ΠΈΒ ΠΏΠΎΠ΄ΠΊΠΎΡ€Π΅Π½Π½Ρ‹ΠΌ числом в стСпСни, Ρ€Π°Π²Π½ΠΎΠΉ Ρ‡ΠΈΡΠ»ΠΈΡ‚Π΅Π»ΡŽ. ΠŸΠΎΠ»ΡƒΡ‡Π°Π΅Ρ‚ΡΡ, Ρ‡Ρ‚ΠΎ ΠΎΠΏΠ΅Ρ€Π°Ρ†ΠΈΡŽ извлСчСния корня всСгда ΠΌΠΎΠΆΠ½ΠΎ Π·Π°ΠΌΠ΅Π½ΠΈΡ‚ΡŒ Π²ΠΎΠ·Π²Π΅Π΄Π΅Π½ΠΈΠ΅ΠΌ Π²Β ΡΡ‚Π΅ΠΏΠ΅Π½ΡŒ. Π­Ρ‚ΠΎ Ρ…ΠΎΡ€ΠΎΡˆΠΎ ΠΈΡΠΏΠΎΠ»ΡŒΠ·ΠΎΠ²Π°Ρ‚ΡŒ для упрощСния записи выраТСния.

    ΠŸΡ€ΠΈΠΌΠ΅Ρ€ использования: 3√a6 = a6 Γ· 3 = a2.

    Π›ΡŽΠ±ΠΎΠ΅ число, Π²ΠΎΠ·Π²Π΅Π΄Π΅Π½Π½ΠΎΠ΅ Π²Β Π½ΡƒΠ»Π΅Π²ΡƒΡŽ ΡΡ‚Π΅ΠΏΠ΅Π½ΡŒ, Π±ΡƒΠ΄Π΅Ρ‚ Ρ€Π°Π²Π½ΠΎ Π΅Π΄ΠΈΠ½ΠΈΡ†Π΅ (Π·Π°Β ΠΈΡΠΊΠ»ΡŽΡ‡Π΅Π½ΠΈΠ΅ΠΌ нуля). ΠΡƒΠ»ΡŒ Π²Β Π½ΡƒΠ»Π΅Π²ΠΎΠΉ стСпСни Π½Π΅Β ΠΎΠΏΡ€Π΅Π΄Π΅Π»Π΅Π½, поэтому ΠΏΠΎΠ΄ΠΎΠ±Π½ΠΎΠ΅ Π²Ρ‹Ρ€Π°ΠΆΠ΅Π½ΠΈΠ΅ Π½Π΅Β ΠΈΠΌΠ΅Π΅Ρ‚ смысла.

    ΠŸΡ€ΠΈΠΌΠ΅Ρ€ использования: 74 Γ— 7βˆ’4 = 74 + (βˆ’4) = 70 = 1.

    Π›ΡŽΠ±ΠΎΠ΅ число, Π²ΠΎΠ·Π²Π΅Π΄Π΅Π½Π½ΠΎΠ΅ Π²Β ΠΎΡ‚Ρ€ΠΈΡ†Π°Ρ‚Π΅Π»ΡŒΠ½ΡƒΡŽ ΡΡ‚Π΅ΠΏΠ΅Π½ΡŒ, Ρ€Π°Π²Π½ΠΎ Π΅Π΄ΠΈΠ½ΠΈΡ†Π΅, Ρ€Π°Π·Π΄Π΅Π»Π΅Π½Π½ΠΎΠΉ на это ТС число, Π½ΠΎΒ Π²Β ΠΏΠΎΠ»ΠΎΠΆΠΈΡ‚Π΅Π»ΡŒΠ½ΠΎΠΉ стСпСни. В частности:

    • aβˆ’1 = 1/a,
    • (a/b)βˆ’n = (b/a)n

    ΠŸΡ€ΠΈΠΌΠ΅Ρ€ использования: 6βˆ’2 = 1/62 = 1/36.

    Π‘Ρ‚Π΅ΠΏΠ΅Π½ΡŒ ΡΒ ΠΎΡ‚Ρ€ΠΈΡ†Π°Ρ‚Π΅Π»ΡŒΠ½Ρ‹ΠΌ основаниСм ΠΈΒ Ρ‡Π΅Ρ‚Π½Ρ‹ΠΌ ΠΏΠΎΠΊΠ°Π·Π°Ρ‚Π΅Π»Π΅ΠΌ Ρ€Π°Π²Π½Π° стСпСни с основаниСм, ΠΏΡ€ΠΎΡ‚ΠΈΠ²ΠΎΠΏΠΎΠ»ΠΎΠΆΠ½Ρ‹ΠΌ Π΄Π°Π½Π½ΠΎΠΌΡƒ, но с тСм ТС ΠΏΠΎΠΊΠ°Π·Π°Ρ‚Π΅Π»Π΅ΠΌ. Π§Ρ‚ΠΎΠ±Ρ‹ возвСсти Π²Β Π½Π΅Ρ‡Π΅Ρ‚Π½ΡƒΡŽ ΡΡ‚Π΅ΠΏΠ΅Π½ΡŒ ΠΎΡ‚Ρ€ΠΈΡ†Π°Ρ‚Π΅Π»ΡŒΠ½ΠΎΠ΅ число, Π½ΡƒΠΆΠ½ΠΎ ΠΏΠΎΡΡ‚Π°Π²ΠΈΡ‚ΡŒ Π·Π½Π°ΠΊ «минус» и возвСсти в эту ΡΡ‚Π΅ΠΏΠ΅Π½ΡŒ число, ΠΏΡ€ΠΎΡ‚ΠΈΠ²ΠΎΠΏΠΎΠ»ΠΎΠΆΠ½ΠΎΠ΅ Π΄Π°Π½Π½ΠΎΠΌΡƒ.

    ΠŸΡ€ΠΈΠΌΠ΅Ρ€ использования: (βˆ’8)4 = 84 = 8 Γ— 8 Γ— 8 Γ— 8 = 4Β 096. Или (βˆ’8)3 = βˆ’83 = βˆ’512.

    КакиС ошибки часто Π΄ΠΎΠΏΡƒΡΠΊΠ°ΡŽΡ‚, ΠΊΠΎΠ³Π΄Π° Ρ€Π΅Ρ‡ΡŒ ΠΈΠ΄Π΅Ρ‚ о свойствах стСпСнСй

    • ΠΠ΅ΠΏΡ€Π°Π²ΠΈΠ»ΡŒΠ½ΠΎ возводят число Π²Β Π½ΡƒΠ»Π΅Π²ΡƒΡŽ, 1-ю ΠΈΒ βˆ’1-ю стСпСни, путая ΠΈΡ…Β ΠΌΠ΅ΠΆΠ΄Ρƒ собой. Π˜ΡΡ…ΠΎΠ΄Ρ ΠΈΠ·Β Π²Ρ‹ΡˆΠ΅ΡƒΠΊΠ°Π·Π°Π½Π½Ρ‹Ρ… свойств стСпСнСй, ΠΌΡ‹Β ΠΏΠΎΠΌΠ½ΠΈΠΌ, Ρ‡Ρ‚ΠΎ a0 = 1, a1 = a, aβˆ’1 = 1/a.
    • ΠΠ΅ΠΏΡ€Π°Π²ΠΈΠ»ΡŒΠ½ΠΎ ΡƒΠΌΠ½ΠΎΠΆΠ°ΡŽΡ‚ ΠΈΒ ΡΠΊΠ»Π°Π΄Ρ‹Π²Π°ΡŽΡ‚ стСпСни. Π§Π°Ρ‰Π΅ всСго это происходит Ρ‚ΠΎΠ³Π΄Π°, ΠΊΠΎΠ³Π΄Π° Π²Β Π²Ρ‹Ρ€Π°ΠΆΠ΅Π½ΠΈΠΈ стСпСни с одинаковыми основаниями. ΠŸΠΎΡΠΌΠΎΡ‚Ρ€ΠΈΡ‚Π΅ Π½Π°Β ΠΊΠ°Ρ€Ρ‚ΠΈΠ½ΠΊΡƒ Π²Ρ‹ΡˆΠ΅: Π²Β ΠΏΠ΅Ρ€Π²ΠΎΠΌ случаС основания Π½Π΅Β ΠΏΠ΅Ρ€Π΅ΠΌΠ½ΠΎΠΆΠ°ΡŽΡ‚ΡΡ, Ρ‚ΠΎΒ Π΅ΡΡ‚ΡŒ Π΄ΠΎΠ»ΠΆΠ½ΠΎ Π±Ρ‹Ρ‚ΡŒ 33 Γ— 33 = 33 + 3 = 36. А во втором случаС ΠΏΡ€ΠΈ слоТСнии ΠΌΡ‹Β Π½Π΅Β ΠΌΠΎΠΆΠ΅ΠΌ ΡΠΊΠ»Π°Π΄Ρ‹Π²Π°Ρ‚ΡŒ ΠΏΠΎΠΊΠ°Π·Π°Ρ‚Π΅Π»ΠΈ стСпСни, Π΄Π°ΠΆΠ΅ Ссли бы ΠΎΠ½ΠΈ Π±Ρ‹Π»ΠΈ ΠΎΠ΄ΠΈΠ½Π°ΠΊΠΎΠ²Ρ‹ΠΌΠΈ: Π½ΡƒΠΆΠ½ΠΎ сначала возвСсти Π²Β ΡΡ‚Π΅ΠΏΠ΅Π½ΡŒ ΠΊΠ°ΠΆΠ΄ΠΎΠ΅ число, Π°Β Π·Π°Ρ‚Π΅ΠΌ произвСсти слоТСниС, Ρ‚ΠΎΒ Π΅ΡΡ‚ΡŒ 53 + 54 = 125 + 625 = 750.
    • ΠΠ΅ΠΏΡ€Π°Π²ΠΈΠ»ΡŒΠ½ΠΎ возводят Π΄Ρ€ΠΎΠ±Π½ΠΎΠ΅ число Π²Β ΠΎΡ‚Ρ€ΠΈΡ†Π°Ρ‚Π΅Π»ΡŒΠ½ΡƒΡŽ ΡΡ‚Π΅ΠΏΠ΅Π½ΡŒ. На картинкС Π²Ρ‹ΡˆΠ΅ Ρ‚ΠΎΠ»ΡŒΠΊΠΎ Π½ΠΈΠΆΠ½ΠΈΠΉ Π²Π°Ρ€ΠΈΠ°Π½Ρ‚ ΠΎΡ‚Π²Π΅Ρ‚Π° соотвСтствуСт ΠΏΡ€Π°Π²ΠΈΠ»Ρƒ (a/b)βˆ’n = (b/a)n.

    Бонус: ΠΏΡ€ΠΎΠ²Π΅Ρ€ΡŒΡ‚Π΅ сСбя, Ρ€Π΅ΡˆΠΈΠ² простой ΠΏΡ€ΠΈΠΌΠ΅Ρ€

    Π­Ρ‚ΠΎΡ‚ ΠΏΡ€ΠΈΠΌΠ΅Ρ€ выглядит Π½Π°ΡΡ‚ΠΎΠ»ΡŒΠΊΠΎ простым, Ρ‡Ρ‚ΠΎ каТСтся, Π±ΡƒΠ΄Ρ‚ΠΎΒ Π±Ρ‹ с ним ΠΌΠΎΠ³Β Π±Ρ‹ ΡΠΏΡ€Π°Π²ΠΈΡ‚ΡŒΡΡ Π΄Π°ΠΆΠ΅ Ρ€Π΅Π±Π΅Π½ΠΎΠΊ. Но на самом Π΄Π΅Π»Π΅ ΠΎΠ½Β Π»Π΅Π³ΠΊΠΎ поставит Π²Β Ρ‚ΡƒΠΏΠΈΠΊ Π±ΠΎΠ»ΡŒΡˆΠΈΠ½ΡΡ‚Π²ΠΎ взрослых. Π“Π»Π°Π²Π½ΠΎΠ΅ Π·Π΄Π΅ΡΡŒΒ β€” это Π½Π΅Β ΠΏΠΎΠΏΠ°ΡΡ‚ΡŒ Π²Β Π»ΠΎΠ²ΡƒΡˆΠΊΡƒ ΠΈΒ Π²Ρ‹ΠΏΠΎΠ»Π½ΠΈΡ‚ΡŒ всС дСйствия Π²Β ΠΏΡ€Π°Π²ΠΈΠ»ΡŒΠ½ΠΎΠΌ порядкС. НачнСм с лСвой стороны ΠΈΒ Π²ΠΎΡΠΏΠΎΠ»ΡŒΠ·ΡƒΠ΅ΠΌΡΡ свойством частного стСпСнСй. Π’ΠΎΠ³Π΄Π° 23/22 = 23 βˆ’ 2 = 21. Π”Π°Π»Π΅Π΅ подставим это число Π²Β Π²Ρ‹Ρ€Π°ΠΆΠ΅Π½ΠΈΠ΅ ΠΈΒ ΠΏΠΎΠ»ΡƒΡ‡ΠΈΠΌ 21 + 21 + 20. На этом шагС ΠΊΠΎΠΌΡƒ-Ρ‚ΠΎ захочСтся просто ΡΠ»ΠΎΠΆΠΈΡ‚ΡŒ ΠΏΠΎΠΊΠ°Π·Π°Ρ‚Π΅Π»ΠΈ стСпСнСй. Но мы помним, Ρ‡Ρ‚ΠΎ это Π½Π΅Π²Π΅Ρ€Π½Ρ‹ΠΉ ΠΏΡƒΡ‚ΡŒ, ΠΈ, воспользовавшись ΠΏΡ€Π°Π²ΠΈΠ»Π°ΠΌΠΈ возвСдСния чисСл Π²Β 1-ю ΠΈΒ Π½ΡƒΠ»Π΅Π²ΡƒΡŽ стСпСни, ΠΏΠΎΠ»ΡƒΡ‡ΠΈΠΌ 21 + 21 + 20 = 2 + 2 + 1 = 5.

    ΠžΡ‚Π²Π΅Ρ‚: 5.

    Π‘Π΅Ρ€ΠΈ ΠΈ Π΄Π΅Π»Π°ΠΉ/ΠšΡƒΠ»ΡŒΡ‚ΡƒΡ€Π°/Бвойства стСпСнСй, ΠΊΠΎΡ‚ΠΎΡ€Ρ‹Π΅ пригодятся Π² ΠΆΠΈΠ·Π½ΠΈ ΠΈ школьникам, ΠΈ ΠΈΡ… родитСлям

    ΠŸΠΎΠΊΠ°Π·Π°Ρ‚Π΅Π»ΠΈ Π² ΠΌΠ°Ρ‚Π΅ΠΌΠ°Ρ‚ΠΈΠΊΠ΅ (Ρ†Π΅Π»Ρ‹Π΅, Π΄Ρ€ΠΎΠ±Π½Ρ‹Π΅ + функция) с ΠΏΡ€ΠΈΠΌΠ΅Ρ€Π°ΠΌΠΈ ΠΈ ΠΎΠ±Ρ€Π°Π·Ρ†Π°ΠΌΠΈ

    ОглавлСниС:

    ΠŸΠΎΠΊΠ°Π·Π°Ρ‚Π΅Π»ΡŒΒ β€” это число, ΠΏΠΎΠΊΠ°Π·Ρ‹Π²Π°ΡŽΡ‰Π΅Π΅, Π² ΠΊΠ°ΠΊΡƒΡŽ ΠΈΠΌΠ΅Π½Π½ΠΎ ΡΡ‚Π΅ΠΏΠ΅Π½ΡŒ возводится основаниС. Β 

    ΠŸΠΎΠΊΠ°Π·Π°Ρ‚Π΅Π»ΡŒΒ β€” Π² Π±ΠΎΠ»ΡŒΡˆΠΈΠ½ΡΡ‚Π²Π΅ случаСв, обобщённая характСристика какого-Π»ΠΈΠ±ΠΎΒ ΠΎΠ±ΡŠΠ΅ΠΊΡ‚Π°, процСсса или Π΅Π³ΠΎ Ρ€Π΅Π·ΡƒΠ»ΡŒΡ‚Π°Ρ‚Π°, понятия или их свойств, ΠΎΠ±Ρ‹Ρ‡Π½ΠΎ, выраТСнная Π² числовой Ρ„ΠΎΡ€ΠΌΠ΅.

    Π¦Π΅Π»Ρ‹Π΅ ΠΏΠΎΠΊΠ°Π·Π°Ρ‚Π΅Π»ΠΈ

    Бвойства Ρ†Π΅Π»Ρ‹Ρ… ΠΏΠΎΠ»ΠΎΠΆΠΈΡ‚Π΅Π»ΡŒΠ½Ρ‹Ρ… ΠΏΠΎΠΊΠ°Π·Π°Ρ‚Π΅Π»Π΅ΠΉ: ΠŸΠΎΠΊΠ°Π·Π°Ρ‚Π΅Π»ΠΈ стСпСни Π΄ΠΎ сСго Π²Ρ€Π΅ΠΌΠ΅Π½ΠΈ ΠΏΡ€Π΅Π΄ΠΏΠΎΠ»Π°Π³Π°Π»ΠΈΡΡŒ Π½Π°ΠΌΠΈ Ρ†Π΅Π»Ρ‹ΠΌΠΈ ΠΈ ΠΏΠΎΠ»ΠΎΠΆΠΈΡ‚Π΅Π»ΡŒΠ½Ρ‹ΠΌΠΈ, ΠΏΡ€ΠΈΡ‡Ρ‘ΠΌ ΠΌΡ‹ ΠΈΠΌ ΠΏΡ€ΠΈΠ΄Π°Π²Π°Π»ΠΈ смысл, Π²Ρ‹Ρ€Π°ΠΆΠ°Π΅ΠΌΡ‹ΠΉ Π² ΡΠ»Π΅Π΄ΡƒΡŽΡ‰Π΅ΠΌ ΠΎΠΏΡ€Π΅Π΄Π΅Π»Π΅Π½ΠΈΠΈ:

    Π’ΠΎΠ·Π²Ρ‹ΡΠΈΡ‚ΡŒ число Π° Π² ΡΡ‚Π΅ΠΏΠ΅Π½ΡŒ с Ρ†Π΅Π»Ρ‹ΠΌ ΠΈ ΠΏΠΎΠ»ΠΎΠΆΠΈΡ‚Π΅Π»ΡŒΠ½Ρ‹ΠΌ ΠΏΠΎΠΊΠ°Π·Π°Ρ‚Π΅Π»Π΅ΠΌ n β€” Π·Π½Π°Ρ‡ΠΈΡ‚ Π½Π°ΠΉΡ‚ΠΈ ΠΏΡ€ΠΎΠΈΠ·Π²Π΅Π΄Π΅Π½ΠΈΠ΅ n ΠΎΠ΄ΠΈΠ½Π°ΠΊΠΎΠ²Ρ‹Ρ… сомноТитСлСй aaa…a.

    ΠŸΠ΅Ρ€Π΅Ρ‡ΠΈΡΠ»ΠΈΠΌ свойства этих ΠΏΠΎΠΊΠ°Π·Π°Ρ‚Π΅Π»Π΅ΠΉ, извСстныС Π½Π°ΠΌ ΠΈΠ· ΠΏΡ€Π΅Π΄Ρ‹Π΄ΡƒΡ‰ΠΈΡ… Π³Π»Π°Π² Π°Π»Π³Π΅Π±Ρ€Ρ‹:

    • 1) ΠΏΡ€ΠΈ ΡƒΠΌΠ½ΠΎΠΆΠ΅Π½ΠΈΠΈ стСпСнСй ΠΎΠ΄Π½ΠΎΠ³ΠΎ ΠΈ Ρ‚ΠΎΠ³ΠΎ ΠΆΠ΅ числа ΠΏΠΎΠΊΠ°Π·Π°Ρ‚Π΅Π»ΠΈ ΠΈΡ… ΡΠΊΠ»Π°Π΄Ρ‹Π²Π°ΡŽΡ‚ΡΡ;
    • 2) ΠΏΡ€ΠΈ Π΄Π΅Π»Π΅Π½ΠΈΠΈ стСпСнСй ΠΎΠ΄Π½ΠΎΠ³ΠΎ ΠΈ Ρ‚ΠΎΠ³ΠΎ ΠΆΠ΅ числа ΠΏΠΎΠΊΠ°Π·Π°Ρ‚Π΅Π»ΡŒ дСлитСля вычитаСтся ΠΈΠ· показатСля Π΄Π΅Π»ΠΈΠΌΠΎΠ³ΠΎ, Ссли ΠΏΠΎΠΊΠ°Π·Π°Ρ‚Π΅Π»ΡŒ дСлитСля Π½Π΅ большС показатСля Π΄Π΅Π»ΠΈΠΌΠΎΠ³ΠΎ;
    • 3) ΠΏΡ€ΠΈ Π²ΠΎΠ·Π²Ρ‹ΡˆΠ΅Π½ΠΈΠΈ ΠΎΡ‚Ρ€ΠΈΡ†Π°Ρ‚Π΅Π»ΡŒΠ½ΠΎΠ³ΠΎ числа Π² ΡΡ‚Π΅ΠΏΠ΅Π½ΡŒ с Ρ‡Ρ‘Ρ‚Π½Ρ‹ΠΌ ΠΏΠΎΠΊΠ°Π·Π°Ρ‚Π΅Π»Π΅ΠΌ получаСтся ΠΏΠΎΠ»ΠΎΠΆΠΈΡ‚Π΅Π»ΡŒΠ½ΠΎΠ΅ число, Π° с Π½Π΅Ρ‡Ρ‘Ρ‚Π½Ρ‹ΠΌ ΠΏΠΎΠΊΠ°Π·Π°Ρ‚Π΅Π»Π΅ΠΌ β€” ΠΎΡ‚Ρ€ΠΈΡ†Π°Ρ‚Π΅Π»ΡŒΠ½ΠΎΠ΅;
    • 4) Ρ‡Ρ‚ΠΎΠ±Ρ‹ Π²ΠΎΠ·Π²Ρ‹ΡΠΈΡ‚ΡŒ Π² ΡΡ‚Π΅ΠΏΠ΅Π½ΡŒ ΠΏΡ€ΠΎΠΈΠ·Π²Π΅Π΄Π΅Π½ΠΈΠ΅, достаточно Π²ΠΎΠ·Π²Ρ‹ΡΠΈΡ‚ΡŒ Π² эту ΡΡ‚Π΅ΠΏΠ΅Π½ΡŒ ΠΊΠ°ΠΆΠ΄Ρ‹ΠΉ ΡΠΎΠΌΠ½ΠΎΠΆΠΈΡ‚Π΅Π»ΡŒ ΠΎΡ‚Π΄Π΅Π»ΡŒΠ½ΠΎ;
    • 5) Ρ‡Ρ‚ΠΎΠ±Ρ‹ Π²ΠΎΠ·Π²Ρ‹ΡΠΈΡ‚ΡŒ ΡΡ‚Π΅ΠΏΠ΅Π½ΡŒ Π² ΡΡ‚Π΅ΠΏΠ΅Π½ΡŒ, достаточно ΠΏΠ΅Ρ€Π΅ΠΌΠ½ΠΎΠΆΠΈΡ‚ΡŒ ΠΏΠΎΠΊΠ°Π·Π°Ρ‚Π΅Π»ΠΈ этих стСпСнСй;
    • 6) Ρ‡Ρ‚ΠΎΠ±Ρ‹ Π²ΠΎΠ·Π²Ρ‹ΡΠΈΡ‚ΡŒ Π² ΡΡ‚Π΅ΠΏΠ΅Π½ΡŒ Π΄Ρ€ΠΎΠ±ΡŒ, достаточно Π²ΠΎΠ·Π²Ρ‹ΡΠΈΡ‚ΡŒ Π² эту ΡΡ‚Π΅ΠΏΠ΅Π½ΡŒ ΠΎΡ‚Π΄Π΅Π»ΡŒΠ½ΠΎ Ρ‡ΠΈΡΠ»ΠΈΡ‚Π΅Π»ΡŒ ΠΈ Π·Π½Π°ΠΌΠ΅Π½Π°Ρ‚Π΅Π»ΡŒ;
    • 7) Ρ‡Ρ‚ΠΎΠ±Ρ‹ Π²ΠΎΠ·Π²Ρ‹ΡΠΈΡ‚ΡŒ Ρ€Π°Π΄ΠΈΠΊΠ°Π» Π² ΡΡ‚Π΅ΠΏΠ΅Π½ΡŒ, достаточно Π²ΠΎΠ·Π²Ρ‹ΡΠΈΡ‚ΡŒ Π² эту ΡΡ‚Π΅ΠΏΠ΅Π½ΡŒ ΠΏΠΎΠ΄ΠΊΠΎΡ€Π΅Π½Π½ΠΎΠ΅ Π²Ρ‹Ρ€Π°ΠΆΠ΅Π½ΠΈΠ΅;
    • 8) Ρ‡Ρ‚ΠΎΠ±Ρ‹ ΠΈΠ·Π²Π»Π΅Ρ‡ΡŒ ΠΊΠΎΡ€Π΅Π½ΡŒ ΠΈΠ· стСпСни, достаточно Ρ€Π°Π·Π΄Π΅Π»ΠΈΡ‚ΡŒ ΠΏΠΎΠΊΠ°Π·Π°Ρ‚Π΅Π»ΡŒ стСпСни Π½Π° ΠΏΠΎΠΊΠ°Π·Π°Ρ‚Π΅Π»ΡŒ корня, Ссли Ρ‚Π°ΠΊΠΎΠ΅ Π΄Π΅Π»Π΅Π½ΠΈΠ΅ выполняСтся Π½Π°Ρ†Π΅Π»ΠΎ.

    Π’Π΅ΠΏΠ΅Ρ€ΡŒ ΠΌΡ‹ Ρ€Π°ΡΡˆΠΈΡ€ΠΈΠΌ понятиС ΠΎ показатСлях, ввСдя ΠΏΠΎΠΊΠ°Π·Π°Ρ‚Π΅Π»ΠΈ ΠΎΡ‚Ρ€ΠΈΡ†Π°Ρ‚Π΅Π»ΡŒΠ½Ρ‹Π΅ ΠΈ Π΄Ρ€ΠΎΠ±Π½Ρ‹Π΅, ΠΊΠΎΡ‚ΠΎΡ€Ρ‹Ρ… Π΄ΠΎ сСго Π²Ρ€Π΅ΠΌΠ΅Π½ΠΈ ΠΌΡ‹ Π½Π΅ употрСбляли. ΠœΡ‹ ΡƒΠ²ΠΈΠ΄ΠΈΠΌ ΠΏΡ€ΠΈ этом, Ρ‡Ρ‚ΠΎ всС свойства Ρ†Π΅Π»Ρ‹Ρ… ΠΏΠΎΠ»ΠΎΠΆΠΈΡ‚Π΅Π»ΡŒΠ½Ρ‹Ρ… ΠΏΠΎΠΊΠ°Π·Π°Ρ‚Π΅Π»Π΅ΠΉ ΡΠΎΡ…Ρ€Π°Π½ΡΡŽΡ‚ΡΡ ΠΈ для ΠΏΠΎΠΊΠ°Π·Π°Ρ‚Π΅Π»Π΅ΠΉ ΠΎΡ‚Ρ€ΠΈΡ†Π°Ρ‚Π΅Π»ΡŒΠ½Ρ‹Ρ… ΠΈ Π΄Ρ€ΠΎΠ±Π½Ρ‹Ρ….

    НулСвой ΠΏΠΎΠΊΠ°Π·Π°Ρ‚Π΅Π»ΡŒ

    ΠŸΡ€ΠΈ Π΄Π΅Π»Π΅Π½ΠΈΠΈ стСпСнСй ΠΎΠ΄Π½ΠΎΠ³ΠΎ ΠΈ Ρ‚ΠΎΠ³ΠΎ ΠΆΠ΅ числа ΠΏΠΎΠΊΠ°Π·Π°Ρ‚Π΅Π»ΡŒ Π΄Π΅Π»ΠΈΠΌΠΎΠ³ΠΎ ΠΌΠΎΠΆΠ΅Ρ‚ ΠΎΠΊΠ°Π·Π°Ρ‚ΡŒΡΡ Ρ€Π°Π²Π½Ρ‹ΠΌ ΠΏΠΎΠΊΠ°Π·Π°Ρ‚Π΅Π»ΡŽ дСлитСля.

    ΠŸΡƒΡΡ‚ΡŒ Π½ΡƒΠΆΠ½ΠΎ Ρ€Π°Π·Π΄Π΅Π»ΠΈΡ‚ΡŒ аⁿ Π½Π° аⁿ.

    ΠŸΡ€ΠΈΠΌΠ΅Π½ΡΡ ΠΏΡ€Π°Π²ΠΈΠ»ΠΎ (2), ΠΏΠΎΠ»ΡƒΡ‡Π°Π΅ΠΌ:
    aⁿ : aⁿ =aⁿ⁻ⁿ = α⁰.

    Но Π½ΡƒΠ»ΡŒ, ΠΊΠ°ΠΊ ΠΏΠΎΠΊΠ°Π·Π°Ρ‚Π΅Π»ΡŒ стСпСни, Π½Π΅ ΠΈΠΌΠ΅Π΅Ρ‚ Ρ‚ΠΎΠ³ΠΎ значСния, ΠΊΠΎΡ‚ΠΎΡ€ΠΎΠ΅ придаётся показатСлям Ρ†Π΅Π»Ρ‹ΠΌ ΠΈ ΠΏΠΎΠ»ΠΎΠΆΠΈΡ‚Π΅Π»ΡŒΠ½Ρ‹ΠΌ, Ρ‚Π°ΠΊ ΠΊΠ°ΠΊ нСльзя ΠΏΠΎΠ²Ρ‚ΠΎΡ€ΠΈΡ‚ΡŒ число сомноТитСлСм Π½ΡƒΠ»ΡŒ Ρ€Π°Π·. Π§Ρ‚ΠΎΠ±Ρ‹ ΠΏΡ€ΠΈΠ΄Π°Ρ‚ΡŒ смысл Π²Ρ‹Ρ€Π°ΠΆΠ΅Π½ΠΈΡŽ α⁰, ΠΏΠΎΠ΄ΠΎΠΉΠ΄Ρ‘ΠΌ ΠΊ вопросу ΠΎ Π΄Π΅Π»Π΅Π½ΠΈΠΈ аⁿ Π½Π° аⁿ с Π΄Ρ€ΡƒΠ³ΠΎΠΉ стороны. ΠœΡ‹ Π·Π½Π°Π΅ΠΌ, Ρ‡Ρ‚ΠΎ ΠΏΡ€ΠΈ Π΄Π΅Π»Π΅Π½ΠΈΠΈ любого (ΠΎΡ‚Π»ΠΈΡ‡Π½ΠΎΠ³ΠΎ ΠΎΡ‚ нуля) числа Π½Π° Ρ€Π°Π²Π½ΠΎΠ΅ Π΅ΠΌΡƒ число частноС Ρ€Π°Π²Π½ΠΎ Π΅Π΄ΠΈΠ½ΠΈΡ†Π΅.

    ΠŸΠΎΡΡ‚ΠΎΠΌΡƒ ΡƒΡΠ»ΠΎΠ²ΠΈΠ»ΠΈΡΡŒ ΡΡ‡ΠΈΡ‚Π°Ρ‚ΡŒ α⁰=l.

    Π’Π°ΠΊΠΈΠΌ ΠΎΠ±Ρ€Π°Π·ΠΎΠΌ, ΠΏΠΎ ΠΎΠΏΡ€Π΅Π΄Π΅Π»Π΅Π½ΠΈΡŽ:
    ВсякоС число (Π·Π° ΠΈΡΠΊΠ»ΡŽΡ‡Π΅Π½ΠΈΠ΅ΠΌ нуля) Π² Π½ΡƒΠ»Π΅Π²ΠΎΠΉ стСпСни Ρ€Π°Π²Π½ΠΎ Π΅Π΄ΠΈΠ½ΠΈΡ†Π΅.

    Π›Π΅Π³ΠΊΠΎ ΡƒΠ±Π΅Π΄ΠΈΡ‚ΡŒΡΡ Π² Ρ‚ΠΎΠΌ, Ρ‡Ρ‚ΠΎ пСрСчислСнныС Π²Ρ‹ΡˆΠ΅ свойства Ρ†Π΅Π»Ρ‹Ρ… ΠΏΠΎΠ»ΠΎΠΆΠΈΡ‚Π΅Π»ΡŒΠ½Ρ‹Ρ… ΠΏΠΎΠΊΠ°Π·Π°Ρ‚Π΅Π»Π΅ΠΉ ΠΏΡ€ΠΈΠΌΠ΅Π½ΠΈΠΌΡ‹ ΠΈ ΠΊ Π½ΡƒΠ»Π΅Π²ΠΎΠΌΡƒ ΠΏΠΎΠΊΠ°Π·Π°Ρ‚Π΅Π»ΡŽ. Π’Π°ΠΊ:

    ΠžΡ‚Ρ€ΠΈΡ†Π°Ρ‚Π΅Π»ΡŒΠ½Ρ‹Π΅ Ρ†Π΅Π»Ρ‹Π΅ ΠΏΠΎΠΊΠ°Π·Π°Ρ‚Π΅Π»ΠΈ

    Условимся ΠΏΡ€ΠΈ Π΄Π΅Π»Π΅Π½ΠΈΠΈ стСпСнСй ΠΎΠ΄Π½ΠΎΠ³ΠΎ ΠΈ Ρ‚ΠΎΠ³ΠΎ ΠΆΠ΅ числа Π²Ρ‹Ρ‡ΠΈΡ‚Π°Ρ‚ΡŒ ΠΏΠΎΠΊΠ°Π·Π°Ρ‚Π΅Π»ΡŒ дСлитСля ΠΈΠ· показатСля Π΄Π΅Π»ΠΈΠΌΠΎΠ³ΠΎ ΠΈ Π² Ρ‚ΠΎΠΌ случаС, Ссли ΠΏΠΎΠΊΠ°Π·Π°Ρ‚Π΅Π»ΡŒ дСлитСля большС показатСля Π΄Π΅Π»ΠΈΠΌΠΎΠ³ΠΎ. Π’ΠΎΠ³Π΄Π° ΠΌΡ‹ ΠΏΠΎΠ»ΡƒΡ‡ΠΈΠΌ Π² частном Π±ΡƒΠΊΠ²Ρƒ с ΠΎΡ‚Ρ€ΠΈΡ†Π°Ρ‚Π΅Π»ΡŒΠ½Ρ‹ΠΌ ΠΏΠΎΠΊΠ°Π·Π°Ρ‚Π΅Π»Π΅ΠΌ, Π½Π°ΠΏΡ€ΠΈΠΌΠ΅Ρ€: Ξ±Β² : α⁡= α⁻³. Π’Π°ΠΊΠΈΠΌ ΠΎΠ±Ρ€Π°Π·ΠΎΠΌ, число с ΠΎΡ‚Ρ€ΠΈΡ†Π°Ρ‚Π΅Π»ΡŒΠ½Ρ‹ΠΌ ΠΏΠΎΠΊΠ°Π·Π°Ρ‚Π΅Π»Π΅ΠΌ ΠΌΡ‹ условимся ΡƒΠΏΠΎΡ‚Ρ€Π΅Π±Π»ΡΡ‚ΡŒ для обозначСния частного ΠΎΡ‚ дСлСния стСпСнСй этого числа Π² Ρ‚ΠΎΠΌ случаС, ΠΊΠΎΠ³Π΄Π° ΠΏΠΎΠΊΠ°Π·Π°Ρ‚Π΅Π»ΡŒ дСлитСля прСвосходит ΠΏΠΎΠΊΠ°Π·Π°Ρ‚Π΅Π»ΡŒ Π΄Π΅Π»ΠΈΠΌΠΎΠ³ΠΎ Π½Π° ΡΡ‚ΠΎΠ»ΡŒΠΊΠΎ Π΅Π΄ΠΈΠ½ΠΈΡ†, сколько ΠΈΡ… находится Π² Π°Π±ΡΠΎΠ»ΡŽΡ‚Π½ΠΎΠΉ Π²Π΅Π»ΠΈΡ‡ΠΈΠ½Π΅ ΠΎΡ‚Ρ€ΠΈΡ†Π°Ρ‚Π΅Π»ΡŒΠ½ΠΎΠ³ΠΎ показатСля. Π’Π°ΠΊ, α⁻² ΠΎΠ·Π½Π°Ρ‡Π°Π΅Ρ‚ частноС Ξ± : Ξ±Β³, ΠΈΠ»ΠΈ Ξ±Β² : α⁡, ΠΈΠ»ΠΈ Ξ±Β³ : α⁡, Π²ΠΎΠΎΠ±Ρ‰Π΅ частноС Ξ± Ν« : Ξ± Ν« ⁺².

    ΠŸΡ€ΠΈΠΌΠ΅Π½ΡΠ΅ΠΌΠΎΠ΅ Π² этом смыслС число с ΠΎΡ‚Ρ€ΠΈΡ†Π°Ρ‚Π΅Π»ΡŒΠ½Ρ‹ΠΌ ΠΏΠΎΠΊΠ°Π·Π°Ρ‚Π΅Π»Π΅ΠΌ Ρ€Π°Π²Π½ΠΎ Π΄Ρ€ΠΎΠ±ΠΈ, Ρƒ ΠΊΠΎΡ‚ΠΎΡ€ΠΎΠΉ Ρ‡ΠΈΡΠ»ΠΈΡ‚Π΅Π»ΡŒ 1, Π° Π·Π½Π°ΠΌΠ΅Π½Π°Ρ‚Π΅Π»ΡŒ β€” Ρ‚ΠΎ ΠΆΠ΅ число, Π½ΠΎ с ΠΏΠΎΠ»ΠΎΠΆΠΈΡ‚Π΅Π»ΡŒΠ½Ρ‹ΠΌ ΠΏΠΎΠΊΠ°Π·Π°Ρ‚Π΅Π»Π΅ΠΌ, Ρ€Π°Π²Π½Ρ‹ΠΌ Π°Π±ΡΠΎΠ»ΡŽΡ‚Π½ΠΎΠΉ Π²Π΅Π»ΠΈΡ‡ΠΈΠ½Π΅ ΠΎΡ‚Ρ€ΠΈΡ†Π°Ρ‚Π΅Π»ΡŒΠ½ΠΎΠ³ΠΎ показатСля.

    Π”Π΅ΠΉΡΡ‚Π²ΠΈΡ‚Π΅Π»ΡŒΠ½ΠΎ, согласно Π½Π°ΡˆΠ΅ΠΌΡƒ ΡƒΡΠ»ΠΎΠ²ΠΈΡŽ, ΠΌΡ‹ Π΄ΠΎΠ»ΠΆΠ½Ρ‹ ΠΈΠΌΠ΅Ρ‚ΡŒ:

    Π‘ΠΎΠΊΡ€Π°Ρ‚ΠΈΠ² Π΄Π²Π΅ ΠΏΠ΅Ρ€Π²Ρ‹Π΅ Π΄Ρ€ΠΎΠ±ΠΈ Π½Π° Π°Ρ‚ ΠΈ Ρ‚Ρ€Π΅Ρ‚ΡŒΡŽ Π΄Ρ€ΠΎΠ±ΡŒ Π½Π° Ρ…Ρ‚ (Ρ‚. Π΅. Π² ΠΎΠ±ΠΎΠΈΡ… случаях сократив Π΄Ρ€ΠΎΠ±ΠΈ Π½Π° Ρ‡ΠΈΡΠ»ΠΈΡ‚Π΅Π»ΡŒ), ΠΏΠΎΠ»ΡƒΡ‡ΠΈΠΌ:

    Π’ΠΎΠΎΠ±Ρ‰Π΅:

    Π—Π°ΠΌΠ΅Ρ‚ΠΈΠΌ, Ρ‡Ρ‚ΠΎ ΠΎΡ‚Ρ€ΠΈΡ†Π°Ρ‚Π΅Π»ΡŒΠ½Ρ‹Π΅ ΠΏΠΎΠΊΠ°Π·Π°Ρ‚Π΅Π»ΠΈ Π΄Π°ΡŽΡ‚ Π²ΠΎΠ·ΠΌΠΎΠΆΠ½ΠΎΡΡ‚ΡŒ ΠΏΡ€Π΅Π΄ΡΡ‚Π°Π²ΠΈΡ‚ΡŒ всякоС Π΄Ρ€ΠΎΠ±Π½ΠΎΠ΅ алгСбраичСскоС Π²Ρ‹Ρ€Π°ΠΆΠ΅Π½ΠΈΠ΅ ΠΏΠΎΠ΄ Π²ΠΈΠ΄ΠΎΠΌ Ρ†Π΅Π»ΠΎΠ³ΠΎ; для этого стоит Ρ‚ΠΎΠ»ΡŒΠΊΠΎ всС ΠΌΠ½ΠΎΠΆΠΈΡ‚Π΅Π»ΠΈ знамСнатСля пСрСнСсти мноТитСлями Π² Ρ‡ΠΈΡΠ»ΠΈΡ‚Π΅Π»ΡŒ, взяв ΠΈΡ… с ΠΎΡ‚Ρ€ΠΈΡ†Π°Ρ‚Π΅Π»ΡŒΠ½Ρ‹ΠΌΠΈ показатСлями. НапримСр:

    ДСйствия Π½Π°Π΄ стСпСнями с ΠΎΡ‚Ρ€ΠΈΡ†Π°Ρ‚Π΅Π»ΡŒΠ½Ρ‹ΠΌΠΈ показатСлями

    УбСдимся Ρ‚Π΅ΠΏΠ΅Ρ€ΡŒ, Ρ‡Ρ‚ΠΎ всС дСйствия Π½Π°Π΄ стСпСнями с ΠΎΡ‚Ρ€ΠΈΡ†Π°Ρ‚Π΅Π»ΡŒΠ½Ρ‹ΠΌΠΈ показатСлями ΠΌΠΎΠΆΠ½ΠΎ ΠΏΡ€ΠΎΠΈΠ·Π²ΠΎΠ΄ΠΈΡ‚ΡŒ ΠΏΠΎ Ρ‚Π΅ΠΌ ΠΆΠ΅ ΠΏΡ€Π°Π²ΠΈΠ»Π°ΠΌ, ΠΊΠ°ΠΊΠΈΠ΅ Π±Ρ‹Π»ΠΈ ΠΏΡ€Π΅ΠΆΠ΄Π΅ Π²Ρ‹Π²Π΅Π΄Π΅Π½Ρ‹ для ΠΏΠΎΠΊΠ°Π·Π°Ρ‚Π΅Π»Π΅ΠΉ ΠΏΠΎΠ»ΠΎΠΆΠΈΡ‚Π΅Π»ΡŒΠ½Ρ‹Ρ…. Достаточно ΠΎΠ±Π½Π°Ρ€ΡƒΠΆΠΈΡ‚ΡŒ это Ρ‚ΠΎΠ»ΡŒΠΊΠΎ для умноТСния ΠΈ Π²ΠΎΠ·Π²Ρ‹ΡˆΠ΅Π½ΠΈΡ Π² ΡΡ‚Π΅ΠΏΠ΅Π½ΡŒ, Ρ‚Π°ΠΊ ΠΊΠ°ΠΊ ΠΏΡ€Π°Π²ΠΈΠ»Π° ΠΎΠ±Ρ€Π°Ρ‚Π½Ρ‹Ρ… дСйствий β€” дСлСния ΠΈ извлСчСния корня β€” ΡΠ²Π»ΡΡŽΡ‚ΡΡ слСдствиями ΠΏΡ€Π°Π²ΠΈΠ» прямых дСйствий.

    Π£ΠΌΠ½ΠΎΠΆΠ΅Π½ΠΈΠ΅

    ΠŸΡ€Π΅Π΄ΡΡ‚ΠΎΠΈΡ‚ ΠΏΠΎΠΊΠ°Π·Π°Ρ‚ΡŒ, Ρ‡Ρ‚ΠΎ ΠΏΡ€ΠΈ ΡƒΠΌΠ½ΠΎΠΆΠ΅Π½ΠΈΠΈ стСпСнСй ΠΏΠΎΠΊΠ°Π·Π°Ρ‚Π΅Π»ΠΈ ΠΎΠ΄ΠΈΠ½Π°ΠΊΠΎΠ²Ρ‹Ρ… Π±ΡƒΠΊΠ² ΡΠΊΠ»Π°Π΄Ρ‹Π²Π°ΡŽΡ‚ΡΡ ΠΈ Π² Ρ‚ΠΎΠΌ случаС, ΠΊΠΎΠ³Π΄Π° эти ΠΏΠΎΠΊΠ°Π·Π°Ρ‚Π΅Π»ΠΈ ΠΎΡ‚Ρ€ΠΈΡ†Π°Ρ‚Π΅Π»ΡŒΠ½Ρ‹Π΅. УбСдимся, Ρ‡Ρ‚ΠΎ , Π³Π΄Π΅ m ΠΈ n β€” Ρ†Π΅Π»Ρ‹Π΅ ΠΏΠΎΠ»ΠΎΠΆΠΈΡ‚Π΅Π»ΡŒΠ½Ρ‹Π΅ числа.

    Π”Π΅ΠΉΡΡ‚Π²ΠΈΡ‚Π΅Π»ΡŒΠ½ΠΎ, Π·Π°ΠΌΠ΅Π½ΠΈΠ² стСпСни с ΠΎΡ‚Ρ€ΠΈΡ†Π°Ρ‚Π΅Π»ΡŒΠ½Ρ‹ΠΌΠΈ показатСлями дробями ΠΈ произвСдя дСйствиС умноТСния ΠΏΠΎ ΠΏΡ€Π°Π²ΠΈΠ»Π°ΠΌ, относящимся ΠΊ дробям, ΠΏΠΎΠ»ΡƒΡ‡ΠΈΠΌ:

    Подобно этому:
    Ρ‚Π°ΠΊ ΠΊΠ°ΠΊ

    Π’ΠΎΠ·Π²Ρ‹ΡˆΠ΅Π½ΠΈΠ΅ Π² ΡΡ‚Π΅ΠΏΠ΅Π½ΡŒ

    Надо ΠΏΠΎΠΊΠ°Π·Π°Ρ‚ΡŒ, Ρ‡Ρ‚ΠΎ ΠΏΡ€ΠΈ Π²ΠΎΠ·Π²Ρ‹ΡˆΠ΅Π½ΠΈΠΈ Π² ΡΡ‚Π΅ΠΏΠ΅Π½ΡŒ ΠΏΠΎΠΊΠ°Π·Π°Ρ‚Π΅Π»ΠΈ этих стСпСнСй ΠΏΠ΅Ρ€Π΅ΠΌΠ½ΠΎΠΆΠ°ΡŽΡ‚ΡΡ ΠΈ Π² Ρ‚ΠΎΠΌ случаС, ΠΊΠΎΠ³Π΄Π° ΠΎΠ½ΠΈ ΠΎΡ‚Ρ€ΠΈΡ†Π°Ρ‚Π΅Π»ΡŒΠ½Ρ‹Π΅. УбСдимся, Ρ‡Ρ‚ΠΎ .

    Π”Π΅ΠΉΡΡ‚Π²ΠΈΡ‚Π΅Π»ΡŒΠ½ΠΎ:

    Подобно этому:
    , ΠΏΠΎΡ‚ΠΎΠΌΡƒ Ρ‡Ρ‚ΠΎ

    ΠŸΡ€ΠΈΠΌΠ΅Ρ€Ρ‹:
    1) (3α⁻ ²b²c⁻ ³) (0,8ab⁻ ³ c⁴)=2,4α⁻ ¹b⁻ ¹ c.
    2) (x⁻ ¹ y³ z²) : (5x²y⁻ ² z³ ) = x⁻ ³y⁡z⁻ ¹ .
    3) (2αx⁻ ³ )⁻ ² =2⁻ ² α⁻ ² x⁢.
    4) (х⁻ Β² β€” у⁻ ΒΉ )Β² =(x⁻ Β² ) Β² β€” 2x⁻ Β² y⁻ ΒΉ +(y⁻ ΒΉ ) Β² =x⁻ ⁴ β€” 2x⁻ Β² y⁻ ΒΉ +y⁻ Β² .
    5) (a⁻ Β² + b⁻ Β³ ) (а⁻ Β² β€” b⁻ Β³ )=a⁻ ⁴ β€” b⁻ ⁢.
    6) =3p⁻ ³ q⁻ ¹.

    Π”Ρ€ΠΎΠ±Π½Ρ‹Π΅ ΠΏΠΎΠΊΠ°Π·Π°Ρ‚Π΅Π»ΠΈ

    Π’ ΠΊΠ°ΠΊΠΎΠΌ смыслС ΡƒΠΏΠΎΡ‚Ρ€Π΅Π±Π»ΡΡŽΡ‚ΡΡ Π΄Ρ€ΠΎΠ±Π½Ρ‹Π΅ ΠΏΠΎΠΊΠ°Π·Π°Ρ‚Π΅Π»ΠΈ: ΠœΡ‹ Π·Π½Π°Π΅ΠΌ, Ρ‡Ρ‚ΠΎ ΠΏΡ€ΠΈ ΠΈΠ·Π²Π»Π΅Ρ‡Π΅Π½ΠΈΠΈ корня ΠΈΠ· стСпСни дСлят ΠΏΠΎΠΊΠ°Π·Π°Ρ‚Π΅Π»ΡŒ стСпСни Π½Π° ΠΏΠΎΠΊΠ°Π·Π°Ρ‚Π΅Π»ΡŒ корня, Ссли Ρ‚Π°ΠΊΠΎΠ΅ Π΄Π΅Π»Π΅Π½ΠΈΠ΅ выполняСтся Π½Π°Ρ†Π΅Π»ΠΎ; Π½Π°ΠΏΡ€ΠΈΠΌΠ΅Ρ€: ΠΈ Ρ‚. Π΄. Условимся Ρ‚Π΅ΠΏΠ΅Ρ€ΡŒ Ρ€Π°ΡΠΏΡ€ΠΎΡΡ‚Ρ€Π°Π½ΡΡ‚ΡŒ это ΠΏΡ€Π°Π²ΠΈΠ»ΠΎ ΠΈ Π½Π° Ρ‚Π΅ случаи, ΠΊΠΎΠ³Π΄Π° ΠΏΠΎΠΊΠ°Π·Π°Ρ‚Π΅Π»ΡŒ стСпСни Π½Π΅ дСлится Π½Π°Ρ†Π΅Π»ΠΎ Π½Π° ΠΏΠΎΠΊΠ°Π·Π°Ρ‚Π΅Π»ΡŒ корня. НапримСр, ΠΌΡ‹ условимся ΠΏΡ€ΠΈΠ½ΠΈΠΌΠ°Ρ‚ΡŒ, Ρ‡Ρ‚ΠΎ:

    Π’ΠΎΠΎΠ±Ρ‰Π΅ ΠΌΡ‹ условимся, Ρ‡Ρ‚ΠΎ:
    Π’Ρ‹Ρ€Π°ΠΆΠ΅Π½ΠΈΠ΅ ΠΎΠ·Π½Π°Ρ‡Π°Π΅Ρ‚ ΠΊΠΎΡ€Π΅Π½ΡŒ, ΠΏΠΎΠΊΠ°Π·Π°Ρ‚Π΅Π»ΡŒ ΠΊΠΎΡ‚ΠΎΡ€ΠΎΠ³ΠΎ Ρ€Π°Π²Π΅Π½ Π·Π½Π°ΠΌΠ΅Π½Π°Ρ‚Π΅Π»ΡŽ, Π° ΠΏΠΎΠΊΠ°Π·Π°Ρ‚Π΅Π»ΡŒ стСпСни ΠΏΠΎΠ΄ΠΊΠΎΡ€Π΅Π½Π½ΠΎΠ³ΠΎ числа Ρ€Π°Π²Π΅Π½ Ρ‡ΠΈΡΠ»ΠΈΡ‚Π΅Π»ΡŽ показатСля

    Условимся ΡƒΠΏΠΎΡ‚Ρ€Π΅Π±Π»ΡΡ‚ΡŒ ΠΎΡ‚Ρ€ΠΈΡ†Π°Ρ‚Π΅Π»ΡŒΠ½Ρ‹Π΅ Π΄Ρ€ΠΎΠ±Π½Ρ‹Π΅ ΠΏΠΎΠΊΠ°Π·Π°Ρ‚Π΅Π»ΠΈ Π² Ρ‚ΠΎΠΌ ΠΆΠ΅ смыслС, Π² ΠΊΠ°ΠΊΠΎΠΌ ΠΌΡ‹ употрСбляли ΠΎΡ‚Ρ€ΠΈΡ†Π°Ρ‚Π΅Π»ΡŒΠ½Ρ‹Π΅ Ρ†Π΅Π»Ρ‹Π΅ ΠΏΠΎΠΊΠ°Π·Π°Ρ‚Π΅Π»ΠΈ; Π½Π°ΠΏΡ€ΠΈΠΌΠ΅Ρ€, условимся, Ρ‡Ρ‚ΠΎ

    ОсновноС свойство Π΄Ρ€ΠΎΠ±Π½ΠΎΠ³ΠΎ показатСля

    Π’Π΅Π»ΠΈΡ‡ΠΈΠ½Π° стСпСни с Π΄Ρ€ΠΎΠ±Π½Ρ‹ΠΌ ΠΏΠΎΠΊΠ°Π·Π°Ρ‚Π΅Π»Π΅ΠΌ Π½Π΅ измСнится, Ссли ΠΌΡ‹ ΡƒΠΌΠ½ΠΎΠΆΠΈΠΌ ΠΈΠ»ΠΈ Ρ€Π°Π·Π΄Π΅Π»ΠΈΠΌ Π½Π° ΠΎΠ΄Π½ΠΎ ΠΈ Ρ‚ΠΎ ΠΆΠ΅ число (ΠΎΡ‚Π»ΠΈΡ‡Π½ΠΎΠ΅ ΠΎΡ‚ нуля) Ρ‡ΠΈΡΠ»ΠΈΡ‚Π΅Π»ΡŒ ΠΈ Π·Π½Π°ΠΌΠ΅Π½Π°Ρ‚Π΅Π»ΡŒ Π΄Ρ€ΠΎΠ±Π½ΠΎΠ³ΠΎ показатСля.
    Π’Π°ΠΊ:

    Π’ΠΎΠΎΠ±Ρ‰Π΅:

    Π”Π΅ΠΉΡΡ‚Π²ΠΈΡ‚Π΅Π»ΡŒΠ½ΠΎ, Π·Π½Π°ΠΌΠ΅Π½Π°Ρ‚Π΅Π»ΡŒ Π΄Ρ€ΠΎΠ±Π½ΠΎΠ³ΠΎ показатСля ΠΎΠ·Π½Π°Ρ‡Π°Π΅Ρ‚ ΠΏΠΎΠΊΠ°Π·Π°Ρ‚Π΅Π»ΡŒ корня, Π° Ρ‡ΠΈΡΠ»ΠΈΡ‚Π΅Π»ΡŒ Π΅Π³ΠΎ ΠΎΠ·Π½Π°Ρ‡Π°Π΅Ρ‚ ΠΏΠΎΠΊΠ°Π·Π°Ρ‚Π΅Π»ΡŒ стСпСни ΠΏΠΎΠ΄ΠΊΠΎΡ€Π΅Π½Π½ΠΎΠ³ΠΎ выраТСния, Π° Ρ‚Π°ΠΊΠΈΠ΅ ΠΏΠΎΠΊΠ°Π·Π°Ρ‚Π΅Π»ΠΈ, ΠΊΠ°ΠΊ ΠΌΡ‹ Π²ΠΈΠ΄Π΅Π»ΠΈ, ΠΌΠΎΠΆΠ½ΠΎ ΡƒΠΌΠ½ΠΎΠΆΠ°Ρ‚ΡŒ ΠΈ Π΄Π΅Π»ΠΈΡ‚ΡŒ Π½Π° ΠΎΠ΄Π½ΠΎ ΠΈ Ρ‚ΠΎ ΠΆΠ΅ число.

    ΠžΡΠ½ΠΎΠ²Ρ‹Π²Π°ΡΡΡŒ Π½Π° этом свойствС, ΠΌΡ‹ ΠΌΠΎΠΆΠ΅ΠΌ ΠΏΡ€Π΅ΠΎΠ±Ρ€Π°Π·ΠΎΠ²Ρ‹Π²Π°Ρ‚ΡŒ Π΄Ρ€ΠΎΠ±Π½Ρ‹ΠΉ ΠΏΠΎΠΊΠ°Π·Π°Ρ‚Π΅Π»ΡŒ ΡΠΎΠ²Π΅Ρ€ΡˆΠ΅Π½Π½ΠΎ Ρ‚Π°ΠΊ ΠΆΠ΅, ΠΊΠ°ΠΊ ΠΈ ΠΎΠ±Ρ‹ΠΊΠ½ΠΎΠ²Π΅Π½Π½ΡƒΡŽ Π΄Ρ€ΠΎΠ±ΡŒ; Π½Π°ΠΏΡ€ΠΈΠΌΠ΅Ρ€, ΠΌΡ‹ ΠΌΠΎΠΆΠ΅ΠΌ ΡΠΎΠΊΡ€Π°Ρ‰Π°Ρ‚ΡŒ Π΄Ρ€ΠΎΠ±Π½Ρ‹ΠΉ ΠΏΠΎΠΊΠ°Π·Π°Ρ‚Π΅Π»ΡŒ ΠΈΠ»ΠΈ ΠΏΡ€ΠΈΠ²ΠΎΠ΄ΠΈΡ‚ΡŒ нСсколько Π΄Ρ€ΠΎΠ±Π½Ρ‹Ρ… ΠΏΠΎΠΊΠ°Π·Π°Ρ‚Π΅Π»Π΅ΠΉ ΠΊ ΠΎΠ΄Π½ΠΎΠΌΡƒ Π·Π½Π°ΠΌΠ΅Π½Π°Ρ‚Π΅Π»ΡŽ.

    ДСйствия Π½Π°Π΄ стСпСнями с Π΄Ρ€ΠΎΠ±Π½Ρ‹ΠΌΠΈ показатСлями

    ΠŸΡ€Π΅Π΄ΡΡ‚ΠΎΠΈΡ‚ Π΄ΠΎΠΊΠ°Π·Π°Ρ‚ΡŒ, Ρ‡Ρ‚ΠΎ ΠΊ Π΄Ρ€ΠΎΠ±Π½Ρ‹ΠΌ показатСлям ΠΏΡ€ΠΈΠΌΠ΅Π½ΠΈΠΌΡ‹ ΠΏΡ€Π°Π²ΠΈΠ»Π°, Π²Ρ‹Π²Π΅Π΄Π΅Π½Π½Ρ‹Π΅ Ρ€Π°Π½ΡŒΡˆΠ΅ для Ρ†Π΅Π»Ρ‹Ρ… ΠΏΠΎΠΊΠ°Π·Π°Ρ‚Π΅Π»Π΅ΠΉ. Π­Ρ‚ΠΎ достаточно ΠΎΠ±Π½Π°Ρ€ΡƒΠΆΠΈΡ‚ΡŒ Ρ‚ΠΎΠ»ΡŒΠΊΠΎ для умноТСния ΠΈ Π²ΠΎΠ·Π²Ρ‹ΡˆΠ΅Π½ΠΈΡ Π² ΡΡ‚Π΅ΠΏΠ΅Π½ΡŒ, Ρ‚Π°ΠΊ ΠΊΠ°ΠΊ ΠΏΡ€Π°Π²ΠΈΠ»Π° дСлСния ΠΈ извлСчСния корня ΡΠ²Π»ΡΡŽΡ‚ΡΡ слСдствиями ΠΏΡ€Π°Π²ΠΈΠ» умноТСния ΠΈ Π²ΠΎΠ·Π²Ρ‹ΡˆΠ΅Π½ΠΈΡ Π² ΡΡ‚Π΅ΠΏΠ΅Π½ΡŒ.

    Π£ΠΌΠ½ΠΎΠΆΠ΅Π½ΠΈΠ΅

    Π”ΠΎΠΊΠ°ΠΆΠ΅ΠΌ, Ρ‡Ρ‚ΠΎ ΠΏΡ€ΠΈ ΡƒΠΌΠ½ΠΎΠΆΠ΅Π½ΠΈΠΈ ΠΏΠΎΠΊΠ°Π·Π°Ρ‚Π΅Π»ΠΈ стСпСнСй ΠΎΠ΄ΠΈΠ½Π°ΠΊΠΎΠ²Ρ‹Ρ… Π±ΡƒΠΊΠ² ΡΠΊΠ»Π°Π΄Ρ‹Π²Π°ΡŽΡ‚ΡΡ ΠΈ Ρ‚ΠΎΠ³Π΄Π°, ΠΊΠΎΠ³Π΄Π° эти ΠΏΠΎΠΊΠ°Π·Π°Ρ‚Π΅Π»ΠΈ Π΄Ρ€ΠΎΠ±Π½Ρ‹Π΅. НапримСр, убСдимся, Ρ‡Ρ‚ΠΎ

    Для этого ΠΈΠ·ΠΎΠ±Ρ€Π°Π·ΠΈΠΌ стСпСни с Π΄Ρ€ΠΎΠ±Π½Ρ‹ΠΌΠΈ показатСлями Π² Π²ΠΈΠ΄Π΅ Ρ€Π°Π΄ΠΈΠΊΠ°Π»ΠΎΠ² ΠΈ ΠΏΡ€ΠΎΠΈΠ·Π²Π΅Π΄Ρ‘ΠΌ ΡƒΠΌΠ½ΠΎΠΆΠ΅Π½ΠΈΠ΅ ΠΏΠΎ ΠΏΡ€Π°Π²ΠΈΠ»Ρƒ умноТСния Ρ€Π°Π΄ΠΈΠΊΠ°Π»ΠΎΠ²:

    Π Π΅Π·ΡƒΠ»ΡŒΡ‚Π°Ρ‚ получился Ρ‚ΠΎΡ‚ ΠΆΠ΅ самый, ΠΊΠ°ΠΊΠΎΠΉ ΠΌΡ‹ ΠΏΠΎΠ»ΡƒΡ‡ΠΈΠ»ΠΈ послС слоТСния ΠΏΠΎΠΊΠ°Π·Π°Ρ‚Π΅Π»Π΅ΠΉ; Π·Π½Π°Ρ‡ΠΈΡ‚, ΠΏΡ€Π°Π²ΠΈΠ»ΠΎ ΠΎ слоТСнии ΠΏΠΎΠΊΠ°Π·Π°Ρ‚Π΅Π»Π΅ΠΉ (ΠΏΡ€ΠΈ ΡƒΠΌΠ½ΠΎΠΆΠ΅Π½ΠΈΠΈ) ΠΌΠΎΠΆΠ½ΠΎ ΠΏΡ€ΠΈΠΌΠ΅Π½ΡΡ‚ΡŒ ΠΈ для Π΄Ρ€ΠΎΠ±Π½Ρ‹Ρ… ΠΏΠΎΠΊΠ°Π·Π°Ρ‚Π΅Π»Π΅ΠΉ.
    Π’Π°ΠΊΠΈΠΌ ΠΎΠ±Ρ€Π°Π·ΠΎΠΌ:

    Π’ΠΎΠ·Π²Ρ‹ΡˆΠ΅Π½ΠΈΠ΅ Π² ΡΡ‚Π΅ΠΏΠ΅Π½ΡŒ

    Π”ΠΎΠΊΠ°ΠΆΠ΅ΠΌ, Ρ‡Ρ‚ΠΎ ΠΏΡ€ΠΈ Π²ΠΎΠ·Π²Ρ‹ΡˆΠ΅Π½ΠΈΠΈ стСпСни Π² ΡΡ‚Π΅ΠΏΠ΅Π½ΡŒ ΠΏΠΎΠΊΠ°Π·Π°Ρ‚Π΅Π»ΠΈ стСпСнСй ΠΌΠΎΠΆΠ½ΠΎ ΠΏΠ΅Ρ€Π΅ΠΌΠ½ΠΎΠΆΠ°Ρ‚ΡŒ ΠΈ Ρ‚ΠΎΠ³Π΄Π°, ΠΊΠΎΠ³Π΄Π° эти ΠΏΠΎΠΊΠ°Π·Π°Ρ‚Π΅Π»ΠΈ Π΄Ρ€ΠΎΠ±Π½Ρ‹Π΅. НапримСр, убСдимся, Ρ‡Ρ‚ΠΎ

    Π”Π΅ΠΉΡΡ‚Π²ΠΈΡ‚Π΅Π»ΡŒΠ½ΠΎ, Π·Π°ΠΌΠ΅Π½ΠΈΠ² Ρ€Π°Π΄ΠΈΠΊΠ°Π»Π°ΠΌΠΈ стСпСни с Π΄Ρ€ΠΎΠ±Π½Ρ‹ΠΌΠΈ показатСлями ΠΈ произвСдя дСйствия Π½Π°Π΄ Ρ€Π°Π΄ΠΈΠΊΠ°Π»Π°ΠΌΠΈ, ΠΏΠΎΠ»ΡƒΡ‡ΠΈΠΌ:

    Если ΠΏΠΎΠΊΠ°Π·Π°Ρ‚Π΅Π»ΠΈ Π½Π΅ Ρ‚ΠΎΠ»ΡŒΠΊΠΎ Π΄Ρ€ΠΎΠ±Π½Ρ‹Π΅ числа, Π½ΠΎ ΠΈ ΠΎΡ‚Ρ€ΠΈΡ†Π°Ρ‚Π΅Π»ΡŒΠ½Ρ‹Π΅, Ρ‚ΠΎ ΠΈ Ρ‚ΠΎΠ³Π΄Π° ΠΊ Π½ΠΈΠΌ ΠΌΠΎΠΆΠ½ΠΎ ΠΏΡ€ΠΈΠΌΠ΅Π½ΡΡ‚ΡŒ ΠΏΡ€Π°Π²ΠΈΠ»Π°, Π΄ΠΎΠΊΠ°Π·Π°Π½Π½Ρ‹Π΅ Ρ€Π°Π½ΡŒΡˆΠ΅ для ΠΏΠΎΠ»ΠΎΠΆΠΈΡ‚Π΅Π»ΡŒΠ½Ρ‹Ρ… ΠΏΠΎΠΊΠ°Π·Π°Ρ‚Π΅Π»Π΅ΠΉ. НапримСр:

    ΠŸΡ€ΠΈΠΌΠ΅Ρ€Ρ‹ Π½Π° дСйствия с Π΄Ρ€ΠΎΠ±Π½Ρ‹ΠΌΠΈ ΠΈ ΠΎΡ‚Ρ€ΠΈΡ†Π°Ρ‚Π΅Π»ΡŒΠ½Ρ‹ΠΌΠΈ показатСлями






    ΠŸΠΎΠ½ΡΡ‚ΠΈΠ΅ ΠΎΠ± ΠΈΡ€Ρ€Π°Ρ†ΠΈΠΎΠ½Π°Π»ΡŒΠ½ΠΎΠΌ ΠΏΠΎΠΊΠ°Π·Π°Ρ‚Π΅Π»Π΅

    Бмысл стСпСни с ΠΈΡ€Ρ€Π°Ρ†ΠΈΠΎΠ½Π°Π»ΡŒΠ½Ρ‹ΠΌ ΠΏΠΎΠΊΠ°Π·Π°Ρ‚Π΅Π»Π΅ΠΌ:

    Рассмотрим стСпСни , Π² ΠΊΠΎΡ‚ΠΎΡ€Ρ‹Ρ… Ξ± β€” ΠΊΠ°ΠΊΠΎΠ΅-Π½ΠΈΠ±ΡƒΠ΄ΡŒ ΠΈΡ€Ρ€Π°Ρ†ΠΈΠΎΠ½Π°Π»ΡŒΠ½ΠΎΠ΅ число, ΠΊΠΎΠ³Π΄Π° основаниС стСпСни Ξ± Π΅ΡΡ‚ΡŒ ΠΊΠ°ΠΊΠΎΠ΅-Π½ΠΈΠ±ΡƒΠ΄ΡŒ ΠΏΠΎΠ»ΠΎΠΆΠΈΡ‚Π΅Π»ΡŒΠ½ΠΎΠ΅ число, Π½Π΅ Ρ€Π°Π²Π½ΠΎΠ΅ 1. ΠŸΡ€ΠΈ этом ΠΌΠΎΠ³ΡƒΡ‚ ΠΏΡ€Π΅Π΄ΡΡ‚Π°Π²ΠΈΡ‚ΡŒΡΡ ΡΠ»Π΅Π΄ΡƒΡŽΡ‰ΠΈΠ΅ Ρ‚Ρ€ΠΈ случая:

    a) Ξ± > 1 ΠΈ Ξ± β€” ΠΏΠΎΠ»ΠΎΠΆΠΈΡ‚Π΅Π»ΡŒΠ½ΠΎΠ΅ ΠΈΡ€Ρ€Π°Ρ†ΠΈΠΎΠ½Π°Π»ΡŒΠ½ΠΎΠ΅ число; Π½Π°ΠΏΡ€ΠΈΠΌΠ΅Ρ€, .

    ΠžΠ±ΠΎΠ·Π½Π°Ρ‡ΠΈΠΌ Ρ‡Π΅Ρ€Π΅Π· α₁ любоС Ρ€Π°Ρ†ΠΈΠΎΠ½Π°Π»ΡŒΠ½ΠΎΠ΅ ΠΏΡ€ΠΈΠ±Π»ΠΈΠΆΡ‘Π½Π½ΠΎΠ΅ Π·Π½Π°Ρ‡Π΅Π½ΠΈΠ΅ числа Ξ±, взятоС с нСдостатком, ΠΈ Ρ‡Π΅Ρ€Π΅Π· Ξ±β‚‚ β€” любоС ΠΏΡ€ΠΈΠ±Π»ΠΈΠΆΡ‘Π½Π½ΠΎΠ΅ Ρ€Π°Ρ†ΠΈΠΎΠ½Π°Π»ΡŒΠ½ΠΎΠ΅ Π·Π½Π°Ρ‡Π΅Π½ΠΈΠ΅ числа Π°, взятоС с ΠΈΠ·Π±Ρ‹Ρ‚ΠΊΠΎΠΌ. Π’ΠΎΠ³Π΄Π° ΡΡ‚Π΅ΠΏΠ΅Π½ΡŒ , ΠΎΠ·Π½Π°Ρ‡Π°Π΅Ρ‚ таксС число, ΠΊΠΎΡ‚ΠΎΡ€ΠΎΠ΅ большС всякой стСпСни , Π½ΠΎ мСньшС всякой стСпСни . МоТно Π΄ΠΎΠΊΠ°Π·Π°Ρ‚ΡŒ, Ρ‡Ρ‚ΠΎ Ρ‚Π°ΠΊΠΎΠ΅ число сущСствуСт, ΠΈ СдинствСнно. НапримСр, ΠΎΠ·Π½Π°Ρ‡Π°Π΅Ρ‚ Ρ‚Π°ΠΊΠΎΠ΅ число, ΠΊΠΎΡ‚ΠΎΡ€ΠΎΠ΅ большС ΠΊΠ°ΠΆΠ΄ΠΎΠ³ΠΎ ΠΈΠ· чисСл ряда:

    Π² ΠΊΠΎΡ‚ΠΎΡ€ΠΎΠΌ ΠΏΠΎΠΊΠ°Π·Π°Ρ‚Π΅Π»ΠΈ β€” дСсятичныС ΠΏΡ€ΠΈΠ±Π»ΠΈΠΆΡ‘Π½Π½Ρ‹Π΅ значСния , взятыС с нСдостатком, Π½ΠΎ мСньшС ΠΊΠ°ΠΆΠ΄ΠΎΠ³ΠΎ ΠΈΠ· чисСл ряда:

    Π² ΠΊΠΎΡ‚ΠΎΡ€ΠΎΠΌ ΠΏΠΎΠΊΠ°Π·Π°Ρ‚Π΅Π»ΠΈ β€” дСсятичныС приблиТСния V 2, взятыС с ΠΈΠ·Π±Ρ‹Ρ‚ΠΊΠΎΠΌ.

    Π±) a < 1 ΠΈ Ξ± β€” ΠΏΠΎ-ΠΏΡ€Π΅ΠΆΠ½Π΅ΠΌΡƒ ΠΏΠΎΠ»ΠΎΠΆΠΈΡ‚Π΅Π»ΡŒΠ½ΠΎΠ΅ ΠΈΡ€Ρ€Π°Ρ†ΠΈΠΎΠ½Π°Π»ΡŒΠ½ΠΎΠ΅ число, Π½Π°ΠΏΡ€ΠΈΠΌΠ΅Ρ€, .

    Π’ΠΎΠ³Π΄Π° ΠΏΠΎΠ΄ ΡΡ‚Π΅ΠΏΠ΅Π½ΡŒΡŽ Ρ€Π°Π·ΡƒΠΌΠ΅ΡŽΡ‚ Ρ‚Π°ΠΊΠΎΠ΅ число, ΠΊΠΎΡ‚ΠΎΡ€ΠΎΠ΅ мСньшС всякой стСпСни , Π½ΠΎ большС всякой стСпСни . Π’Π°ΠΊ, Π΅ΡΡ‚ΡŒ число, мСньшСС ΠΊΠ°ΠΆΠ΄ΠΎΠ³ΠΎ ΠΈΠ· чисСл ряда:

    Π½ΠΎ большСС ΠΊΠ°ΠΆΠ΄ΠΎΠ³ΠΎ ΠΈΠ· чисСл ряда:

    Π’Π°ΠΊΠΈΠΌ ΠΎΠ±Ρ€Π°Π·ΠΎΠΌ, Ссли ΠΈΡ€Ρ€Π°Ρ†ΠΈΠΎΠ½Π°Π»ΡŒΠ½ΠΎΠ΅ число Π° Π·Π°ΠΊΠ»ΡŽΡ‡Π΅Π½ΠΎ ΠΌΠ΅ΠΆΠ΄Ρƒ двумя Ρ€Π°Ρ†ΠΈΠΎΠ½Π°Π»ΡŒΠ½Ρ‹ΠΌΠΈ числами α₁ ΠΈ Ξ±β‚‚, Ρ‚ΠΎ ΡΡ‚Π΅ΠΏΠ΅Π½ΡŒ Π·Π°ΠΊΠ»ΡŽΡ‡Π΅Π½Π° ΠΌΠ΅ΠΆΠ΄Ρƒ стСпСнями ΠΈ ΠΈ Ρ‚ΠΎΠ³Π΄Π°, ΠΊΠΎΠ³Π΄Π° Ξ± > 1, ΠΈ Ρ‚ΠΎΠ³Π΄Π°, ΠΊΠΎΠ³Π΄Π° Ξ± < l.

    Π²) ΠΈ Ξ± β€” ΠΎΡ‚Ρ€ΠΈΡ†Π°Ρ‚Π΅Π»ΡŒΠ½ΠΎΠ΅ ΠΈΡ€Ρ€Π°Ρ†ΠΈΠΎΠ½Π°Π»ΡŒΠ½ΠΎΠ΅ число; Π½Π°ΠΏΡ€ΠΈΠΌΠ΅Ρ€:

    Π’ΠΎΠ³Π΄Π° Π²Ρ‹Ρ€Π°ΠΆΠ΅Π½ΠΈΡŽ ΠΏΡ€ΠΈΠ΄Π°ΡŽΡ‚ Ρ‚ΠΎΡ‚ ΠΆΠ΅ смысл, ΠΊΠ°ΠΊΠΎΠΉ ΠΈΠΌΠ΅ΡŽΡ‚ стСпСни с ΠΎΡ‚Ρ€ΠΈΡ†Π°Ρ‚Π΅Π»ΡŒΠ½Ρ‹ΠΌΠΈ Ρ€Π°Ρ†ΠΈΠΎΠ½Π°Π»ΡŒΠ½Ρ‹ΠΌΠΈ показатСлями. Π’Π°ΠΊ:

    ΠŸΡ€ΠΈ ΠΏΠΎΠ΄Ρ€ΠΎΠ±Π½ΠΎΠΌ рассмотрСнии Ρ‚Π΅ΠΎΡ€ΠΈΠΈ ΠΈΡ€Ρ€Π°Ρ†ΠΈΠΎΠ½Π°Π»ΡŒΠ½Ρ‹Ρ… ΠΏΠΎΠΊΠ°Π·Π°Ρ‚Π΅Π»Π΅ΠΉ обнаруТиваСтся, Ρ‡Ρ‚ΠΎ всС свойства ΠΏΠΎΠΊΠ°Π·Π°Ρ‚Π΅Π»Π΅ΠΉ Ρ€Π°Ρ†ΠΈΠΎΠ½Π°Π»ΡŒΠ½Ρ‹Ρ… ΠΏΡ€ΠΈΠΌΠ΅Π½ΠΈΠΌΡ‹ ΠΈ ΠΊ показатСлям ΠΈΡ€Ρ€Π°Ρ†ΠΈΠΎΠ½Π°Π»ΡŒΠ½Ρ‹ΠΌ; Ρ‚Π°ΠΊ:

    ΠŸΠΎΠΊΠ°Π·Π°Ρ‚Π΅Π»ΡŒΠ½Π°Ρ функция

    ΠžΠΏΡ€Π΅Π΄Π΅Π»Π΅Π½ΠΈΠ΅:

    ΠŸΠΎΠΊΠ°Π·Π°Ρ‚Π΅Π»ΡŒΠ½ΠΎΠΉ Ρ„ΡƒΠ½ΠΊΡ†ΠΈΠ΅ΠΉ называСтся функция, ΠΏΡ€Π΅Π΄ΡΡ‚Π°Π²Π»ΡΡŽΡ‰Π°Ρ собой ΡΡ‚Π΅ΠΏΠ΅Π½ΡŒ, Ρƒ ΠΊΠΎΡ‚ΠΎΡ€ΠΎΠΉ основаниС Π° Π΅ΡΡ‚ΡŒ ΠΊΠ°ΠΊΠΎΠ΅-Π½ΠΈΠ±ΡƒΠ΄ΡŒ постоянноС ΠΏΠΎΠ»ΠΎΠΆΠΈΡ‚Π΅Π»ΡŒΠ½ΠΎΠ΅ число, Π½Π΅ Ρ€Π°Π²Π½ΠΎΠ΅ 1, Π° ΠΏΠΎΠΊΠ°Π·Π°Ρ‚Π΅Π»ΡŒ Ρ… β€” нСзависимоС ΠΏΠ΅Ρ€Π΅ΠΌΠ΅Π½Π½ΠΎΠ΅, ΠΌΠΎΠ³ΡƒΡ‰Π΅Π΅ ΠΏΡ€ΠΈΠ½ΠΈΠΌΠ°Ρ‚ΡŒ всСвозмоТныС значСния, ΠΏΠΎΠ»ΠΎΠΆΠΈΡ‚Π΅Π»ΡŒΠ½Ρ‹Π΅ ΠΈ ΠΎΡ‚Ρ€ΠΈΡ†Π°Ρ‚Π΅Π»ΡŒΠ½Ρ‹Π΅, Ρ†Π΅Π»Ρ‹Π΅ ΠΈ Π΄Ρ€ΠΎΠ±Π½Ρ‹Π΅, Ρ€Π°Ρ†ΠΈΠΎΠ½Π°Π»ΡŒΠ½Ρ‹Π΅ ΠΈ ΠΈΡ€Ρ€Π°Ρ†ΠΈΠΎΠ½Π°Π»ΡŒΠ½Ρ‹Π΅. ΠŸΡ€ΠΈ этом прСдполагаСтся, Ρ‡Ρ‚ΠΎ Π² Ρ‚ΠΎΠΌ случаС, ΠΊΠΎΠ³Π΄Π° ΠΏΠΎΠΊΠ°Π·Π°Ρ‚Π΅Π»ΡŒ Ρ… Ρ€Π°Π²Π΅Π½ Π΄Ρ€ΠΎΠ±ΠΈ ΠΈ, ΡΠ»Π΅Π΄ΠΎΠ²Π°Ρ‚Π΅Π»ΡŒΠ½ΠΎ, ΠΊΠΎΠ³Π΄Π° ΠΎΠ·Π½Π°Ρ‡Π°Π΅Ρ‚ Ρ€Π°Π΄ΠΈΠΊΠ°Π» Π½Π΅ΠΊΠΎΡ‚ΠΎΡ€ΠΎΠΉ стСпСни, Ρ‚ΠΎ ΠΈΠ· всСх Π·Π½Π°Ρ‡Π΅Π½ΠΈΠΉ Ρ€Π°Π΄ΠΈΠΊΠ°Π»Π° бСрётся Ρ‚ΠΎΠ»ΡŒΠΊΠΎ ΠΎΠ΄Π½ΠΎ арифмСтичСскоС, Ρ‚. Π΅. ΠΏΠΎΠ»ΠΎΠΆΠΈΡ‚Π΅Π»ΡŒΠ½ΠΎΠ΅.

    Из Ρ‚ΠΎΠ³ΠΎ, Ρ‡Ρ‚ΠΎ ΠΌΡ‹ Π·Π½Π°Π΅ΠΌ ΠΎ показатСлях стСпСни, слСдуСт, Ρ‡Ρ‚ΠΎ функция ΠΏΡ€ΠΈ всяком Π·Π½Π°Ρ‡Π΅Π½ΠΈΠΈ Ρ… ΠΈΠΌΠ΅Π΅Ρ‚ СдинствСнноС Π·Π½Π°Ρ‡Π΅Π½ΠΈΠ΅ (благодаря ΡƒΡΠ»ΠΎΠ²ΠΈΡŽ Π±Ρ€Π°Ρ‚ΡŒ для Ρ€Π°Π΄ΠΈΠΊΠ°Π»ΠΎΠ² Ρ‚ΠΎΠ»ΡŒΠΊΠΎ арифмСтичСскоС Π·Π½Π°Ρ‡Π΅Π½ΠΈΠ΅).

    Бвойства ΠΏΠΎΠΊΠ°Π·Π°Ρ‚Π΅Π»ΡŒΠ½ΠΎΠΉ Ρ„ΡƒΠ½ΠΊΡ†ΠΈΠΈ

    Рассмотрим Π½Π΅ΠΊΠΎΡ‚ΠΎΡ€Ρ‹Π΅ свойства ΠΏΠΎΠΊΠ°Π·Π°Ρ‚Π΅Π»ΡŒΠ½ΠΎΠΉ Ρ„ΡƒΠ½ΠΊΡ†ΠΈΠΈ, помня, Ρ‡Ρ‚ΠΎ Π° ΠΌΡ‹ считаСм ΠΏΠΎΠ»ΠΎΠΆΠΈΡ‚Π΅Π»ΡŒΠ½Ρ‹ΠΌ числом.

    1. ΠŸΡ€ΠΈ всяком ΠΏΠΎΠ»ΠΎΠΆΠΈΡ‚Π΅Π»ΡŒΠ½ΠΎΠΌ основании функция ΠΏΠΎΠ»ΠΎΠΆΠΈΡ‚Π΅Π»ΡŒΠ½Π°, Ρ‚. Π΅. > 0.
    ΠŸΡ€ΠΈ Ρ… Ρ†Π΅Π»ΠΎΠΌ ΠΏΠΎΠ»ΠΎΠΆΠΈΡ‚Π΅Π»ΡŒΠ½ΠΎΠΌ >0, ΠΊΠ°ΠΊΠΎΠ²ΠΎ Π±Ρ‹ Π½ΠΈ Π±Ρ‹Π»ΠΎ ΠΏΠΎΠ»ΠΎΠΆΠΈΡ‚Π΅Π»ΡŒΠ½ΠΎΠ΅ число Π°; ΡΠ»Π΅Π΄ΠΎΠ²Π°Ρ‚Π΅Π»ΡŒΠ½ΠΎ, высказанноС Π½Π°ΠΌΠΈ ΠΏΠΎΠ»ΠΎΠΆΠ΅Π½ΠΈΠ΅ Π² этом случаС справСдливо.

    ΠŸΡƒΡΡ‚ΡŒ Ρ‚Π΅ΠΏΠ΅Ρ€ΡŒ Ρ… Ρ€Π°Π²Π½ΠΎ Π½Π΅ΠΊΠΎΡ‚ΠΎΡ€ΠΎΠΉ ΠΏΠΎΠ»ΠΎΠΆΠΈΡ‚Π΅Π»ΡŒΠ½ΠΎΠΉ Π΄Ρ€ΠΎΠ±ΠΈ, Π½Π°ΠΏΡ€ΠΈΠΌΠ΅Ρ€
    Π’ΠΎΠ³Π΄Π°:

    Но > 0, ΡΠ»Π΅Π΄ΠΎΠ²Π°Ρ‚Π΅Π»ΡŒΠ½ΠΎ, ΠΈ > 0, Ρ‚Π°ΠΊ ΠΊΠ°ΠΊ ΠΌΡ‹ ΡƒΡΠ»ΠΎΠ²ΠΈΠ»ΠΈΡΡŒ Π±Ρ€Π°Ρ‚ΡŒ лишь арифмСтичСскоС Π·Π½Π°Ρ‡Π΅Π½ΠΈΠ΅ корня.

    ΠŸΡƒΡΡ‚ΡŒ Ρ… β€” ΠΏΠΎΠ»ΠΎΠΆΠΈΡ‚Π΅Π»ΡŒΠ½ΠΎΠ΅ ΠΈΡ€Ρ€Π°Ρ†ΠΈΠΎΠ½Π°Π»ΡŒΠ½ΠΎΠ΅ число. ΠžΠ±ΠΎΠ·Π½Π°Ρ‡ΠΈΠΌ Ρ‡Π΅Ρ€Π΅Π· α₁ ΠΈ aβ‚‚ ΠΏΡ€ΠΈΠ±Π»ΠΈΠΆΡ‘Π½Π½Ρ‹Π΅ Ρ€Π°Ρ†ΠΈΠΎΠ½Π°Π»ΡŒΠ½Ρ‹Π΅ значСния Ρ… ΠΏΠΎ нСдостатку ΠΈ ΠΈΠ·Π±Ρ‹Ρ‚ΠΊΡƒ. Π­Ρ‚ΠΈ ΠΏΡ€ΠΈΠ±Π»ΠΈΠΆΡ‘Π½Π½Ρ‹Π΅ значСния ΠΌΠΎΠΆΠ½ΠΎ Π²Ρ‹Π±Ρ€Π°Ρ‚ΡŒ ΠΏΠΎΠ»ΠΎΠΆΠΈΡ‚Π΅Π»ΡŒΠ½Ρ‹ΠΌΠΈ. Π’ΠΎΠ³Π΄Π° Π·Π½Π°Ρ‡Π΅Π½ΠΈΠ΅ , Π±ΡƒΠ΄ΡƒΡ‡ΠΈ Π·Π°ΠΊΠ»ΡŽΡ‡Ρ‘Π½Π½Ρ‹ΠΌ ΠΌΠ΅ΠΆΠ΄Ρƒ двумя ΠΏΠΎΠ»ΠΎΠΆΠΈΡ‚Π΅Π»ΡŒΠ½Ρ‹ΠΌΠΈ числами ΠΈ , являСтся ΠΏΠΎΠ»ΠΎΠΆΠΈΡ‚Π΅Π»ΡŒΠ½Ρ‹ΠΌ числом.

    ΠŸΡƒΡΡ‚ΡŒ, Π½Π°ΠΊΠΎΠ½Π΅Ρ†, Ρ… Ρ€Π°Π²Π½ΠΎ Π½Π΅ΠΊΠΎΡ‚ΠΎΡ€ΠΎΠΌΡƒ ΠΎΡ‚Ρ€ΠΈΡ†Π°Ρ‚Π΅Π»ΡŒΠ½ΠΎΠΌΡƒ числу, Π½Π°ΠΏΡ€ΠΈΠΌΠ΅Ρ€ x=β€”Ρ€. Π’ΠΎΠ³Π΄Π°:

    Каково Π±Ρ‹ Π½ΠΈ Π±Ρ‹Π»ΠΎ ΠΏΠΎΠ»ΠΎΠΆΠΈΡ‚Π΅Π»ΡŒΠ½ΠΎΠ΅ число Ρ€, согласно ΠΏΡ€Π΅Π΄Ρ‹Π΄ΡƒΡ‰Π΅ΠΌΡƒ > 0, Π½ΠΎ Ρ‚ΠΎΠ³Π΄Π° ΠΈ .

    Π’Π°ΠΊΠΈΠΌ ΠΎΠ±Ρ€Π°Π·ΠΎΠΌ, высказанноС Π½Π°ΠΌΠΈ ΠΏΠΎΠ»ΠΎΠΆΠ΅Π½ΠΈΠ΅ справСдливо для всякого Ρ….

    2. ΠŸΡ€ΠΈ a > 1 функция >l, Ссли Ρ…>0, ΠΈ < l, Ссли x < 0 (ΠΏΡ€ΠΈ a < l Π·Π½Π°ΠΊΠΈ нСравСнства для ΠΏΡ€ΠΎΡ‚ΠΈΠ²ΠΎΠΏΠΎΠ»ΠΎΠΆΠ½Ρ‹).

    ΠŸΡƒΡΡ‚ΡŒ Ρ… β€” Ρ†Π΅Π»ΠΎΠ΅ ΠΏΠΎΠ»ΠΎΠΆΠΈΡ‚Π΅Π»ΡŒΠ½Π΅Π΅ число. Π’ΠΎΠ³Π΄Π°:

    ΠŸΡƒΡΡ‚ΡŒ Ρ… β€” ΠΏΠΎΠ»ΠΎΠΆΠΈΡ‚Π΅Π»ΡŒΠ½Π°Ρ Π΄Ρ€ΠΎΠ±ΡŒ, Π½Π°ΠΏΡ€ΠΈΠΌΠ΅Ρ€ . Π’ΠΎΠ³Π΄Π°:

    Если Ρ… β€” ΠΏΠΎΠ»ΠΎΠΆΠΈΡ‚Π΅Π»ΡŒΠ½ΠΎΠ΅ ΠΈΡ€Ρ€Π°Ρ†ΠΈΠΎΠ½Π°Π»ΡŒΠ½ΠΎΠ΅ число, Ρ‚ΠΎ > 1, Π³Π΄Π΅ a₁ β€” ΠΏΡ€ΠΈΠ±Π»ΠΈΠΆΡ‘Π½Π½ΠΎΠ΅ Ρ€Π°Ρ†ΠΈΠΎΠ½Π°Π»ΡŒΠ½ΠΎΠ΅ Π·Π½Π°Ρ‡Π΅Π½ΠΈΠ΅ Ρ… ΠΏΠΎ нСдостатку, Π° поэтому ΠΈ . Π’Π°ΠΊΠΈΠΌ ΠΎΠ±Ρ€Π°Π·ΠΎΠΌ, ΠΏΡ€ΠΈ всяком ΠΏΠΎΠ»ΠΎΠΆΠΈΡ‚Π΅Π»ΡŒΠ½ΠΎΠΌ Ρ…

    ΠŸΡƒΡΡ‚ΡŒ Ρ‚Π΅ΠΏΠ΅Ρ€ΡŒ Ρ… Π΅ΡΡ‚ΡŒ ΠΊΠ°ΠΊΠΎΠ΅-Π»ΠΈΠ±ΠΎ ΠΎΡ‚Ρ€ΠΈΡ†Π°Ρ‚Π΅Π»ΡŒΠ½ΠΎΠ΅ число, Π½Π°ΠΏΡ€ΠΈΠΌΠ΅Ρ€ x = β€”Ρ€. Π’ΠΎΠ³Π΄Π°:

    Но согласно ΠΏΡ€Π΅Π΄Ρ‹Π΄ΡƒΡ‰Π΅ΠΌΡƒ > 1. Π‘Π»Π΅Π΄ΠΎΠ²Π°Ρ‚Π΅Π»ΡŒΠ½ΠΎ:

    ΠŸΡ€ΠΈ a>1 функция возрастаСт ΠΏΡ€ΠΈ возрастании Ρ….

    Если x₁ ΠΈ xβ‚‚ β€” Π΄Π²Π° Ρ†Π΅Π»Ρ‹Ρ… ΠΏΠΎΠ»ΠΎΠΆΠΈΡ‚Π΅Π»ΡŒΠ½Ρ‹Ρ… числа ΠΈ xβ‚‚ > x₁, Ρ‚ΠΎ ΠΎΡ‡Π΅Π²ΠΈΠ΄Π½ΠΎ, Ρ‡Ρ‚ΠΎ ΠΏΡ€ΠΈ 1 Π±ΡƒΠ΄Π΅ΠΌ ΠΈΠΌΠ΅Ρ‚ΡŒ:

    ΠŸΡƒΡΡ‚ΡŒ Ρ‚Π΅ΠΏΠ΅Ρ€ΡŒ x₁ ΠΈ xβ‚‚ β€” ΠΏΠΎΠ»ΠΎΠΆΠΈΡ‚Π΅Π»ΡŒΠ½Ρ‹Π΅ Π΄Ρ€ΠΎΠ±ΠΈ, Π½Π°ΠΏΡ€ΠΈΠΌΠ΅Ρ€ x₁= ΠΈ xβ‚‚ =. ΠŸΡƒΡΡ‚ΡŒ Ρ‚Π°ΠΊΠΆΠ΅ . Π’ΠΎΠ³Π΄Π°:

    Или ΠΏΠΎ ΠΏΡ€ΠΈΠ²Π΅Π΄Π΅Π½ΠΈΠΈ Π΄Ρ€ΠΎΠ±Π΅ΠΉ ΠΊ ΠΎΠ΄Π½ΠΎΠΌΡƒ Π·Π½Π°ΠΌΠ΅Π½Π°Ρ‚Π΅Π»ΡŽ:

    Из Π΄Π²ΡƒΡ… Π½Π΅Ρ€Π°Π²Π½Ρ‹Ρ… Π΄Ρ€ΠΎΠ±Π΅ΠΉ с ΠΎΠ΄ΠΈΠ½Π°ΠΊΠΎΠ²Ρ‹ΠΌΠΈ знамСнатСлями Ρ‚Π° большС, Ρƒ ΠΊΠΎΡ‚ΠΎΡ€ΠΎΠΉ Ρ‡ΠΈΡΠ»ΠΈΡ‚Π΅Π»ΡŒ большС. Π‘Π»Π΅Π΄ΠΎΠ²Π°Ρ‚Π΅Π»ΡŒΠ½ΠΎ:

    Π’Π°ΠΊ ΠΊΠ°ΠΊ Ρ€n ΠΈ mq β€” Ρ†Π΅Π»Ρ‹Π΅ числа, Ρ‚ΠΎ ΠΊ Π½ΠΈΠΌ ΠΌΠΎΠΆΠ½ΠΎ ΠΏΡ€ΠΈΠΌΠ΅Π½ΠΈΡ‚ΡŒ ΠΏΡ€Π΅Π΄Ρ‹Π΄ΡƒΡ‰ΠΈΠ΅ рассуТдСния ΠΈ ΠΌΡ‹ ΠΏΠΎΠ»ΡƒΡ‡ΠΈΠΌ:

    Π˜Π·Π²Π»Π΅Ρ‡Ρ‘ΠΌ ΠΈΠ· ΠΈ ΠΊΠΎΡ€Π΅Π½ΡŒ стСпСни qn. ΠœΡ‹ Π·Π½Π°Π΅ΠΌ, Ρ‡Ρ‚ΠΎ ΠΈΠ· Π΄Π²ΡƒΡ… ΠΊΠΎΡ€Π½Π΅ΠΉ ΠΎΠ΄ΠΈΠ½Π°ΠΊΠΎΠ²ΠΎΠΉ стСпСни Ρ‚ΠΎΡ‚ большС, Ρƒ ΠΊΠΎΡ‚ΠΎΡ€ΠΎΠ³ΠΎ большС ΠΏΠΎΠ΄ΠΊΠΎΡ€Π΅Π½Π½ΠΎΠ΅ число. Π‘Π»Π΅Π΄ΠΎΠ²Π°Ρ‚Π΅Π»ΡŒΠ½ΠΎ:

    Бокращая ΠΏΠΎΠΊΠ°Π·Π°Ρ‚Π΅Π»ΠΈ, ΠΏΠΎΠ»ΡƒΡ‡ΠΈΠΌ:

    ΠŸΡƒΡΡ‚ΡŒ x₁ ΠΈ xβ‚‚ β€” Π΄Π²Π° вСщСствСнных числа, ΠΈΠ· ΠΊΠΎΡ‚ΠΎΡ€Ρ‹Ρ… ΠΎΠ΄Π½ΠΎ ΠΈΠ»ΠΈ ΠΎΠ±Π° ΠΈΡ€Ρ€Π°Ρ†ΠΈΠΎΠ½Π°Π»ΡŒΠ½Ρ‹.

    ΠžΠ±ΠΎΠ·Π½Π°Ρ‡ΠΈΠΌ Ρ‡Π΅Ρ€Π΅Π· Ξ² ΠΏΡ€ΠΈΠ±Π»ΠΈΠΆΡ‘Π½Π½ΠΎΠ΅ Ρ€Π°Ρ†ΠΈΠΎΠ½Π°Π»ΡŒΠ½ΠΎΠ΅ Π·Π½Π°Ρ‡Π΅Π½ΠΈΠ΅ x₁ ΠΏΠΎ ΠΈΠ·Π±Ρ‹Ρ‚ΠΊΡƒ, Π° Ρ‡Π΅Ρ€Π΅Π· Π° ΠΏΡ€ΠΈΠ±Π»ΠΈΠΆΡ‘Π½Π½ΠΎΠ΅ Π·Π½Π°Ρ‡Π΅Π½ΠΈΠ΅ xβ‚‚ ΠΏΠΎ нСдостатку. Если x₁ < xβ‚‚, Ρ‚ΠΎ ΠΌΠΎΠΆΠ½ΠΎ Π²Ρ‹Π±Ρ€Π°Ρ‚ΡŒ Π° ΠΈ Ξ² ΠΏΡ€ΠΈ условии Ξ² < Ξ±. Π’ΠΎΠ³Π΄Π° Π±ΡƒΠ΄Π΅ΠΌ ΠΈΠΌΠ΅Ρ‚ΡŒ Но Ρ‚Π°ΠΊ ΠΊΠ°ΠΊ , Ρ‚ΠΎ .

    Π“Ρ€Π°Ρ„ΠΈΠΊ ΠΏΠΎΠΊΠ°Π·Π°Ρ‚Π΅Π»ΡŒΠ½ΠΎΠΉ Ρ„ΡƒΠ½ΠΊΡ†ΠΈΠΈ. ΠŸΠΎΡΡ‚Ρ€ΠΎΠΈΠΌ Π³Ρ€Π°Ρ„ΠΈΠΊ ΡΠ»Π΅Π΄ΡƒΡŽΡ‰ΠΈΡ… Ρ‚Ρ€Ρ‘Ρ… ΠΏΠΎΠΊΠ°Π·Π°Ρ‚Π΅Π»ΡŒΠ½Ρ‹Ρ… Ρ„ΡƒΠ½ΠΊΡ†ΠΈΠΉ:

    Для построСния Π³Ρ€Π°Ρ„ΠΈΠΊΠΎΠ² ΠΏΠ΅Ρ€Π²Ρ‹Ρ… Π΄Π²ΡƒΡ… Ρ„ΡƒΠ½ΠΊΡ†ΠΈΠΉ ΠΌΡ‹ Π΄Π°Π΄ΠΈΠΌ ΠΏΠ΅Ρ€Π΅ΠΌΠ΅Π½Π½ΠΎΠΌΡƒ числу Ρ… ряд Ρ†Π΅Π»Ρ‹Ρ… Π·Π½Π°Ρ‡Π΅Π½ΠΈΠΉ:
    -3; -2; -1; 0; 1; 2; 3.

    ΠŸΡ€ΠΈ x=β€”3 ΠΌΡ‹ ΠΏΠΎΠ»ΡƒΡ‡ΠΈΠΌ:

    Подобно этому вычислим значСния Ρƒ ΠΈ для всСх ΠΎΡΡ‚Π°Π»ΡŒΠ½Ρ‹Ρ… Π·Π½Π°Ρ‡Π΅Π½ΠΈΠΉ x.

    Для Ρ„ΡƒΠ½ΠΊΡ†ΠΈΠΈ Π½Π΅ΡƒΠ΄ΠΎΠ±Π½ΠΎ Π±Ρ€Π°Ρ‚ΡŒ ΡƒΠΊΠ°Π·Π°Π½Π½Ρ‹Π΅ значСния числа Ρ…, Ρ‚Π°ΠΊ ΠΊΠ°ΠΊ ΠΌΡ‹ ΠΏΠΎΠ»ΡƒΡ‡ΠΈΠ»ΠΈ Π±Ρ‹ Ρ‚ΠΎΠ³Π΄Π° для Ρƒ Ρ‚Π°ΠΊΠΈΠ΅ большиС числа, ΠΊΠΎΡ‚ΠΎΡ€Ρ‹Π΅ Π½Π° Ρ‡Π΅Ρ€Ρ‚Π΅ΠΆΠ΅ 29 Π½Π΅ ΡƒΠΌΠ΅Ρ‰Π°ΡŽΡ‚ΡΡ (Π½Π°ΠΏΡ€ΠΈΠΌΠ΅Ρ€, ΠΏΡ€ΠΈ Ρ… = 3 ΠΌΡ‹ ΠΏΠΎΠ»ΡƒΡ‡ΠΈΠ»ΠΈ Π±Ρ‹ y= 10Β³ = 1000). Для этой Ρ„ΡƒΠ½ΠΊΡ†ΠΈΠΈ ΠΌΡ‹ Π²ΠΎΠ·ΡŒΠΌΡ‘ΠΌ Ρ‚Π°ΠΊΠΈΠ΅ Π΄Ρ€ΠΎΠ±Π½Ρ‹Π΅ значСния (Π·Π°ΠΊΠ»ΡŽΡ‡Π°ΡŽΡ‰ΠΈΠ΅ΡΡ ΠΌΠ΅ΠΆΠ΄Ρƒ β€”1 ΠΈ +1):

    Π‘ΠΎΠΎΡ‚Π²Π΅Ρ‚ΡΡ‚Π²ΡƒΡŽΡ‰ΠΈΠ΅ значСния Ρƒ вычислим Π² Ρ‚Π°ΠΊΠΎΠΉ ΠΏΠΎΡΠ»Π΅Π΄ΠΎΠ²Π°Ρ‚Π΅Π»ΡŒΠ½ΠΎΡΡ‚ΠΈ:

    Π”Π°Π»Π΅Π΅ простым ΡƒΠΌΠ½ΠΎΠΆΠ΅Π½ΠΈΠ΅ΠΌ ΠΈ Π΄Π΅Π»Π΅Π½ΠΈΠ΅ΠΌ Π½Π°Ρ…ΠΎΠ΄ΠΈΠΌ:



    Π’Ρ‹ΠΏΠΈΡˆΠ΅ΠΌ всС Π½Π°ΠΉΠ΄Π΅Π½Π½Ρ‹Π΅ значСния Π² ΡΠ»Π΅Π΄ΡƒΡŽΡ‰ΠΈΠ΅ Ρ‚Ρ€ΠΈ Ρ‚Π°Π±Π»ΠΈΡ†Ρ‹:

    x =возрастаСт-3-2-10123 возрастаСт
    y = возрастаСт1248 возрастаСт

    x =возрастаСт-3-2-10123 возрастаСт
    y = возрастаСт8421 возрастаСт

    x = возрастаСт-101 возрастаСт
    y = возрастаСт0,10,170,320,5611,783,165,6210 возрастаСт

    (Π² послСднСй Ρ‚Π°Π±Π»ΠΈΡ†Π΅ числа ΠΎΠΊΡ€ΡƒΠ³Π»Π΅Π½Ρ‹).

    НанСся эти значСния Π½Π° Ρ‡Π΅Ρ€Ρ‚Ρ‘ΠΆ ΠΈ соСдиняя ΠΏΠΎΠ»ΡƒΡ‡Π΅Π½Π½Ρ‹Π΅ Ρ‚ΠΎΡ‡ΠΊΠΈ ΠΊΡ€ΠΈΠ²Ρ‹ΠΌΠΈ, ΠΌΡ‹ ΠΏΠΎΠ»ΡƒΡ‡ΠΈΠΌ (Ρ‡Π΅Ρ€Ρ‚. 29) Ρ‚Ρ€ΠΈ Π³Ρ€Π°Ρ„ΠΈΠΊΠ° взятых Ρ„ΡƒΠ½ΠΊΡ†ΠΈΠΉ (ΡƒΠ΄ΠΎΠ±Π½ΠΎ Ρ‡Π΅Ρ€Ρ‚Ρ‘ΠΆ Π²Ρ‹ΠΏΠΎΠ»Π½ΠΈΡ‚ΡŒ Π½Π° ΠΌΠΈΠ»Π»ΠΈΠΌΠ΅Ρ‚Ρ€ΠΎΠ²ΠΎΠΉ Π±ΡƒΠΌΠ°Π³Π΅, бСря Π·Π° Π΅Π΄ΠΈΠ½ΠΈΡ†Ρƒ Π΄Π»ΠΈΠ½Ρ‹ сантимСтр).

    Рассматривая Π³Ρ€Π°Ρ„ΠΈΠΊΠΈ ΠΏΠΎΠΊΠ°Π·Π°Ρ‚Π΅Π»ΡŒΠ½Ρ‹Ρ… Ρ„ΡƒΠ½ΠΊΡ†ΠΈΠΉ, ΠΌΡ‹ Π²ΠΈΠ΄ΠΈΠΌ Π½Π° Π½ΠΈΡ… Π² наглядном ΠΈΠ·ΠΎΠ±Ρ€Π°ΠΆΠ΅Π½ΠΈΠΈ ΡΠ»Π΅Π΄ΡƒΡŽΡ‰ΠΈΠ΅ свойства:

    1. ΠŸΡ€ΠΈ всяком ΠΏΠΎΠ»ΠΎΠΆΠΈΡ‚Π΅Π»ΡŒΠ½ΠΎΠΌ основании функция ΠΏΠΎΠ»ΠΎΠΆΠΈΡ‚Π΅Π»ΡŒΠ½Π° (всС ΠΊΡ€ΠΈΠ²Ρ‹Π΅ располоТСны Π²Ρ‹ΡˆΠ΅ оси Ρ…-ΠΎΠ²).
    2. ΠŸΡ€ΠΈ Ξ± > 1 функция > 1, Ссли Ρ… > 0, ΠΈ < 1, Ссли Ρ… < 0 (ΠΏΡ€ΠΈ Π° < 1 Π·Π½Π°ΠΊΠΈ нСравСнств для ΠΏΡ€ΠΎΡ‚ΠΈΠ²ΠΎΠΏΠΎΠ»ΠΎΠΆΠ½Ρ‹).
    3. ΠŸΡ€ΠΈ возрастании Ρ… функция Π°Ρ… возрастаСт, Ссли Π° > 1 (ΠΈ ΡƒΠ±Ρ‹Π²Π°Π΅Ρ‚, Ссли a < l).
    4. Если Ρ…=0, Ρ‚ΠΎ =1 ΠΏΡ€ΠΈ всяком Π° (всС ΠΊΡ€ΠΈΠ²Ρ‹Π΅ проходят Ρ‡Π΅Ρ€Π΅Π· ΠΎΠ΄Π½Ρƒ ΠΈ Ρ‚Ρƒ ΠΆΠ΅ Ρ‚ΠΎΡ‡ΠΊΡƒ, Π»Π΅ΠΆΠ°Ρ‰ΡƒΡŽ Π½Π° оси Ρƒ-ΠΎΠ² ΠΈ ΠΎΡ‚ΡΡ‚ΠΎΡΡ‰ΡƒΡŽ ΠΎΡ‚ Ρ‚ΠΎΡ‡ΠΊΠΈ 0 Π½Π° +1).
    5. ΠŸΡ€ΠΈ a > 1 функция ΠΏΡ€ΠΈ возрастании Ρ… возрастаСт Ρ‚Π΅ΠΌ быстрСС, Ρ‡Π΅ΠΌ большС Π° (кривая ΠΏΡ€ΠΈ a = 10 поднимаСтся Π²Π²Π΅Ρ€Ρ… Π·Π½Π°Ρ‡ΠΈΡ‚Π΅Π»ΡŒΠ½ΠΎ большС, Ρ‡Π΅ΠΌ ΠΏΡ€ΠΈ a=2).
    Π§Π΅Ρ€Ρ‚. 29.

    РСшСниС Π·Π°Π΄Π°Π½ΠΈΠΉ ΠΈ Π·Π°Π΄Π°Ρ‡ ΠΏΠΎ ΠΏΡ€Π΅Π΄ΠΌΠ΅Ρ‚Π°ΠΌ:

    • ΠœΠ°Ρ‚Π΅ΠΌΠ°Ρ‚ΠΈΠΊΠ°
    • Π’Ρ‹ΡΡˆΠ°Ρ ΠΌΠ°Ρ‚Π΅ΠΌΠ°Ρ‚ΠΈΠΊΠ°
    • ΠœΠ°Ρ‚Π΅ΠΌΠ°Ρ‚ΠΈΡ‡Π΅ΡΠΊΠΈΠΉ Π°Π½Π°Π»ΠΈΠ·
    • ЛинСйная Π°Π»Π³Π΅Π±Ρ€Π°

    Π”ΠΎΠΏΠΎΠ»Π½ΠΈΡ‚Π΅Π»ΡŒΠ½Ρ‹Π΅ Π»Π΅ΠΊΡ†ΠΈΠΈ ΠΏΠΎ Π²Ρ‹ΡΡˆΠ΅ΠΉ ΠΌΠ°Ρ‚Π΅ΠΌΠ°Ρ‚ΠΈΠΊΠ΅:

    1. ВоТдСствСнныС прСобразования алгСбраичСских Π²Ρ‹Ρ€Π°ΠΆΠ΅Π½ΠΈΠΉ
    2. Π€ΡƒΠ½ΠΊΡ†ΠΈΠΈ ΠΈ Π³Ρ€Π°Ρ„ΠΈΠΊΠΈ
    3. ΠŸΡ€Π΅ΠΎΠ±Ρ€Π°Π·ΠΎΠ²Π°Π½ΠΈΡ Π³Ρ€Π°Ρ„ΠΈΠΊΠΎΠ² Ρ„ΡƒΠ½ΠΊΡ†ΠΈΠΉ
    4. ΠšΠ²Π°Π΄Ρ€Π°Ρ‚Π½Π°Ρ функция ΠΈ Π΅Ρ‘ Π³Ρ€Π°Ρ„ΠΈΠΊΠΈ
    5. АлгСбраичСскиС нСравСнства
    6. НСравСнства
    7. НСравСнства с ΠΏΠ΅Ρ€Π΅ΠΌΠ΅Π½Π½Ρ‹ΠΌΠΈ
    8. ΠŸΡ€ΠΎΠ³Ρ€Π΅ΡΡΠΈΠΈ Π² ΠΌΠ°Ρ‚Π΅ΠΌΠ°Ρ‚ΠΈΠΊΠ΅
    9. АрифмСтичСская прогрСссия
    10. ГСомСтричСская прогрСссия
    11. Π›ΠΎΠ³Π°Ρ€ΠΈΡ„ΠΌΡ‹ Π² ΠΌΠ°Ρ‚Π΅ΠΌΠ°Ρ‚ΠΈΠΊΠ΅
    12. ИсслСдованиС ΡƒΡ€Π°Π²Π½Π΅Π½ΠΈΠΉ
    13. УравнСния Π²Ρ‹ΡΡˆΠΈΡ… стСпСнСй
    14. УравнСния Π²Ρ‹ΡΡˆΠΈΡ… стСпСнСй с ΠΎΠ΄Π½ΠΈΠΌ нСизвСстным
    15. ΠšΠΎΠΌΠΏΠ»Π΅ΠΊΡΠ½Ρ‹Π΅ числа
    16. НСпрСрывная Π΄Ρ€ΠΎΠ±ΡŒ (цСпная Π΄Ρ€ΠΎΠ±ΡŒ)
    17. АлгСбраичСскиС уравнСния
    18. НСопрСдСлСнныС уравнСния
    19. БоСдинСния
    20. Π‘ΠΈΠ½ΠΎΠΌ ΠΡŒΡŽΡ‚ΠΎΠ½Π°
    21. Число С
    22. НСпрСрывныС Π΄Ρ€ΠΎΠ±ΠΈ
    23. Ѐункция
    24. ИсслСдованиС Ρ„ΡƒΠ½ΠΊΡ†ΠΈΠΉ
    25. ΠŸΡ€Π΅Π΄Π΅Π»
    26. Π˜Π½Ρ‚Π΅Π³Ρ€Π°Π»
    27. Π”Π²ΠΎΠΉΠ½ΠΎΠΉ ΠΈΠ½Ρ‚Π΅Π³Ρ€Π°Π»
    28. Π’Ρ€ΠΎΠΉΠ½ΠΎΠΉ ΠΈΠ½Ρ‚Π΅Π³Ρ€Π°Π»
    29. Π˜Π½Ρ‚Π΅Π³Ρ€ΠΈΡ€ΠΎΠ²Π°Π½ΠΈΠ΅
    30. НСопрСдСлённый ΠΈΠ½Ρ‚Π΅Π³Ρ€Π°Π»
    31. ΠžΠΏΡ€Π΅Π΄Π΅Π»Π΅Π½Π½Ρ‹ΠΉ ΠΈΠ½Ρ‚Π΅Π³Ρ€Π°Π»
    32. ΠšΡ€ΠΈΠ²ΠΎΠ»ΠΈΠ½Π΅ΠΉΠ½Ρ‹Π΅ ΠΈΠ½Ρ‚Π΅Π³Ρ€Π°Π»Ρ‹
    33. ΠŸΠΎΠ²Π΅Ρ€Ρ…Π½ΠΎΡΡ‚Π½Ρ‹Π΅ ΠΈΠ½Ρ‚Π΅Π³Ρ€Π°Π»Ρ‹
    34. НСсобствСнныС ΠΈΠ½Ρ‚Π΅Π³Ρ€Π°Π»Ρ‹
    35. ΠšΡ€Π°Ρ‚Π½Ρ‹Π΅ ΠΈΠ½Ρ‚Π΅Π³Ρ€Π°Π»Ρ‹
    36. Π˜Π½Ρ‚Π΅Π³Ρ€Π°Π»Ρ‹, зависящиС ΠΎΡ‚ ΠΏΠ°Ρ€Π°ΠΌΠ΅Ρ‚Ρ€Π°
    37. ΠšΠ²Π°Π΄Ρ€Π°Ρ‚Π½Ρ‹ΠΉ Ρ‚Ρ€Π΅Ρ…Ρ‡Π»Π΅Π½
    38. ΠŸΡ€ΠΎΠΈΠ·Π²ΠΎΠ΄Π½Π°Ρ
    39. ΠŸΡ€ΠΈΠΌΠ΅Π½Π΅Π½ΠΈΠ΅ ΠΏΡ€ΠΎΠΈΠ·Π²ΠΎΠ΄Π½ΠΎΠΉ ΠΊ исслСдованию Ρ„ΡƒΠ½ΠΊΡ†ΠΈΠΉ
    40. ΠŸΡ€ΠΈΠ»ΠΎΠΆΠ΅Π½ΠΈΡ ΠΏΡ€ΠΎΠΈΠ·Π²ΠΎΠ΄Π½ΠΎΠΉ
    41. Π”ΠΈΡ„Ρ„Π΅Ρ€Π΅Π½Ρ†ΠΈΠ°Π» Ρ„ΡƒΠ½ΠΊΡ†ΠΈΠΈ
    42. Π”ΠΈΡ„Ρ„Π΅Ρ€Π΅Π½Ρ†ΠΈΡ€ΠΎΠ²Π°Π½ΠΈΠ΅ Π² ΠΌΠ°Ρ‚Π΅ΠΌΠ°Ρ‚ΠΈΠΊΠ΅
    43. Π€ΠΎΡ€ΠΌΡƒΠ»Ρ‹ ΠΈ ΠΏΡ€Π°Π²ΠΈΠ»Π° диффСрСнцирования
    44. Π”ΠΈΡ„Ρ„Π΅Ρ€Π΅Π½Ρ†ΠΈΠ°Π»ΡŒΠ½ΠΎΠ΅ исчислСниС
    45. Π”ΠΈΡ„Ρ„Π΅Ρ€Π΅Π½Ρ†ΠΈΠ°Π»ΡŒΠ½Ρ‹Π΅ уравнСния
    46. Π”ΠΈΡ„Ρ„Π΅Ρ€Π΅Π½Ρ†ΠΈΠ°Π»ΡŒΠ½Ρ‹Π΅ уравнСния ΠΏΠ΅Ρ€Π²ΠΎΠ³ΠΎ порядка
    47. Π”ΠΈΡ„Ρ„Π΅Ρ€Π΅Π½Ρ†ΠΈΠ°Π»ΡŒΠ½Ρ‹Π΅ уравнСния Π²Ρ‹ΡΡˆΠΈΡ… порядков
    48. Π”ΠΈΡ„Ρ„Π΅Ρ€Π΅Π½Ρ†ΠΈΠ°Π»ΡŒΠ½Ρ‹Π΅ уравнСния Π² частных ΠΏΡ€ΠΎΠΈΠ·Π²ΠΎΠ΄Π½Ρ‹Ρ…
    49. ВригономСтричСскиС Ρ„ΡƒΠ½ΠΊΡ†ΠΈΠΈ
    50. ВригономСтричСскиС уравнСния ΠΈ нСравСнства
    51. ΠŸΠΎΠΊΠ°Π·Π°Ρ‚Π΅Π»ΡŒΠ½Π°Ρ функция
    52. ΠŸΠΎΠΊΠ°Π·Π°Ρ‚Π΅Π»ΡŒΠ½Ρ‹Π΅ уравнСния
    53. ΠžΠ±ΠΎΠ±Ρ‰Π΅Π½Π½Π°Ρ ΡΡ‚Π΅ΠΏΠ΅Π½ΡŒ
    54. Π’Π·Π°ΠΈΠΌΠ½ΠΎ ΠΎΠ±Ρ€Π°Ρ‚Π½Ρ‹Π΅ Ρ„ΡƒΠ½ΠΊΡ†ΠΈΠΈ
    55. ЛогарифмичСская функция
    56. УравнСния ΠΈ нСравСнства
    57. ΠŸΠΎΠ»ΠΎΠΆΠΈΡ‚Π΅Π»ΡŒΠ½Ρ‹Π΅ ΠΈ ΠΎΡ‚Ρ€ΠΈΡ†Π°Ρ‚Π΅Π»ΡŒΠ½Ρ‹Π΅ числа
    58. АлгСбраичСскиС выраТСния
    59. Π˜Ρ€Ρ€Π°Ρ†ΠΈΠΎΠ½Π°Π»ΡŒΠ½Ρ‹Π΅ алгСбраичСскиС выраТСния
    60. ΠŸΡ€Π΅ΠΎΠ±Ρ€Π°Π·ΠΎΠ²Π°Π½ΠΈΠ΅ алгСбраичСских Π²Ρ‹Ρ€Π°ΠΆΠ΅Π½ΠΈΠΉ
    61. ΠŸΡ€Π΅ΠΎΠ±Ρ€Π°Π·ΠΎΠ²Π°Π½ΠΈΠ΅ Π΄Ρ€ΠΎΠ±Π½Ρ‹Ρ… алгСбраичСских Π²Ρ‹Ρ€Π°ΠΆΠ΅Π½ΠΈΠΉ
    62. Π Π°Π·Π»ΠΎΠΆΠ΅Π½ΠΈΠ΅ ΠΌΠ½ΠΎΠ³ΠΎΡ‡Π»Π΅Π½ΠΎΠ² Π½Π° ΠΌΠ½ΠΎΠΆΠΈΡ‚Π΅Π»ΠΈ
    63. ΠœΠ½ΠΎΠ³ΠΎΡ‡Π»Π΅Π½Ρ‹ ΠΎΡ‚ ΠΎΠ΄Π½ΠΎΠ³ΠΎ ΠΏΠ΅Ρ€Π΅ΠΌΠ΅Π½Π½ΠΎΠ³ΠΎ
    64. АлгСбраичСскиС Π΄Ρ€ΠΎΠ±ΠΈ
    65. ΠŸΡ€ΠΎΠΏΠΎΡ€Ρ†ΠΈΠΈ
    66. УравнСния
    67. БистСмы ΡƒΡ€Π°Π²Π½Π΅Π½ΠΈΠΉ
    68. БистСмы ΡƒΡ€Π°Π²Π½Π΅Π½ΠΈΠΉ Π²Ρ‹ΡΡˆΠΈΡ… стСпСнСй
    69. БистСмы алгСбраичСских ΡƒΡ€Π°Π²Π½Π΅Π½ΠΈΠΉ
    70. БистСмы Π»ΠΈΠ½Π΅ΠΉΠ½Ρ‹Ρ… ΡƒΡ€Π°Π²Π½Π΅Π½ΠΈΠΉ
    71. БистСмы Π΄ΠΈΡ„Ρ„Π΅Ρ€Π΅Π½Ρ†ΠΈΠ°Π»ΡŒΠ½Ρ‹Ρ… ΡƒΡ€Π°Π²Π½Π΅Π½ΠΈΠΉ
    72. АрифмСтичСский ΠΊΠ²Π°Π΄Ρ€Π°Ρ‚Π½Ρ‹ΠΉ ΠΊΠΎΡ€Π΅Π½ΡŒ
    73. ΠšΠ²Π°Π΄Ρ€Π°Ρ‚Π½Ρ‹Π΅ ΠΈ кубичСскиС ΠΊΠΎΡ€Π½ΠΈ
    74. Π˜Π·Π²Π»Π΅Ρ‡Π΅Π½ΠΈΠ΅ ΠΊΠ²Π°Π΄Ρ€Π°Ρ‚Π½ΠΎΠ³ΠΎ корня
    75. Π Π°Ρ†ΠΈΠΎΠ½Π°Π»ΡŒΠ½Ρ‹Π΅ числа
    76. Π˜Ρ€Ρ€Π°Ρ†ΠΈΠΎΠ½Π°Π»ΡŒΠ½Ρ‹Π΅ числа
    77. АрифмСтичСский ΠΊΠΎΡ€Π΅Π½ΡŒ
    78. ΠšΠ²Π°Π΄Ρ€Π°Ρ‚Π½Ρ‹Π΅ уравнСния
    79. Π˜Ρ€Ρ€Π°Ρ†ΠΈΠΎΠ½Π°Π»ΡŒΠ½Ρ‹Π΅ уравнСния
    80. ΠŸΠΎΡΠ»Π΅Π΄ΠΎΠ²Π°Ρ‚Π΅Π»ΡŒΠ½ΠΎΡΡ‚ΡŒ
    81. Ряды сходящиСся ΠΈ расходящиСся
    82. ВригономСтричСскиС Ρ„ΡƒΠ½ΠΊΡ†ΠΈΠΈ ΠΏΡ€ΠΎΠΈΠ·Π²ΠΎΠ»ΡŒΠ½ΠΎΠ³ΠΎ ΡƒΠ³Π»Π°
    83. ВригономСтричСскиС Ρ„ΠΎΡ€ΠΌΡƒΠ»Ρ‹
    84. ΠžΠ±Ρ€Π°Ρ‚Π½Ρ‹Π΅ тригономСтричСскиС Ρ„ΡƒΠ½ΠΊΡ†ΠΈΠΈ
    85. Π’Π΅ΠΎΡ€Π΅ΠΌΠ° Π‘Π΅Π·Ρƒ
    86. ΠœΠ°Ρ‚Π΅ΠΌΠ°Ρ‚ΠΈΡ‡Π΅ΡΠΊΠ°Ρ индукция
    87. ΠŸΠΎΠΊΠ°Π·Π°Ρ‚Π΅Π»ΡŒ стСпСни
    88. ΠŸΠΎΠΊΠ°Π·Π°Ρ‚Π΅Π»ΡŒΠ½Ρ‹Π΅ Ρ„ΡƒΠ½ΠΊΡ†ΠΈΠΈ ΠΈ Π»ΠΎΠ³Π°Ρ€ΠΈΡ„ΠΌΡ‹
    89. ΠœΠ½ΠΎΠΆΠ΅ΡΡ‚Π²ΠΎ
    90. ΠœΠ½ΠΎΠΆΠ΅ΡΡ‚Π²ΠΎ Π΄Π΅ΠΉΡΡ‚Π²ΠΈΡ‚Π΅Π»ΡŒΠ½Ρ‹Ρ… чисСл
    91. ЧисловыС мноТСства
    92. ΠŸΡ€Π΅ΠΎΠ±Ρ€Π°Π·ΠΎΠ²Π°Π½ΠΈΠ΅ Ρ€Π°Ρ†ΠΈΠΎΠ½Π°Π»ΡŒΠ½Ρ‹Ρ… Π²Ρ‹Ρ€Π°ΠΆΠ΅Π½ΠΈΠΉ
    93. ΠŸΡ€Π΅ΠΎΠ±Ρ€Π°Π·ΠΎΠ²Π°Π½ΠΈΠ΅ ΠΈΡ€Ρ€Π°Ρ†ΠΈΠΎΠ½Π°Π»ΡŒΠ½Ρ‹Ρ… Π²Ρ‹Ρ€Π°ΠΆΠ΅Π½ΠΈΠΉ
    94. ГСомСтрия
    95. Π”Π΅ΠΉΡΡ‚Π²ΠΈΡ‚Π΅Π»ΡŒΠ½Ρ‹Π΅ числа
    96. Π‘Ρ‚Π΅ΠΏΠ΅Π½ΠΈ ΠΈ ΠΊΠΎΡ€Π½ΠΈ
    97. Π‘Ρ‚Π΅ΠΏΠ΅Π½ΡŒ с Ρ€Π°Ρ†ΠΈΠΎΠ½Π°Π»ΡŒΠ½Ρ‹ΠΌ ΠΏΠΎΠΊΠ°Π·Π°Ρ‚Π΅Π»Π΅ΠΌ
    98. ВригономСтричСскиС Ρ„ΡƒΠ½ΠΊΡ†ΠΈΠΈ ΡƒΠ³Π»Π°
    99. ВригономСтричСскиС Ρ„ΡƒΠ½ΠΊΡ†ΠΈΠΈ числового Π°Ρ€Π³ΡƒΠΌΠ΅Π½Ρ‚Π°
    100. ВригономСтричСскиС выраТСния ΠΈ ΠΈΡ… прСобразования
    101. ΠŸΡ€Π΅ΠΎΠ±Ρ€Π°Π·ΠΎΠ²Π°Π½ΠΈΠ΅ тригономСтричСских Π²Ρ‹Ρ€Π°ΠΆΠ΅Π½ΠΈΠΉ
    102. ΠšΠΎΠΌΠ±ΠΈΠ½Π°Ρ‚ΠΎΡ€ΠΈΠΊΠ°
    103. Π’Ρ‹Ρ‡ΠΈΡΠ»ΠΈΡ‚Π΅Π»ΡŒΠ½Π°Ρ ΠΌΠ°Ρ‚Π΅ΠΌΠ°Ρ‚ΠΈΠΊΠ°
    104. ΠŸΡ€ΡΠΌΠ°Ρ линия Π½Π° плоскости ΠΈ Π΅Π΅ уравнСния
    105. ΠŸΡ€ΡΠΌΠ°Ρ ΠΈ ΠΏΠ»ΠΎΡΠΊΠΎΡΡ‚ΡŒ
    106. Π›ΠΈΠ½ΠΈΠΈ ΠΈ уравнСния
    107. ΠŸΡ€ΡΠΌΠ°Ρ линия
    108. УравнСния прямой ΠΈ плоскости Π² пространствС
    109. ΠšΡ€ΠΈΠ²Ρ‹Π΅ Π²Ρ‚ΠΎΡ€ΠΎΠ³ΠΎ порядка
    110. ΠšΡ€ΠΈΠ²Ρ‹Π΅ ΠΈ повСрхности Π²Ρ‚ΠΎΡ€ΠΎΠ³ΠΎ порядка
    111. ЧисловыС ряды
    112. Π‘Ρ‚Π΅ΠΏΠ΅Π½Π½Ρ‹Π΅ ряды
    113. Ряды Π€ΡƒΡ€ΡŒΠ΅
    114. ΠŸΡ€Π΅ΠΎΠ±Ρ€Π°Π·ΠΎΠ²Π°Π½ΠΈΠ΅ Π€ΡƒΡ€ΡŒΠ΅
    115. Π€ΡƒΠ½ΠΊΡ†ΠΈΠΎΠ½Π°Π»ΡŒΠ½Ρ‹Π΅ ряды
    116. Π€ΡƒΠ½ΠΊΡ†ΠΈΠΈ ΠΌΠ½ΠΎΠ³ΠΈΡ… ΠΏΠ΅Ρ€Π΅ΠΌΠ΅Π½Π½Ρ‹Ρ…
    117. ΠœΠ΅Ρ‚ΠΎΠ΄ ΠΊΠΎΠΎΡ€Π΄ΠΈΠ½Π°Ρ‚
    118. ГармоничСский Π°Π½Π°Π»ΠΈΠ·
    119. ВСщСствСнныС числа
    120. ΠŸΡ€Π΅Π΄Π΅Π» ΠΏΠΎΡΠ»Π΅Π΄ΠΎΠ²Π°Ρ‚Π΅Π»ΡŒΠ½ΠΎΡΡ‚ΠΈ
    121. АналитичСская гСомСтрия
    122. АналитичСская гСомСтрия Π½Π° плоскости
    123. АналитичСская гСомСтрия Π² пространствС
    124. Π€ΡƒΠ½ΠΊΡ†ΠΈΠΈ ΠΎΠ΄Π½ΠΎΠΉ ΠΏΠ΅Ρ€Π΅ΠΌΠ΅Π½Π½ΠΎΠΉ
    125. Π’Ρ‹ΡΡˆΠ°Ρ Π°Π»Π³Π΅Π±Ρ€Π°
    126. ВСкторная Π°Π»Π³Π΅Π±Ρ€Π°
    127. Π’Π΅ΠΊΡ‚ΠΎΡ€Π½Ρ‹ΠΉ Π°Π½Π°Π»ΠΈΠ·
    128. Π’Π΅ΠΊΡ‚ΠΎΡ€Ρ‹
    129. БкалярноС ΠΏΡ€ΠΎΠΈΠ·Π²Π΅Π΄Π΅Π½ΠΈΠ΅ Π²Π΅ΠΊΡ‚ΠΎΡ€ΠΎΠ²
    130. Π’Π΅ΠΊΡ‚ΠΎΡ€Π½ΠΎΠ΅ ΠΏΡ€ΠΎΠΈΠ·Π²Π΅Π΄Π΅Π½ΠΈΠ΅ Π²Π΅ΠΊΡ‚ΠΎΡ€ΠΎΠ²
    131. БмСшанноС ΠΏΡ€ΠΎΠΈΠ·Π²Π΅Π΄Π΅Π½ΠΈΠ΅ Π²Π΅ΠΊΡ‚ΠΎΡ€ΠΎΠ²
    132. ΠžΠΏΠ΅Ρ€Π°Ρ†ΠΈΠΈ Π½Π°Π΄ Π²Π΅ΠΊΡ‚ΠΎΡ€Π°ΠΌΠΈ
    133. ΠΠ΅ΠΏΡ€Π΅Ρ€Ρ‹Π²Π½ΠΎΡΡ‚ΡŒ Ρ„ΡƒΠ½ΠΊΡ†ΠΈΠΉ
    134. ΠŸΡ€Π΅Π΄Π΅Π» ΠΈ Π½Π΅ΠΏΡ€Π΅Ρ€Ρ‹Π²Π½ΠΎΡΡ‚ΡŒ Ρ„ΡƒΠ½ΠΊΡ†ΠΈΠΉ Π½Π΅ΡΠΊΠΎΠ»ΡŒΠΊΠΈΡ… ΠΏΠ΅Ρ€Π΅ΠΌΠ΅Π½Π½Ρ‹Ρ…
    135. ΠŸΡ€Π΅Π΄Π΅Π» ΠΈ Π½Π΅ΠΏΡ€Π΅Ρ€Ρ‹Π²Π½ΠΎΡΡ‚ΡŒ Ρ„ΡƒΠ½ΠΊΡ†ΠΈΠΈ ΠΎΠ΄Π½ΠΎΠΉ ΠΏΠ΅Ρ€Π΅ΠΌΠ΅Π½Π½ΠΎΠΉ
    136. ΠŸΡ€ΠΎΠΈΠ·Π²ΠΎΠ΄Π½Ρ‹Π΅ ΠΈ Π΄ΠΈΡ„Ρ„Π΅Ρ€Π΅Π½Ρ†ΠΈΠ°Π»Ρ‹ Ρ„ΡƒΠ½ΠΊΡ†ΠΈΠΈ ΠΎΠ΄Π½ΠΎΠΉ ΠΏΠ΅Ρ€Π΅ΠΌΠ΅Π½Π½ΠΎΠΉ
    137. ЧастныС ΠΏΡ€ΠΎΠΈΠ·Π²ΠΎΠ΄Π½Ρ‹Π΅ ΠΈ Π΄ΠΈΡ„Ρ„Π΅Ρ€Π΅Π½Ρ†ΠΈΡ€ΡƒΠ΅ΠΌΠΎΡΡ‚ΡŒ Ρ„ΡƒΠ½ΠΊΡ†ΠΈΠΉ Π½Π΅ΡΠΊΠΎΠ»ΡŒΠΊΠΈΡ… ΠΏΠ΅Ρ€Π΅ΠΌΠ΅Π½Π½Ρ‹Ρ…
    138. Π”ΠΈΡ„Ρ„Π΅Ρ€Π΅Π½Ρ†ΠΈΠ°Π»ΡŒΠ½ΠΎΠ΅ исчислСниС Ρ„ΡƒΠ½ΠΊΡ†ΠΈΠΈ ΠΎΠ΄Π½ΠΎΠΉ ΠΏΠ΅Ρ€Π΅ΠΌΠ΅Π½Π½ΠΎΠΉ
    139. ΠœΠ°Ρ‚Ρ€ΠΈΡ†Ρ‹
    140. Π›ΠΈΠ½Π΅ΠΉΠ½Ρ‹Π΅ ΠΈ Π΅Π²ΠΊΠ»ΠΈΠ΄ΠΎΠ²Ρ‹ пространства
    141. Π›ΠΈΠ½Π΅ΠΉΠ½Ρ‹Π΅ отобраТСния
    142. Π”ΠΈΡ„Ρ„Π΅Ρ€Π΅Π½Ρ†ΠΈΠ°Π»ΡŒΠ½Ρ‹Π΅ Ρ‚Π΅ΠΎΡ€Π΅ΠΌΡ‹ ΠΎ срСднСм
    143. ВСория устойчивости Π΄ΠΈΡ„Ρ„Π΅Ρ€Π΅Π½Ρ†ΠΈΠ°Π»ΡŒΠ½Ρ‹Ρ… ΡƒΡ€Π°Π²Π½Π΅Π½ΠΈΠΉ
    144. Π€ΡƒΠ½ΠΊΡ†ΠΈΠΈ комплСксного ΠΏΠ΅Ρ€Π΅ΠΌΠ΅Π½Π½ΠΎΠ³ΠΎ
    145. ΠŸΡ€Π΅ΠΎΠ±Ρ€Π°Π·ΠΎΠ²Π°Π½ΠΈΠ΅ Лапласа
    146. Π’Π΅ΠΎΡ€ΠΈΠΈ поля
    147. ΠžΠΏΠ΅Ρ€Π°Ρ†ΠΈΠΎΠ½Π½ΠΎΠ΅ исчислСниС
    148. БистСмы ΠΊΠΎΠΎΡ€Π΄ΠΈΠ½Π°Ρ‚
    149. Π Π°Ρ†ΠΈΠΎΠ½Π°Π»ΡŒΠ½Π°Ρ функция
    150. Π˜Π½Ρ‚Π΅Π³Ρ€Π°Π»ΡŒΠ½ΠΎΠ΅ исчислСниС
    151. Π˜Π½Ρ‚Π΅Π³Ρ€Π°Π»ΡŒΠ½ΠΎΠ΅ исчислСниС Ρ„ΡƒΠ½ΠΊΡ†ΠΈΠΉ ΠΎΠ΄Π½ΠΎΠΉ ΠΏΠ΅Ρ€Π΅ΠΌΠ΅Π½Π½ΠΎΠΉ
    152. Π”ΠΈΡ„Ρ„Π΅Ρ€Π΅Π½Ρ†ΠΈΠ°Π»ΡŒΠ½ΠΎΠ΅ исчислСниС Ρ„ΡƒΠ½ΠΊΡ†ΠΈΠΉ Π½Π΅ΡΠΊΠΎΠ»ΡŒΠΊΠΈΡ… ΠΏΠ΅Ρ€Π΅ΠΌΠ΅Π½Π½Ρ‹Ρ…
    153. ΠžΡ‚Π½ΠΎΡˆΠ΅Π½ΠΈΠ΅ Π² ΠΌΠ°Ρ‚Π΅ΠΌΠ°Ρ‚ΠΈΠΊΠ΅
    154. ΠœΠ°Ρ‚Π΅ΠΌΠ°Ρ‚ΠΈΡ‡Π΅ΡΠΊΠ°Ρ Π»ΠΎΠ³ΠΈΠΊΠ°
    155. Π“Ρ€Π°Ρ„Ρ‹ Π² ΠΌΠ°Ρ‚Π΅ΠΌΠ°Ρ‚ΠΈΠΊΠ΅
    156. Π›ΠΈΠ½Π΅ΠΉΠ½Ρ‹Π΅ пространства
    157. ΠŸΠ΅Ρ€Π²ΠΎΠΎΠ±Ρ€Π°Π·Π½Π°Ρ ΠΈ Π½Π΅ΠΎΠΏΡ€Π΅Π΄Π΅Π»Π΅Π½Π½Ρ‹ΠΉ ΠΈΠ½Ρ‚Π΅Π³Ρ€Π°Π»
    158. ЛинСйная функция
    159. Π’Ρ‹ΠΏΡƒΠΊΠ»Ρ‹Π΅ мноТСства Ρ‚ΠΎΡ‡Π΅ΠΊ
    160. БистСма ΠΊΠΎΠΎΡ€Π΄ΠΈΠ½Π°Ρ‚

    Π‘Π»ΠΎΠΆΠ΅Π½ΠΈΠ΅ ΠΈ Π²Ρ‹Ρ‡ΠΈΡ‚Π°Π½ΠΈΠ΅ с показатСлями (Π²ΠΈΠ΄Π΅ΠΎ ΠΈ ΠΏΡ€Π°ΠΊΡ‚ΠΈΠΊΠ°)

    TranscriptPractice

    ΠŸΡ€ΠΈΠ²Π΅Ρ‚, рСбята! Π”ΠΎΠ±Ρ€ΠΎ ΠΏΠΎΠΆΠ°Π»ΠΎΠ²Π°Ρ‚ΡŒ Π² это Π²ΠΈΠ΄Π΅ΠΎ ΠΎ слоТСнии ΠΈ Π²Ρ‹Ρ‡ΠΈΡ‚Π°Π½ΠΈΠΈ с показатСлями стСпСни.

    Для Π½Π°Ρ‡Π°Π»Π°, Ρ‡Ρ‚ΠΎΠ±Ρ‹ ΠΌΡ‹ всС Π±Ρ‹Π»ΠΈ Π½Π° ΠΎΠ΄Π½ΠΎΠΉ страницС, я ΡΠΎΠ±ΠΈΡ€Π°ΡŽΡΡŒ ΠΎΠΏΡ€Π΅Π΄Π΅Π»ΠΈΡ‚ΡŒ экспонСнты , Π° Ρ‚Π°ΠΊΠΆΠ΅ нСсколько Π΄Ρ€ΡƒΠ³ΠΈΡ… Π²Π΅Ρ‰Π΅ΠΉ, Ρ‡Ρ‚ΠΎΠ±Ρ‹ Π² дальнСйшСм, надСюсь, Π½Π΅ Π±Ρ‹Π»ΠΎ Ρ‚Π°ΠΊΠΎΠΉ ΠΏΡƒΡ‚Π°Π½ΠΈΡ†Ρ‹.

    Π˜Ρ‚Π°ΠΊ, я Π½Π°Ρ‡Π½Ρƒ с Π±Π°Π·Ρ‹ (ΠΈΠ»ΠΈ ΠΏΠ΅Ρ€Π΅ΠΌΠ΅Π½Π½ΠΎΠΉ Π±Π°Π·Ρ‹ Π² Π΄Π°Π½Π½ΠΎΠΌ случаС). Π±Π°Π·Π° β€” это число, ΠΊΠΎΡ‚ΠΎΡ€ΠΎΠ΅ возводится Π² ΡΡ‚Π΅ΠΏΠ΅Π½ΡŒ Ρ‡Π΅Π³ΠΎ-Ρ‚ΠΎ, ΠΈ это число, Π½Π° ΠΊΠΎΡ‚ΠΎΡ€ΠΎΠ΅ ΠΎΠ½ΠΎ возводится, называСтся ΠΏΠΎΠΊΠ°Π·Π°Ρ‚Π΅Π»Π΅ΠΌ стСпСни . Π’Π΅ΠΏΠ΅Ρ€ΡŒ ΠΏΠΎΠΊΠ°Π·Π°Ρ‚Π΅Π»ΡŒ стСпСни прСдставляСт собой Ρ‚ΠΎ, сколько Ρ€Π°Π· это основаниС умноТаСтся само Π½Π° сСбя. Π˜Ρ‚Π°ΠΊ, Π² этом случаС \(x\), наша Π±Π°Π·Π°, умноТаСтся сама Π½Π° сСбя Π΄Π²Π°ΠΆΠ΄Ρ‹ (\(x \cdot x\)). Число ΠΏΠ΅Ρ€Π΅Π΄ основаниСм ΠΈΠ»ΠΈ ΠΏΠ΅Ρ€Π΅ΠΌΠ΅Π½Π½ΠΎΠΉ называСтся коэффициСнтом . НадСюсь, Ρ‚Π΅ΠΏΠ΅Ρ€ΡŒ, ΠΏΡ€ΠΎΠ΄Π²ΠΈΠ³Π°ΡΡΡŒ Π²ΠΏΠ΅Ρ€Π΅Π΄, Π²Ρ‹ смоТСтС Π»ΡƒΡ‡ΡˆΠ΅ ΠΏΠΎΠ½ΡΡ‚ΡŒ, ΠΎ Ρ‡Π΅ΠΌ я Π³ΠΎΠ²ΠΎΡ€ΡŽ, ΠΊΠΎΠ³Π΄Π° я ΡΡΡ‹Π»Π°ΡŽΡΡŒ Π½Π° эти Ρ‚Π΅Ρ€ΠΌΠΈΠ½Ρ‹. 93\)

    Β 

    Π˜Ρ‚Π°ΠΊ, Ρ‡Ρ‚ΠΎΠ±Ρ‹ ΡƒΠΏΡ€ΠΎΡΡ‚ΠΈΡ‚ΡŒ это, ΠΌΡ‹ Π΄ΠΎΠ»ΠΆΠ½Ρ‹ ΠΈΡΠΊΠ°Ρ‚ΡŒ Π΄Π²Π΅ Π²Π΅Ρ‰ΠΈ. Π’ΠΎ-ΠΏΠ΅Ρ€Π²Ρ‹Ρ…, наши Π±Π°Π·Ρ‹ (ΠΈΠ»ΠΈ ΠΏΠ΅Ρ€Π΅ΠΌΠ΅Π½Π½Ρ‹Π΅) ΠΎΠ΄ΠΈΠ½Π°ΠΊΠΎΠ²Ρ‹; ΠΈ Π΄Π²Π°, наши ΠΏΠΎΠΊΠ°Π·Π°Ρ‚Π΅Π»ΠΈ ΠΎΠ΄ΠΈΠ½Π°ΠΊΠΎΠ²Ρ‹? ОбС Π²Π΅Ρ‰ΠΈ Π΄ΠΎΠ»ΠΆΠ½Ρ‹ Π±Ρ‹Ρ‚ΡŒ истинными, Ρ‡Ρ‚ΠΎΠ±Ρ‹ ΠΌΡ‹ ΠΌΠΎΠ³Π»ΠΈ ΡΠ»ΠΎΠΆΠΈΡ‚ΡŒ эти Π΄Π²Π° Ρ‚Π΅Ρ€ΠΌΠΈΠ½Π° вмСстС. Π­Ρ‚ΠΎΡ‚ ΠΆΠ΅ процСсс слоТСния ΠΈ вычитания с показатСлями Ρ‚Π°ΠΊΠΆΠ΅ называСтся объСдинСниСм ΠΎΠ΄ΠΈΠ½Π°ΠΊΠΎΠ²Ρ‹Ρ… Ρ‚Π΅Ρ€ΠΌΠΈΠ½ΠΎΠ², Ρ‡Ρ‚ΠΎ ΠΌΠΎΠΆΠ΅Ρ‚ ΠΏΠΎΠΊΠ°Π·Π°Ρ‚ΡŒΡΡ Π²Π°ΠΌ Π±ΠΎΠ»Π΅Π΅ Π·Π½Π°ΠΊΠΎΠΌΡ‹ΠΌ. Ну, это ΠΎΠ΄Π½ΠΎ ΠΈ Ρ‚ΠΎ ΠΆΠ΅.

    Π˜Ρ‚Π°ΠΊ, ΠΎΠ³Π»ΡΠ΄Ρ‹Π²Π°ΡΡΡŒ Π½Π°Π·Π°Π΄ Π½Π° Π½Π°ΡˆΡƒ Π·Π°Π΄Π°Ρ‡Ρƒ, ΠΌΡ‹ Π²ΠΈΠ΄ΠΈΠΌ, Ρ‡Ρ‚ΠΎ наши основания ΠΎΠ΄ΠΈΠ½Π°ΠΊΠΎΠ²Ρ‹, ΠΈ наши ΠΏΠΎΠΊΠ°Π·Π°Ρ‚Π΅Π»ΠΈ ΠΎΠ΄ΠΈΠ½Π°ΠΊΠΎΠ²Ρ‹. Π­Ρ‚ΠΎ ΠΎΠ·Π½Π°Ρ‡Π°Π΅Ρ‚, Ρ‡Ρ‚ΠΎ Ρƒ нас Π΅ΡΡ‚ΡŒ Π΄Π²Π° ΠΎΠ΄ΠΈΠ½Π°ΠΊΠΎΠ²Ρ‹Ρ… Ρ‚Π΅Ρ€ΠΌΠΈΠ½Π°, ΠΊΠΎΡ‚ΠΎΡ€Ρ‹Π΅ ΠΌΠΎΠΆΠ½ΠΎ ΠΎΠ±ΡŠΠ΅Π΄ΠΈΠ½ΠΈΡ‚ΡŒ вмСстС. Π˜Ρ‚Π°ΠΊ, Ρ‡Ρ‚ΠΎΠ±Ρ‹ Π½Π° самом Π΄Π΅Π»Π΅ ΠΎΠ±ΡŠΠ΅Π΄ΠΈΠ½ΠΈΡ‚ΡŒ ΠΈΡ…, Π²ΠΎΡ‚ Ρ‡Ρ‚ΠΎ Π²Π°ΠΌ Π½ΡƒΠΆΠ½ΠΎ ΡΠ΄Π΅Π»Π°Ρ‚ΡŒ: 92+15\), ΠΊΠΎΡ‚ΠΎΡ€Ρ‹ΠΉ являСтся ΠΏΠ΅Ρ€ΠΈΠΌΠ΅Ρ‚Ρ€ΠΎΠΌ Ρ‚Ρ€Π΅ΡƒΠ³ΠΎΠ»ΡŒΠ½ΠΈΠΊΠ°.

    Π‘ΠΊΡ€Ρ‹Ρ‚ΡŒ ΠΎΡ‚Π²Π΅Ρ‚

    Β 

    Π’Π΅Ρ€Π½ΡƒΡ‚ΡŒΡΡ ΠΊ Π²ΠΈΠ΄Π΅ΠΎ ΠΏΠΎ Π°Π»Π³Π΅Π±Ρ€Π΅ I

    875756

    Π‘Π»ΠΎΠΆΠ΅Π½ΠΈΠ΅ ΠΏΠΎΠΊΠ°Π·Π°Ρ‚Π΅Π»Π΅ΠΉ стСпСни — ΠΎΠΏΡ€Π΅Π΄Π΅Π»Π΅Π½ΠΈΠ΅, шаги, ΠΌΠ΅Ρ‚ΠΎΠ΄, ΠΏΡ€ΠΈΠΌΠ΅Ρ€Ρ‹

    Π”Ρ€ΡƒΠ³ΠΈΠΌΠΈ словами, Π΄ΠΎΠ±Π°Π²Π»Π΅Π½ΠΈΠ΅ основания ΠΈ ΠΏΠΎΠΊΠ°Π·Π°Ρ‚Π΅Π»Π΅ΠΉ стСпСни β€” это Π΄ΠΎΠ±Π°Π²Π»Π΅Π½ΠΈΠ΅ ΠΏΠΎΠΊΠ°Π·Π°Ρ‚Π΅Π»Π΅ΠΉ стСпСни. ΠŸΠΎΠΊΠ°Π·Π°Ρ‚Π΅Π»ΡŒ стСпСни Ρ‚Π°ΠΊΠΆΠ΅ называСтся ΡΡ‚Π΅ΠΏΠ΅Π½ΡŒΡŽ числа ΠΈ ΠΏΠΎΠΊΠ°Π·Ρ‹Π²Π°Π΅Ρ‚, сколько Ρ€Π°Π· число умноТаСтся само Π½Π° сСбя. Π’ ΠΎΠ±Ρ‰Π΅ΠΌ Ρ… n ΠΎΠ·Π½Π°Ρ‡Π°Π΅Ρ‚, Ρ‡Ρ‚ΠΎ x умноТаСтся сам Π½Π° сСбя n Ρ€Π°Π·. Π”ΠΎΠ±Π°Π²Π»Π΅Π½ΠΈΠ΅ ΠΏΠΎΠΊΠ°Π·Π°Ρ‚Π΅Π»Π΅ΠΉ выполняСтся Π² Ρ€Π°Π·Π½Ρ‹Ρ… Ρ‚ΠΈΠΏΠ°Ρ…, Π΄Π°Π²Π°ΠΉΡ‚Π΅ посмотрим, Ρ‡Ρ‚ΠΎ это Π·Π° Ρ‚ΠΈΠΏΡ‹, ΠΈ Ρ€Π΅ΡˆΠΈΠΌ нСсколько ΠΏΡ€ΠΈΠΌΠ΅Ρ€ΠΎΠ², Ρ‡Ρ‚ΠΎΠ±Ρ‹ Π»ΡƒΡ‡ΡˆΠ΅ ΠΏΠΎΠ½ΡΡ‚ΡŒ эту ΠΊΠΎΠ½Ρ†Π΅ΠΏΡ†ΠΈΡŽ.

    1. Π§Ρ‚ΠΎ Ρ‚Π°ΠΊΠΎΠ΅ Π΄ΠΎΠ±Π°Π²Π»Π΅Π½ΠΈΠ΅ ΠΏΠΎΠΊΠ°Π·Π°Ρ‚Π΅Π»Π΅ΠΉ?
    2. Π¨Π°Π³ΠΈ добавлСния ΠΏΠΎΠΊΠ°Π·Π°Ρ‚Π΅Π»Π΅ΠΉ
    3. Π”ΠΎΠ±Π°Π²Π»Π΅Π½ΠΈΠ΅ ΠΌΠ΅Ρ‚ΠΎΠ΄ΠΎΠ² экспонСнты
    4. Часто Π·Π°Π΄Π°Π²Π°Π΅ΠΌΡ‹Π΅ вопросы ΠΎ Π΄ΠΎΠ±Π°Π²Π»Π΅Π½ΠΈΠΈ экспонСнтов

    Π§Ρ‚ΠΎ Ρ‚Π°ΠΊΠΎΠ΅ Π΄ΠΎΠ±Π°Π²Π»Π΅Π½ΠΈΠ΅ ΠΏΠΎΠΊΠ°Π·Π°Ρ‚Π΅Π»Π΅ΠΉ?

    Π‘Π»ΠΎΠΆΠ΅Π½ΠΈΠ΅ ΠΏΠΎΠΊΠ°Π·Π°Ρ‚Π΅Π»Π΅ΠΉ стСпСни β€” это процСсс слоТСния ΠΏΠΎΠΊΠ°Π·Π°Ρ‚Π΅Π»Π΅ΠΉ стСпСни ΠΈΠ»ΠΈ стСпСни числа нСзависимо ΠΎΡ‚ Ρ‚ΠΎΠ³ΠΎ, совпадаСт Π»ΠΈ основаниС. ΠŸΠΎΠΊΠ°Π·Π°Ρ‚Π΅Π»ΡŒ стСпСни Ρ‚Π°ΠΊΠΆΠ΅ ΠΌΠΎΠΆΠ½ΠΎ Π½Π°Π·Π²Π°Ρ‚ΡŒ ΡΡ‚Π΅ΠΏΠ΅Π½ΡŒΡŽ числа, ΠΏΠΎΡΠΊΠΎΠ»ΡŒΠΊΡƒ ΠΎΠ½ прСдставляСт собой количСство Ρ€Π°Π·, ΠΊΠΎΠ³Π΄Π° число умноТаСтся само Π½Π° сСбя. НапримСр, 3 2 = 3 Γ— 3, Π³Π΄Π΅ 3 β€” основаниС, Π° 2 β€” ΠΏΠΎΠΊΠ°Π·Π°Ρ‚Π΅Π»ΡŒ стСпСни.

    Π—Π΄Π΅ΡΡŒ Π² Ρ‡Π»Π΅Π½Π΅ Ρ… n ,

    • x называСтся Π±Π°Π·ΠΎΠΉ
    • n называСтся ΠΏΠΎΠΊΠ°Π·Π°Ρ‚Π΅Π»Π΅ΠΌ стСпСни ΠΈΠ»ΠΈ ΡΡ‚Π΅ΠΏΠ΅Π½ΡŒΡŽ
    • x n читаСтся ΠΊΠ°ΠΊ x Π² стСпСни n (ΠΈΠ»ΠΈ) x Π² стСпСни n

    ΠŸΡ€ΠΈ слоТСнии ΠΏΠΎΠΊΠ°Π·Π°Ρ‚Π΅Π»Π΅ΠΉ слСдуСт ΠΏΠΎΠΌΠ½ΠΈΡ‚ΡŒ ΠΎΠ΄Π½ΠΎ Π³Π»Π°Π²Π½ΠΎΠ΅ ΠΏΡ€Π°Π²ΠΈΠ»ΠΎ: основаниС ΠΈ ΠΏΠΎΠΊΠ°Π·Π°Ρ‚Π΅Π»ΡŒ стСпСни Π΄ΠΎΠ»ΠΆΠ½Ρ‹ ΡΠΎΠ²ΠΏΠ°Π΄Π°Ρ‚ΡŒ, Π° слоТСниС выполняСтся с коэффициСнтом. ΠŸΠ΅Ρ€Π΅ΠΌΠ΅Π½Π½Ρ‹Π΅, ΠΊΠΎΡ‚ΠΎΡ€Ρ‹Π΅ ΠΎΠ±ΡŠΠ΅Π΄ΠΈΠ½ΡΡŽΡ‚ΡΡ, ΠΈΠΌΠ΅ΡŽΡ‚ ΠΎΠ΄Π½ΠΎ ΠΈ Ρ‚ΠΎ ΠΆΠ΅ основаниС ΠΈ ΠΎΠ΄ΠΈΠ½Π°ΠΊΠΎΠ²ΡƒΡŽ ΠΌΠΎΡ‰Π½ΠΎΡΡ‚ΡŒ. Π­Ρ‚ΠΎ ΠΏΡ€Π°Π²ΠΈΠ»ΠΎ ΠΏΡ€ΠΈΠΌΠ΅Π½ΠΈΠΌΠΎ ΠΈ ΠΊΠΎ всСм Π΄Ρ€ΡƒΠ³ΠΈΠΌ Ρ„ΠΎΡ€ΠΌΠ°ΠΌ ΠΏΠΎΠΊΠ°Π·Π°Ρ‚Π΅Π»Π΅ΠΉ стСпСни, Ρ‚ΠΎ Π΅ΡΡ‚ΡŒ ΠΊ Π²Ρ‹Ρ‡ΠΈΡ‚Π°Π½ΠΈΡŽ, ΡƒΠΌΠ½ΠΎΠΆΠ΅Π½ΠΈΡŽ ΠΈ дСлСнию.

    Π¨Π°Π³ΠΈ добавлСния экспонСнты

    Π”ΠΎΠ±Π°Π²Π»Π΅Π½ΠΈΠ΅ ΠΏΠΎΠΊΠ°Π·Π°Ρ‚Π΅Π»Π΅ΠΉ стСпСни ΠΌΠΎΠΆΠ΅Ρ‚ Π±Ρ‹Ρ‚ΡŒ Π²Ρ‹ΠΏΠΎΠ»Π½Π΅Π½ΠΎ, ΠΊΠΎΠ³Π΄Π° основаниС ΠΈ ΡΡ‚Π΅ΠΏΠ΅Π½ΡŒ ΡΠΎΠ²ΠΏΠ°Π΄Π°ΡŽΡ‚. Π‘Ρ‹Π²Π°ΡŽΡ‚ случаи, ΠΊΠΎΠ³Π΄Π° основаниС ΠΈ ΠΏΠΎΠΊΠ°Π·Π°Ρ‚Π΅Π»ΠΈ стСпСни Ρ€Π°Π·Π»ΠΈΡ‡Π°ΡŽΡ‚ΡΡ, Π½ΠΎ ΠΌΡ‹ всС Ρ€Π°Π²Π½ΠΎ ΠΌΠΎΠΆΠ΅ΠΌ Π²Ρ‹ΠΏΠΎΠ»Π½ΡΡ‚ΡŒ слоТСниС для этих Π²Ρ‹Ρ€Π°ΠΆΠ΅Π½ΠΈΠΉ. Π”Π°Π²Π°ΠΉΡ‚Π΅ посмотрим Π½Π° шаги добавлСния ΠΏΠΎΠΊΠ°Π·Π°Ρ‚Π΅Π»Π΅ΠΉ.

    • Π¨Π°Π³ 1: ΠŸΡ€ΠΎΠ²Π΅Ρ€ΡŒΡ‚Π΅ Ρ‡Π»Π΅Π½Ρ‹ Π² Π²Ρ‹Ρ€Π°ΠΆΠ΅Π½ΠΈΠΈ, ΠΈΠΌΠ΅ΡŽΡ‚ Π»ΠΈ ΠΎΠ½ΠΈ ΠΎΠ΄ΠΈΠ½Π°ΠΊΠΎΠ²ΠΎΠ΅ основаниС ΠΈ ΠΎΠ΄ΠΈΠ½Π°ΠΊΠΎΠ²Ρ‹Π΅ ΠΏΠΎΠΊΠ°Π·Π°Ρ‚Π΅Π»ΠΈ стСпСни. НапримСр, 2 2 + 2 2 . Как ΠΌΡ‹ Π²ΠΈΠ΄ΠΈΠΌ, ΠΈ основаниС, ΠΈ ΠΏΠΎΠΊΠ°Π·Π°Ρ‚Π΅Π»ΡŒ стСпСни Ρ€Π°Π²Π½Ρ‹ 2,9.0281
    • Π¨Π°Π³ 2: Если основаниС ΠΈ ΠΏΠΎΠΊΠ°Π·Π°Ρ‚Π΅Π»ΠΈ Ρ€Π°Π·Π»ΠΈΡ‡Π½Ρ‹, вычислитС Π²Ρ‹Ρ€Π°ΠΆΠ΅Π½ΠΈΠ΅ с ΠΎΡ‚Π΄Π΅Π»ΡŒΠ½Ρ‹ΠΌΠΈ Ρ‡Π»Π΅Π½Π°ΠΌΠΈ. НапримСр, 5 3 + 4 2 . Π‘Π°Π·Π° ΠΈ ΠΏΠΎΠΊΠ°Π·Π°Ρ‚Π΅Π»ΠΈ Ρ€Π°Π·Π½Ρ‹Π΅.
    • Π¨Π°Π³ 3. Π‘Π»ΠΎΠΆΠΈΡ‚Π΅ Ρ€Π΅Π·ΡƒΠ»ΡŒΡ‚Π°Ρ‚Ρ‹.

    Π”ΠΎΠ±Π°Π²Π»Π΅Π½ΠΈΠ΅ ΠΌΠ΅Ρ‚ΠΎΠ΄ΠΎΠ² экспонСнты

    Π‘Π»ΠΎΠΆΠ΅Π½ΠΈΠ΅ ΠΏΠΎΠΊΠ°Π·Π°Ρ‚Π΅Π»Π΅ΠΉ ΠΌΠΎΠΆΠ΅Ρ‚ Π²Ρ‹ΠΏΠΎΠ»Π½ΡΡ‚ΡŒΡΡ Ρ€Π°Π·Π»ΠΈΡ‡Π½Ρ‹ΠΌΠΈ способами. ОсновноС ΠΏΡ€Π°Π²ΠΈΠ»ΠΎ ΠΏΡ€ΠΈ Π΄ΠΎΠ±Π°Π²Π»Π΅Π½ΠΈΠΈ ΠΏΠΎΠΊΠ°Π·Π°Ρ‚Π΅Π»Π΅ΠΉ стСпСни состоит Π² Ρ‚ΠΎΠΌ, Ρ‡Ρ‚ΠΎ основаниС ΠΈ ΡΡ‚Π΅ΠΏΠ΅Π½ΡŒ Π΄ΠΎΠ»ΠΆΠ½Ρ‹ Π±Ρ‹Ρ‚ΡŒ ΠΎΠ΄ΠΈΠ½Π°ΠΊΠΎΠ²Ρ‹ΠΌΠΈ. Однако ΠΈΠ½ΠΎΠ³Π΄Π° основаниС ΠΈ ΠΏΠΎΠΊΠ°Π·Π°Ρ‚Π΅Π»ΠΈ ΠΌΠΎΠ³ΡƒΡ‚ Π½Π΅ ΡΠΎΠ²ΠΏΠ°Π΄Π°Ρ‚ΡŒ, поэтому Π½Π°ΠΌ Π½ΡƒΠΆΠ½ΠΎ Π²Ρ‹Ρ‡ΠΈΡΠ»ΡΡ‚ΡŒ Ρ‡Π»Π΅Π½Ρ‹ ΠΏΠΎ ΠΎΡ‚Π΄Π΅Π»ΡŒΠ½ΠΎΡΡ‚ΠΈ, Ρ‡Ρ‚ΠΎΠ±Ρ‹ Π²Ρ‹Ρ‡ΠΈΡΠ»ΠΈΡ‚ΡŒ Π²Ρ‹Ρ€Π°ΠΆΠ΅Π½ΠΈΠ΅. Π”Π°Π²Π°ΠΉΡ‚Π΅ посмотрим Π½Π° Ρ€Π°Π·Π»ΠΈΡ‡Π½Ρ‹Π΅ ΠΌΠ΅Ρ‚ΠΎΠ΄Ρ‹.

    ΠœΠ΅Ρ‚ΠΎΠ΄ 1: Π‘Π»ΠΎΠΆΠ΅Π½ΠΈΠ΅ ΠΏΠΎΠΊΠ°Π·Π°Ρ‚Π΅Π»Π΅ΠΉ стСпСни с ΠΎΠ΄ΠΈΠ½Π°ΠΊΠΎΠ²Ρ‹ΠΌΠΈ основаниСм ΠΈ показатСлями

    Π‘Π»ΠΎΠΆΠ΅Π½ΠΈΠ΅ ΠΏΠΎΠΊΠ°Π·Π°Ρ‚Π΅Π»Π΅ΠΉ стСпСни, ΠΊΠΎΠ³Π΄Π° основаниС ΠΈ ΠΏΠΎΠΊΠ°Π·Π°Ρ‚Π΅Π»ΠΈ ΡΠΎΠ²ΠΏΠ°Π΄Π°ΡŽΡ‚, выполняСтся ΠΎΡ‡Π΅Π½ΡŒ простым ΠΌΠ΅Ρ‚ΠΎΠ΄ΠΎΠΌ. ΠžΠ±Ρ‰Π°Ρ Ρ„ΠΎΡ€ΠΌΠ° слоТСния ΠΏΠΎΠΊΠ°Π·Π°Ρ‚Π΅Π»Π΅ΠΉ с ΠΎΠ΄ΠΈΠ½Π°ΠΊΠΎΠ²Ρ‹ΠΌ основаниСм ΠΈ показатСлями Ρ‚Π°ΠΊΠΎΠ²Π°: n + n = 2a n . Π”Π°Π²Π°ΠΉΡ‚Π΅ посмотрим Π½Π° ΠΏΡ€ΠΈΠΌΠ΅Ρ€, Ρ‡Ρ‚ΠΎΠ±Ρ‹ ΠΏΠΎΠ½ΡΡ‚ΡŒ это Π»ΡƒΡ‡ΡˆΠ΅. НапримСр: 4 3 + 4 3 = 2(4 3 ) = 2 Γ— 4 Γ— 4 Γ— 4 = 128.

    ΠœΠ΅Ρ‚ΠΎΠ΄ 2. Π‘Π»ΠΎΠΆΠ΅Π½ΠΈΠ΅ ΠΏΠΎΠΊΠ°Π·Π°Ρ‚Π΅Π»Π΅ΠΉ стСпСни с Ρ€Π°Π·Π½Ρ‹ΠΌ основаниСм ΠΈ ΡΡ‚Π΅ΠΏΠ΅Π½ΡŒΡŽ

    Если основаниС ΠΈ ΠΏΠΎΠΊΠ°Π·Π°Ρ‚Π΅Π»ΡŒ стСпСни ΠΈΠΌΠ΅ΡŽΡ‚ Ρ€Π°Π·Π½Ρ‹Π΅ значСния, ΠΌΡ‹ сначала добавляСм ΠΊΠ°ΠΆΠ΄Ρ‹ΠΉ ΠΏΠΎΠΊΠ°Π·Π°Ρ‚Π΅Π»ΡŒ стСпСни, Π° Π·Π°Ρ‚Π΅ΠΌ вычисляСм всС Π²Ρ‹Ρ€Π°ΠΆΠ΅Π½ΠΈΠ΅. ΠžΠ±Ρ‰Π°Ρ Ρ„ΠΎΡ€ΠΌΠ° вычислСния Ρ€Π°Π·Π»ΠΈΡ‡Π½Ρ‹Ρ… оснований ΠΈ ΠΏΠΎΠΊΠ°Π·Π°Ρ‚Π΅Π»Π΅ΠΉ стСпСни Ρ‚Π°ΠΊΠΎΠ²Π°: a n + b m . Π”Π°Π²Π°ΠΉΡ‚Π΅ посмотрим Π½Π° ΠΏΡ€ΠΈΠΌΠ΅Ρ€, Ρ‡Ρ‚ΠΎΠ±Ρ‹ ΠΏΠΎΠ½ΡΡ‚ΡŒ это Π»ΡƒΡ‡ΡˆΠ΅. НапримСр: 3 3 + 5 2 = 3 Γ— 3 Γ— 3 + 5 Γ— 5 = 27 + 25 = 52.

    ΠœΠ΅Ρ‚ΠΎΠ΄ 3: слоТСниС ΠΎΡ‚Ρ€ΠΈΡ†Π°Ρ‚Π΅Π»ΡŒΠ½Ρ‹Ρ… стСпСнСй с Ρ€Π°Π·Π½Ρ‹ΠΌΠΈ основаниями

    срок ΠΏΠΎ ΠΎΡ‚Π΄Π΅Π»ΡŒΠ½ΠΎΡΡ‚ΠΈ, Π° Π·Π°Ρ‚Π΅ΠΌ Π΄ΠΎΠ±Π°Π²ΠΈΡ‚ΡŒ ΠΎΠ±Ρ‰Π΅Π΅ количСство. Π’Π΅Ρ€ΠΌΠΈΠ½Ρ‹ Π·Π°ΠΏΠΈΡΡ‹Π²Π°ΡŽΡ‚ΡΡ Π² Π΄Ρ€ΠΎΠ±Π½ΠΎΠΉ Ρ„ΠΎΡ€ΠΌΠ΅, Π° Π·Π°Ρ‚Π΅ΠΌ Π΄ΠΎΠ±Π°Π²Π»ΡΡŽΡ‚ΡΡ. ΠžΠ±Ρ‰Π°Ρ Ρ„ΠΎΡ€ΠΌΠ° вычислСния ΠΎΡ‚Ρ€ΠΈΡ†Π°Ρ‚Π΅Π»ΡŒΠ½Ρ‹Ρ… ΠΏΠΎΠΊΠ°Π·Π°Ρ‚Π΅Π»Π΅ΠΉ с Ρ€Π°Π·Π½Ρ‹ΠΌΠΈ основаниями — это -Π½ + Π± -ΠΌ = 1/Π° Π½ + 1/Π± ΠΌ . Π”Π°Π²Π°ΠΉΡ‚Π΅ ΠΏΡ€ΠΈΠΌΠ΅Π½ΠΈΠΌ ΠΎΠ±Ρ‰ΡƒΡŽ Ρ„ΠΎΡ€ΠΌΡƒ Π² ΠΏΡ€ΠΈΠΌΠ΅Ρ€Π΅, Ρ‡Ρ‚ΠΎΠ±Ρ‹ ΠΏΠΎΠ½ΡΡ‚ΡŒ это Π»ΡƒΡ‡ΡˆΠ΅. НапримСр: 6 -2 + 3 -3 = 1/6 2 + 1/3 3 = 1/36 + 1/27 = 0,0648.

    ΠœΠ΅Ρ‚ΠΎΠ΄ 4: Π΄ΠΎΠ±Π°Π²Π»Π΅Π½ΠΈΠ΅ Π΄Ρ€ΠΎΠ±Π½Ρ‹Ρ… ΠΏΠΎΠΊΠ°Π·Π°Ρ‚Π΅Π»Π΅ΠΉ стСпСни с ΠΎΠ΄ΠΈΠ½Π°ΠΊΠΎΠ²Ρ‹ΠΌ основаниСм ΠΈ показатСлями стСпСни

    ΠŸΠΎΠΊΠ°Π·Π°Ρ‚Π΅Π»ΠΈ стСпСни Ρ‚Π°ΠΊΠΆΠ΅ ΠΌΠΎΠ³ΡƒΡ‚ Π±Ρ‹Ρ‚ΡŒ Π²Ρ‹Ρ€Π°ΠΆΠ΅Π½Ρ‹ Π² Ρ„ΠΎΡ€ΠΌΠ΅ Π΄Ρ€ΠΎΠ±ΠΈ. Π’Π°ΠΊΠΈΠΌ ΠΎΠ±Ρ€Π°Π·ΠΎΠΌ, слоТСниС этих Π΄Ρ€ΠΎΠ±Π΅ΠΉ Π² Π²ΠΈΠ΄Π΅ ΠΏΠΎΠΊΠ°Π·Π°Ρ‚Π΅Π»Π΅ΠΉ стСпСни ΠΌΠΎΠΆΠ΅Ρ‚ Π±Ρ‹Ρ‚ΡŒ Π²Ρ‹ΠΏΠΎΠ»Π½Π΅Π½ΠΎ с использованиСм этой ΠΎΠ±Ρ‰Π΅ΠΉ Ρ„ΠΎΡ€ΠΌΡ‹, Π½/ΠΌ + a Π½/ΠΌ = 2a Π½/ΠΌ . Для Π΄Ρ€ΠΎΠ±Π΅ΠΉ ΠΌΡ‹ ΠΌΠΎΠΆΠ΅ΠΌ ΠΏΡ€Π΅ΠΎΠ±Ρ€Π°Π·ΠΎΠ²Π°Ρ‚ΡŒ Π΄Ρ€ΠΎΠ±Π½Ρ‹Π΅ ΠΏΠΎΠΊΠ°Π·Π°Ρ‚Π΅Π»ΠΈ Π² Ρ„ΠΎΡ€ΠΌΡƒ корня, Ρ‚ΠΎ Π΅ΡΡ‚ΡŒ Π»ΠΈΠ±ΠΎ ΠΊΠ²Π°Π΄Ρ€Π°Ρ‚Π½Ρ‹ΠΉ ΠΊΠΎΡ€Π΅Π½ΡŒ, Π»ΠΈΠ±ΠΎ кубичСский ΠΊΠΎΡ€Π΅Π½ΡŒ, Π² зависимости ΠΎΡ‚ Π΄Ρ€ΠΎΠ±ΠΈ. Π”Π°Π²Π°ΠΉΡ‚Π΅ ΠΏΡ€ΠΈΠΌΠ΅Π½ΠΈΠΌ эту ΠΎΠ±Ρ‰ΡƒΡŽ Ρ„ΠΎΡ€ΠΌΡƒ Π² ΠΏΡ€ΠΈΠΌΠ΅Ρ€Π΅ для Π»ΡƒΡ‡ΡˆΠ΅Π³ΠΎ понимания. НапримСр: 4 1/2 + 4 1/2 = 2(4 1/2 ) = 2 Γ— √4 = 2 Γ— 2 = 4.

    ΠœΠ΅Ρ‚ΠΎΠ΄ 5. Π‘Π»ΠΎΠΆΠ΅Π½ΠΈΠ΅ Π΄Ρ€ΠΎΠ±Π½Ρ‹Ρ… ΠΏΠΎΠΊΠ°Π·Π°Ρ‚Π΅Π»Π΅ΠΉ с Ρ€Π°Π·Π½Ρ‹ΠΌ основаниСм ΠΈ ΠŸΠΎΠΊΠ°Π·Π°Ρ‚Π΅Π»ΡŒ стСпСни

    ΠŸΠΎΠΊΠ°Π·Π°Ρ‚Π΅Π»ΡŒ стСпСни, записанный Π² Π΄Ρ€ΠΎΠ±Π½ΠΎΠΉ Ρ„ΠΎΡ€ΠΌΠ΅ с Ρ€Π°Π·Π½Ρ‹ΠΌ основаниСм ΠΈ Ρ€Π°Π·Π½Ρ‹ΠΌΠΈ показатСлями стСпСни, выраТаСтся Π² ΠΎΠ±Ρ‰Π΅ΠΉ Ρ„ΠΎΡ€ΠΌΠ΅ ΠΊΠ°ΠΊ Π½/ΠΌ + Π± Π΄/ΠΊ . Π—Π΄Π΅ΡΡŒ сначала вычисляСтся ΠΊΠ°ΠΆΠ΄Ρ‹ΠΉ Ρ‡Π»Π΅Π½, Π° Π·Π°Ρ‚Π΅ΠΌ вычисляСтся вСсь Ρ€Π΅Π·ΡƒΠ»ΡŒΡ‚Π°Ρ‚. Дробная Ρ„ΠΎΡ€ΠΌΠ° прСобразуСтся Π² Π΅Π΅ ΠΊΠΎΡ€Π΅Π½ΡŒ, Π° Π·Π°Ρ‚Π΅ΠΌ вычисляСтся. Π”Π°Π²Π°ΠΉΡ‚Π΅ рассмотрим ΠΏΡ€ΠΈΠΌΠ΅Ρ€: 27 1/3 + 4 1/2 = 3 √27 + √4 = 3 + 2 = 5. Π°Π½Π°Π»ΠΎΠ³ΠΈΡ‡Π½ΠΎ ΠΌΠ΅Ρ‚ΠΎΠ΄Ρƒ слоТСния ΠΏΠΎΠΊΠ°Π·Π°Ρ‚Π΅Π»Π΅ΠΉ стСпСни с ΠΎΠ΄ΠΈΠ½Π°ΠΊΠΎΠ²Ρ‹ΠΌ основаниСм ΠΈ ΠΎΠ΄ΠΈΠ½Π°ΠΊΠΎΠ²Ρ‹ΠΌΠΈ показатСлями стСпСни. ΠžΠ±Ρ‰Π°Ρ Ρ„ΠΎΡ€ΠΌΠ° Ρ… Π½ + Ρ… Π½ = 2x n . Π”Π°Π²Π°ΠΉΡ‚Π΅ посмотрим Π½Π° ΠΏΡ€ΠΈΠΌΠ΅Ρ€, 7 2 + 7 2 = 2(7 2 ) = 2 Γ— 7 Γ— 7 = 98.

    ΠœΠ΅Ρ‚ΠΎΠ΄ 7: Π”ΠΎΠ±Π°Π²Π»Π΅Π½ΠΈΠ΅ ΠΏΠ΅Ρ€Π΅ΠΌΠ΅Π½Π½ΠΎΠΉ с Ρ€Π°Π·Π½Ρ‹ΠΌΠΈ показатСлями стСпСни

    ΠžΠ±Ρ‰Π°Ρ Ρ„ΠΎΡ€ΠΌΠ° для вычислСния ΠΏΠ΅Ρ€Π΅ΠΌΠ΅Π½Π½Ρ‹Ρ… с Ρ€Π°Π·Π½Ρ‹ΠΌΠΈ показатСлями составляСт x n + x m . Π”Π°Π²Π°ΠΉΡ‚Π΅ рассмотрим ΠΏΡ€ΠΈΠΌΠ΅Ρ€: 4 2 + 4 3 = 4 2 Γ— 3 = 4 6 = 4096. ВзглянСм.

    • Как ΠΎΠΏΡ€Π΅Π΄Π΅Π»ΠΈΡ‚ΡŒ стСпСни ΠΈ стСпСни
    • Π­ΠΊΡΠΏΠΎΠ½Π΅Π½Ρ†ΠΈΠ°Π»ΡŒΠ½Ρ‹Π΅ Ρ„ΡƒΠ½ΠΊΡ†ΠΈΠΈ
    • Π˜Ρ€Ρ€Π°Ρ†ΠΈΠΎΠ½Π°Π»ΡŒΠ½Ρ‹Π΅ ΠŸΠΎΠΊΠ°Π·Π°Ρ‚Π΅Π»ΠΈ

    Часто Π·Π°Π΄Π°Π²Π°Π΅ΠΌΡ‹Π΅ вопросы ΠΎ Π΄ΠΎΠ±Π°Π²Π»Π΅Π½ΠΈΠΈ экспонСнтов

    Π§Ρ‚ΠΎ ΠΎΠ·Π½Π°Ρ‡Π°Π΅Ρ‚ Π΄ΠΎΠ±Π°Π²Π»Π΅Π½ΠΈΠ΅ ΠΏΠΎΠΊΠ°Π·Π°Ρ‚Π΅Π»Π΅ΠΉ?

    Π”ΠΎΠ±Π°Π²Π»Π΅Π½ΠΈΠ΅ ΠΏΠΎΠΊΠ°Π·Π°Ρ‚Π΅Π»Π΅ΠΉ стСпСни β€” это процСсс добавлСния Ρ‚Π΅Ρ€ΠΌΠΈΠ½ΠΎΠ², ΠΈΠΌΠ΅ΡŽΡ‰ΠΈΡ… ΠΎΠ΄Π½ΠΎ ΠΈ Ρ‚ΠΎ ΠΆΠ΅ основаниС ΠΈ ΠΏΠΎΠΊΠ°Π·Π°Ρ‚Π΅Π»ΡŒ стСпСни. Π‘Π»ΠΎΠΆΠ΅Π½ΠΈΠ΅ Π²ΠΎΠ·ΠΌΠΎΠΆΠ½ΠΎ Ρ‚ΠΎΠ»ΡŒΠΊΠΎ Π² Ρ‚ΠΎΠΌ случаС, Ссли основаниС ΠΈ ΠΏΠΎΠΊΠ°Π·Π°Ρ‚Π΅Π»ΡŒ стСпСни ΡΠΎΠ²ΠΏΠ°Π΄Π°ΡŽΡ‚. Π•ΡΡ‚ΡŒ случаи, ΠΊΠΎΠ³Π΄Π° это Π½Π΅ Ρ‚Π°ΠΊ, это ΠΌΠΎΠΆΠ½ΠΎ Ρ€Π΅ΡˆΠΈΡ‚ΡŒ, ΡƒΠ²ΠΈΠ΄Π΅Π², ΠΎΠ΄ΠΈΠ½Π°ΠΊΠΎΠ²Ρ‹ Π»ΠΈ ΠΏΠΎΠΊΠ°Π·Π°Ρ‚Π΅Π»ΠΈ Π΄Π²ΡƒΡ… Ρ‚Π΅Ρ€ΠΌΠΈΠ½ΠΎΠ² ΠΈΠ»ΠΈ ΠΎΠ΄ΠΈΠ½Π°ΠΊΠΎΠ²ΠΎΠ΅ основаниС Π΄Π²ΡƒΡ… Ρ‚Π΅Ρ€ΠΌΠΈΠ½ΠΎΠ². Π’ ΠΏΡ€ΠΎΡ‚ΠΈΠ²Π½ΠΎΠΌ случаС Π΄ΠΎΠ±Π°Π²Π»Π΅Π½ΠΈΠ΅ ΠΏΠΎΠΊΠ°Π·Π°Ρ‚Π΅Π»Π΅ΠΉ стСпСни Π½Π΅Π²ΠΎΠ·ΠΌΠΎΠΆΠ½ΠΎ.

    ΠšΠ°ΠΊΠΎΠ²Ρ‹ шаги ΠΏΠΎ добавлСнию ΠΏΠΎΠΊΠ°Π·Π°Ρ‚Π΅Π»Π΅ΠΉ?

    Π”ΠΎΠ±Π°Π²Π»Π΅Π½ΠΈΠ΅ ΠΏΠΎΠΊΠ°Π·Π°Ρ‚Π΅Π»Π΅ΠΉ стСпСни выполняСтся Π² 3 простых шага, Π° ΠΈΠΌΠ΅Π½Π½ΠΎ:

    • ΠŸΡ€ΠΎΠ²Π΅Ρ€ΡŒΡ‚Π΅ Ρ‡Π»Π΅Π½Ρ‹ Π² Π²Ρ‹Ρ€Π°ΠΆΠ΅Π½ΠΈΠΈ, ΠΈΠΌΠ΅ΡŽΡ‚ Π»ΠΈ ΠΎΠ½ΠΈ ΠΎΠ΄Π½ΠΎ ΠΈ Ρ‚ΠΎ ΠΆΠ΅ основаниС ΠΈ ΠΎΠ΄ΠΈΠ½Π°ΠΊΠΎΠ²Ρ‹Π΅ стСпСни.
    • Если основаниС ΠΈ ΠΏΠΎΠΊΠ°Π·Π°Ρ‚Π΅Π»ΠΈ Ρ€Π°Π·Π»ΠΈΡ‡Π½Ρ‹, вычислитС Π²Ρ‹Ρ€Π°ΠΆΠ΅Π½ΠΈΠ΅ с ΠΎΡ‚Π΄Π΅Π»ΡŒΠ½Ρ‹ΠΌΠΈ Ρ‡Π»Π΅Π½Π°ΠΌΠΈ.
    • Π‘Π»ΠΎΠΆΠΈΡ‚Π΅ Ρ€Π΅Π·ΡƒΠ»ΡŒΡ‚Π°Ρ‚Ρ‹ вмСстС.

    Каково ΠΏΡ€Π°Π²ΠΈΠ»ΠΎ добавлСния ΠΏΠΎΠΊΠ°Π·Π°Ρ‚Π΅Π»Π΅ΠΉ?

    НаиболСС Π²Π°ΠΆΠ½Ρ‹ΠΌ ΠΏΡ€Π°Π²ΠΈΠ»ΠΎΠΌ слоТСния ΠΏΠΎΠΊΠ°Π·Π°Ρ‚Π΅Π»Π΅ΠΉ стСпСни являСтся Ρ‚ΠΎ, Ρ‡Ρ‚ΠΎ основаниС ΠΈ ΡΡ‚Π΅ΠΏΠ΅Π½ΡŒ Ρ‡Π»Π΅Π½ΠΎΠ², ΠΊΠΎΡ‚ΠΎΡ€Ρ‹Π΅ ΠΏΠΎΠΌΠ΅Ρ‰Π°ΡŽΡ‚ΡΡ для слоТСния, Π΄ΠΎΠ»ΠΆΠ½Ρ‹ Π±Ρ‹Ρ‚ΡŒ ΠΎΠ΄ΠΈΠ½Π°ΠΊΠΎΠ²Ρ‹ΠΌΠΈ. Если ΠΎΠ½ΠΈ ΠΎΠ΄ΠΈΠ½Π°ΠΊΠΎΠ²Ρ‹Π΅, коэффициСнты Π±ΡƒΠ΄ΡƒΡ‚ ΡΠΊΠ»Π°Π΄Ρ‹Π²Π°Ρ‚ΡŒΡΡ вмСстС, ΠΏΡ€ΠΈ этом основаниС ΠΈ ΠΏΠΎΠΊΠ°Π·Π°Ρ‚Π΅Π»ΡŒ стСпСни ΠΎΠ΄ΠΈΠ½Π°ΠΊΠΎΠ²Ρ‹.

    МоТно Π»ΠΈ ΡΠΊΠ»Π°Π΄Ρ‹Π²Π°Ρ‚ΡŒ числа с Ρ€Π°Π·Π½Ρ‹ΠΌΠΈ показатСлями стСпСни?

    НСт, слоТСниС чисСл с Ρ€Π°Π·Π½Ρ‹ΠΌΠΈ показатСлями стСпСни ΠΏΡ€ΠΈΠΌΠ΅Π½ΠΈΠΌΠΎ, ΠΏΠΎΡΠΊΠΎΠ»ΡŒΠΊΡƒ ΠΏΡ€Π°Π²ΠΈΠ»ΠΎ слоТСния ΠΏΠΎΠΊΠ°Π·Π°Ρ‚Π΅Π»Π΅ΠΉ стСпСни состоит Π² Ρ‚ΠΎΠΌ, Ρ‡Ρ‚ΠΎ основаниС ΠΈ ΠΏΠΎΠΊΠ°Π·Π°Ρ‚Π΅Π»ΡŒ стСпСни Π΄ΠΎΠ»ΠΆΠ½Ρ‹ Π±Ρ‹Ρ‚ΡŒ ΠΎΠ΄ΠΈΠ½Π°ΠΊΠΎΠ²Ρ‹ΠΌΠΈ. Π’ Ρ‚ΠΎ врСмя ΠΊΠ°ΠΊ Ρ‡Ρ‚ΠΎ-Ρ‚ΠΎ основаниС ΠΈ ΠΏΠΎΠΊΠ°Π·Π°Ρ‚Π΅Π»ΡŒ стСпСни ΠΌΠΎΠ³ΡƒΡ‚ Π±Ρ‹Ρ‚ΡŒ Ρ€Π°Π·Π½Ρ‹ΠΌΠΈ, Π½ΠΎ Ρ‚ΠΎΠ»ΡŒΠΊΠΎ ΠΎΠ΄Π½ΠΎ ΠΎΡ‚Π»ΠΈΡ‡ΠΈΠ΅ Π½Π΅ ΠΏΡ€ΠΈΠΌΠ΅Π½ΠΈΠΌΠΎ.

    ДобавляСтС Π»ΠΈ Π²Ρ‹ ΠΏΠΎΠΊΠ°Π·Π°Ρ‚Π΅Π»ΠΈ ΠΏΡ€ΠΈ Π΄ΠΎΠ±Π°Π²Π»Π΅Π½ΠΈΠΈ?

    Для добавлСния ΠΏΠΎΠΊΠ°Π·Π°Ρ‚Π΅Π»Π΅ΠΉ стСпСни основаниС ΠΈ ΠΏΠΎΠΊΠ°Π·Π°Ρ‚Π΅Π»ΡŒ стСпСни Π΄ΠΎΠ»ΠΆΠ½Ρ‹ ΡΠΎΠ²ΠΏΠ°Π΄Π°Ρ‚ΡŒ. ΠšΠΎΡΡ„Ρ„ΠΈΡ†ΠΈΠ΅Π½Ρ‚ ΠΏΠ΅Ρ€Π΅ΠΌΠ΅Π½Π½ΠΎΠΉ добавляСтся, оставляя ΠΏΠΎΠΊΠ°Π·Π°Ρ‚Π΅Π»ΡŒ стСпСни Π½Π΅ΠΈΠ·ΠΌΠ΅Π½Π½Ρ‹ΠΌ. Π’ Π²Ρ‹Ρ€Π°ΠΆΠ΅Π½ΠΈΠ΅ Π΄ΠΎΠ±Π°Π²Π»ΡΡŽΡ‚ΡΡ Ρ‡Π»Π΅Π½Ρ‹ с ΠΎΠ΄ΠΈΠ½Π°ΠΊΠΎΠ²Ρ‹ΠΌΠΈ ΠΏΠ΅Ρ€Π΅ΠΌΠ΅Π½Π½Ρ‹ΠΌΠΈ ΠΈ стСпСнями. Π­Ρ‚ΠΎ ΠΏΡ€Π°Π²ΠΈΠ»ΠΎ ΠΏΡ€ΠΈΠΌΠ΅Π½ΠΈΠΌΠΎ ΠΊΠ°ΠΊ ΠΊ ΡƒΠΌΠ½ΠΎΠΆΠ΅Π½ΠΈΡŽ, Ρ‚Π°ΠΊ ΠΈ ΠΊ дСлСнию.

    Как ΡΠΊΠ»Π°Π΄Ρ‹Π²Π°Ρ‚ΡŒ ΠΎΠ΄Π½ΠΎΡ‡Π»Π΅Π½Ρ‹ с показатСлями стСпСни?

    Бкладывая Π΄Π²Π° ΠΈΠ»ΠΈ Π±ΠΎΠ»Π΅Π΅ ΠΏΠΎΠ΄ΠΎΠ±Π½Ρ‹Ρ… ΠΎΠ΄Π½ΠΎΡ‡Π»Π΅Π½Π°, ΠΌΡ‹ сначала складываСм коэффициСнты, ΠΏΠΎΠΊΠ° ΠΏΠ΅Ρ€Π΅ΠΌΠ΅Π½Π½Ρ‹Π΅ ΠΈ ΠΏΠΎΠΊΠ°Π·Π°Ρ‚Π΅Π»ΠΈ стСпСни ΠΏΠ΅Ρ€Π΅ΠΌΠ΅Π½Π½ΠΎΠΉ ΠΎΠ΄ΠΈΠ½Π°ΠΊΠΎΠ²Ρ‹. ΠœΡ‹ ΠΌΠΎΠΆΠ΅ΠΌ ΠΏΠΎΠ»ΡƒΡ‡ΠΈΡ‚ΡŒ Ρ€Π΅Π·ΡƒΠ»ΡŒΡ‚Π°Ρ‚, добавляя ΠΏΠΎΠ΄ΠΎΠ±Π½Ρ‹Π΅ ΠΎΠ΄Π½ΠΎΡ‡Π»Π΅Π½Ρ‹.

    Π”ΠΎΠ±Π°Π²Π»Π΅Π½ΠΈΠ΅ ΠΏΠΎΠΊΠ°Π·Π°Ρ‚Π΅Π»Π΅ΠΉ – ΠΌΠ΅Ρ‚ΠΎΠ΄Ρ‹ ΠΈ ΠΏΡ€ΠΈΠΌΠ΅Ρ€Ρ‹

    АлгСбра β€” ΠΎΠ΄ΠΈΠ½ ΠΈΠ· основных курсов ΠΌΠ°Ρ‚Π΅ΠΌΠ°Ρ‚ΠΈΠΊΠΈ. Π§Ρ‚ΠΎΠ±Ρ‹ ΠΏΠΎΠ½ΡΡ‚ΡŒ Π°Π»Π³Π΅Π±Ρ€Ρƒ, Π²Π°ΠΆΠ½ΠΎ Π·Π½Π°Ρ‚ΡŒ, ΠΊΠ°ΠΊ ΠΈΡΠΏΠΎΠ»ΡŒΠ·ΠΎΠ²Π°Ρ‚ΡŒ экспонСнты ΠΈ Ρ€Π°Π΄ΠΈΠΊΠ°Π»Ρ‹. Π”ΠΎΠ±Π°Π²Π»Π΅Π½ΠΈΠ΅ ΠΏΠΎΠΊΠ°Π·Π°Ρ‚Π΅Π»Π΅ΠΉ являСтся Ρ‡Π°ΡΡ‚ΡŒΡŽ ΠΏΡ€ΠΎΠ³Ρ€Π°ΠΌΠΌΡ‹ ΠΏΠΎ Π°Π»Π³Π΅Π±Ρ€Π΅, ΠΈ ΠΏΠΎ этой ΠΏΡ€ΠΈΡ‡ΠΈΠ½Π΅ для учащихся Π²Π°ΠΆΠ½ΠΎ ΠΈΠΌΠ΅Ρ‚ΡŒ Π±ΠΎΠ»Π΅Π΅ ΠΏΡ€ΠΎΡ‡Π½ΡƒΡŽ основу Π² ΠΌΠ°Ρ‚Π΅ΠΌΠ°Ρ‚ΠΈΠΊΠ΅.

    МногиС ΡƒΡ‡Π΅Π½ΠΈΠΊΠΈ часто ΠΏΡƒΡ‚Π°ΡŽΡ‚ слоТСниС ΠΏΠΎΠΊΠ°Π·Π°Ρ‚Π΅Π»Π΅ΠΉ стСпСни с слоТСниСм чисСл , ΠΈ, ΡΠ»Π΅Π΄ΠΎΠ²Π°Ρ‚Π΅Π»ΡŒΠ½ΠΎ, Π΄Π΅Π»Π°ΡŽΡ‚ ошибки. Π­Ρ‚ΠΈ нСдоразумСния ΠΎΠ±Ρ‹Ρ‡Π½ΠΎ Π²Π»Π΅ΠΊΡƒΡ‚ Π·Π° собой Ρ€Π°Π·Π½ΠΈΡ†Ρƒ Π² Π·Π½Π°Ρ‡Π΅Π½ΠΈΠΈ Ρ‚Π°ΠΊΠΈΡ… Ρ‚Π΅Ρ€ΠΌΠΈΠ½ΠΎΠ², ΠΊΠ°ΠΊ Π²ΠΎΠ·Π²Π΅Π΄Π΅Π½ΠΈΠ΅ Π² ΡΡ‚Π΅ΠΏΠ΅Π½ΡŒ ΠΈ ΠΏΠΎΠΊΠ°Π·Π°Ρ‚Π΅Π»ΡŒ стСпСни.

    ΠŸΡ€Π΅ΠΆΠ΄Π΅ Ρ‡Π΅ΠΌ ΡƒΠ³Π»ΡƒΠ±Π»ΡΡ‚ΡŒΡΡ Π² совСты ΠΎ Ρ‚ΠΎΠΌ, ΠΊΠ°ΠΊ Π΄ΠΎΠ±Π°Π²Π»ΡΡ‚ΡŒ ΠΏΠΎΠΊΠ°Π·Π°Ρ‚Π΅Π»ΠΈ стСпСни, Π΄Π°Π²Π°ΠΉΡ‚Π΅ Π½Π°Ρ‡Π½Π΅ΠΌ с опрСдСлСния Ρ‚Π΅Ρ€ΠΌΠΈΠ½ΠΎΠ² для ΠΏΠΎΠΊΠ°Π·Π°Ρ‚Π΅Π»Π΅ΠΉ стСпСни. НачнСм с Ρ‚ΠΎΠ³ΠΎ, Ρ‡Ρ‚ΠΎ ΠΏΠΎΠΊΠ°Π·Π°Ρ‚Π΅Π»ΡŒ стСпСни β€” это просто ΠΌΠ½ΠΎΠ³ΠΎΠΊΡ€Π°Ρ‚Π½ΠΎΠ΅ ΡƒΠΌΠ½ΠΎΠΆΠ΅Π½ΠΈΠ΅ числа само Π½Π° сСбя. Π’ ΠΌΠ°Ρ‚Π΅ΠΌΠ°Ρ‚ΠΈΠΊΠ΅ эта опСрация называСтся Π²ΠΎΠ·Π²Π΅Π΄Π΅Π½ΠΈΠ΅ΠΌ Π² ΡΡ‚Π΅ΠΏΠ΅Π½ΡŒ. Π’Π°ΠΊΠΈΠΌ ΠΎΠ±Ρ€Π°Π·ΠΎΠΌ, Π²ΠΎΠ·Π²Π΅Π΄Π΅Π½ΠΈΠ΅ Π² ΡΡ‚Π΅ΠΏΠ΅Π½ΡŒ β€” это опСрация с числами Π² Ρ„ΠΎΡ€ΠΌΠ΅ b n , Π³Π΄Π΅ b называСтся основаниСм, Π° число nΒ β€” ΠΏΠΎΠΊΠ°Π·Π°Ρ‚Π΅Π»Π΅ΠΌ стСпСни, индСксом ΠΈΠ»ΠΈ ΡΡ‚Π΅ΠΏΠ΅Π½ΡŒΡŽ . НапримСр, x 4 содСрТат 4 Π² качСствС показатСля стСпСни, Π° x Π½Π°Π·Ρ‹Π²Π°ΡŽΡ‚ΡΡ основаниСм.

    ЭкспонСнты ΠΈΠ½ΠΎΠ³Π΄Π° Π½Π°Π·Ρ‹Π²Π°ΡŽΡ‚ стСпСнями чисСл. ΠŸΠΎΠΊΠ°Π·Π°Ρ‚Π΅Π»ΡŒ стСпСни прСдставляСт количСство Ρ€Π°Π·, ΠΊΠΎΡ‚ΠΎΡ€ΠΎΠ΅ число Π΄ΠΎΠ»ΠΆΠ½ΠΎ Π±Ρ‹Ρ‚ΡŒ ΡƒΠΌΠ½ΠΎΠΆΠ΅Π½ΠΎ само Π½Π° сСбя. НапримСр, x 4 Β =Β xΒ Γ—Β xΒ Γ—Β xΒ Γ—Β x.

    Π§Ρ‚ΠΎΠ±Ρ‹ Π΄ΠΎΠ±Π°Π²ΠΈΡ‚ΡŒ ΠΏΠΎΠΊΠ°Π·Π°Ρ‚Π΅Π»ΠΈ стСпСни, ΠΈ ΠΏΠΎΠΊΠ°Π·Π°Ρ‚Π΅Π»ΠΈ стСпСни, ΠΈ ΠΏΠ΅Ρ€Π΅ΠΌΠ΅Π½Π½Ρ‹Π΅ Π΄ΠΎΠ»ΠΆΠ½Ρ‹ Π±Ρ‹Ρ‚ΡŒ ΠΎΠ΄ΠΈΠ½Π°ΠΊΠΎΠ²Ρ‹ΠΌΠΈ. Π’Ρ‹ добавляСтС коэффициСнты ΠΏΠ΅Ρ€Π΅ΠΌΠ΅Π½Π½Ρ‹Ρ…, оставляя ΠΏΠΎΠΊΠ°Π·Π°Ρ‚Π΅Π»ΠΈ Π±Π΅Π· ΠΈΠ·ΠΌΠ΅Π½Π΅Π½ΠΈΠΉ. Π”ΠΎΠ±Π°Π²Π»ΡΡŽΡ‚ΡΡ Ρ‚ΠΎΠ»ΡŒΠΊΠΎ Ρ‚Π΅Ρ€ΠΌΠΈΠ½Ρ‹ с ΠΎΠ΄ΠΈΠ½Π°ΠΊΠΎΠ²Ρ‹ΠΌΠΈ ΠΏΠ΅Ρ€Π΅ΠΌΠ΅Π½Π½Ρ‹ΠΌΠΈ ΠΈ стСпСнями. Π­Ρ‚ΠΎ ΠΏΡ€Π°Π²ΠΈΠ»ΠΎ Ρ‚Π°ΠΊΠΆΠ΅ согласуСтся с ΡƒΠΌΠ½ΠΎΠΆΠ΅Π½ΠΈΠ΅ΠΌ ΠΈ Π΄Π΅Π»Π΅Π½ΠΈΠ΅ΠΌ ΠΏΠΎΠΊΠ°Π·Π°Ρ‚Π΅Π»Π΅ΠΉ.

    НиТС ΠΏΡ€ΠΈΠ²Π΅Π΄Π΅Π½Ρ‹ шаги для добавлСния стСпСнСй:

    • ΠŸΡ€ΠΎΠ²Π΅Ρ€ΡŒΡ‚Π΅ Ρ‡Π»Π΅Π½Ρ‹, Ссли ΠΎΠ½ΠΈ ΠΈΠΌΠ΅ΡŽΡ‚ ΠΎΠ΄ΠΈΠ½Π°ΠΊΠΎΠ²Ρ‹Π΅ основания ΠΈ стСпСни

    НапримСр, 4 2 +4 2 , эти Ρ‡Π»Π΅Π½Ρ‹ ΠΈΠΌΠ΅ΡŽΡ‚ ΠΎΠ΄ΠΈΠ½Π°ΠΊΠΎΠ²ΠΎΠ΅ основаниС 4 ΠΈ ΠΏΠΎΠΊΠ°Π·Π°Ρ‚Π΅Π»ΡŒ стСпСни 2.

    • ВычислитС ΠΊΠ°ΠΆΠ΄Ρ‹ΠΉ Ρ‡Π»Π΅Π½ ΠΎΡ‚Π΄Π΅Π»ΡŒΠ½ΠΎ, Ссли ΠΎΠ½ΠΈ ΠΈΠΌΠ΅ΡŽΡ‚ Ρ€Π°Π·Π½ΠΎΠ΅ основаниС ΠΈΠ»ΠΈ ΠΏΠΎΠΊΠ°Π·Π°Ρ‚Π΅Π»ΡŒ стСпСни

    НапримСр, 3 2 + 4 3 , эти Ρ‡Π»Π΅Π½Ρ‹ ΠΈΠΌΠ΅ΡŽΡ‚ Ρ€Π°Π·Π½Ρ‹Π΅ ΠΏΠΎΠΊΠ°Π·Π°Ρ‚Π΅Π»ΠΈ стСпСни ΠΈ основания.

    • Π‘Π»ΠΎΠΆΠΈΡ‚Π΅ Ρ€Π΅Π·ΡƒΠ»ΡŒΡ‚Π°Ρ‚Ρ‹ вмСстС.

    Π‘Π»ΠΎΠΆΠ΅Π½ΠΈΠ΅ ΠΏΠΎΠΊΠ°Π·Π°Ρ‚Π΅Π»Π΅ΠΉ стСпСни с Ρ€Π°Π·Π½Ρ‹ΠΌΠΈ показатСлями стСпСни ΠΈ основаниями

    Π‘Π»ΠΎΠΆΠ΅Π½ΠΈΠ΅ ΠΏΠΎΠΊΠ°Π·Π°Ρ‚Π΅Π»Π΅ΠΉ стСпСни выполняСтся ΠΏΡƒΡ‚Π΅ΠΌ вычислСния сначала ΠΊΠ°ΠΆΠ΄ΠΎΠΉ стСпСни, Π° Π·Π°Ρ‚Π΅ΠΌ добавлСния: ΠžΠ±Ρ‰Π°Ρ Ρ„ΠΎΡ€ΠΌΠ° Ρ‚Π°ΠΊΠΈΡ… ΠΏΠΎΠΊΠ°Π·Π°Ρ‚Π΅Π»Π΅ΠΉ стСпСни: a n Β +Β b m .

    ΠŸΡ€ΠΈΠΌΠ΅Ρ€ 1

    1. 4 2 + 2 5 = 4–4 + 2=2 ОВ 2-2 = 16 + 32 = 48
    2. 8 3 3 3 39. 2 = (8)(8)(8) + (9)(9) = 512 + 81 = 593
    3. 3 2 + 5 3 = (3)(3) + (5)(5)(5) = 9 + 125 = 134
    4. 6 2 + 6 3 9 252.
    5. 3 4 + 3 6 = 81 + 729 = 810.

    Π”ΠΎΠ±Π°Π²Π»Π΅Π½ΠΈΠ΅ экспонСнтов с Ρ‚Π΅ΠΌΠΈ ΠΆΠ΅ основаниями ΠΈ экспонСнтами

    . n

    ΠŸΡ€ΠΈΠΌΠ΅Ρ€ 2

    1. 4 2 + 4 2 = 2β‹…4 2 = 2 % 4 = 32
    2. 8 3 + 8 3 + 8 3 = 3 (8 3 ) = 3 * 512 = 1536
    3. 3 3 3 3 3 3 3 3 3 3 3 3 3 3 3 3 3 3 3 3 3 3 3 ) = 2 = 2(3 2 ) = 2 * 9 = 18
    4. 5 2 + 5 2 = 2(5 2 ) = 2 * 25 = 50. с Ρ€Π°Π·Π½Ρ‹ΠΌΠΈ Π±Π°Π·Π°ΠΌΠΈ?

      Π”ΠΎΠ±Π°Π²Π»Π΅Π½ΠΈΠ΅ ΠΎΡ‚Ρ€ΠΈΡ†Π°Ρ‚Π΅Π»ΡŒΠ½Ρ‹Ρ… ΠΏΠΎΠΊΠ°Π·Π°Ρ‚Π΅Π»Π΅ΠΉ выполняСтся ΠΏΡƒΡ‚Π΅ΠΌ вычислСния ΠΊΠ°ΠΆΠ΄ΠΎΠ³ΠΎ показатСля ΠΎΡ‚Π΄Π΅Π»ΡŒΠ½ΠΎ, Π° Π·Π°Ρ‚Π΅ΠΌ добавлСния:

      a -n Β +Β b -m Β =Β 1/a n Β +Β 1/b m

      Example 3

      4 -2 Β + 2 -5 Β = 1/4 2 Β + 1/2 5 Β = 1/(4β‹…4)+1/(2β‹…2β‹…2β‹…2β‹…2) = 1/16+1/32 = 0,09375

      Как ΡΠ»ΠΎΠΆΠΈΡ‚ΡŒ Π΄Ρ€ΠΎΠ±ΡŒ с Ρ€Π°Π·Π½Ρ‹ΠΌΠΈ основаниями ΠΈ показатСлями?

      Π”ΠΎΠ±Π°Π²Π»Π΅Π½ΠΈΠ΅ Π΄Ρ€ΠΎΠ±Π½Ρ‹Ρ… ΠΏΠΎΠΊΠ°Π·Π°Ρ‚Π΅Π»Π΅ΠΉ стСпСни выполняСтся ΠΏΡƒΡ‚Π΅ΠΌ вычислСния ΠΊΠ°ΠΆΠ΄ΠΎΠ³ΠΎ показатСля стСпСни ΠΎΡ‚Π΄Π΅Π»ΡŒΠ½ΠΎ, Π° Π·Π°Ρ‚Π΅ΠΌ добавлСния:

      a n/m Β +Β b k/j .

      ΠŸΡ€ΠΈΠΌΠ΅Ρ€ 4

      3 3/2 + 2 5/2 = √ (3 3 ) + √ (2 5 ) = √ (27) + √ (32). = 5,196 + 5,657 = 10,853

      Как ΡΠ»ΠΎΠΆΠΈΡ‚ΡŒ Π΄Ρ€ΠΎΠ±Π½Ρ‹Π΅ ΠΏΠΎΠΊΠ°Π·Π°Ρ‚Π΅Π»ΠΈ с ΠΎΠ΄ΠΈΠ½Π°ΠΊΠΎΠ²Ρ‹ΠΌΠΈ основаниями ΠΈ ΠΎΠ΄ΠΈΠ½Π°ΠΊΠΎΠ²Ρ‹ΠΌΠΈ Π΄Ρ€ΠΎΠ±Π½Ρ‹ΠΌΠΈ показатСлями?

      b n/m Β +Β b n/m Β = 2b n/m

      Example 5

      4 2/3 Β + 4 2/3 Β = 2β‹…4 2/3 Β = 2 β‹… 3 √ (4 2 ) = 5,04

      Как ΡΠ»ΠΎΠΆΠΈΡ‚ΡŒ ΠΏΠ΅Ρ€Π΅ΠΌΠ΅Π½Π½Ρ‹Π΅ с Ρ€Π°Π·Π½Ρ‹ΠΌΠΈ показатСлями стСпСни?

      Π”ΠΎΠ±Π°Π²Π»Π΅Π½ΠΈΠ΅ ΠΏΠΎΠΊΠ°Π·Π°Ρ‚Π΅Π»Π΅ΠΉ выполняСтся ΠΏΡƒΡ‚Π΅ΠΌ вычислСния ΠΊΠ°ΠΆΠ΄ΠΎΠ³ΠΎ показатСля ΠΎΡ‚Π΄Π΅Π»ΡŒΠ½ΠΎ, Π° Π·Π°Ρ‚Π΅ΠΌ слоТСния:

      x n Β +Β x m

      Как ΡΠ»ΠΎΠΆΠΈΡ‚ΡŒ ΠΏΠ΅Ρ€Π΅ΠΌΠ΅Π½Π½Ρ‹Π΅ с ΠΎΠ΄ΠΈΠ½Π°ΠΊΠΎΠ²Ρ‹ΠΌΠΈ показатСлями?

      x N +x N = 2x N

      ΠŸΡ€ΠΈΠΌΠ΅Ρ€ 6

      x 2 + x 2 + x 2 + x 2 + x 2 + x 2 + x 2

      10. 0096 x 2

      ΠŸΡ€ΠΈΠΌΠ΅Ρ€ 7

      (4 -1 Β + 8 -1 ) Γ· (2/3) -1 1 900Γ·1 9000

      0 (3/2)

      = (2 + 1)/8 Γ· 3/2

      = (3/8 Γ· 3/2)

      = (3/8 Γ· 2/3)

      = ΒΌ

      ΠŸΡ€ΠΈΠΌΠ΅Ρ€ 8

      Π£ΠΏΡ€ΠΎΡ‰Π΅Π½ΠΈΠ΅: (1/2) -2 Β + (1/3) -2 Β + (1/4) -2
      РСшСниС:
      (1/2) -2 Β + (1/3) -2 Β + (1/4) -2
      = (2/1) 2 Β + (3/1) 2 Β + (4/1) 2
      = (2 2 Β + 3 2 Β + 4 5) 90 90 + 9 + 16)
      = 29

      слоТСниС, Π²Ρ‹Ρ‡ΠΈΡ‚Π°Π½ΠΈΠ΅, ΡƒΠΌΠ½ΠΎΠΆΠ΅Π½ΠΈΠ΅ ΠΈ Π΄Π΅Π»Π΅Π½ΠΈΠ΅

      Π’ΠΎΠ·Π²Π΅Π΄Π΅Π½ΠΈΠ΅ Π² ΡΡ‚Π΅ΠΏΠ΅Π½ΡŒ β€” матСматичСская опСрация с двумя числами: β€” основаниС $x$ ΠΈ β€” ΠΏΠΎΠΊΠ°Π·Π°Ρ‚Π΅Π»ΡŒ стСпСни $a$ . Когда $a$ являСтся ΠΏΠΎΠ»ΠΎΠΆΠΈΡ‚Π΅Π»ΡŒΠ½Ρ‹ΠΌ Ρ†Π΅Π»Ρ‹ΠΌ числом, Π²ΠΎΠ·Π²Π΅Π΄Π΅Π½ΠΈΠ΅ Π² ΡΡ‚Π΅ΠΏΠ΅Π½ΡŒ соотвСтствуСт -ΠΊΡ€Π°Ρ‚Π½ΠΎΠΌΡƒ ΡƒΠΌΠ½ΠΎΠΆΠ΅Π½ΠΈΡŽ основания . 93$. Π­Ρ‚ΠΎ ΠΎΠ·Π½Π°Ρ‡Π°Π΅Ρ‚, Ρ‡Ρ‚ΠΎ ΠΌΡ‹ Ρ‚Ρ€ΠΈΠΆΠ΄Ρ‹ ΡƒΠΌΠ½ΠΎΠΆΠΈΠ»ΠΈ число $2$ само Π½Π° сСбя ΠΈΠ»ΠΈ: $2\cdot{2}\cdot{2}$, Ρ‡Ρ‚ΠΎ равняСтся $8$.

      Β 

      По ΠΎΠΏΡ€Π΅Π΄Π΅Π»Π΅Π½ΠΈΡŽ ΠΊΠ°ΠΆΠ΄ΠΎΠ΅ число, ΠΈΠΌΠ΅ΡŽΡ‰Π΅Π΅ Π² качСствС показатСля стСпСни 0, Ρ€Π°Π²Π½ΠΎ 1. Π­Ρ‚ΠΎ ΠΎΠ·Π½Π°Ρ‡Π°Π΅Ρ‚, Ρ‡Ρ‚ΠΎ нСзависимо ΠΎΡ‚ Ρ‚ΠΎΠ³ΠΎ, насколько Π²Π΅Π»ΠΈΠΊΠΎ основаниС, Ссли ΠΈΡ… ΠΏΠΎΠΊΠ°Π·Π°Ρ‚Π΅Π»ΡŒ стСпСни Ρ€Π°Π²Π΅Π½ 0, это число всСгда Ρ€Π°Π²Π½ΠΎ 1.

      КаТдоС число, ΠΊ ΠΊΠΎΡ‚ΠΎΡ€ΠΎΠΌΡƒ Π½Π΅ присоСдинСн ΠΏΠΎΠΊΠ°Π·Π°Ρ‚Π΅Π»ΡŒ стСпСни, Π½Π° самом Π΄Π΅Π»Π΅ ΠΈΠΌΠ΅Π΅Ρ‚ число 1 Π² качСствС показатСля стСпСни. Число 1 являСтся ΠΏΠΎΠΊΠ°Π·Π°Ρ‚Π΅Π»Π΅ΠΌ стСпСни ΠΊΠ°ΠΆΠ΄ΠΎΠ³ΠΎ числа ΠΏΠΎ ΡƒΠΌΠΎΠ»Ρ‡Π°Π½ΠΈΡŽ, поэтому Π΅Π³ΠΎ Π½Π΅ ΠΎΠ±ΡΠ·Π°Ρ‚Π΅Π»ΡŒΠ½ΠΎ Π·Π°ΠΏΠΈΡΡ‹Π²Π°Ρ‚ΡŒ, Π½ΠΎ Π² Π½Π΅ΠΊΠΎΡ‚ΠΎΡ€Ρ‹Ρ… Π·Π°Π΄Π°Ρ‡Π°Ρ… это ΠΌΠΎΠΆΠ΅Ρ‚ Π±Ρ‹Ρ‚ΡŒ ΠΏΠΎΠ»Π΅Π·Π½ΠΎ. 94 $

      Π ΠΠ‘ΠžΠ’Π« Π”Π•Π™Π‘Π’Π’Π˜Π’Π•Π›Π¬ΠΠ«Π• ΠŸΠ•Π Π•Π”ΠΠ§Π˜

      Π‘Π’ΠžΠ™Π‘Π’Π’Π Π­ΠšΠ‘ΠŸΠžΠΠ•Π Π‘. (166,4 ΠšΠΈΠ‘, 1976 совпадСний)

      Β Β  Π­ΠΊΡΠΏΠΎΠ½Π΅Π½Ρ†ΠΈΠ°Π»ΡŒΠ½ΠΎΠ΅ прСдставлСниС β€” запись Π² стандартной Π½ΠΎΡ‚Π°Ρ†ΠΈΠΈ (187,0 ΠšΠΈΠ‘, 1740 совпадСний)

      ΠžΠΏΠ΅Ρ€Π°Ρ†ΠΈΠΈ с показатСлями стСпСни

      Β Β  Π£ΠΌΠ½ΠΎΠΆΠ΅Π½ΠΈΠ΅ (195,3 ΠΊΠΈΠ±, 2281 Ρ…ΠΈΡ‚Ρ‹)

      Π”ΠΈΠ²ΠΈΠ·ΠΈΠΎΠ½ (197,0 ΠΊΠΈΠ±, 1993 Ρ…ΠΈΡ‚Ρ‹)

      ΠŸΠΎΠ²Ρ‹ΡˆΠ΅Π½Π½Ρ‹Π΅ Π΄ΠΎ власти (174,1 ΠΊΠΈΠ±, 2123 Π₯ΠΈΡ‚Ρ‹)

      ЭкспонСнты.

      Π£ΠΌΠ½ΠΎΠΆΠ΅Π½ΠΈΠ΅

      ОбновлСно 14 дСкабря 2020 Π³.

      Π›ΠΈ ДТонсон

      Π’Ρ‹ΠΏΠΎΠ»Π½Π΅Π½ΠΈΠ΅ вычислСний ΠΈ Ρ€Π°Π±ΠΎΡ‚Π° с показатСлями стСпСни ΡΠ²Π»ΡΡŽΡ‚ΡΡ Π²Π°ΠΆΠ½ΠΎΠΉ Ρ‡Π°ΡΡ‚ΡŒΡŽ ΠΌΠ°Ρ‚Π΅ΠΌΠ°Ρ‚ΠΈΠΊΠΈ высокого уровня. Π₯отя выраТСния, Π²ΠΊΠ»ΡŽΡ‡Π°ΡŽΡ‰ΠΈΠ΅ нСсколько стСпСнСй, ΠΎΡ‚Ρ€ΠΈΡ†Π°Ρ‚Π΅Π»ΡŒΠ½Ρ‹Π΅ стСпСни ΠΈ Ρ‚. Π΄., ΠΌΠΎΠ³ΡƒΡ‚ ΠΏΠΎΠΊΠ°Π·Π°Ρ‚ΡŒΡΡ ΠΎΡ‡Π΅Π½ΡŒ Π·Π°ΠΏΡƒΡ‚Π°Π½Π½Ρ‹ΠΌΠΈ, всС, Ρ‡Ρ‚ΠΎ Π²Π°ΠΌ Π½ΡƒΠΆΠ½ΠΎ ΡΠ΄Π΅Π»Π°Ρ‚ΡŒ для Ρ€Π°Π±ΠΎΡ‚Ρ‹ с Π½ΠΈΠΌΠΈ, ΠΌΠΎΠΆΠ½ΠΎ ΡΡƒΠΌΠΌΠΈΡ€ΠΎΠ²Π°Ρ‚ΡŒ с ΠΏΠΎΠΌΠΎΡ‰ΡŒΡŽ Π½Π΅ΡΠΊΠΎΠ»ΡŒΠΊΠΈΡ… простых ΠΏΡ€Π°Π²ΠΈΠ». Π£Π·Π½Π°ΠΉΡ‚Π΅, ΠΊΠ°ΠΊ ΡΠΊΠ»Π°Π΄Ρ‹Π²Π°Ρ‚ΡŒ, Π²Ρ‹Ρ‡ΠΈΡ‚Π°Ρ‚ΡŒ, ΡƒΠΌΠ½ΠΎΠΆΠ°Ρ‚ΡŒ ΠΈ Π΄Π΅Π»ΠΈΡ‚ΡŒ числа с показатСлями стСпСни ΠΈ ΠΊΠ°ΠΊ ΡƒΠΏΡ€ΠΎΡΡ‚ΠΈΡ‚ΡŒ Π»ΡŽΠ±Ρ‹Π΅ выраТСния, Π² ΠΊΠΎΡ‚ΠΎΡ€Ρ‹Ρ… ΠΎΠ½ΠΈ ΠΈΡΠΏΠΎΠ»ΡŒΠ·ΡƒΡŽΡ‚ΡΡ, ΠΈ Π²Ρ‹ почувствуСтС сСбя Π½Π°ΠΌΠ½ΠΎΠ³ΠΎ Π±ΠΎΠ»Π΅Π΅ ΠΊΠΎΠΌΡ„ΠΎΡ€Ρ‚Π½ΠΎ, Ρ€Π΅ΡˆΠ°Ρ Π·Π°Π΄Π°Ρ‡ΠΈ с показатСлями стСпСни.

      TL;DR (слишком Π΄Π»ΠΈΠ½Π½Ρ‹ΠΉ; Π½Π΅ Ρ‡ΠΈΡ‚Π°Π»)

      Π£ΠΌΠ½ΠΎΠΆΡŒΡ‚Π΅ Π΄Π²Π° числа с показатСлями стСпСни ΠΏΡƒΡ‚Π΅ΠΌ слоТСния ΠΏΠΎΠΊΠ°Π·Π°Ρ‚Π΅Π»Π΅ΠΉ стСпСни: + N

      Π Π°Π·Π΄Π΅Π»ΠΈΡ‚Π΅ Π΄Π²Π° числа с показатСлями, вычитая ΠΎΠ΄ΠΈΠ½ ΠΏΠΎΠΊΠ°Π·Π°Ρ‚Π΅Π»ΡŒ ΠΈΠ· ΠΎΡΡ‚Π°Π»ΡŒΠ½Ρ‹Ρ…: x M Γ· x N = Γ· x N = x x N = x M N = x M N = X M N . ​ βˆ’ ​ ΠΏ ​

      Когда экспонСнт ΠΏΠΎΠ²Ρ‹ΡˆΠ°Π΅Ρ‚ΡΡ Π½Π° ΠΌΠΎΡ‰Π½ΠΎΡΡ‚ΡŒ, ΡƒΠΌΠ½ΠΎΠΆΡŒΡ‚Π΅ экспонСнты вмСстС: ( x Y ) Z = x Y Γ—

      6969649634 Γ— 9696969634 Γ—

      69696969997 Γ— 9696969997 Γ— 969696999797 Γ—

      696969 . z ​

      Π›ΡŽΠ±ΠΎΠ΅ число, Π²ΠΎΠ·Π²Π΅Π΄Π΅Π½Π½ΠΎΠ΅ Π² Π½ΡƒΠ»Π΅Π²ΡƒΡŽ ΡΡ‚Π΅ΠΏΠ΅Π½ΡŒ, Ρ€Π°Π²Π½ΠΎ Π΅Π΄ΠΈΠ½ΠΈΡ†Π΅: ​ x ​ 0 = 1

      Π§Ρ‚ΠΎ Ρ‚Π°ΠΊΠΎΠ΅ экспонСнта?

      ΠŸΠΎΠΊΠ°Π·Π°Ρ‚Π΅Π»ΡŒ стСпСни относится ΠΊ числу, Π² ΡΡ‚Π΅ΠΏΠ΅Π½ΡŒ ΠΊΠΎΡ‚ΠΎΡ€ΠΎΠ³ΠΎ Ρ‡Ρ‚ΠΎ-Ρ‚ΠΎ возводится. НапримСр, 94 = x Γ— x Γ— x Γ— x

      ΠŸΠΎΠΊΠ°Π·Π°Ρ‚Π΅Π»ΠΈ Ρ‚Π°ΠΊΠΆΠ΅ ΠΌΠΎΠ³ΡƒΡ‚ Π±Ρ‹Ρ‚ΡŒ ΠΏΠ΅Ρ€Π΅ΠΌΠ΅Π½Π½Ρ‹ΠΌΠΈ; Π½Π°ΠΏΡ€ΠΈΠΌΠ΅Ρ€, 4 x прСдставляСт Ρ‡Π΅Ρ‚Ρ‹Ρ€Π΅, ΡƒΠΌΠ½ΠΎΠΆΠ΅Π½Π½Ρ‹Π΅ Π½Π° сСбя x Ρ€Π°Π·.

      ΠŸΡ€Π°Π²ΠΈΠ»Π° для ΠΏΠΎΠΊΠ°Π·Π°Ρ‚Π΅Π»Π΅ΠΉ стСпСни

      Π’Ρ‹ΠΏΠΎΠ»Π½Π΅Π½ΠΈΠ΅ вычислСний с показатСлями стСпСни Ρ‚Ρ€Π΅Π±ΡƒΠ΅Ρ‚ понимания основных ΠΏΡ€Π°Π²ΠΈΠ», Ρ€Π΅Π³ΡƒΠ»ΠΈΡ€ΡƒΡŽΡ‰ΠΈΡ… ΠΈΡ… использованиС. Π’Π°ΠΌ Π½ΡƒΠΆΠ½ΠΎ ΠΏΠΎΠ΄ΡƒΠΌΠ°Ρ‚ΡŒ ΠΎ Ρ‡Π΅Ρ‚Ρ‹Ρ€Π΅Ρ… основных Π²Π΅Ρ‰Π°Ρ…: слоТСнии, Π²Ρ‹Ρ‡ΠΈΡ‚Π°Π½ΠΈΠΈ, ΡƒΠΌΠ½ΠΎΠΆΠ΅Π½ΠΈΠΈ ΠΈ Π΄Π΅Π»Π΅Π½ΠΈΠΈ.

      Π‘Π»ΠΎΠΆΠ΅Π½ΠΈΠ΅ ΠΈ Π²Ρ‹Ρ‡ΠΈΡ‚Π°Π½ΠΈΠ΅ стСпСнСй

      Π‘Π»ΠΎΠΆΠ΅Π½ΠΈΠ΅ ΠΏΠΎΠΊΠ°Π·Π°Ρ‚Π΅Π»Π΅ΠΉ стСпСни ΠΈ Π²Ρ‹Ρ‡ΠΈΡ‚Π°Π½ΠΈΠ΅ ΠΏΠΎΠΊΠ°Π·Π°Ρ‚Π΅Π»Π΅ΠΉ стСпСни Π½Π° самом Π΄Π΅Π»Π΅ Π½Π΅ Ρ‚Ρ€Π΅Π±ΡƒΠ΅Ρ‚ ΠΏΡ€Π°Π²ΠΈΠ»Π°. Если число Π²ΠΎΠ·Π²Π΅Π΄Π΅Π½ΠΎ Π² ΡΡ‚Π΅ΠΏΠ΅Π½ΡŒ, Π΄ΠΎΠ±Π°Π²ΡŒΡ‚Π΅ Π΅Π³ΠΎ ΠΊ Π΄Ρ€ΡƒΠ³ΠΎΠΌΡƒ числу, Π²ΠΎΠ·Π²Π΅Π΄Π΅Π½Π½ΠΎΠΌΡƒ Π² ΡΡ‚Π΅ΠΏΠ΅Π½ΡŒ (Π»ΠΈΠ±ΠΎ с Π΄Ρ€ΡƒΠ³ΠΈΠΌ основаниСм, Π»ΠΈΠ±ΠΎ с Π΄Ρ€ΡƒΠ³ΠΈΠΌ ΠΏΠΎΠΊΠ°Π·Π°Ρ‚Π΅Π»Π΅ΠΌ стСпСни), вычислив Ρ€Π΅Π·ΡƒΠ»ΡŒΡ‚Π°Ρ‚ Ρ‡Π»Π΅Π½Π° стСпСни, Π° Π·Π°Ρ‚Π΅ΠΌ Π½Π°ΠΏΡ€ΡΠΌΡƒΡŽ Π΄ΠΎΠ±Π°Π²ΠΈΠ² Π΅Π³ΠΎ ΠΊ Π΄Ρ€ΡƒΠ³ΠΎΠΌΡƒ. Когда Π²Ρ‹ Π²Ρ‹Ρ‡ΠΈΡ‚Π°Π΅Ρ‚Π΅ стСпСни, примСняСтся Ρ‚ΠΎΡ‚ ΠΆΠ΅ Π²Ρ‹Π²ΠΎΠ΄: просто вычислитС Ρ€Π΅Π·ΡƒΠ»ΡŒΡ‚Π°Ρ‚, Ссли ΠΌΠΎΠΆΠ΅Ρ‚Π΅, Π° Π·Π°Ρ‚Π΅ΠΌ Π²Ρ‹ΠΏΠΎΠ»Π½ΠΈΡ‚Π΅ Π²Ρ‹Ρ‡ΠΈΡ‚Π°Π½ΠΈΠ΅ ΠΊΠ°ΠΊ ΠΎΠ±Ρ‹Ρ‡Π½ΠΎ. Если ΠΈ ΠΏΠΎΠΊΠ°Π·Π°Ρ‚Π΅Π»ΠΈ стСпСни, ΠΈ основания ΡΠΎΠ²ΠΏΠ°Π΄Π°ΡŽΡ‚, Π²Ρ‹ ΠΌΠΎΠΆΠ΅Ρ‚Π΅ ΡΠΊΠ»Π°Π΄Ρ‹Π²Π°Ρ‚ΡŒ ΠΈ Π²Ρ‹Ρ‡ΠΈΡ‚Π°Ρ‚ΡŒ ΠΈΡ…, ΠΊΠ°ΠΊ Π»ΡŽΠ±Ρ‹Π΅ Π΄Ρ€ΡƒΠ³ΠΈΠ΅ ΡΠΎΠ²ΠΏΠ°Π΄Π°ΡŽΡ‰ΠΈΠ΅ символы Π² Π°Π»Π³Π΅Π±Ρ€Π΅. НапримСр: 9{10} \end{aligned}

      Π‘Π»ΠΎΠΆΠ΅Π½ΠΈΠ΅ ΠΈ Π²Ρ‹Ρ‡ΠΈΡ‚Π°Π½ΠΈΠ΅ ΠΏΠΎΠΊΠ°Π·Π°Ρ‚Π΅Π»Π΅ΠΉ стСпСни — GeeksforGeeks

      Π’ ΠΌΠ°Ρ‚Π΅ΠΌΠ°Ρ‚ΠΈΠΊΠ΅ ΠΏΠΎΠΊΠ°Π·Π°Ρ‚Π΅Π»ΠΈ стСпСни ΠΈΡΠΏΠΎΠ»ΡŒΠ·ΡƒΡŽΡ‚ΡΡ для прСдставлСния большСго числа с Ρ‚ΠΎΡ‡ΠΊΠΈ зрСния мощности. Он ΠΏΠΎΠΊΠ°Π·Ρ‹Π²Π°Π΅Ρ‚, Π²ΠΎ сколько Ρ€Π°Π· основаниС умноТаСтся само Π½Π° сСбя. Π“Π΄Π΅ основаниСм являСтся любоС число ΠΈΠ»ΠΈ любоС матСматичСскоС Π²Ρ‹Ρ€Π°ΠΆΠ΅Π½ΠΈΠ΅. НапримСр, A 3 здСсь основаниС Ρ€Π°Π²Π½ΠΎ A, Π° ΡΡ‚Π΅ΠΏΠ΅Π½ΡŒ Ρ€Π°Π²Π½Π° 3, Ρ‡Ρ‚ΠΎ ΠΎΠ·Π½Π°Ρ‡Π°Π΅Ρ‚, Ρ‡Ρ‚ΠΎ A умноТится само Π½Π° сСбя Ρ‚Ρ€ΠΈ Ρ€Π°Π·Π°, Ρ‚ΠΎ Π΅ΡΡ‚ΡŒ A 3 = A x A x A. ΠžΠ±Ρ‰ΠΈΠΉ Ρ‡Π»Π΅Π½ показатСля стСпСни Ρ€Π°Π²Π΅Π½

      Y n = Y Γ— Y Γ— Y ×………n Ρ€Π°Π·

      Π—Π΄Π΅ΡΡŒ y называСтся основаниСм, Π° n β€” ΡΡ‚Π΅ΠΏΠ΅Π½ΡŒΡŽ ΠΈΠ»ΠΈ ΠΏΠΎΠΊΠ°Π·Π°Ρ‚Π΅Π»Π΅ΠΌ стСпСни. Π’ΠΈΠΏΡ‹ ΠΏΠΎΠΊΠ°Π·Π°Ρ‚Π΅Π»Π΅ΠΉ стСпСни:

      • ΠžΡ‚Ρ€ΠΈΡ†Π°Ρ‚Π΅Π»ΡŒΠ½Π°Ρ ΡΡ‚Π΅ΠΏΠ΅Π½ΡŒ: ΠžΡ‚Ρ€ΠΈΡ†Π°Ρ‚Π΅Π»ΡŒΠ½Ρ‹Π΅ стСпСни — это Ρ‚Π΅ стСпСни, ΠΊΠΎΡ‚ΠΎΡ€Ρ‹Π΅ ΠΏΠΎΠΊΠ°Π·Ρ‹Π²Π°ΡŽΡ‚, Π²ΠΎ сколько Ρ€Π°Π· обратная Π²Π΅Π»ΠΈΡ‡ΠΈΠ½Π° основания умноТаСтся сама Π½Π° сСбя. Он прСдставлСн ΠΊΠ°ΠΊ -n ΠΈΠ»ΠΈ 1/a n . НапримСр, 23 -2 , 4 -2 .
      • Π”Ρ€ΠΎΠ±Π½Ρ‹ΠΉ ΠΏΠΎΠΊΠ°Π·Π°Ρ‚Π΅Π»ΡŒ стСпСни: Когда ΠΏΠΎΠΊΠ°Π·Π°Ρ‚Π΅Π»ΡŒ стСпСни прСдставлСн Π² Π²ΠΈΠ΄Π΅ Π΄Ρ€ΠΎΠ±ΠΈ, Ρ‚Π°ΠΊΠΈΠ΅ Ρ‚ΠΈΠΏΡ‹ ΠΏΠΎΠΊΠ°Π·Π°Ρ‚Π΅Π»Π΅ΠΉ Π½Π°Π·Ρ‹Π²Π°ΡŽΡ‚ΡΡ Π΄Ρ€ΠΎΠ±Π½Ρ‹ΠΌΠΈ показатСлями. Он прСдставлСн ΠΊΠ°ΠΊ 1/Π½ . НапримСр, 3 1/2 , 4 1/3 .
      • ДСсятичный ΠΏΠΎΠΊΠ°Π·Π°Ρ‚Π΅Π»ΡŒ стСпСни: Когда ΠΏΠΎΠΊΠ°Π·Π°Ρ‚Π΅Π»ΡŒ стСпСни прСдставлСн дСсятичными Ρ†ΠΈΡ„Ρ€Π°ΠΌΠΈ, Ρ‚Π°ΠΊΠΈΠ΅ Ρ‚ΠΈΠΏΡ‹ ΠΏΠΎΠΊΠ°Π·Π°Ρ‚Π΅Π»Π΅ΠΉ Π½Π°Π·Ρ‹Π²Π°ΡŽΡ‚ΡΡ дСсятичными показатСлями стСпСни. Он прСдставлСн ΠΊΠ°ΠΊ 1.3 . НапримСр, 3 1,5 , 4 12,3 .

      Π‘Π»ΠΎΠΆΠ΅Π½ΠΈΠ΅ ΠΈ Π²Ρ‹Ρ‡ΠΈΡ‚Π°Π½ΠΈΠ΅ ΠΏΠΎΠΊΠ°Π·Π°Ρ‚Π΅Π»Π΅ΠΉ стСпСни

      Π‘Π»ΠΎΠΆΠ΅Π½ΠΈΠ΅ ΠΈ Π²Ρ‹Ρ‡ΠΈΡ‚Π°Π½ΠΈΠ΅ β€” Π΄Π²Π΅ основныС матСматичСскиС ΠΎΠΏΠ΅Ρ€Π°Ρ†ΠΈΠΈ. Π‘Π»ΠΎΠΆΠ΅Π½ΠΈΠ΅ ΠΎΠ·Π½Π°Ρ‡Π°Π΅Ρ‚ Π½Π°Ρ…ΠΎΠΆΠ΄Π΅Π½ΠΈΠ΅ суммы Π΄Π²ΡƒΡ… Ρ†ΠΈΡ„Ρ€, Π° Π²Ρ‹Ρ‡ΠΈΡ‚Π°Π½ΠΈΠ΅ β€” Π½Π°Ρ…ΠΎΠΆΠ΄Π΅Π½ΠΈΠ΅ Ρ€Π°Π·Π½ΠΈΡ†Ρ‹ ΠΌΠ΅ΠΆΠ΄Ρƒ двумя Ρ†ΠΈΡ„Ρ€Π°ΠΌΠΈ. Но ΠΌΡ‹ Π½Π΅ ΠΌΠΎΠΆΠ΅ΠΌ Π½Π°ΠΏΡ€ΡΠΌΡƒΡŽ ΡΠΊΠ»Π°Π΄Ρ‹Π²Π°Ρ‚ΡŒ ΠΈΠ»ΠΈ Π²Ρ‹Ρ‡ΠΈΡ‚Π°Ρ‚ΡŒ ΠΏΠΎΠΊΠ°Π·Π°Ρ‚Π΅Π»ΠΈ стСпСни, ΠΌΡ‹ ΠΌΠΎΠΆΠ΅ΠΌ Π²Ρ‹ΠΏΠΎΠ»Π½ΡΡ‚ΡŒ слоТСниС ΠΈΠ»ΠΈ Π²Ρ‹Ρ‡ΠΈΡ‚Π°Π½ΠΈΠ΅ Ρ‚ΠΎΠ»ΡŒΠΊΠΎ с коэффициСнтами ΠΈΠ»ΠΈ ΠΏΠ΅Ρ€Π΅ΠΌΠ΅Π½Π½Ρ‹ΠΌΠΈ, ΠΈΠΌΠ΅ΡŽΡ‰ΠΈΠΌΠΈ ΠΎΠ΄ΠΈΠ½Π°ΠΊΠΎΠ²ΠΎΠ΅ основаниС ΠΈ ΠΎΠ΄ΠΈΠ½Π°ΠΊΠΎΠ²ΡƒΡŽ ΠΌΠΎΡ‰Π½ΠΎΡΡ‚ΡŒ. ΠœΡ‹ ΠΌΠΎΠΆΠ΅ΠΌ Ρ‚ΠΎΠ»ΡŒΠΊΠΎ ΡΠΊΠ»Π°Π΄Ρ‹Π²Π°Ρ‚ΡŒ ΠΏΠΎΠΊΠ°Π·Π°Ρ‚Π΅Π»ΠΈ стСпСни ΠΏΡ€ΠΈ ΡƒΠΌΠ½ΠΎΠΆΠ΅Π½ΠΈΠΈ ΠΈ Π²Ρ‹Ρ‡ΠΈΡ‚Π°Ρ‚ΡŒ стСпСни стСпСни ΠΏΡ€ΠΈ Π΄Π΅Π»Π΅Π½ΠΈΠΈ.

      Π¨Π°Π³ΠΈ для слоТСния ΠΈΠ»ΠΈ вычитания ΠΌΠ΅ΠΆΠ΄Ρƒ показатСлями стСпСни Π² Π°Π»Π³Π΅Π±Ρ€Π΅:

      Π§Ρ‚ΠΎΠ±Ρ‹ Π²Ρ‹ΠΏΠΎΠ»Π½ΠΈΡ‚ΡŒ слоТСниС ΠΈΠ»ΠΈ Π²Ρ‹Ρ‡ΠΈΡ‚Π°Π½ΠΈΠ΅, ΠΌΡ‹ Π±ΡƒΠ΄Π΅ΠΌ ΡΠ»Π΅Π΄ΠΎΠ²Π°Ρ‚ΡŒ Ρ‚Π΅ΠΌ ΠΆΠ΅ шагам:

      Π¨Π°Π³ 1: основаниС ΠΈ ΠΏΠΎΠΊΠ°Π·Π°Ρ‚Π΅Π»ΠΈ ΡΠΎΠ²ΠΏΠ°Π΄Π°ΡŽΡ‚ ΠΈΠ»ΠΈ Π½Π΅Ρ‚.

      Π¨Π°Π³ 2: РасполоТитС ΠΏΠΎΡ…ΠΎΠΆΠΈΠ΅ ΠΏΠ΅Ρ€Π΅ΠΌΠ΅Π½Π½Ρ‹Π΅/Ρ‡Π»Π΅Π½Ρ‹ вмСстС.

      Π¨Π°Π³ 3: Π’Π΅ΠΏΠ΅Ρ€ΡŒ ΠΏΡ€ΠΈ нСобходимости Π²Ρ‹ΠΏΠΎΠ»Π½ΠΈΡ‚Π΅ слоТСниС ΠΈΠ»ΠΈ Π²Ρ‹Ρ‡ΠΈΡ‚Π°Π½ΠΈΠ΅ ΠΌΠ΅ΠΆΠ΄Ρƒ коэффициСнтами Ρ‡Π»Π΅Π½ΠΎΠ².

      ΠŸΡ€ΠΈΠΌΠ΅Ρ€ 1: Ρ€Π΅ΡˆΠΈΡ‚ΡŒ 6x 3 + 12x 3 .

      РСшСниС:

      Π—Π΄Π΅ΡΡŒ основаниС ΠΎΠ΄ΠΈΠ½Π°ΠΊΠΎΠ²ΠΎΠ΅, Ρ‚. Π΅. x ΠΈ ΠΏΠΎΠΊΠ°Π·Π°Ρ‚Π΅Π»ΠΈ стСпСни Π΄Π²ΡƒΡ… слагаСмых Ρ‚Π°ΠΊΠΆΠ΅ ΠΎΠ΄ΠΈΠ½Π°ΠΊΠΎΠ²Ρ‹, Ρ‚. Π΅. 3

      Π’Π°ΠΊΠΈΠΌ ΠΎΠ±Ρ€Π°Π·ΠΎΠΌ, ΠΌΡ‹ ΠΌΠΎΠΆΠ΅ΠΌ ΡΠ»ΠΎΠΆΠΈΡ‚ΡŒ коэффициСнты Π΄Π²ΡƒΡ… слагаСмых, Ρ‡Ρ‚ΠΎΠ±Ρ‹ ΠΏΠΎΠ»ΡƒΡ‡ΠΈΡ‚ΡŒ Ρ€Π΅Π·ΡƒΠ»ΡŒΡ‚Π°Ρ‚.

      6x 3 + 12x 3 = (6 + 12) x 3

      = 18x 3

      . ΠŸΡ€ΠΈΠΌΠ΅Ρ€ 2: Π‘ΠΎΡ€Π΅Π² 3 -135.13x 3.

      РСшСниС:

      Π—Π΄Π΅ΡΡŒ основаниС ΠΎΠ΄ΠΈΠ½Π°ΠΊΠΎΠ²ΠΎΠ΅, Ρ‚. Π΅. x ΠΈ ΠΏΠΎΠΊΠ°Π·Π°Ρ‚Π΅Π»ΠΈ стСпСни Π΄Π²ΡƒΡ… слагаСмых Ρ‚Π°ΠΊΠΆΠ΅ ΠΎΠ΄ΠΈΠ½Π°ΠΊΠΎΠ²Ρ‹, Ρ‚. Π΅. 3

      Π’Π°ΠΊΠΈΠΌ ΠΎΠ±Ρ€Π°Π·ΠΎΠΌ, ΠΌΡ‹ ΠΌΠΎΠΆΠ΅ΠΌ Π²Ρ‹Ρ‡Π΅ΡΡ‚ΡŒ коэффициСнты Π΄Π²ΡƒΡ… слагаСмых, Ρ‡Ρ‚ΠΎΠ±Ρ‹ ΠΏΠΎΠ»ΡƒΡ‡ΠΈΡ‚ΡŒ Ρ€Π΅Π·ΡƒΠ»ΡŒΡ‚Π°Ρ‚.

      9x 3 -13x 3 = (9 -13) x 3

      = -4x 3

      Π¨Π°Π³ΠΈ для выполнСния ΠΈΠ»ΠΈ субтракции ΠΌΠ΅ΠΆΠ΄Ρƒ эксплу слоТСниС ΠΈΠ»ΠΈ Π²Ρ‹Ρ‡ΠΈΡ‚Π°Π½ΠΈΠ΅ ΠΌΡ‹ Π±ΡƒΠ΄Π΅ΠΌ ΡΠ»Π΅Π΄ΠΎΠ²Π°Ρ‚ΡŒ Ρ‚Π΅ΠΌ ΠΆΠ΅ шагам:

      Π¨Π°Π³ 1: ΠŸΠ΅Ρ€Π΅Π΄ Π²Ρ‹ΠΏΠΎΠ»Π½Π΅Π½ΠΈΠ΅ΠΌ любого слоТСния ΠΈΠ»ΠΈ вычитания ΠΌΠ΅ΠΆΠ΄Ρƒ показатСлями стСпСни ΠΌΡ‹ Π΄ΠΎΠ»ΠΆΠ½Ρ‹ Π½Π°Π±Π»ΡŽΠ΄Π°Ρ‚ΡŒ, ΡΠΎΠ²ΠΏΠ°Π΄Π°ΡŽΡ‚ Π»ΠΈ основаниС ΠΈ ΠΏΠΎΠΊΠ°Π·Π°Ρ‚Π΅Π»ΠΈ стСпСни.

      Π¨Π°Π³ 2: РасполоТитС ΠΎΠ΄ΠΈΠ½Π°ΠΊΠΎΠ²Ρ‹Π΅ Ρ‡Π»Π΅Π½Ρ‹ основания ΠΈ стСпСни вмСстС. Если Ρ‡Π»Π΅Π½Ρ‹ ΠΈΠΌΠ΅ΡŽΡ‚ Ρ€Π°Π·Π½Ρ‹Π΅ основаниС ΠΈ ΠΏΠΎΠΊΠ°Π·Π°Ρ‚Π΅Π»ΡŒ стСпСни, Ρ‚ΠΎ Ρ€Π΅ΡˆΠΈΡ‚Π΅ ΠΈΡ… ΠΏΠΎ ΠΎΡ‚Π΄Π΅Π»ΡŒΠ½ΠΎΡΡ‚ΠΈ.

      Π¨Π°Π³ 3: Π’Π΅ΠΏΠ΅Ρ€ΡŒ Π²Ρ‹ΠΏΠΎΠ»Π½ΠΈΡ‚Π΅ слоТСниС ΠΈΠ»ΠΈ Π²Ρ‹Ρ‡ΠΈΡ‚Π°Π½ΠΈΠ΅ ΠΏΠΎ ΠΌΠ΅Ρ€Π΅ нСобходимости ΠΌΠ΅ΠΆΠ΄Ρƒ Π±Π°Π·ΠΎΠΉ Ρ‚Π΅Ρ€ΠΌΠΈΠ½ΠΎΠ².

      ΠŸΡ€ΠΈΠΌΠ΅Ρ€ 1: Ρ€Π΅ΡˆΠΈΡ‚ΡŒ 6 3 + 6 3 .

      РСшСниС:

      Π—Π΄Π΅ΡΡŒ основаниС ΠΎΠ΄ΠΈΠ½Π°ΠΊΠΎΠ²ΠΎΠ΅, Ρ‚. Π΅. 6 ΠΈ ΠΏΠΎΠΊΠ°Π·Π°Ρ‚Π΅Π»ΠΈ стСпСни Π΄Π²ΡƒΡ… Ρ‡Π»Π΅Π½ΠΎΠ² Ρ‚ΠΎΠΆΠ΅ ΠΎΠ΄ΠΈΠ½Π°ΠΊΠΎΠ²Ρ‹Π΅, Ρ‚. Π΅. 3

      . 2(6) 3

      = 2 x 6 x 6 x 6

      = 432

      ΠŸΡ€ΠΈΠΌΠ΅Ρ€ 2: Ρ€Π΅ΡˆΠΈΡ‚ΡŒ 9 2 – 13 3 .

      РСшСниС:

      Π—Π΄Π΅ΡΡŒ основаниС ΠΈ ΠΏΠΎΠΊΠ°Π·Π°Ρ‚Π΅Π»ΠΈ Ρ€Π°Π·Π½Ρ‹Π΅

      Π˜Ρ‚Π°ΠΊ, ΠΌΡ‹ Ρ€Π΅ΡˆΠ°Π΅ΠΌ ΠΈΡ… ΠΏΠΎ ΠΎΡ‚Π΄Π΅Π»ΡŒΠ½ΠΎΡΡ‚ΠΈ.

      9 2 -13 3 = 9 x 9 -13 x 13 x 13

      = 81 -2197

      = -2116

      АналогичныС вопросы

      Вопрос 1: Solve 5x 3

      53535353 3x 3x 3x 3x 3x 3x 3x 3x 3x 3x 3x 3x 3x 3x 3x 3x 3x 3x 3x 3x 3x 3x 3x 3x 3x 3x 3x 3x 3x 3x 3x

      . 3 .

      РСшСниС:

      Π—Π΄Π΅ΡΡŒ основаниС ΠΎΠ΄ΠΈΠ½Π°ΠΊΠΎΠ²ΠΎΠ΅, Ρ‚. Π΅. x ΠΈ ΠΏΠΎΠΊΠ°Π·Π°Ρ‚Π΅Π»ΠΈ стСпСни Π΄Π²ΡƒΡ… слагаСмых Ρ‚Π°ΠΊΠΆΠ΅ ΠΎΠ΄ΠΈΠ½Π°ΠΊΠΎΠ²Ρ‹, Ρ‚. Π΅. 3

      Π’Π°ΠΊΠΈΠΌ ΠΎΠ±Ρ€Π°Π·ΠΎΠΌ, ΠΌΡ‹ ΠΌΠΎΠΆΠ΅ΠΌ ΡΠ»ΠΎΠΆΠΈΡ‚ΡŒ коэффициСнты Π΄Π²ΡƒΡ… слагаСмых, Ρ‡Ρ‚ΠΎΠ±Ρ‹ ΠΏΠΎΠ»ΡƒΡ‡ΠΈΡ‚ΡŒ Ρ€Π΅Π·ΡƒΠ»ΡŒΡ‚Π°Ρ‚.

      5x 3 + 3x 3 = (5 + 3)x 3

      = 8x 3

      So, 5x 3 + 3x 3 = 8x 3

      Question 2: Π§Ρ‚ΠΎ являСтся Ρ€Π΅Π·ΡƒΠ»ΡŒΡ‚Π°Ρ‚ΠΎΠΌ выраТСния -11a 2 + 4a 2 .

      РСшСниС:

      Π—Π΄Π΅ΡΡŒ основаниС ΠΎΠ΄ΠΈΠ½Π°ΠΊΠΎΠ²ΠΎΠ΅, Ρ‚. Π΅. a ΠΈ ΠΏΠΎΠΊΠ°Π·Π°Ρ‚Π΅Π»ΠΈ стСпСни Π΄Π²ΡƒΡ… слагаСмых Ρ‚Π°ΠΊΠΆΠ΅ ΠΎΠ΄ΠΈΠ½Π°ΠΊΠΎΠ²Ρ‹, Ρ‚. Π΅. 2

      Π’Π°ΠΊΠΈΠΌ ΠΎΠ±Ρ€Π°Π·ΠΎΠΌ, ΠΌΡ‹ ΠΌΠΎΠΆΠ΅ΠΌ ΡΠ»ΠΎΠΆΠΈΡ‚ΡŒ коэффициСнты Π΄Π²ΡƒΡ… слагаСмых, Ρ‡Ρ‚ΠΎΠ±Ρ‹ ΠΏΠΎΠ»ΡƒΡ‡ΠΈΡ‚ΡŒ Ρ€Π΅Π·ΡƒΠ»ΡŒΡ‚Π°Ρ‚.

      -11a 2 + 4A 2 = (-11 + 4) A 2

      = (4 -11) A 2

      = -7a 2

      SO, -1134. 2 + 4a 2 = -7a 2

      Question 3: Solve the expression 4x 3 + 4x 2 – 2x 3 + x 2 – x + 1

      РСшСниС:

      Π—Π΄Π΅ΡΡŒ Ρƒ нас Ρ€Π°Π·Π½Ρ‹Π΅ Ρ‚Π΅Ρ€ΠΌΠΈΠ½Ρ‹ (x 3 , x 2 , Ρ…), Ρ‚. Π΅. основания ΠΎΠ΄ΠΈΠ½Π°ΠΊΠΎΠ²Ρ‹Π΅, Π½ΠΎ Ρ€Π°Π·Π½Ρ‹Π΅ ΠΏΠΎΠΊΠ°Π·Π°Ρ‚Π΅Π»ΠΈ.

      Π˜Ρ‚Π°ΠΊ, ΠΈΠ΄Π΅Π½Ρ‚ΠΈΡ„ΠΈΡ†ΠΈΡ€ΡƒΠΉΡ‚Π΅ ΠΏΠΎΡ…ΠΎΠΆΠΈΠ΅ ΠΏΠ΅Ρ€Π΅ΠΌΠ΅Π½Π½Ρ‹Π΅, сгруппируйтС ΠΈΡ… ΠΈ Π²Ρ‹ΠΏΠΎΠ»Π½ΠΈΡ‚Π΅ слоТСниС/Π²Ρ‹Ρ‡ΠΈΡ‚Π°Π½ΠΈΠ΅ Π½Π° основС Π·Π½Π°ΠΊΠΎΠ² ΠΈ располоТитС Π² полиномиальном порядкС, Ρ‚. Π΅. основания с большСй экспонСнтой сначала ΠΈ мСньшСй Π² ΠΊΠΎΠ½Ρ†Π΅.

      4x 3 + 4x 2 — 2x 3 + x 2 — x + 1 = (4x 3 — 2x 3 ) + (4x 2 3535353353333333333333333333333334 333533333333333333333333 333533333333333333333333333333334) + (4x 2 353533533333333333333333333333334). Ρ… + 1

      = (4 – 2)Ρ… 3 + (4 + 1)Ρ… 2 – Ρ… + 1

      = 2 Ρ… 3 + 5 Ρ… 2 – Ρ… + 1

      РСшСниС:

      Π—Π΄Π΅ΡΡŒ Ρƒ нас Π΅ΡΡ‚ΡŒ 2 Ρ€Π°Π·Π½Ρ‹Π΅ ΠΏΠ΅Ρ€Π΅ΠΌΠ΅Π½Π½Ρ‹Π΅ Π² 2 Ρ‚Π΅Ρ€ΠΌΠΈΠ½Π°Ρ… x, y ΠΈ стСпСни x, y Π² Π΄Π²ΡƒΡ… Ρ‚Π΅Ρ€ΠΌΠΈΠ½Π°Ρ… ΠΎΠ΄ΠΈΠ½Π°ΠΊΠΎΠ²Ρ‹, Ρ‚. Π΅. 3,1 соотвСтствСнно.

      Π’Π°ΠΊΠΈΠΌ ΠΎΠ±Ρ€Π°Π·ΠΎΠΌ, ΠΌΡ‹ ΠΌΠΎΠΆΠ΅ΠΌ ΡΡ‡ΠΈΡ‚Π°Ρ‚ΡŒ, Ρ‡Ρ‚ΠΎ эти 2 Ρ‚Π΅Ρ€ΠΌΠΈΠ½Π° ΠΈΠΌΠ΅ΡŽΡ‚ ΡΠΎΠ²ΠΏΠ°Π΄Π°ΡŽΡ‰ΠΈΠ΅ ΠΏΠ΅Ρ€Π΅ΠΌΠ΅Π½Π½Ρ‹Π΅, ΠΈ ΠΌΡ‹ ΠΌΠΎΠΆΠ΅ΠΌ Π΄ΠΎΠ±Π°Π²Π»ΡΡ‚ΡŒ/Π²Ρ‹Ρ‡ΠΈΡ‚Π°Ρ‚ΡŒ коэффициСнт 2 Ρ‚Π΅Ρ€ΠΌΠΈΠ½ΠΎΠ² Π² зависимости ΠΎΡ‚ Ρ‚Ρ€Π΅Π±ΠΎΠ²Π°Π½ΠΈΠΉ.

      x 3 Y + 4x 3 Y = (1 + 4) x 3 y

      = 5x 3 Y

      Вопрос 5: Π‘ΠΎΡ€Π΅Π²Ρ‚ΠΎ y 2 + 4x – x + 1

      РСшСниС:

      Π—Π΄Π΅ΡΡŒ Ρƒ нас Π΅ΡΡ‚ΡŒ 2 Ρ€Π°Π·Π½Ρ‹Π΅ ΠΏΠ΅Ρ€Π΅ΠΌΠ΅Π½Π½Ρ‹Π΅ Π² 2 Ρ‚Π΅Ρ€ΠΌΠΈΠ½Π°Ρ… x, y ΠΈ ΠΏΠΎΠΊΠ°Π·Π°Ρ‚Π΅Π»ΡŒ стСпСни x Π² 2 Ρ‚Π΅Ρ€ΠΌΠΈΠ½Π°Ρ… ΠΎΠ΄ΠΈΠ½Π°ΠΊΠΎΠ²Ρ‹, Ρ‚. Π΅. 3, Π½ΠΎ ΠΏΠΎΠΊΠ°Π·Π°Ρ‚Π΅Π»ΡŒ стСпСни y Π½Π΅ ΠΎΠ΄Π½ΠΎ ΠΈ Ρ‚ΠΎ ΠΆΠ΅, поэтому ΠΌΡ‹ Π½Π΅ ΠΌΠΎΠΆΠ΅ΠΌ ΡΡ‡ΠΈΡ‚Π°Ρ‚ΡŒ Π΄Π²Π° Ρ‚Π΅Ρ€ΠΌΠΈΠ½Π° ΠΎΠ΄ΠΈΠ½Π°ΠΊΠΎΠ²Ρ‹ΠΌΠΈ ΠΈ Π½Π΅ Π±ΡƒΠ΄Π΅ΠΌ Π²Ρ‹ΠΏΠΎΠ»Π½ΡΡ‚ΡŒ ΠΌΠ΅ΠΆΠ΄Ρƒ Π½ΠΈΠΌΠΈ Π½ΠΈΠΊΠ°ΠΊΠΈΡ… ΠΎΠΏΠ΅Ρ€Π°Ρ†ΠΈΠΉ. (ΠΎΡΡ‚Π°Π»ΠΎΡΡŒ ΠΊΠ°ΠΊ Π΅ΡΡ‚ΡŒ)

      Но Π΅ΡΡ‚ΡŒ Π΅Ρ‰Π΅ Π΄Π²Π° Ρ‡Π»Π΅Π½Π° с Ρ‚ΠΎΠΉ ΠΆΠ΅ Π±Π°Π·ΠΎΠ²ΠΎΠΉ ΠΏΠ΅Ρ€Π΅ΠΌΠ΅Π½Π½ΠΎΠΉ x ΠΈ ΠΏΠΎΠΊΠ°Π·Π°Ρ‚Π΅Π»Π΅ΠΌ стСпСни, Ρ‡Ρ‚ΠΎ ΠΈ 1. ΠŸΠΎΡΡ‚ΠΎΠΌΡƒ ΠΌΡ‹ Π³Ρ€ΡƒΠΏΠΏΠΈΡ€ΡƒΠ΅ΠΌ ΠΈΡ… ΠΈ выполняСм вычислСния с ΠΈΡ… коэффициСнтами.

      x 3 Y + 4x 3 Y 2 + 4x — x + 1 = 4x 3 y 2 + x 3 Y + (4x — x) + 1

      = 40234 x 3 3 .

    Π”ΠΎΠ±Π°Π²ΠΈΡ‚ΡŒ ΠΊΠΎΠΌΠΌΠ΅Π½Ρ‚Π°Ρ€ΠΈΠΉ

    Π’Π°Ρˆ адрСс email Π½Π΅ Π±ΡƒΠ΄Π΅Ρ‚ ΠΎΠΏΡƒΠ±Π»ΠΈΠΊΠΎΠ²Π°Π½. ΠžΠ±ΡΠ·Π°Ρ‚Π΅Π»ΡŒΠ½Ρ‹Π΅ поля ΠΏΠΎΠΌΠ΅Ρ‡Π΅Π½Ρ‹ *